Skip to main content

Full text of "casefilemicrobiology"

See other formats


SECOND EDITION 



TM 



CASE FILES 



MICROBIOLOGY 



• 50 clinical cases with USMLE-style questions 
help you ace course exams and the boards 

• Microbiology pearls highlight key points 

• Primer teaches you how to approach clinical 
problems 

• Proven teaming system maximizes shelf exam scores 



TOY»DeBORD«WANGER 
CASTRO • KETTERING • BRISCOE 




ANGE 



Case Files 

Microbiology 



Second Edition 



f 



Medical 



New York Chicago San Francisco 

Lisbon London Madrid Mexico City 

Milan New Delhi San Juan Seoul 

Singapore Sydney Toronto 



EUGENE C.TOY, MD 

The John S. Dunn, Senior Academic Chief and Program 

Director 

The Methodist Hospital-Houston 

Obstetrics and Gynecology Residency 

Houston,Texas 

Clerkship Director; Clinical Associate Professor 

Department of Obstetrics/Gynecology 

University ofTexas Medical School at Houston 

Houston,Texas 

CYNTHIA DEBORD, PHD 

Former Instructor and Course Director of Medical 

Microbiology 

Department of Microbiology and Molecular Genetics 

University ofTexas Medical School at Houston 

Houston,Texas 

AUDREY WAN GER, PHD 

Associate Professor 

Department of Pathology 

University ofTexas Medical School at Houston 

Houston, Texas 

GILBERT CASTRO, PHD 

Professor (Retired) 

Department of Integrative Biology and Pharmacology 

University ofTexas Medical School at Houston 

Vice President for Inter-Institutional Relations (Retired) 

University ofTexas Health Science Center at Houston 

Houston, Texas 

JAMES D. KETTERING, PHD 

Emeritus Professor 

Department of Biochemistry and Microbiology 
Loma Linda University School of Medicine 
Loma Linda, California 

DONALD BRISCOE, MD 

Program Director 

Family Medicine Residency 

The Methodist Hospital-Houston 

Medical Director Houston Community Health Centers, Inc. 

Denver Harbor Clinic 

Houston, Texas 



TJie McGraw-Hill Companies 



Copyright © 2008 by The McGraw-Hill Companies, Inc. All rights reserved. Manufactured in the 
United States of America. Except as permitted under the United States Copyright Act of 1976, no 
part of this publication may be reproduced or distributed in any form or by any means, or stored 
in a database or retrieval system, without the prior written permission of the publisher. 

0-07-1 643 15-X 

The material in this eBook also appears in the print version of this title: 0-07-149258-5. 

All trademarks are trademarks of their respective owners. Rather than put a trademark 
symbol after every occurrence of a trademarked name, we use names in an editorial fashion only, 
and to the benefit of the trademark owner, with no intention of infringement of the trademark. 
Where such designations appear in this book, they have been printed with initial caps. 

McGraw-Hill eBooks are available at special quantity discounts to use as premiums and sales 
promotions, or for use in corporate training programs. For more information, please contact 
George Hoare, Special Sales, at george_hoare@mcgraw-hill.com or (212) 904-4069. 

TERMS OF USE 

This is a copyrighted work and The McGraw-Hill Companies, Inc. ("McGraw-Hill") and its 
licensors reserve all rights in and to the work. Use of this work is subject to these terms. Except as 
permitted under the Copyright Act of 1976 and the right to store and retrieve one copy of the work, 
you may not decompile, disassemble, reverse engineer, reproduce, modify, create derivative works 
based upon, transmit, distribute, disseminate, sell, publish or sublicense the work or any part of it 
without McGraw-Hill's prior consent. You may use the work for your own noncommercial and 
personal use; any other use of the work is strictly prohibited. Your right to use the work may be 
terminated if you fail to comply with these terms. 

THE WORK IS PROVIDED 'AS IS." McGRAW-HILL AND ITS LICENSORS MAKE NO 
GUARANTEES OR WARRANTIES AS TO THE ACCURACY, ADEQUACY OR COMPLETE- 
NESS OF OR RESULTS TO BE OBTAINED FROM USING THE WORK, INCLUDING ANY 
INFORMATION THAT CAN BE ACCESSED THROUGH THE WORK VIA HYPERLINK OR 
OTHERWISE, AND EXPRESSLY DISCLAIM ANY WARRANTY, EXPRESS OR IMPLIED, 
INCLUDING BUT NOT LIMITED TO IMPLIED WARRANTIES OF MERCHANTABILITY 
OR FITNESS FOR A PARTICULAR PURPOSE. McGraw-Hill and its licensors do not warrant 
or guarantee that the functions contained in the work will meet your requirements or that its 
operation will be uninterrupted or error free. Neither McGraw-Hill nor its licensors shall be liable 
to you or anyone else for any inaccuracy, error or omission, regardless of cause, in the work or for 
any damages resulting therefrom. McGraw-Hill has no responsibility for the content of any 
information accessed through the work. Under no circumstances shall McGraw-Hill and/or its 
licensors be liable for any indirect, incidental, special, punitive, consequential or similar damages 
that result from the use of or inability to use the work, even if any of them has been advised of the 
possibility of such damages. This limitation of liability shall apply to any claim or cause 
whatsoever whether such claim or cause arises in contract, tort or otherwise. 

DOI: 10.1036/0071492585 




Professional 




Want to learn more? 

We hope you enjoy this 
McGraw-Hill eBook! If 
you'd like more information about this book, 
its author, or related books and websites, 
please click here. 



<♦ DEDICATION 



To Phil Robinson, a modern day miracle worker; he came to 

St. Joseph in 2006 amidst turmoil and uncertainty, and with 

leadership, vision, and commitment, directed us to a bright and 

exciting future. 

— ECT 

To my parents, Darrell and Ruth, for their ongoing support; 
to my loving husband, Wes, and children Emily, Elliot, and Evan. 

— CD 



To my wife, Georgia, and daughters, Theresa and Mitzi, who have always 
been the ultimate motivation for me to do my work, and enjoy it. 

— GC 

To my patient wife, Betty, and children, Brian, Pamela, and David. 

— JK 

To Cal, Casey, Peter, Ben, Kristen, Leonard, Eric, and all of their parents. 

— DB 



Copyright © 2008 by The McGraw-Hill Companies, Inc. Click here for terms of use. 



This page intentionally left blank 



Phil Robinson 

Chief Operating Officer, Hospital Partners of America 




Case Files: Microbiology is dedicated to Phil Robinson, FACHE, MHA, who 
serves as Chief Operating Officer of Hospital Partners of America and CEO of 
St. Joseph Medical Center, Houston. With a career in healthcare that has 
spanned 30 years, Phil Robinson's wide experience has stretched across the 
country and in for-profit hospitals and not-for-profit facilities. A graduate of 
Texas A & M University, Phil received his Masters in Health Administration 
from Washington University in St. Louis. He remains active with mentoring 
and teaching at both universities. 

Mr. Robinson has been listed in Modern Healthcare as an "Up and Comer," 
is a graduate of Leadership Houston, and recently received the Distinguished 
Alumnus Award from the Washington University Health Administration 
Program. He also provides leadership and insight to boards of several nation- 
al and local healthcare and civic organizations. He has held executive positions 
with The Methodist Hospital System in Houston, the Alton Ochsner Medical 
Foundation and Ochsner Foundation Hospital in Louisiana, JFK Medical 
Center in Florida, and with HCA. 

In April 2006, Phil accepted the role of interim CEO of St. Joseph Medical 
Center in Houston, Texas, while serving as the Vice President of Operations for 
the Texas Region. Most recently, Phil was promoted to COO for Hospital 
Partners of America and oversees the operational activities of the seven HPA 
medical centers in the United States. For our hospital, St. Joseph, he literally 
resurrected the institution from being money losing to a now dynamic, money 
making, strong hospital. We at St. Joseph, from doctors to nurses, from pharma- 
cists to technicians, and from patients to their families, are grateful to Phil 
Robinson. He is our modern day hero. 



This page intentionally left blank 



For more information about this title, click here 



* CONTENTS 



CONTRIBUTORS IX 

ACKNOWLEDGMENTS xi 

INTRODUCTION xiii 

SECTION I 

Applying the Basic Sciences to Clinical Medicine 1 

Part 1 . Approach to Learning Microbiology 3 

Part 2. Approach to Disease 6 

Part 3. Approach to Reading 6 

SECTION II 

Antimicrobial Therapy 1 1 

Part 1. Antibacterial Agents 13 

Part 2. Antiviral Agents 16 

Part 3. Antifungal Agents 19 

Part 4. Antiparasitic Agents 21 

SECTION III 

Clinical Cases 25 

Fifty Case Scenarios 27 



viii CONTENTS 

SECTION IV 

Listing of Cases 359 

Listing by Case Number 361 

Listing by Disorder (Alphabetical) 362 

INDEX 365 



►> CONTRIBUTORS 



♦> 



Ashley Gullet 

Medical Student, Class of 2008 

University of Texas Medical School at Houston 

Houston, Texas 

Cytomegalovirus 

Hepatitis 

David Parker 

Medical Student, Class of 2008 

University of Texas Medical School at Houston 

Houston, Texas 

Clamydia trachomatis 

Herpes simplex virus 

Human papillomavirus 

Treponema pallidum 



Copyright © 2008 by The McGraw-Hill Companies, Inc. Click here for terms of use. 



This page intentionally left blank 



♦♦♦ 



ACKNOWLEDGMENTS 



The inspiration for this basic science series occurred at an educational retreat 
led by Dr. Maximilian Buja, who at the time was the Dean of the University 
of Texas Medical School at Houston. It has been such a joy to work together 
with Drs. DeBord, Wanger, and Castro, who are accomplished scientists and 
teachers, as well as the other excellent authors and contributors. It has been 
rewarding to collaborate with Dr. Donald Briscoe, a brilliant faculty member. 
Dr. James Kettering is an inspiring author, phenomenal scientist, and virolo- 
gist, and has achieved what all of us strive for a retirement in beautiful 
Northern California. I would like to thank McGraw-Hill for believing in the 
concept of teaching by clinical cases. I owe a great debt to Catherine Johnson, 
who has been a fantastically encouraging and enthusiastic editor. At the 
University of Texas Medical School at Houston, I would like to recognize 
Dr. Samuel Kaplan, Professor and Chair of the Department of Microbiology 
and Molecular Genetics, for his support. At Methodist Hospital, I appreciate 
Drs. Mark Boom, Karin Pollock-Larsen, H. Dirk Sostman, and Judy Paukert 
and Mr. John Lyle and Mr. Reggie Abraham. At St. Joseph Medical Center, 
I would like to recognize our outstanding administrators: Phil Robinson, Pat 
Mathews, Laura Fortin, Dori Upton, Cecile Reynolds, and Drs. John Bertini 
and Thomas V. Taylor. I appreciate Maria Buffington's advice and assistance. 
Without the help from my colleagues, this book could not have been written. 
Most important, I am humbled by the love, affection, and encouragement from 
my lovely wife Terri and our four children, Andy, Michael, Allison, and 
Christina. 

Eugene C. Toy 



Copyright © 2008 by The McGraw-Hill Companies, Inc. Click here for terms of use. 



This page intentionally left blank 



♦♦♦ 



INTRODUCTION 



Often, the medical student will cringe at the "drudgery" of the basic science 
courses and see little connection between a field such as microbiology and 
clinical problems. Clinicians, however, often wish they knew more about the 
basic sciences, because it is through the science that we can begin to under- 
stand the complexities of the human body and thus have rational methods of 
diagnosis and treatment. 

Mastering the knowledge in a discipline such as microbiology is a formi- 
dable task. It is even more difficult to retain this information and to recall it 
when the clinical setting is encountered. To accomplish this synthesis, micro- 
biology is optimally taught in the context of medical situations, and this is 
reinforced later during the clinical rotations. The gulf between the basic sci- 
ences and the patient arena is wide. Perhaps one way to bridge this gulf is with 
carefully constructed clinical cases that ask basic science-oriented questions. 
In an attempt to achieve this goal, we have designed a collection of patient 
cases to teach microbiological related points. More important, the explanations 
for these cases emphasize the underlying mechanisms and relate the clinical 
setting to the basic science data. We explore the principles rather than empha- 
size rote memorization. 

This book is organized for versatility: to allow the student "in a rush" to go 
quickly through the scenarios and check the corresponding answers and to 
provide more detailed information for the student who wants thought- 
provoking explanations. The answers are arranged from simple to complex: a 
summary of the pertinent points, the bare answers, a clinical correlation, an 
approach to the microbiology topic, a comprehension test at the end for rein- 
forcement or emphasis, and a list of references for further reading. The clini- 
cal cases are arranged by system to better reflect the organization within the 
basic science. Finally, to encourage thinking about mechanisms and relation- 
ships, we intentionally used open-ended questions with the clinical cases. 
Nevertheless, several multiple-choice questions are included at the end of each 
scenario to reinforce concepts or introduce related topics. 

HOW TO GET THE MOST OUT OF THIS BOOK 

Each case is designed to introduce a clinically related issue and includes open- 
ended questions usually asking a basic science question, but at times, to break 
up the monotony, there will be a clinical question. The answers are organized 
into four different parts: 



Copyright © 2008 by The McGraw-Hill Companies, Inc. Click here for terms of use. 



xiv INTRODUCTION 

PARTI 

1 . Summary 

2. A straightforward answer is given for each open-ended question 

3. Clinical Correlation — A discussion of the relevant points relating the 
basic science to the clinical manifestations, and perhaps introducing 
the student to issues such as diagnosis and treatment. 

PART II 

An approach to the basic science concept consisting of three parts 

1 . Objectives — A listing of the two to four main principles that are criti- 
cal for understanding the underlying microbiology to answer the ques- 
tion and relate to the clinical situation 

2. Definitions of basic terminology 

3. Discussion of topic 

PART III 

Comprehension Questions — Each case includes several multiple-choice 
questions that reinforce the material or introduces new and related concepts. 
Questions about the material not found in the text are explained in the answers. 

PART IV 

Microbiology Pearls — A listing of several important points, many clinically 
relevant reiterated as a summation of the text and to allow for easy review, 
such as before an examination. 



SECTION I 



Applying the Basic 
Sciences to Clinical 
Medicine 



Part 1 . Approach to Learning Microbiology 
Part 2. Approach to Disease 
Part 3. Approach to Reading 



Copyright © 2008 by The McGraw-Hill Companies, Inc. Click here for terms of use. 



This page intentionally left blank 



APPLYING THE BASIC SCIENCES TO CLINICAL MEDICINE 3 

PART 1. APPROACH TO LEARNING MICROBIOLOGY 

The student of microbiology should be aware of the scientific characteristics 
of each microbe, with a particular interest in the relevance to clinical manifes- 
tations. The following is a systematic three-pronged approach: 

1. How does one know that a person is infected? 

2. Where and how is a person infected? 

3. What can be done if a person is infected? 

1. How does one know that a person is infected? The clinician may have 
a suspicion of a certain etiologic agent based on clinical clues, but this 
educated guess must be corroborated by laboratory confirmation. This 
necessitates an understanding of the basis for presumptive and definitive 
diagnosis. Possible laboratory tests include culture, polymerase chain 
reaction (PCR) of DNA or RNA, antigen tests, or antibody tests. 
2. Where and how is a person infected? This question translates to 
understanding about the mechanisms of disease transmission. For 
example, if a patient is found to be infected with the hepatitis B virus, 
the student should be aware that the most common methods of disease 
acquisition are intravenous drug use, sexual transmission, and vertical 
transmission. Blood transfusion at one time was a common modality, 
but now with screening of banked blood, the incidence is very low. 

3. What can be done if a person is infected? This translates to knowing 
the best treatment and method of prevention of infection. In other 
words, once a patient is known to be infected with a certain microbe, 
what is the best treatment? The student is best served by learning more 
than one antimicrobial therapy and some of the advantages and disad- 
vantages of each therapeutic choice. For example, a urinary tract infec- 
tion caused by Escherichia coli may be treated empirically with a 
variety of antibiotics; however, a quinolone antibiotic such as 
ciprofloxacin is contraindicated in pediatric patients, and gentamicin is 
relatively contraindicated in those with renal insufficiency. 

Likewise, the student should have a systematic approach to classifying 
microorganisms: viruses, bacteria, protozoa, and fungi. 

Virus: A noncellular organism having genetic nucleic acid that requires a host 
to replicate. They are usually 15 to 450 nanometers (nm) in diameter. 
Viruses do not have a cell membrane or cell wall, but they have a rigid pro- 
tein coat called the "capsid." The inner cavity contains DNA or RNA. 
Viruses come in various shapes including spherical, tetrahedral, polygonal, 
rod shaped, and polyhedral. One end is usually broader (head), and one end 
narrower (tail). The tail often has antigenic proteins for attachment to the 
host. Because viruses do not reproduce without a host, they are considered 
obligate parasites and not living. See Figure 1-1 for a schematic of viruses. 



CASE FILES: MICROBIOLOGY 





Genome 
size (kb) 


Envelope 


Capsid 
symmetry 


@ Picornaviridae 


7.2-8.4 


No 


Icosahedral 


@ Caliciviridae 


8 


No 


Icosahedral 


RNA (§) Togaviridae 


12 


Yes 


Icosahedral 


(§} Flaviviridae 


10 


Yes 


Icosahedral 


fffat Coronaviridae 


16-21 


Yes 


Helical 



Retroviruses %^ Retroviridae 


3-9 


Yes 


Icosahedral 



^'\Smsiii Rhabdoviridae 


13-16 


Yes 


Helical 




13 


Yes 


Helical 


uS'I fil < ,, WWVWWV\'> Filnviririae 


R NA Vir uses ' r 


j[£_ Jt Paramyxoviridae 


16-20 


Yes 


Helical 



Orthomyxoviridae 



Segmented 

Negative-strand 

RNA Viruses 



Segmented 
Double-strand 
RNA Viruses 



Bunyaviridae 



Arenaviridae 



Reoviridae 



14 



13-21 



10-14 



16-27 



Yes 



Yes 



Yes 



No 



Helical 



Helical 



Helical 



Icosahedral 



Parvoviridae 



DNA Viruses 




Papovaviridae 



Adenoviridae 



Herpesviridae 



Poxviridae 



5-9 



36-38 



100-250 



240 



No 



No 



No 



Yes 



Yes 



Icosahedral 



Icosahedral 



Icosahedral 



Icosahedral 



Complex 



Figure 1-1. Schematic diagram of selected virus families that are pathogenic 
to humans, approximately to size. 



Bacteria: These single-celled organisms belong in the kingdom 
Prokaryotae, and they usually have a cell wall as an outer covering con- 
sisting of a complex of sugar and amino acids, and often a cell mem- 
brane surrounding the cytoplasm. Being prokaryotes, bacteria do not 



APPLYING THE BASIC SCIENCES TO CLINICAL MEDICINE 5 

have a membrane around their nuclei. Some bacteria have flagella, 
which are cytoplasmic fibrous structures for locomotion. Bacteria may 
be classified according to shape (cocci, bacilli, or vibrio [comma- 
shaped], or spirilla [corkscrew]). Bacteria may also be classified by 
Gram stain characteristics, metabolism requirements (anaerobic versus 
aerobic), and presence or absence of cell wall (Mycoplasma do not have 
a cell wall). See Figure 1-2 for cell wall characteristics of gram-negative 
versus gram-positive bacteria. 

Parasites: Usually consisting of the protozoa and helminths. Helminths are 
parasitic worms usually subdivided into flatworms or platyhelminths 
and roundworms or nemathelminths. 

Protozoa: Parasites in humans belonging to the kingdom Protozoa are clas- 
sified into three phyla: Sarcomastigophora (flagellates and amebas), 
Ciliophora (ciliates), and Apicomplexa (sporozoans). 

Fungi: Eukaryotic organisms growing in two basic forms: yeasts and 
molds. The mold form usually consists of multicellular filamentous 
colonies. Branching cylinder-like tubules form, called hyphae. The yeast 
form are single cells, usually spherical or ellipsoid in shape. Most yeast 
reproduce by budding. When the yeast cells bud but fail to break off, 
they can form elongated yeast cells, called pseudohyphae. Fungi can be 
classified according to their ability to produce superficial versus deep 
invasive infection, or by their appearance or sexual reproduction charac- 
teristics. 




Figure 1-2. Schematic diagram of cell walls of gram-negative versus gram- 
positive bacteria. 



CASE FILES: MICROBIOLOGY 



PART 2. APPROACH TO DISEASE 

Physicians usually approach clinical situations by taking a history (asking ques- 
tions), performing a physical examination, obtaining selective laboratory and 
imaging tests, and then formulating a diagnosis. The conglomeration of the his- 
tory, physician examination, and laboratory tests is called the clinical data- 
base. After reaching a diagnosis, a treatment plan is usually initiated, and the 
patient is followed for a clinical response. Rational understanding of disease 
and plans for treatment are best acquired by learning about the normal human 
processes on a basic science level, and likewise, being aware of how disease 
alters the normal physiological processes is understood on a basic science level. 

Clinicians should be aware of the laboratory methods of diagnosis, includ- 
ing the advantages and disadvantages, cost, time requirements, and potential 
morbidity to the patient. Various laboratory techniques include detecting DNA 
or RNA sequences, identifying certain protein components of the microorgan- 
ism (antigen), or unique enzyme or toxin; microscopic examination such as 
Gram stain (most bacteria), acid-fast stain (Mycobacterium), and immunoflu- 
orescence techniques (used to detect difficult-to-culture organisms such as 
Legionella). Cultures are the traditional method of diagnosis, and they must be 
taken in such a way as to minimize contamination and placed on the appro- 
priate media (or mammalian cell for viruses), with temperature and atmos- 
pheric conditions for optimal amplification. Thereafter, the correct 
identification process should be used to assess characteristics such as colony 
morphology (both grossly and under the microscope), hemolytic pattern on agar, 
fermentation profile, Gram stain appearance, and the like. 

Once the organism is identified, then susceptibility testing is generally per- 
formed to assess the likelihood that certain antimicrobial agents will be effec- 
tive against the particular strain of pathogen. For example, isolates of 
Staphylococcus aureus should be tested against (3-lactam antibiotics such as 
methicillin to aid the clinician in treating with methicillin versus vancomycin. 
Susceptibility is generally performed in a qualitative manner (susceptible, 
intermediate, resistant), or quantitative with minimum inhibitory concentra- 
tions (MIC) or minimum bactericidal concentrations (MBC) as determined by 
successive dilutions of the isolate bathed in antimicrobial mixtures. 

PART 3. APPROACH TO READING 

There are seven key questions that help to stimulate the application of basic 
science information to the clinical setting. 

1. Given a particular microorganism, what is the most likely clinical 
manifestation? 

2. Given a particular microorganism, what is the mechanism 
whereby clinical or subclinical findings arise? 

3. Given clinical symptoms of infection, what is the most likely causative 
microorganism? 



APPLYING THE BASIC SCIENCES TO CLINICAL MEDICINE 7 

4. Given clinical findings, what are the most likely associated features 
of the microorganism (such as cell wall characteristics or viral 
genome)? 

5. Given the clinical findings, what is the most likely vector of 
transmission? 

6. Given the clinical findings, what is the most likely laboratory culture 
findings? 

7. Given a particular microorganism, what is the most likely mecha- 
nism of resistance acquisition? 

1. Given a particular microorganism, what is the most likely clinical 
manifestation? 

This is the fundamental knowledge that the student must learn in the 
broad scope of microbiology, that is, the most likely presentation of 
clinical disease. Each organism has certain typical patterns of clinical 
manifestations, based on its characteristics. The interaction between 
microbe and host, including replication cycle, enzymes released, and 
host immune response all play roles in the overt symptoms and signs. 
The student should have an understanding of the common presenta- 
tions of disease. Likewise, the student should be aware of the mecha- 
nisms of the clinical manifestations. 

2. Given a particular microorganism, what is the mechanism 
whereby clinical or subclinical findings arise? 

The student of microbiology is often tempted to memorize the exten- 
sive list of microorganisms and clinical disease. Unfortunately, this hap- 
hazard approach leads to quick forgetfulness and lack of understanding of 
the basic science underlying this discipline. Instead, the student should 
approach the field of microbiology by linking the microorganism, and its 
particular characteristics, to the mechanisms of disease, such as the inter- 
action with host physiological state, cellular abnormalities that develop, 
and even cell death. It is the ability of the student to understand the mech- 
anisms that allows for rational approaches to diagnosis and treatment. 

3. Given clinical symptoms of infection, what is the most likely 
causative microorganism? 

The student of microbiology first learns antegrade from microbe 
toward disease, but patients present instead with disease symptoms and 
signs. Thus, the student must be able to work backward from clinical 
presentation to a differential diagnosis (a list of the most likely etiolo- 
gies) to the probable causative organism. Again, rather than memo- 
rization, the student should incorporate mechanisms in the learning 
process. Thus, the student would best be served to give a reason why 
the suspected microorganism causes a certain clinical presentation. 

Microorganism — » Host defenses/response to invading organism 

— > Clinical signs 

Clinical signs — > Most likely microbe(s) 



CASE FILES: MICROBIOLOGY 



4. Given clinical findings, what are the most likely associated features 
of the microorganism (such as cell wall characteristics or viral 
genome)? 

This is similar to question 3, but it goes back to the underlying basic 
science peculiarities of the microbe. First, the student must use the clin- 
ical information to discern the likely microorganism, and then the stu- 
dent must be able to relate the characteristics of the microbe. Because a 
common and effective method of classifying viruses includes the viral 
genome, this is an important differentiating point. Likewise, bacteria are 
often subdivided by their cell wall characteristics, which lead to their 
Gram stain findings. For example, the clinical information may be: "A 
66-year-old male complains of blisters erupting on the right chest wall 
region associated with pain and tingling." The student should be able to 
remember that a vesicular rash that is unilaterally associated with pain 
and tingling is characteristic for herpes zoster. The etiologic agent is 
varicella-zoster, which causes chickenpox. The virus can lay dormant in 
the dorsal root ganglia and, then during times of stress or immunocom- 
promised states, travel down the nerve and cause local eruption on the 
dermatome distribution. The pain and tingling are caused by the stimu- 
lation of the nerve. The student may recall that varicella is a her- 
pesvirus, and thus is a double-stranded DNA virus. 

5. Given the clinical findings, what is the most likely vector of 
transmission? 

Again, the student must first discern the most likely etiologic agent 
based on the clinical findings. Then, an understanding of how the 
microbe causes disease, such as vector of transmission, is important. A 
related topic is preventive or treatment related, such as how to sterilize 
equipment exposed to the patient, or the best antibiotic therapy for the 
described patient. 

6. Given the clinical findings, what are the most likely laboratory 
findings? 

Based on the clinical presentation, the student should discern the 
most likely microbe and then its laboratory characteristics. These lab- 
oratory findings should be correlated to its mechanisms of disease. For 
example, the clinical findings are: "A 10-year-old boy presents with 
sore throat and fever. On examination, there is exudate in the orophar- 
ynx. A Gram stain reveals gram-positive cocci in chains." The student 
should be aware that a culture would reveal (3-hemolytic pattern on 
blood agar media. Furthermore, the mechanism should be learned, that 
is, that the group A Streptococcus gives off hemolysin, which leads to 
hemolysis of the red blood cells. 

7. Given a particular microorganism, what is the most likely mecha- 
nism of resistance acquisition? 

Patterns of antimicrobial resistance are enormous concerns for all 
involved in medical sciences. Bacteria and viruses are increasingly 



APPLYING THE BASIC SCIENCES TO CLINICAL MEDICINE 9 

acquiring mechanisms of resistance, which spurs scientists to design 
new antibiotics or chemicals that disable the microbial method 
of resistance. For example, if the mechanism of resistance is a 
p-lactamase enzyme, then the addition of a (3-lactamase inhibitor such 
as sulbactam may lead to increased efficacy of a p-lactam such as 
ampicillin. 



MICROBIAL PEARLS 

*♦* There are seven key questions to stimulate the application of basic 
science information to the clinical arena. 

*♦* The student of microbiology should approach the discipline in a sys- 
tematic manner, by organizing first by bacteria, viruses, fungi, 
and protozoa, and then subdividing by major characteristics. 

*♦* Learning microbiology can be summarized as a threefold approach: 
(1) Basis for presumptive and definitive diagnosis (i.e., understand- 
ing how one knows that a person is infected), (2) disease transmis- 
sion (i.e., where and how a person is infected), (3) treatment and 
prevention of infection (i.e., what can be done if a person is 
infected). 

*♦* The skilled clinician must be able to translate back and forth 
between the basic sciences and the clinical sciences. 






REFERENCES 

Brooks GF, Butel JS, Morse SA. The science of microbiology. In: Medical Micro- 
biology. New York: McGraw-Hill, 2004:1-7. 

Madoff LC, Kasper DL. Introduction to infectious disease. In: Kasper DL, Fauci 
AS, Longo DL, eds. Harrison's Principles of Internal Medicine, 16th ed. New 
York: McGraw-Hill, 2005:695-99. 



This page intentionally left blank 



SECTION II 



Antimicrobial Therapy 



Part 1 . Antibacterial Agents 

Part 2. Antiviral Agents 

Part 3. Antifungal Agents 

Part 4. Antiparasitic Agents 



Copyright © 2008 by The McGraw-Hill Companies, Inc. Click here for terms of use. 



This page intentionally left blank 



ANTIMICROBIAL THERAPY 13 

Antimicrobial agents consist of antibacterial, antiviral, antifungal, and 
antiparasitic medications. These drugs take advantage of the different struc- 
tures or metabolism of the microbes versus human cells. The student should 
group classes of drugs together rather than memorize each individual agent. A 
systematic approach includes learning the agent, the mechanism of action, and 
the spectrum of activity. 

PART 1. ANTIBACTERIAL AGENTS 

Antibacterial agents, which target specific components of microorganisms that 
are unique or more essential to their function than they are to humans, are clas- 
sified according to their mechanisms of action. The component targets include 
enzymes necessary for bacterial cell wall synthesis, the bacterial ribosome, 
and enzymes necessary for nucleotide synthesis and DNA replication. 

Resistance of pathogens to antibacterial and other chemotherapeutic agents 
may be the result of a natural resistance or may be acquired. In either case, it 
occurs through mutation, adaptation, or gene transfer. The mechanisms of 
resistance for any antibacterial agent vary, but are consequences of either 
changes in uptake of drug into, or its removal from, the bacterial cell, or to 
changes in the bacterial cell target site of the drug from a gene mutation. 
Multiple drug resistance is also a major impediment to antibacterial therapy 
and may be chromosomal or plasmid mediated where genetic elements from 
resistant bacteria that code for enzymes that inactivate antibacterial agents are 
transferred to nonresistant bacteria. The emergence of drug resistance is to a 
large degree the result of the widespread and often unnecessary or inappropri- 
ate use of antibiotics in humans. 

The penicillins include natural penicillins, penicillins that are resistant 
to staphylococcal [3-lactamase, and extended-spectrum penicillins (see 
Table II-l). The cephalosporins are classified as first to fourth generation 
according to their antibacterial spectrum (see Table II-2). Aztreonam, which 
is relatively (3-lactamase resistant, is the only available monobactam. It is 
nonallergenic and is active only against aerobic gram-negative bacilli (e.g., 
Pseudomonas, Serratia). See Table II-3 for a listing of selected antibacterial 
agents. The carbapenems (imipenem, meropenem, and ertapenem), which are 
resistant to most [3-lactamases, have a wide spectrum of activity against gram- 
positive and gram-negative rods and anaerobes. To prevent its metabolism to 
a nephrotoxic metabolite, imipenem is administered with an inhibitor of 
renal tubule dehydropeptidase, cilastatin. Vancomycin, which is unaffected 
by (3-lactamases, inhibits bacterial cell wall synthesis by covalent binding to 
the terminal two D-alanine residues of nascent peptidoglycan pentapeptide to 
prevent their elongation and cross-linking, thus increasing the susceptibility of 
the cell to lysis. It is active against gram-positive bacteria. 



14 



CASE FILES: MICROBIOLOGY 



Table II-l 

PARTIAL LISTING OF PENCILLINS 



Natural penicillins: Penicillins G (prototype); Penicillin V 
B-lactamase resistant: Nafcillin; Oxacillin; Cloxacillin; Dicloxacillin 



Extended-spectrum 

Aminopenicillin: Ampicillin; Amoxicillin 
Ureidopenicillins: Mezlocillin; Piperacillin 
Carboxypenicillin: Ticarcillin 



Table II-2 

SELECTED LISTING OF CEPHALOSPORINS 



REPRESENTATIVE 
CEPHALOSPORINS (ROUTE) 


NOTES 


1 st generation 
-Cefazolin (iv) 
-Cephalexin (po) 
-Cefadroxil (po) 


Active against gram-positive cocci, including 
staphylococci, pneumococci, and streptococci. 
They are particularly good for soft tissue and 
skin infections. 


2nd generation 
-Cefuroxime (po) 
-Cefoxitin (iv) 
-Cefotetan (iv) 


Marked differences in spectrum of activity; 
generally, active against certain aerobic 
gram-negative bacteria and somewhat less 
active against many gram-positive organisms 
sensitive to 1st generation cephalosporins. 
Cefuroxime more active against Haemophilus 
influenza, and cefoxitin more active against 
Bacteroides fragilis. 


3rd generation 
-Cefotaxime (iv) 
-Ceftazidime (iv) 
-Ceftriaxone (iv) 


Expanded aerobic gram-negative spectrum 
and cross and blood-brain barrier. Useful to 
treat bacterial strains resistant to other drugs. 


4th generation 
-Cefepime (iv) 


Generally similar activity to 3rd generation 
cephalosporins but more resistant to 
P-lactamases. 



iv = intravenous; po = oral. 



ANTIMICROBIAL THERAPY 



75 



Table II-3 

PARTIAL LISTING OF ANTIBACTERIAL AGENTS 



ANTIBACTERIAL AGENTS 


MECHANISM OF ACTION 


P-LACTAM ANTIBIOTICS 
Penicillins 
Cephalosporins 
Monobactams 

- aztreonam (p) 
Carbapenems 

- imipenem (p) 

- meropenem (p) 

- ertapenem (p) 


Inhibit synthesis of the bacterial cell wall. 


Vancomycin (o,p) 


Inhibits synthesis of bacterial cell wall. 


Chloramphenicol 
Tetracyclines 

- tetracycline (o.p) 

- oxytetracycline (o,p) 

- doxycycline (o.p) 

- methacycline (o) 

- minocycline (o.p) 
Macrolides 

- erythromycin (o.p) 

- clarithromycin (o) 

- azithromycin (o) 
Ketolides 

- telithroinycin (o) 
Oxazolidones 

- linezolid (o,p) 
Aminoglycosides 
Spectinomycin (p) 
Lincomycins 

- clindamycin (o,p) 


Bind to bacterial ribosomes to inhibit 
protein synthesis. 


SULFONAMIDES 

- sulfadiazine (o) 

- sulfamethizole (o) 

- sulfamethoxazole (o) 

- sulfanilamide (t) 

- sulfisoxazole (t.o) 
Trimethoprim 


Sulfonamides: Structural analogs of 
p-aminobenzoic acid that inhibit bacterial 
dihydropteroate synthase to block folic acid 
synthesis and cell growth. Trimethoprim: 
Selectively inhibits dihydrofolic acid eductase 
to block folic acid synthesis and cell growth. 
Acts synergistically with sulfmethoxazole. 


Fluoroquinolones (selected) 

- ciprofloxacin (t.o.p) 

- levofloxacin (t.o.p) 

- ofloxacin (t.o.p) 

- gatifloxacin (o,p) 

- moxifloxacin (o.p) 


Inhibit activity of bacterial topoisomerase 
(DNA gyrase) that is necessary 
for replication. 



t = topical, o = oral, p = parenteral 



16 



CASE FILES: MICROBIOLOGY 



PART 2. ANTIVIRAL AGENTS 

The three major classes of antiviral agents are DNA polymerase 
inhibitors, reverse transcriptase inhibitors, and protease inhibitors. It 

should be noted that HIV treatment usually includes the use of at least two 
reverse transcriptase inhibitors and one protease inhibitor. DNA polymerase 
inhibitors are subdivided into nucleoside and nonnucleoside. Drugs may target 
viral nucleic acid replication such as DNA polymerase either via nucleoside 
(purine or pyrimidine analogs) such as acyclovir or ribavirin, or by attacking 
a unique viral process needed in nucleic acid synthesis such as viral pyrophos- 
phate (nonnucleoside type). Antiviral drugs used to treat herpes simplex virus 
(HSV), varicella-zoster virus (VZV), and Cytomegalovirus (CMV), can be 
classified as either nucleosides or nonnucleosides, or according to their site of 
action in the viral replicative cycle, or according to their clinical use. 



Common Antiviral Agents 

Influenza. Amantadine and rimantadine are primarily used against infec- 
tions caused by the influenza A virus. Their mechanism of action is interfer- 
ing with viral uncoating. Both agents are fairly well absorbed orally and 
cause some minor central nervous system (CNS) effects (rimantadine less so) 
and minor gastrointestinal (GI) effects (see Table II-4 for partial listing). 



Table II-4 

PARTIAL LISTING OF ANTIVIRAL MEDICATIONS 



Hepatitis B and C 


Lamivudine, adefovir, interferon alfa, and ribavirin. 


Influenza 


Amantadine and rimantadine (nonnucleosides that inhibit 
uncoating), zanamivir and oseltamivir (nonnucleosides that 
inhibit release and budding). 


HIV-1 


Nucleoside reverse transcriptase inhibitors (NRTIs; abacavir, 

didanosine, lamivudine, stavudine, zalcitabine, zidovudine) 

Nucleotide inhibitors (tenofovir) 

Nonnucleotide reverse transcriptase inhibitors (NNRTIs; 

delavirdine, efavirenz, nevirapine) 

Protease inhibitors (amprenavir, indinavir, nelfinavir, ritonavir 

saquinavir) 

Fusion inhibitors (enfuvirtide) 



ANTIMICROBIAL THERAPY 



17 



Herpes Virus 

Acyclovir is used against herpes simplex virus 1 and 2. Acyclovir, a nucle- 
oside DNA polymerase inhibitor, is a deoxyguanosine triphosphate 
(dGTP) analog, which is incorporated into the viral DNA and causes DNA 
chain termination. Its specificity is a result of the presence of herpes specific 
thymidine kinase in infected cells, which phosphorylates acyclovir 100 times 
more efficiently than by uninfected cells. Acyclovir can be used topically, orally 
for recurrent genital herpes, and intravenously for immunocompromised patients 
or herpes encephalitis. Its adverse effects include headache, nausea, and rarely 
nephrotoxicity with intravenous use. Valacyclovir is an analog of acyclovir and 
is converted to acyclovir in the body. Its advantage is better bioavailability. See 
Table II-5 for listing of agents against herpes viruses. 



Table II-5 

AGENTS USED TO TREAT HERPES VIRUSES 
(ROUTE OF ADMINISTRATION) 



AGENTS 


VIRAL INFECTIONS 


Acyclovir (t,o,p) 


HSV, VZV 


Cidofovir (p) 


VZV, CMV 


Famciclovir (o) 


HSV, VZV 


Ganciclovir (o,p) 


CMV 


Penciclovir (t) 


HSV, VZV 


Idoxuridine (o) 


HSV 


Valacyclovir (o) 


HSV, VZV 


Trifluridine (t) 


HSV 


Foscarnet (p) 


HSV, CMV 


Fomivirsen (p) 


CMV 



HSV = herpes simplex virus; CMV = cytomegalovirus; VZV = varicella-zoster virus; t = topical, 
o = oral, p = parenteral 



18 CASE FILES: MICROBIOLOGY 

Penciclovir is converted to the triphosphate form and inhibits viral DNA 
polymerase. Famciclovir is converted to the active agent penciclovir in the 
body. Penciclovir is used topically to treat herpes labialis "cold sores;" famci- 
clovir is used for genital herpes or herpes zoster. Headache and GI effects are 
common. Ganciclovir is structurally similar to acyclovir and must be con- 
verted to the triphosphate form to be active; it competes with dGTP for incor- 
poration into viral DNA, thereby inhibiting DNA polymerase. Its primary role 
is against Cytomegalovirus and is far more effective than acyclovir against 
CMV. Ganciclovir can induce serious myelosuppression. 

Foscarnet is a synthetic nonnucleoside analog of pyrophosphate and 
inhibits DNA polymerase or HIV reverse transcriptase by directly binding to 
the pyrophosphate binding site. Its use is usually for acyclovir resistant her- 
pes or CMV retinitis. Significant nephrotoxicity may occur with its use. 

Sorivudine is a pyrimidine nucleoside analog and, on being converted to 
the triphosphate form, is active against herpes DNA synthesis. It is effective 
against varicella-zoster and is usually well tolerated. Idoxuridine is an iodi- 
nated thymidine analog, which inhibits herpes DNA synthesis in its 
triphosphate form. It is used primarily topically for herpes keratitis. Adverse 
effects include pain and inflammation. Vidarabine is an adenosine analog, 
which also needs to be in its triphosphate form and blocks herpes specific 
DNA polymerase. It has been used for herpes encephalitis or zoster in 
immunocompromised individuals, but because of its nephrotoxicity, it has 
largely been supplanted by acyclovir. Trifluridine is a fluorinated pyrimi- 
dine nucleoside analog. Its monophosphate form inhibits thymidylate syn- 
thetase and triphosphate form inhibits DNA polymerase. It is active against 
herpes 1 and 2 and CMV, and it is used primarily against keratoconjunctivitis 
and recurrent keratitis. 

Anti-HIV Agents 

Retrovir (azidothymidine [AZT] or zidovudine [ZDV]) inhibits viral reverse 
transcriptase when its triphosphate form is incorporated into the nucleic acid 
and blocks further DNA chain elongation, leading to termination of the 
DNA. Also, the monophosphate form of Retrovir may block deoxythymidine 
kinase and inhibit the production of normal dTTp. Its principal role is treating 
HIV infection, and adverse effects include headache, bone marrow suppres- 
sion, fever, and abdominal pain. 

Didanosine is also a nucleoside reverse transcriptase inhibitor, primarily 
used as an adjunct to Retrovir, or for those patients with HIV infection intol- 
erant or unresponsive to zidovudine. Peripheral neuropathy and pancreatic 
damage are its side effects. Stavudine is a thymidine nucleoside analog that 
inhibits HIV-1 replication and that is used in HIV patients unresponsive to 
other therapies. 



ANTIMICROBIAL THERAPY 19 

Protease Inhibitors 

Invirase or saquinavir blocks HIV protease activity, rendering the virus 
unable to generate essential proteins and enzymes including reverse transcrip- 
tase. It is used in combination with a conventional reverse transcriptase 
inhibitor. 

PART 3. ANTIFUNGAL AGENTS 

In addition to the pyrimidine analog, flucytosine, and the penicillium-derived 
antifungal agent, griseofulvin, the three major classes of antifungal agents are 
the polyene macrolides, azoles, and allylamines (Table II-6). 

Of all the available antifungal agents, amphotericin B has the broadest 
spectrum of activity, including activity against yeast, mycoses, and molds. 
It is the drug of choice for disseminated or invasive fungal infections in 
immunocompromised patients. The major adverse effect resulting from 



Table II-6 

SELECTED ANTIFUNGAL DRUGS 

POLYENE MACROLIDES 

Nystatin (o) 

Natamycin (t) 

Amphotericin B (t, o for GI tract, p) 

AZOLES 

Miconazole (t) 
Ketoconazole (t,o) 
Clotrimazole (t) 
Itraconazole (o,p) 
Fluconazole (o,p) 
Voriconazole (o,p) 

ALLYLAMINES 

Naftifine (t) 
Terbinafine (o,t) 

OTHER ANTIFUNGAL AGENTS 

Flucytosine (o) 
Griseofulvin (o) 

t = topical; o = oral; p = parenteral 



20 CASE FILES: MICROBIOLOGY 

amphotericin B administration is the almost invariable renal toxicity that 
results from decreased renal blood flow and from tubular and basement mem- 
brane destruction that may be irreversible and may require dialysis. Other 
adverse effects of amphotericin B relate to its intravenous infusion and include 
fever, chills, vomiting, hypotension, and headache that can be ameliorated 
somewhat by careful monitoring and slow infusion. 

The azole antifungal agents have a broad spectrum of activity, including 
activity against candidiasis, mycoses, and dermatophytes, among many oth- 
ers. As topical agents they are relatively safe. Administered orally, their most 
common adverse effect is GI dysfunction. Hepatic dysfunction may rarely 
occur. Oral azoles are contraindicated for use with midazolam and triazo- 
lam because of potentiation of their hypnotic and sedative effects, and with 
p-hydroxy-p-methylglutaryl-coenzyme A (HMG CoA) reductase inhibitors 
because of an increased risk of rhabdomyolysis. Itraconazole has been 
associated with heart failure when used to treat onychomycosis and, there- 
fore, should not be used in patients with ventricular abnormalities. 
Monitoring patients who receive itraconazole for potential hepatic toxicity is 
also highly recommended. Voriconazole frequently causes an acute blurring 
of vision with changes in color perception that resolves quickly. 

The allylamine antifungal agents, naftifine and terbinafine, are used 
topically to treat dermatophytes. Contact with mucous membranes may lead 
to local irritation and erythema and should be avoided. Terbinafine adminis- 
tered orally is effective against the onychomycosis. Monitoring for potential 
hepatic toxicity is highly recommended. 

Flucytosine is active against only a relatively restricted range of fungal 
infections. Because of rapid development of resistance, it is used concomi- 
tantly for its synergistic effects with other antifungal agents. The most com- 
monly reported adverse effect is bone marrow suppression, probably 
because of the toxicity of the metabolite fluorouracil, that should be contin- 
uously monitored. Other reported less common adverse effects include 
reversible hepatotoxicity, enterocolitis, and hair loss. 

Griseofulvin, the use of which is declining relative to terbinafine and itra- 
conazole, is an effective antifungal agent that is used only systemically to treat 
a very limited range of dermatophyte infections. The most common adverse 
effects include hypersensitivity (fever, skin rash, serum-sickness-like syn- 
drome) and headache. It is teratogenic. 

Mechanism of Action 

Nystatin and amphotericin B bind to ergosterol, a major component of 
fungal cell membranes. This disrupts the stability of the cell by forming 
pores in the cell membrane that result in leakage of intracellular con- 
stituents. Bacteria are not susceptible to these agents because they lack 
ergosterol. 



ANTIMICROBIAL THERAPY 21 

Azoles (imidazoles less so) have a greater affinity for fungal than human 
cytochrome P 450 enzymes and, therefore, more effectively reduce the 
synthesis of fungal cell ergosterol than human cell cholesterol. 

The allylamine antifungal agents, naftifine and terbinafine, decrease ergos- 
terol synthesis and increase fungal membrane disruption by inhibiting 
the enzyme squalene epoxidase. 

Flucytosine must first be transported into fungal cells via a cytosine per- 
mease and converted to 5-fluorouracil and then sequentially converted to 
5-fluorodeoxyuridylic acid, which disrupts DNA synthesis by inhibiting 
thymidylate synthetase. Human cells are unable to synthesize the active 
flucytosine metabolites. 

The mechanism of antifungal action of griseofulvin is not definitely known. 
It acts only on growing skin cells and has been reported to inhibit cell 
wall synthesis, interfere with nucleic acid synthesis, and disrupt micro- 
tubule function, among other activities. 

Administration 

Amphotericin B is insoluble in water and, therefore, is generally administered 
as a colloidal suspension with sodium deoxycholate. Because of its poor 
absorption from the GI tract, amphotericin B must be given intravenously to 
treat systemic disease, although it is effective orally for fungal infections 
within the GI lumen. Likewise, nystatin is poorly absorbed but may also be 
used for fungal infection of the GI tract. It is too toxic for systemic use and, 
therefore, is mostly used topically to treat fungal infections of the skin and 
mucous membranes (e.g., oropharyngeal thrush, vaginal candidiasis). Costly 
lipid formulations of amphotericin B are available for intravenous use which 
reduce its nonspecific binding to cholesterol of human cell membranes and, 
therefore, its potential to cause renal damage. Griseofulvin is administered in 
a microparticulate form to improve absorption. 

PART 4. ANTIPARASITIC AGENTS 

Parasitic infections affect half the world's population and are particularly 
prevalent in developing countries. Immunocompromised individuals such as 
those with HIV infection are also prone to parasitic disease. These medications 
can be categorized as active against malaria, toxoplasmosis, cyclospora, Cryp- 
tosporidia, Pneumocystis, amebiasis, leishmaniasis, helminths, trematodes, 
and cestodes (see Table II-7). 



22 



CASE FILES: MICROBIOLOGY 



Table II-7 

DRUGS FOR PARASITES 





PROPHYLAXIS 


rREATMENT 


Malaria 


• Chloroquine « 


Chloroquine 2nd drug for falciparum: 


(erythrocytic) 


• Mefloquine 


1 Quinine • Fansidar 




• Doxycycline 


1 Quinidine • Doxycycline 
• Clindamycin 


Malaria 


• Primaquine 


1 Primaquine 


(hepatic) 


• Etaquine 




Toxoplasmosis 


• Trimethoprim/ 


1 Pyrimethamine/Sulfadiazine 




Sulfamethoxazole 


1 Pyrimethamine/Clindamycin 


Cyclospora 


• Trimethoprim/ 


1 Trimethoprim/Sulfamethoxazole 




Sulfamethoxazole 


1 Pyrimethamine/Sulfadiazine 


Cryptosporidiosis 




1 Antiretrovirals 

1 Paromomycin/ Azithromycin 


Pneumocystis 


• Trimethoprim/ 
Sulfamethoxazole 


1 Trimethoprim/Sulfamethoxazole 




• Dapsone 


1 Trimethoprim/Dapsone 




• Pentamidine (inhaled) 


1 Clindamycin/Primaquine 




• Atovaquone 


1 Atozaquine 

1 Pentamidine (IV) 


Amebiasis 




1 Metronidazole 
1 Iodoquinol 


Giardiasis 




1 Metronidazole 
1 Paromomycin 


Leishmaniasis 




1 Pentamidine 

1 Amphotericin B 

1 Azole antifungal drugs 


Antihelminthics 




1 Mebendazole 


(intestinal worms) 




1 Albendazole 
1 Thiabendazole 
1 Pyrantel pamoate 


Antitrematodes & 




1 Praziquantel 


Anticestodes 




1 Albendazole 


(Flukes, 






Schistosomiases) 







Source: Harrison 's Internal Medicine, 2004;1202-8. 



ANTIMICROBIAL THERAPY 23 



ANTIMICROBIAL PEARLS 

*♦* (3-lactam antibiotics inactivate bacterial transpeptidases and prevent 

the cross-linking of peptidoglycan polymers essential for cell 

wall integrity. 

*♦* Both penicillin and amoxicillin are susceptible to (3-lactamases. 
4A 



V* To prevent its metabolism to a nephrotoxic metabolite, imipenem is 

administered with an inhibitor of renal tubule dehydropeptidase, 

cilastatin. 
*♦* Vancomycin, which is unaffected by (3-lactamases, is active against 

gram-positive bacteria. 
*♦* Chloramphenicol can cause GI disturbances, reversible suppression 

of bone marrow, and rarely plastic anemia. 
*♦* Aminoglycosides may cause ototoxicity or nephrotoxicity and 

should be used with caution in those patients who have renal 

insufficiency or who are elderly. 
*♦* The primary strategy of antiviral agents is to attack a unique but vital 

viral enzyme or process. 
*♦* The three major types of antiviral agents include DNA polymerase 

inhibitors, reverse transcriptase inhibitors, and protease inhibitors. 
*♦* HIV therapy usually uses at least two reverse transcriptase inhibitors 

and one protease inhibitor. 
*♦* Didanosine is also a nucleoside reverse transcriptase inhibitor for 

HIV infections and is associated with peripheral neuropathy and 

pancreatic damage. 
*♦* Foscarnet is a synthetic nonnucleoside analog of pyrophosphate and 

is associated with reversible nephrotoxicity; hypo- or hypercalcemia 

and phosphatemia that may lead to neural and cardiac dysfunction. 

Also, hallucinations, genital ulceration, and anemia may occur. 
*♦* Itraconazole has been associated with heart failure when used to 

treat onychomycosis and, therefore, should not be used in 

patients with ventricular abnormalities. 
*♦* A common side effect of griseofulvin is hypersensitivity. 
*♦* Because of renal toxicity, amphotericin B is often used to initiate a 

clinical response before substituting a continuing maintenance 

dose of an azole. 



24 CASE FILES: MICROBIOLOGY 

REFERENCES 

Gale EF, Cundliffe E, Reynolds PE, et al. The Molecular Basis of Antibiotic Action, 

2nd ed. London: Wiley, 1981. 
Groll A, Piscitelli SC, Walsh TJ. Clinical pharmacology of systemic antifungal 

agents: a comprehensive review of agents in clinical use, current investigational 

compounds, and putative targets for antifungal drug development. Adv 

Pharmacol 1998;44:343. 
Levy SB. The challenge of antimicrobial resistance. Sci Am 1998;278:46. 
Sarosi GA, Davies SF. Therapy for fungal infections. Mayo Clin Proc 

1994;69:1111. 
Stevens DA, Bennett JE. Antifungal agents. In: Mandell GL, Bennett JE, Dolin R, 

eds. Principles and Practices of Infectious Diseases, 5th ed. Philadelphia, PA: 

Churchill Livingstone, 2000:448. 
Wright AJ. The penicillins. Mayo Clin Proc 1999;74:290. 



SECTION III 



Clinical Cases 



Copyright © 2008 by The McGraw-Hill Companies, Inc. Click here for terms of use. 



This page intentionally left blank 



♦♦♦ CASE1 

A 53-year-old male farmer presents for evaluation of a growth on his arm. 
About a week ago, he noticed some mildly itchy red bumps on his arm. They 
started to blister a day or two later and then ruptured. During this time he had 
a low-grade fever, but otherwise felt well. Further questioning reveals that he 
has had no ill contacts and never had anything like this before. He has cows, 
horses, goats, sheep, and chickens on his farm. On examination of his right 
upper arm, you find a 4.5-cm circular black eschar surrounded by several 
vesicles (blisters) and edema. He has tender axillary lymph node enlargement 
(adenopathy). A Gram stain of fluid drained from a vesicle and a biopsy from 
the eschar both show chains of gram-positive bacilli on microscopy. 

^ What organism is the likely cause of this disease? 

^ What are the primary virulence factors of this organism? 



28 CASE FILES: MICROBIOLOGY 

ANSWERS TO CASE 1: Bacillus anthracis 

Summary: A 53-year-old male farmer has a 4.5 cm circular skin lesion of black 
eschar surrounded by vesicles and edema. The Gram stain on microscopy 
shows gram-positive bacilli in chains. 

^ Organism most likely causing disease: Bacillus anthracis 

^ Primarily virulence factors: Capsular polypeptide and anthrax toxin 

CLINICAL CORRELATION 

Bacillus anthracis is the etiologic agent of cutaneous, gastrointestinal, and 
inhalational anthrax. Approximately 95 percent of anthrax disease is cuta- 
neous. Bacillus anthracis is distributed worldwide, and all animals are sus- 
ceptible, but it is more prevalent in herbivores. Infected animals often develop 
a fatal infection and contaminate the soil and water with B. anthracis that can 
sporulate and continue to survive in the environment for many years. Oxygen 
is required for sporulation, and the spores will grow on culture plates, in soil, 
or in the tissue of dead animals. Human infections are caused by either pene- 
tration of these spores through the skin barrier (cutaneous), ingestion of the 
spores (rare), or inhalation of the spores (so-called wool-sorters' disease), 
which usually occurs while processing animal products. Person-to-person 
transmission of anthrax has not been described. Cutaneous anthrax, the most 
common clinical manifestation, occurs within 2-3 days of exposure to an 
infected animal or animal product. A papule develops at the site of inoculation, 
which progresses to form a vesicle. A characteristic black eschar is formed 
after rupture of the vesicle and development of necrosis in the area. In rare 
cases the disease progresses and becomes systemic with local edema and bac- 
teremia, which can be fatal if untreated. 

The only other Bacillus species frequently associated with human disease 
is B. cereus, which is a cause of gastroenteritis following ingestion of a con- 
taminated food product, most commonly fried rice. The spores of B. cereus 
can also survive in the soil and be responsible for traumatic wound infections, 
particularly to the eye, when soil contamination is involved. 

APPROACH TO SUSPECTED ANTHRAX INFECTION 
Objectives 

1. Know the structure and characteristics of B. anthracis. 

2. Know the clinical diseases caused by and virulence of B. anthracis. 

3. Know the structure and characteristics of B. cereus. 

4. Know the clinical diseases caused by and virulence of B. cereus. 



CLINICAL CASES 29 

Definitions 

Eschar: Skin lesion associated with cutaneous anthrax and resembling a 

black, necrotic sore. 
Wool-sorters' disease: Disease associated with inhalation of anthrax 

spores from infected animal products, most often associated with sheep 

wool. 
Differential diagnosis: Listing of the possible diseases or conditions that 

may be responsible for the patient's clinical presentation. 

DISCUSSION 

Characteristics of Bacillus Species 

Bacillus species are large, motile, facultative anaerobic, gram-positive rods 

with a central spore. The spore is quite resistant to extreme conditions and 
can survive in nature for prolonged periods of time. Bacillus anthracis is 
nonmotile and on Gram stain is often seen in chains. The virulent forms of 
B. anthracis are more likely to be surrounded by a capsule. The organism 
can be cultured as large colonies on blood agar plates within 24 hours, often 
resembling a "Medusa head" (irregular appearance to the colony with swirling 
projections). The principal virulence factors of B. anthracis are the capsu- 
lar polypeptide and anthrax toxin. The capsule consists of poly-D-glutamic 
acid, which is thought to allow the organism to resist phagocytosis. 
Anthrax toxin consists of three proteins: protective antigen, edema factor, 
and lethal factor. Protective antigen is named for its ability to confer immu- 
nity in experimental situations. Edema factor and lethal factor bind to protec- 
tive antigen to form edema toxin and lethal toxin. The bound proteins are 
transported across cell membranes and are released in the cytoplasm where 
they exert their effects. Once the spores enter the body they are taken up by 
macrophages. Because of both lethal and edema factors, the spores survive 
killing, and subsequently germinate. 

Diagnosis 

The differential diagnosis of a patient (farmer) with fever, adenopathy, and 
black eschar include other cutaneous lesions such as furuncles (staphylo- 
cocci), ecthyma gangrenosum (Pseudomonas aeruginosa), and spider bites. 
However, none of these etiologies are known to cause eschar formation with 
surrounding edema. The specific diagnosis of anthrax is made by growth of 
the organism from blood (inhalation anthrax), or wound (cutaneous anthrax). 
Bacillus anthracis grows easily on most bacteriological culture media within 
18-24 hours at 35 °C (95 °F). The organism is a nonmotile, spore-forming gram- 
positive bacillus that is nonhemolytic when grown on blood containing agar 
medium and produces lecithinase on egg yolk agar. Lecithinase is an enzyme 



30 CASE FILES: MICROBIOLOGY 

produced by both B. anthracis and B. cereus that degrades the lecithin in the 
egg yolk agar leaving a white precipitate. 

Careful review of a Gram stain from a primary specimen of a patient with 
suspected anthrax is necessary, because the organisms have the propensity to 
easily decolorize and appear gram-negative. However, the presence of spores 
is a key to the identification of the organism as a gram-positive bacillus. Based 
on these few tests (large gram-positive bacilli, nonhemolytic, lecithinase pos- 
itive) a presumptive identification of B. anthracis can be made. As a result of 
the recent events in the world leading to concerns over bioterrorism, definitive 
diagnosis of anthrax must be performed in a public health laboratory. 
Confirmatory testing involves the use of fluorescently labeled monoclonal 
antibodies as well as DNA amplification assays. The use of India ink can 
also help to determine the presence of a capsule, a relatively unique aspect. 
The capsule of B. anthracis is not stained by the India ink, which can be 
easily visualized against the dark background. 

Treatment and Prevention 

Ciprofloxacin is the drug of choice for anthrax, following the identification 
of weaponized strains that were resistant to penicillin as a result of the 
production of a (3-lactamase. Prevention of anthrax involves vaccination of 

animals as well as humans at high risk of exposure (military personnel). 
Prophylaxis is not recommended for asymptomatic persons. When deemed 
necessary, prophylaxis with ciprofloxacin must be maintained for up to 
30 days because of the potential delay in germination of inhaled spores. 

COMPREHENSION QUESTIONS 

[1.1] A wound specimen obtained from a person working with wool from a 
Caribbean island demonstrated a large gram-positive rod from a non- 
hemolytic colony with swirling projections on blood agar. The most 
likely method to demonstrate spores would be which of the following? 

A. Acid-fast stain 

B. Gram stain 

C. India ink stain 

D. Malachite green stain 

[1.2] Which of the following is the current preferred antimicrobial treatment 
of cutaneous anthrax? 

A. Aminoglycosides 

B. Ciprofloxacin 

C. Penicillin 

D. Tetracyclines 



CLINICAL CASES 31 

[1.3] Bacillus anthracis is unique to other bacteria. It is the only bacteria to 
possess which of the following? 

A. An endotoxin 

B. An exotoxin 

C. A polypeptide capsule 

D. A polysaccharide capsule 

E. Lipopolysaccharide in its outer cell wall 

F. Teichoic acid in its outer cell wall 

Answers 

[1.1] D. Spores can be observed as intracellular refractile bodies in 
unstained cell suspensions. Also, they are commonly observed by 
staining with malachite green or carbolfuchsin. The spore wall is 
relatively impermeable, but heating of the preparation allows dyes to 
penetrate. Alcohol treatment then serves to prevent spore decoloriza- 
tion. Finally, the spores are counterstained. 

[1.2] B. Penicillin G was considered to be the first choice treatment for 
patients with cutaneous anthrax and when used should be continued 
for 7-10 days. However, because some naturally occurring isolates 
have been reported to be penicillin resistant (but still ciprofloxacin 
sensitive) and some patients are allergic to penicillin, ciprofloxacin is 
now considered to be the drug of choice for cutaneous anthrax. 
Ciprofloxacin belongs to the family of quinolones. As a fluorinated 
quinolone, it has greater antibacterial activity, lower toxicity, and is 
able to achieve clinically useful levels in blood and tissues compared 
to nonfluorinated quinolones. They act against many gram-positive 
and gram-negative bacteria by inhibiting bacterial DNA synthesis via 
the blockage of DNA gyrase. Despite the use of antibiotics in the 
treatment of anthrax, clinically manifested inhalational anthrax is 
usually fatal. If anthrax is suspected, public health authorities should 
be notified immediately. Aminoglycosides and tetracyclines have dif- 
ferent mechanisms of action and have preferred uses in other disease 
states and infections. Aminoglycosides inhibit bacterial protein syn- 
thesis by attaching to and inhibiting the function of the 30S subunit 
of the bacterial ribosome. Their clinical usefulness has declined with 
the advent of cephalosporins and quinolones. Tetracyclines also 
inhibit bacterial protein synthesis; however they do so by inhibiting 
the binding of aminoacyl-tRNA to the 30S subunit of bacterial 
ribosomes. 



32 CASE FILES: MICROBIOLOGY 

[1.3] C. Virulent forms of B. anthracis, the causative agent of anthrax, are 
more likely to be surrounded by a capsule. This capsule is a polypep- 
tide, composed of a polymer of glutamic acid, and is a unique feature 
of B. anthracis. Lipopolysaccharides (LPS/endotoxin) are unique to 
gram-negative bacteria (B. anthracis is a gram-positive rod). In addi- 
tion, whereas B. anthracis is associated with both teichoic acid (cell 
wall) and a potent exotoxin, these are not unique features of this 
bacterium. Other gram-positives (i.e., staphylococci) release exotox- 
ins and have teichoic acid in their cell walls. 



♦!♦ 



MICROBIOLOGY PEARLS 



*♦* The most common form of anthrax is cutaneous anthrax in which pen- 
etration of the skin by B. anthracis spores causes eschar formation 
with regional lymphadenopathy. 

*♦* The organism is a nonmotile, spore-forming gram-positive bacilli 
that is nonhemolytic and produces lecithinase. 

*♦* Inhalation anthrax is a matter of public health concern. 

*♦* The drug of choice for treating anthrax is ciprofloxacin. 



The two main methods of anthrax virulence are its capsule and toxin. 



V* An eschar surrounded by edema is suspicious for anthrax. 



REFERENCES 

Logan NA, Turnbull PCB. Bacillus and other aerobic endospore-forming bacteria. In: 

Murray PR, Baron EJ, Jorgensen JH, et al., eds. Manual of Clinical Microbiology, 

8th ed. Washington, DC: ASM Press 2003. 
Murray PR, Rosenthal KS, Pfaller MA. Bacillus. In: Murray PR, Rosenthal KS, 

Pfaller MA. Medical Microbiology, 5th ed. St. Louis, MO: Mosby, 2005:265-71. 
Ryan KJ. Sherris Medical Microbiology: An Introduction to Infectious Diseases, 

3rd ed. New York: McGraw-Hill, 2003. 
Swartz MN. Recognition and management of anthrax — an update. N Engl J Med 

2001:345:1621-6. 



♦♦♦ CASE 2 

A 60-year-old man presents to the emergency room with severe abdominal 
pain. He has had mild, left lower abdominal cramping pain for about 3 days, 
which has worsened in the past 8 hours. He has also had nausea, fever, and 
chills. On examination, he is in obvious pain, has a fever of 38.6°C (101. 5°F) 
and has an elevated heart rate (tachycardia). His abdominal examination is 
notable for absent bowel sounds, diffuse tenderness, and rigidity when pal- 
pated. An x-ray reveals the presence of free air in the abdominal cavity. He is 
taken for emergency surgery and found to have severe diverticulitis with a per- 
forated colon. Cloudy peritoneal fluid is collected. An anaerobic culture grows 
Bacteroides fragilis. 

^ What characteristics are noted on Gram staining of B. fragilis? 

^ What are its primary mechanisms for resisting phagocytosis? 



34 CASE FILES: MICROBIOLOGY 

ANSWERS TO CASE 2: Bacteroides fragilis 

Summary: A 60-year-old man has a ruptured diverticulitis leading to peritoni- 
tis. The cultures of the purulent drainage reveal B. fragilis. 

^ Characteristics on gram staining: Bacteroides fragilis appears 

encapsulated, with irregular staining, pleomorphism, and vacuolization. 

^ Primary mechanisms of resisting phagocytosis: The capsular 
polysaccharide and succinic acid production. 

CLINICAL CORRELATION 

Bacteroides fragilis is one of the most clinically significant anaerobic organ- 
isms. It is part of the normal flora of the gastrointestinal (GI) tract and causes 
clinical infections when it escapes from this environment following surgery, 
traumatic bowel perforation, or other diseases, such as diverticulitis. Although 
many anaerobes are part of the normal gastrointestinal flora, B. fragilis is the 
most common cause of intraabdominal infections. Bacteroides fragilis is 
also associated with respiratory tract infections (sinusitis, otitis), genital tract 
infections, brain, skin, and soft tissue infections. 

Diverticulitis is an inflammation of a small food and particle collecting sac 
in the large intestine; it may lead to colonic rupture and therefore allow the 
organisms normally present in the GI tract to penetrate the peritoneal cavity 
and possibly the bloodstream. These infections usually involve a mixture of 
both aerobes and anaerobes. 

APPROACH TO Bacteroides fragilis 
Objectives 

1 . Know the microbiologic characteristics of B. fragilis and other Bacteroides 
species. 

2. Know the virulence factors associated with B. fragilis. 

Definitions 

Anaerobes: Organisms that do not require oxygen for growth and may die 

in its presence. 
Bacteroides bile esculin (BBE) agar: Media selective for B. fragilis on 

which the colonies appear black. 
Diverticulitis: Inflammation of a diverticulum, which is a small bulging 

sac in the colon wall which can trap food particles and become inflamed 

and painful. 



CLINICAL CASES 35 



DISCUSSION 

Characteristics of Bacteroides Species 

Bacteroides species include the B. fragilis group as well as many other species. 
Two new genera were recently created, Prevotella and Porphyromas, to remove 
the pigmented, bile-sensitive anaerobes previously in the genus Bacteroides. 
All are small, anaerobic, gram-negative bacilli and many strains are encap- 
sulated. Vacuolization, irregular staining, and pleomorphism are common. 
Bacteroides fragilis has a distinct capsule composed of two polysaccha- 
rides, which appears to inhibit phagocytosis and allow adherence to 
peritoneal surfaces. Other virulence factors for this bacterium include the pres- 
ence of pili, which promote adherence to epithelial cells and the production of 
succinic acid, which inhibits phagocytosis. Bacteroides fragilis produces an 
endotoxin that has little biologic activity. It also produces superoxide dismu- 
tase, an enzyme, which allows the organism to survive in the presence of 
small amounts of oxygen. 

Diagnosis 

Anaerobes are not usually the primary cause of an infection, but are involved in 
a mixed aerobic, anaerobic infection. Often diagnosis of anaerobic infections 
is based on clinical features including a foul smelling wound with the pres- 
ence of gas in the involved tissue usually located in close proximity to a 
mucosal surface. Infections that involve spillage of GI material into the peri- 
toneum are likely to involve aerobes and anaerobes. The most commonly asso- 
ciated anaerobe is B. fragilis. Patients with severe diverticulitis, appendicitis, 
or colonic injury often develop B. fragilis peritonitis. 

To increase the chances of recovery of anaerobes from a specimen, the sam- 
ple must be appropriately collected to allow survival of anaerobes. Anaerobes 
are organisms that do not require oxygen for growth. Sensitivity of the anaer- 
obic organism can vary from those that cannot tolerate any oxygen (strict 
anaerobes) to those that can grow in the presence of small amounts of oxy- 
gen (oxygen tolerant). Anaerobes are therefore grown under atmospheric 
conditions that limit the presence of oxygen and include predominantly nitro- 
gen, as well as hydrogen and carbon dioxide. Tissues or fluids collected and 
transported under anaerobic conditions are the most optimal; however, if nec- 
essary an anaerobic transport swab can also be used. 

Bacteroides species produce small colonies on anaerobic blood agar 
medium within 24 hours. Selective media such as kanamycin/gentamicin laked 
blood agar will support growth of gram-negative anaerobes only. Presumptive 
identification of B. fragilis can be made by growth of black pigmented colonies 
on Bacteroides bile esculin agar, and resistance to kanamycin, colistin, and van- 
comycin special potency antimicrobial disks. Definitive identification of anaer- 
obes or B. fragilis is made with commercial identification systems that are based 



36 CASE FILES: MICROBIOLOGY 

on the presence of preformed enzymes or in reference laboratories using gas liq- 
uid chromatography to determine the specific gases produced by the organism. 

Treatment and Prevention 

Surgical debridement is usually necessary at least in part for the treatment of 
anaerobic infections. fJ-Lactamase activity is common in Bacteroides 
species, especially B. fragilis, which results in resistance to penicillin and 
cephalosporin antibiotics. Drugs of choice for Bacteroides species include 
(3-lactam-(3-lactamase inhibitor combinations, such as piperacillin/tazobactam, 
metronidazole, and imipenem. 

COMPREHENSION QUESTIONS 

[2.1] During an emergency surgery, a 60-year-old male is found to have severe 
peritonitis and a perforated colon. Foul-smelling cloudy peritoneal fluid 
is collected. Subsequent analysis reveals the growth of black pigmented 
colonies on Bacteroides bile esculin agar. No growth is detected in the 
presence of kanamycin, colistin, or vancomycin. Which of the following 
microorganisms is most likely involved in this case? 

A. Actinomyces israelii 

B . Bacteroides frag His 

C. Clostridium difficile 

D. Enterococcus faecalis 

E. Porphyromonas gingivalis 

F. Prevotella melaninogenica 

[2.2] Which of the following is the treatment of choice to control this infec- 
tion in this patient (described in Question [2.1])? 

A. Cephalothin 

B. Erythromycin 

C. Metronidazole 

D. Penicillin 

[2.3] Among the many virulence factors produced, B. fragilis produces an 
enzyme that allows the organism to survive in the presence of small 
amounts of oxygen. Which of the enzymes listed below catalyzes the 
following reaction? 

20" + 2H + -> H 2 2 + 2 

A. [^-Lactamase 

B. Myeloperoxidase 

C. Nicotinamide adenine dinucleotide phosphate (NADPH) oxidase 

D. NO synthase 

E. Oxidase 

F. Superoxide dismutase 



CLINICAL CASES 37 

[2.4] A foul-smelling specimen was obtained from a 26-year-old woman with 
a pelvic abscess. Culture grew both aerobic and anaerobic gram-negative 
bacteria. The most likely organisms are which of the following? 

A. Actinomyces israelii and Escherichia coli 

B. Bacteroides fragilis and Listeria monocytogenes 

C. Bacteroides fragilis and Neisseria gonorrhoeae 

D. Clostridium perfringens and Bacteroides fragilis 

E. Escherichia coli and Peptostreptococcus 

Answers 

[2.1] B. Bacteroides species are normal inhabitants of the bowel and other 
sites. Normal stools contain large numbers of B. fragilis (10" organ- 
isms per gram). As a result, they are very important anaerobes that 
can cause human infection. Members of the B. fragilis group are most 
commonly isolated from infections associated with contamination by 
the contents of the colon, where they may cause suppuration, for 
example, peritonitis after bowel injury. Classification is based on 
colonial and biochemical features and on characteristic short-chain 
fatty acid patterns in gas chromatography. These short-chain fatty 
acids also contribute to the foul-smelling odor emanating from the 
wound in the above case. 

[2.2] C. Metronidazole, mainly used as an antiprotozoal agent, is also highly 
effective against anaerobic bacterial infections, such as those infections 
caused by Bacteroides species. It is the drug of first choice for GI 
strains of Bacteroides. Two other effective antibiotics are imipenem 
and piperacillin/tazobactam. Bacteroides species, such as B. fragilis, 
commonly possess P-lactamase activity resulting in resistance to peni- 
cillin and cephalosporin (e.g., cephalothin) antibiotics. Erythromycin is 
not indicated in the treatment of Bacteroides species. 

[2.3] F. A key feature of obligate anaerobes such as Clostridium, Bacteroides, 
and Actinomyces is that they lack catalase and/or super-oxide dismutase 
(SOD) and are therefore susceptible to oxidative damage. Bacteroides 
fragilis, however, is able to survive (not grow) in environments with low 
oxygen content because of its ability to produce small amounts of both 
SOD and catalase. Anaerobes that possess SOD and/or catalase are able 
to negate the toxic effects of oxygen radicals and hydrogen peroxide and 
thus tolerate oxygen. Other common enzymes listed above catalyze the 
following reactions: 

Catalase/superoxide dismutase catalyzes: 2H 2 7 — > 2H 9 + O, 

Myeloperoxidase catalyzes: CI" + H 2 9 — > CIO" + H 2 

NADPH oxidase catalyzes: NADPH + 20 2 ~-> 20" + H + NADP+ 

NO synthase catalyzes: V.O, + arginine — > NO + citrulline 

Oxidase catalyzes: 2H + + 2e" + V 2 0, -> H 2 



38 CASE FILES: MICROBIOLOGY 

[2.4] C. In infections, such as intra-abdominal abscesses, Bacteroides 
species are often associated with other organisms. The only other 
organism in the list above that is solely aerobic and gram-negative is 
N. gonorrhoeae. Clostridium and Listeria are both gram-positive. 
E. coli is gram-negative and a facultative anaerobe. 



MICROBIOLOGY PEARLS 



*♦* The treatment of choice of B. fragilis is surgical debridement in 

addition to metronidazole, imipenem, or piperacillin/tazobactam. 
*♦* Most anaerobes are part of mixed infections at mucosal surfaces. 
*♦* Bacteroides fragilis is the most common anaerobe in the human 

GI tract. 
*♦* Bacteroides fragilis usually express superoxide dismutase, an 

enzyme, which allows the organism to survive in the presence of 

small amounts of oxygen. 



REFERENCES 

Brook I, Frazier EH. Aerobic and anaerobic microbiology in intraabdominal infec- 
tions associated with diverticulitis. J Med Microbiol 2000;49:827-30. 

Engelkirk PG, Duben-Engelkirk JD, Dowell VR. Principles and Practice of Clinical 
Anaerobic Bacteriology. Belmont, CA: Star Publishing, 1992. 

Murray PR, Rosenthal KS, Pfaller MA. Anaerobic gram negative bacilli. In: Murray 
PR, Rosenthal KS, Pfaller MA. Medical Microbiology, 5th ed. St. Louis, MO: 
Mosby, 2005:421-26. 



♦♦♦ CASE 3 

A 28-year-old woman presents to the office for the evaluation of a rash. She had 
just returned from a weeklong camping trip in the New England area, when she 
noted the presence of a circular, red rash on her lower abdomen. Also, she has 
had a low-grade fever, and some achiness and fatigue. Examination of her 
abdomen reveals a 10-cm flat, red, circular patch with some central clearing. 
No other skin rashes are noted, and the remainder of the examination is normal. 
The blood cultures are negative. You make the presumptive diagnosis of 
erythema migrans and send blood for confirmatory serologic studies. 

^ What organism is the etiologic agent of erythema migrans? 

^ What are the primary reservoir and vector of transmission of this 
agent? 



40 CASE FILES: MICROBIOLOGY 

ANSWERS TO CASE 3: Borrelia burgdorferi 

Summary: A 28-year-old woman who has been recently camping in the New 
England area complains of fever and a skin rash consistent with erythema 
migrans. Confirmatory serologies are sent. 

^ Etiologic agent of erythema migrans: Borrelia burgdorferi 

^ Primary reservoir of infection: Small rodents, primarily the white- 
footed mouse 

^ Primary vector of transmission: Ixodes tick 

CLINICAL CORRELATION 

Borrelia burgdorferi is the causative agent of Lyme disease and is transmitted 
to humans by Ixodes ticks. This disease was first recognized in Old Lyme, 
Connecticut, with the identification of cluster cases of arthritis in children. The 
infection is characterized by a "bull's eye" skin lesion, which develops from 
the site of the tick bite, 1-4 weeks postinfection. Additional initial symptoms 
include fever, fatigue, headache, joint pain, or mild stiffness of the neck. Lyme 
disease is the most common vector-born disease in the United States, and 
if left undiagnosed and untreated, the infection usually progresses to involve 
the nervous or vascular systems and cause fluctuating or chronic arthritis. 

APPROACH TO Borrelia SPECIES 
Objectives 

1 . Know the characteristics and virulence factors of B. burgdorferi. 

2. Know the reservoir, vector, and host involved in the transmission of 
B. burgdorferi. 

Definitions 

Erythema migrans: Skin lesion composed of redness (erythema) with 

central clearing (target lesion). 
Spirochetes: Thin spiral bacteria of which three genera cause significant 

disease in humans: Leptospira, Borrelia, and Treponema, which lead to 

leptospirosis, Lyme disease, and syphilis, respectively (see Table 3-1 for 

an abbreviated listing). 



CLINICAL CASES 



41 



Table 3-1 

ABBREVIATED LISTING OF SPIROCHETES 



SPIROCHETE 


DISEASE 


TREATMENT 


COMMENT 


Treponema 


Venereal syphilis, 


Penicillin 


Veneral syphilis usually is 




Yaws, pinta, endemic 




sexually or vertically (to 




syphilis 




fetus/neonate) transmitted, 
whereas yaws, pinta, 
endemic syphillis is by 
skin or shared utensils, 
usually affecting children 


Leptospiras 


Weil syndrome 


Doxycycline or 


Coiled, highly motile with 




(severe hepatic renal 


ampicillin 


hooked ends and flagella 




failure) Anicteric 




allowing burrowing into 




leptospirosis (without 




tissue 




jaundice) 






Borrelia 


Relapsing fever Lyme 


Erythromycin, 


Usually human or rodent 




disease 


tetracycline 


reservoir transmitted by 
tick bite 



DISCUSSION 



Characteristics of Borrelia Species 

Borrelia burgdorferi belongs to the spirochete family of prokaryotes. It stains 
gram-negative, although spirochetes are considered neither gram-positive nor 
gram-negative. Spirochetes consist of a flexible, multilayer outer cell mem- 
brane and a more rigid, peptidogly can-containing cytoplasmic membrane. 

Between these two layers are endoflagella that insert at the ends of the 
spirochete. Rotation of these flagella creates the characteristic cork-screw 

shape of these organisms. This provides for motility of the organism and hides 
the normally antigenic flagella from host defenses. These organisms are 
microaerophilic and have a doubling time of 8-24 hours. The disease is 
endemic in several regions of the United States including Northeastern, 
Midwest, and Pacific coast states. However, most reported cases occur in 
New York, Connecticut, Pennsylvania, and New Jersey. 

The spirochetes that cause Lyme disease have been divided into 
genospecies. Three genospecies, B. burgdorferi sensu stricto, B. garinii, and 
B. afzelii, are known to cause Lyme disease and are known collectively as 
B. burgdorferi sensu lato. The outer membrane of B. burgdorferi contains 
unique outer surface proteins (Osps), which are thought to play a role in their 
virulence. Small rodents, particularly the white-footed mouse, are the 



42 CASE FILES: MICROBIOLOGY 

primary reservoirs of B. burgdorferi, and the vector of transmission is the 
Ixodes tick. The larva of the tick is born uninfected. The ticks become 
infected with the spirochete on feeding on an infected animal. This usually 
occurs during the nymph stage of the tick's life cycle. The spirochetes multi- 
ply in the gastrointestinal (GI) tract of the tick, and then are transmitted to 
the animal host by regurgitation or salivation during a subsequent feeding. 
Borrelia burgdorferi are next transmitted to humans via tick bite followed 
by dissemination through the bloodstream to the joints, heart, and central nerv- 
ous system (CNS). The nymphal stage of the tick is more infective than the 
adult and larval stages. Most exposures to Borrelia occur between the months 
of May and July, when the nymphs are most active. 

Clinically there are three stages of B. burgdorferi infection: stage 1, which 
occurs in the first 4 weeks postinfection, involves the initial characteristic skin 
lesion referred to as "erythema migrans"; stage 2 follows for months postin- 
fection with neurologic and cardiac involvement; and stage 3 results in 
chronic arthritis of the joints. 

Diagnosis 

The diagnosis of Lyme disease is made primarily by clinical presentation and 
patient history of exposure. Confirmation of a clinical diagnosis is made sero- 
logically via the detection of antibody by enzyme-linked immunoabsorbance 
or indirect immunofluorescence. However, serologic tests are most reliable 
2-4 weeks postinfection, because of cross-reactivity with normal flora, and 
Western blot analysis should be used to confirm a positive serologic test. 
Alternately, new PCR-based tests are available to detect B. burgdorferi DNA. 
Borrelia burgdorferi is difficult to grow in culture, requiring complex culture 
media and a microaerophilic environment. It is also difficult to visualize under 
light microscopy, but can be seen under darkfield microscopy or when stained 
with Giemsa or silver stains. 

Treatment and Prevention 

Initial stages of infection with B. burgdorferi can be effectively treated with 
doxycycline or amoxicillin, while later stages of disease are better treated 
with penicillin G or ceftriaxone. Prevention of infection involves limiting 
exposure to ticks by wearing protective clothing in endemic areas, includ- 
ing long sleeves and long pants tucked into socks. Careful search for and 
removal of ticks is also an important preventative measure. Use of repellants 
is also helpful and administration of insecticides may reduce the number of 
active nymphal ticks for a given season. A vaccine containing recombinant 
OspA protein was developed for persons with the highest risk of exposure. The 
vaccine is approved for adults and shows approximately 75 percent efficacy. 



CLINICAL CASES 43 

COMPREHENSION QUESTIONS 

[3.1] A 9-year-old boy presents with a migratory rash with central clearing on 
the back of his neck. The child had recently been on vacation with his 
family in Oregon and had gone hiking. The child's pediatrician observes 
the rash and suspects an infection with B. burgdorferi. Which of the fol- 
lowing is thought to be a virulence factor of this organism? 

A. Intracellular growth in leukocytes 

B. Endotoxin release 

C. Localization in reticuloendothelial cells 

D. Antiphagocytic capsular antigen 

E. Expression of outer surface proteins 

[3.2] If the child's infection is left untreated, which of the following symp- 
toms would most likely appear? 

A. Urethritis 

B. Centripetal spread of rash 

C. Biphasic illness with fever and chills 

D. Stiffness in the knees 

E. Swelling of lymph nodes 

[3.3] A small tick, of the genus Ixodes, most commonly transmits B. burgdorferi. 
Which of the following diseases is also transmitted by a tick? 

A. Q fever 

B. Leptospirosis 

C. Ehrlichiosis 

D. Yellow fever 

E. Eastern equine encephalitis 

Answers 

[3.1] E. Differential expression of outer surface proteins is thought to be 
involved with virulence; answers A, B, C, and D are incorrect: 
(A) intracellular growth in leukocytes is a virulence factor of 
Ehrlichia; (B) endotoxins are characteristic of gram-negative organ- 
isms, not Borrelia; (C) localization in reticuloendothelial cells occurs 
in infections with Francisella tularensis; (D) an antiphagocytic cap- 
sular antigen is not a virulence factor of Borrelia. 

[3.2] D. Later stages of infection with B. burgdorferi include arthritis, 
meningitis, nerve palsies, and cardiovascular abnormalities; answers 
A, B, C, E, are incorrect: (A) arthritis, not urethritis, is a later mani- 
festation of infection with B. burgdorferi; (B) the skin rash or ery- 
thema migrans expands centrifugally, not centripetally; (C) biphasic 
illness with fever and chills occurs more commonly with Leptospira 
infections; (E) swelling of lymph nodes is more commonly associ- 
ated with Yersinia infections. 



44 CASE FILES: MICROBIOLOGY 

[3.3] C. Similar to Lyme disease, ehrlichiosis is also transmitted via a tick 
vector; answers A, B, D, and E are incorrect: (A) Q fever is most 
commonly transmitted via inhalation of dried feces or urine contam- 
inated with rickettsiae; (B) Leptospirosis is typically transmitted via 
ingestion of contaminated food or water; (D and E) both yellow fever 
and eastern equine encephalitis are transmitted by mosquitoes. 



MICROBIOLOGY PEARLS 



*♦* Borrelia burgdorferi is a microaerophilic spirochete. 
*♦* Primary reservoirs of B. burgdorferi are small rodents (e.g., white- 
footed mouse), and the vector of transmission is the Ixodes tick. 
*♦* States with highest incidence include: New York, Connecticut, 



Pennsylvania, and New Jersey, 
imary treatment is doxycycline i 
evention consists of wearing prot 

lants or insecticides, and a recombinant OspA protein vaccine. 



*♦* Primary treatment is doxycycline or amoxicillin. 

*♦* Prevention consists of wearing protective clothing, use of insect repel- 



REFERENCES 

Brooks GF, Butel JS, Morse SA. Jawetz, Melnick, & Adelberg's Medical 

Microbiology, 23rd ed. New York: McGraw-Hill, 2004:336-338. 
Murray PR, Rosenthal KS, Pfaller MA. Medical Microbiology, 5th ed. St. Louis, 

MO: Mosby, 2005. 
Ryan JR, Ray CG. Sherris Medical Microbiology, 4th ed. New York: McGraw-Hill, 

2004:431-434. 
Wilske B, Schriefer ME. Borrelia. In: Murray PR, Baron EJ, Jorgensen JH, et al., 

eds. Manual of Clinical Microbiology, 8th ed. ASM Press, 2003. 



♦♦♦ CASE 4 

A 19-year-old male college student presents to the student health department 
with abdominal pain, diarrhea, and fever. He says that his symptoms started a 
day ago. He has had 10 stools in the past day and has noted blood mixed in 
with the stool on several occasions. He usually eats at home but reports having 
eaten chicken in the college cafeteria 3 days ago. He has no history of gas- 
trointestinal (GI) disease. On examination, he has a temperature of 37.8°C 
(100°F) and appears to be in pain. His abdomen has hyperactive bowel sounds 
and is diffusely tender but without rigidity, rebound tenderness, or guarding. 
A general surgeon is consulted and is considering the diagnosis of acute 
appendicitis versus bacterial gastroenteritis possibly related to the chicken 
eaten. A stool sample tests positive for blood and fecal leukocytes. Stool cul- 
tures are sent and are subsequently positive for a pathologic organism. 

^ What is the most likely pathologic organism? 

^ In what atmospheric environment does this organism grow? 



46 CASE FILES: MICROBIOLOGY 

ANSWERS TO CASE 4: Campylobacter jejuni 

Summary: A 19-year-old man presents with a bacterial gastroenteritis that 
mimics appendicitis. 

^ Most likely etiology of this infection: Campylobacter jejuni 

^ Preferred atmospheric environment: Microaerophilic (presence of 
increased levels of carbon dioxide) 

CLINICAL CORRELATION 

More than 50 serotypes of C. jejuni have been identified based on heat labile 
(capsular and flagellar) antigens. Campylobacter jejuni is endemic worldwide, 
and most cases of infection are associated with eating poorly cooked 
chicken, although milk, water and other meats have also been implicated. 
Human-to -human transmission is rare. Campylobacter jejuni is one of the 
most frequent causes of bacterial diarrhea occurring most often in the 
summer or early fall. The incubation period is 1-3 days followed initially by 
symptoms of fever, malaise, and abdominal pain. Campylobacter jejuni can 
cause bloody diarrhea, mucosal inflammation, and bacteremia, suggesting that 
it is invasive to the lining of the intestine. Most cases of Campylobacter 
gastroenteritis is self-limited, with symptoms resolving within 7 days; however, 
relapses can occur in 5-10 percent of cases which are untreated. 

Complications of Campylobacter gastroenteritis include pancreatitis, peri- 
tonitis, or more uncommonly arthritis, osteomyelitis, and sepsis. A serious 
postinfection sequelae of Campylobacter gastroenteritis is Guillain-Barre 
syndrome, an acute demyelinating disease. Antigenic similarities between the 
lipopolysaccharides on the surface of some serotypes of C. jejuni and myelin 
proteins are thought to be responsible for causing Guillain-Barre disease. 

Other Campylobacter species such as C. coli also cause gastroenteritis, 
which is clinically indistinguishable from C. jejuni infection. Campylobacter 
fetus is primarily a cause of bacteremia, septic arthritis, peritonitis, abscesses, 
meningitis, and endocarditis in immunocompromised patients. 

APPROACH TO SUSPECTED Campylobacter 
INFECTION 

Objectives 

1 . Know the characteristics, virulence factors, and preferred growth envi- 
ronments of C. jejuni. 

2. Know the sources of infection with and mechanism of transmission of 
C. jejuni. 



CLINICAL CASES 47 

Definitions 

Guillain-Barre syndrome: A demyelinating disease resulting from similar- 
ities between the host and the surface of the Campylobacter organism. 

Fecal leukocytes: White blood cells present in the stool, which correlate 
loosely with the presence of an invasive pathogen. 

DISCUSSION 

Characteristics of Campylobacter 

Campylobacter species are small motile, nonspore-forming, comma-shaped, 
gram-negative bacilli. Its motility is the result of a single flagellum located 
at one or both poles of the organism. Campylobacter does not grow in aerobic 
or anaerobic environments. It is microaerophilic, requiring 5-10 percent oxy- 
gen and high concentrations of carbon dioxide for growth. Campylobacter 
jejuni grows better at 42°C (107.6°F) than 37°C (98.6°F). Campylobacter 
jejuni multiplies more slowly than other enteric bacteria, making isolation 
difficult from stool samples unless selective media are used. When selective 
media are used, the colonies that grow tend to be gray, mucoid, and wet 
appearing. Its outer membrane contains lipopolysaccharides with endotoxic 
activity. Extracellular toxins with cytopathic activity have also been found; 
however, little is known regarding the pathogenesis of this organism and the 
role of these putative virulence factors in disease. The organisms are sensitive 
to decreased pH, so it is hypothesized that factors that neutralize gastric acid 
enhance the organisms chances for survival. 

Diagnosis 

The differential diagnosis of acute gastroenteritis would include Salmonella, 
Shigella, Yersinia, as well as Campylobacter. Because of the feature of abdom- 
inal pain and cramps, sometimes in the absence of diarrhea, Campylobacter 
gastroenteritis can be misdiagnosed as appendicitis or irritable bowel syn- 
drome. The presence of bloody diarrhea may also suggest enterohemorrhagic 
Escherichia coli. 

Definitive diagnosis would be made by culture of the stool and growth 
of Campylobacter. Campylobacter are more fastidious than most other causes 
of bacterial gastroenteritis and specimens should be transported to the labora- 
tory in media such as Cary-Blair. Selective media such as campy blood agar 
or Skirrow medium, which includes antibiotics to inhibit the normal stool 
flora, allows for growth of Campylobacter within 48-72 hours. Presumptive 
identification can be made by growth of oxidase positive colonies on selec- 
tive media at 42°C (107.6°F) after 48-72 hours with characteristic comma- 
shaped, small, gram-negative bacilli seen on Gram stain. Confirmation of 
identification of either C. jejuni or C. coli can be made by resistance to 



48 CASE FILES: MICROBIOLOGY 

cephalothin and susceptibility to nalidixic acid antimicrobial disks. As a result 
of the fastidious nature of these pathogens, a commercial assay for detection 
of Campylobacter antigen in the stool is frequently used for diagnosis. 

TREATMENT AND PREVENTION 

Most often C. jejuni infection is self-limited and does not require specific 
antimicrobial therapy. Supportive care, that is, hydration, is often the only 
treatment needed. If specific therapy is needed for severe disease, or infection 
in immunocompromised patients, erythromycin is the drug of choice, because 
of the recent increase in resistance to fluoroquinolones. 

Prevention involves care in food preparation. Foods, especially chicken, 
should be completely cooked, and exposure to raw or undercooked chicken or 
unpasteurized milk should be limited, especially in pregnant women or immuno- 
compromised persons. 

COMPREHENSION QUESTIONS 

[4.1] Which of the following are the special laboratory conditions needed to 
recover C. jejuni? 

A. 37°C (98.6°F) aerobic on blood agar plates 

B. 37°C (98.6°F) anaerobic on blood agar plates 

C. 42°C (107. 6°F) microaerophilic on Skirrow medium 

D. 42°C (107. 6°F) aerobic on Skirrow medium 

[4.2] A 21 -year-old woman presents to the emergency room with shortness 
of breath 2 weeks after recovering from a "stomach flu." Physical exam 
reveals ascending muscle weakness that began in her toes. Cardiac 
irregularities are also notable. A review of the patient's chart revealed 
that a bacterial stool culture 2 weeks earlier, during the patient's "flu" 
episode, found comma-shaped organisms growing at 42°C (107.6°F). 
Which of the following pairs represents the causative agent of this 
patient's flu and the postflu condition, respectively? 

A. Campylobacter jejuni, Guillain-Barre syndrome 

B. Clostridium botulinum, botulism 

C. JC virus, progressive multifocal leukoencephalopathy (PML) 

D. Poliovirus, poliomyelitis 

[4.3] In a nonimmunocompromised patient with C. jejuni as the causative 
agent of their food poisoning, which of the following is the treatment 
most often required? 

A. Metronidazole 

B. Vancomycin 

C. Cephalosporin 

D. TMP-SMZ 

E. Supportive care and hydration 



CLINICAL CASES 49 

[4.4] A 20-year-old college student develops diarrhea that lasts for approxi- 
mately 1 week. Stool cultures reveal a motile, microaerophilic gram- 
negative rod that is isolated by incubation at 41°C (105. 8°F) on medium 
containing antibiotics. This organism is most likely to be which of the 
following? 

A. Escherichia coli 

B. Vibrio parahaemolyticus 

C. Yersinia enterocolitica 

D. Campylobacter jejuni 

E. Proteus vulgaris 

Answers 

[4.1] C. The isolation and identification of C. jejuni can be achieved using 
special culture characteristics. Three requirements must be met. First, 
a selective medium is needed. There are several widely used selective 
media: Skirrow's medium uses vancomycin, polymyxin B, and trimetho- 
prim; other selective media contain cefoperazone, other antimicrobials, 
and inhibitory compounds. The selective media are suitable for isola- 
tion of C. jejuni at 42°C (107.6°F); when incubated at 36-37°C 
(96.8-98.6°F), other Campylobacters and bacteria may be isolated. 
Finally, incubation must be in an atmosphere with reduced oxygen and 
added carbon dioxide. The colonies appear to be colorless or gray and 
may be watery and spreading or round and convex. 

[4.2] A. Guillain-Barre syndrome (acute idiopathic polyneuritis) is associated 
with infections such as herpesvirus and C. jejuni (comma-shaped bacte- 
ria that grows at 42°C [107. 6°F]). It is believed that some C. jejuni 
serotypes have surface lipopolysaccharides that are antigenically simi- 
lar to myelin protein leading to the inflammation and demyelination of 
peripheral nerves and ventral root motor fibers. Suspected Guillain- 
Barre in a patient is always a medical emergency because respiratory 
distress or failure can ensue, and the patient should always be admitted 
to the hospital for careful treatment and observation. The other answers 
contain correct matching of the causative agent with the resulting con- 
dition, but do not reflect the clinical scenario described. 

[4.3] E. Most infections with C. jejuni are self-limiting and thus do not 
require specific antimicrobial therapy, except in cases of severe disease 
and infection in immunocompromised individuals. Therefore, most 
often the only required therapy is hydration and supportive care. When 
specific antimicrobial therapy is indicated, the drug of first choice is 
erythromycin, with alternate drugs being tetracycline, ciprofloxacin, 
and ofloxacin. 

[4.4] D. Based on the culture characteristics indicated above, the only possible 
answer is C. jejuni. Please also refer to the discussion for Question [4.1]. 



50 CASE FILES: MICROBIOLOGY 



MICROBIOLOGY PEARLS 

*♦* Campylobacter species are small motile, non-spore-forming, comma- 
shaped, gram-negative bacilli, best grown in a microaerophilic envi- 
ronment at 42°C (107.6°F). 

*♦* Guillain-Barre is a rare neurological complication of C. jejuni 
gastroenteritis. 

*♦* Campylobacter jejuni gastroenteritis is usually self-limited; however, 
if necessary, erythromycin is the drug of choice. 

*♦* Campylobacter infection most often occurs several days after 
consumption of undercooked chicken. 

*♦* Symptom of fever and abdominal pain may initially mimic appen- 



| 



dicitis. 



REFERENCES 

Alios BM. Campylobacter jejuni infections: update on emerging issues and 

trends. Clin Infect Dis 2001;32:1201-6. 
Murray PR, Rosenthal KS, Pfaller MA. Campylobacter and Helicobacter. In: 

Murray PR, Rosenthal KS, Pfaller MA. Medical Microbiology, 5th ed. St. Louis, 

MO: Mosby, 2005:347-55. 
Winn W, Allen S, Janda W, Koneman E, et al. Curved gram negative bacilli and 

oxidase-positive fermenters; Campylobacteraceae and Vibrionaceae in Koneman's 

Color Atlas and Textbook of Diagnostic Microbiology, 6th ed., Baltimore, MD: 

Lippincott, Williams and Wilkins, 2006:392-428. 



♦♦♦ CASE 5 

A 19-year-old woman presents for the evaluation of pelvic pain. The pain has 
progressively worsened over the past week. She has also been having some 
burning with urination and a vaginal discharge. She is sexually active, has had 
four lifetime partners, takes oral contraceptive pills, and occasionally uses 
condoms. On examination, she appears in no acute distress and does not have 
a fever. Her abdomen is soft with moderate lower abdominal tenderness. On 
pelvic examination, she is noted to have a yellow cervical discharge and 
significant cervical motion tenderness. No uterine or adnexal masses are pal- 
pated, but mild tenderness is also noted. A Gram stain of the cervical discharge 
reveals only multiple polymorphonuclear leukocytes. A direct DNA probe test 
subsequently comes back positive for Chlamydia trachomatis. 

^ How does C. trachomatis enter a target cell? 

^ What are the two stages of the C. trachomatis life cycle? 



52 CASE FILES: MICROBIOLOGY 

ANSWERS TO CASE 5: Chlamydia trachomatis 

Summary: A 19-year-old woman with probable pelvic inflammatory disease 
has a positive DNA probe assay for C. trachomatis. 

How C. trachomatis enters a target cell: The elementary body of 
C. trachomatis binds to receptors on the host and induces endocytosis. 

Two stages of the C. trachomatis life cycle: The elementary body and 
the reticulate body. 

CLINICAL CORRELATION 

Chlamydia trachomatis is the causative agent of the most common sexually 
transmitted disease in the United States, and it is also the greatest cause of pre- 
ventable blindness around the world. Chlamydial disease affects women five 
times more often than men, and approximately two-thirds of those affected 
lack symptoms and thus, do not know that they are infected. Many of those 
infected with gonorrheal disease are also infected with Chlamydia, as both 
organisms infect the columnar epithelial cells of the mucous membranes. 
Chlamydial disease usually affects those of lower socioeconomic standing and 
is prevalent in underdeveloped countries. Children are also a main reservoir, 
transmitting the disease by hand-to-hand transfer of infected eye fluids or by 
sharing contaminated towels or clothing. 

APPROACH TO SUSPECTED CHLAMYDIAL 
INFECTION 

Objectives 

1. Know the characteristics of the Chlamydia species. 

2. Know the virulence factors and diseases associated with Chlamydia 
bacteria. 

Definitions 

Elementary body: Nondividing 300-nm infectious particle. This particle 

has an outer membrane with disulfide linkages which allows it to survive 

extracellularly. 
Chandelier sign: Cervical motion tenderness during the bimanual exam, 

characteristic of pelvic inflammatory disease (PID). 
Exudate: Material, such as fluids, cells or debris, which has extravasated 

from vessels and has been deposited on tissue surfaces or in tissue. 
Papule: Small palpable elevated lesion that is less than 1 cm. 



CLINICAL CASES 53 

DISCUSSION 

Characteristics of Chlamydia trachomatis 

Chlamydia trachomatis is a gram-negative obligate intracellular parasite 

with a unique life cycle. It is coccoid in morphology and is very small, usually 
about 350 nm in diameter. Although C. trachomatis is classified as gram- 
negative bacteria, it lacks a peptidoglycan layer and muramic acid, which 
are present in other gram-negative organisms. There are many disulfide link- 
ages present in the outer membrane which stabilize the organism. Its extra- 
cellular form is called the elementary body, which has a small, spore-like 
structure. It attaches to columnar, cuboidal, or transitional epithelial cells in 
structures lined by mucous membranes. The elementary body binds to receptors 
on susceptible cells and induces endocytosis into the host. These membrane- 
protected structures are known as inclusions. The elementary body undergoes 
reorganization into a larger, more metabolically active form known as the 
reticulate body. Reticulate bodies grow and multiply by binary fission to 
create larger intracellular inclusions. Reticulate bodies transform back into 
elementary bodies, which are released from the epithelial cell by exocytosis 
and which can then infect other cells. The life cycle of C trachomatis lasts 
approximately 48-72 hours. Table 5-1 lists in sequential order are the stages 
of the life cycle. 

Chlamydia trachomatis appears to be an obligate human pathogen with 
approximately 15 serotypes. It is the most common bacterial cause of sexually 
transmitted diseases in humans and also causes conjunctivitis and ocular 
trachoma. Infection of the conjunctiva by C. trachomatis results in scarring 
and inflammation. This fibrosis pulls the eyelid inward causing the eyelashes 
to rub against the cornea. Because the eyelid is rolled inward, the individual is 
unable to completely close the eye resulting in the inability to maintain 
moisture on the surface of the eye. It is the combination of the lack of surface 



Table 5-1 

LIFE CYCLE OF Chlamydia trachomatis 

1 . Elementary body attaches to host cell. 

2. Host cell phagocytizes the elementary body residing in a vacuole, inhibiting 
phagosome-lysome fusion. 

3. The elementary body reorganizes to form a reticulate body. 

4. The reticulate body divides by binary fusion. 

5. Some reticulate bodies convert back into elementary bodies; elementary bodies are 
released into host cell. 



54 CASE FILES: MICROBIOLOGY 

moisture and constant abrasion by the eyelashes that causes corneal scarring 
and blindness. Ocular trachoma is one of the leading causes of blindness 
worldwide. 

Chlamydia trachomatis also causes other diseases including pneumonia, 
urethritis, epididymitis, lymphogranuloma venereum, cervicitis, and pelvic 
inflammatory disease. Lymphogranuloma venereum presents with a pain- 
less papule on the genitalia that heals spontaneously. The infection is then 
localized to regional lymph nodes where it resides for approximately 
2 months. As time progresses, the lymph nodes begin to swell, causing pain, 
and may rupture and expel an exudate. Men with epididymitis present with 
fever, unilateral scrotal swelling, and pain. Women with cervicitis present with 
a swollen, inflamed cervix. There may also be a yellow purulent discharge 
present. PID occurs when the infection spreads to the uterus, fallopian tubes, 
and ovaries. PID presents with lower abdominal pain, dyspareunia, vaginal 
discharge, uterine bleeding, nausea, vomiting, and fever. Cervical motion ten- 
derness during the bimanual exam is known as the "chandelier sign." 
Recurrent PID may scar the fallopian tubes, resulting in infertility or ectopic 
pregnancy. Children may acquire chlamydial disease during birth via passage 
through an infected birth canal. Inflammation of the infant's conjunctiva 
may occur with a yellow discharge and swelling of the eyelids within 2 weeks 
after birth. The presence of basophilic hit racy top lasmic inclusion bodies 
from the conjunctiva is a helpful diagnostic clue. Neonatal pneumonia may 
also occur from passage through an infected birth canal. An infected child may 
present 4-1 1 weeks after birth with respiratory distress, cough, and tachypnea. 
The direct destruction of host cells due to chlamydial infection and then host's 
inflammatory response produces the clinical symptoms associated with the 
various forms of chlamydial disease. 

Other Chlamydial species are known to cause disease in humans. Atypical 
pneumonia is caused by Chlamydophila pneumonia, and presents with fever, 
headache, and a dry hacking cough. Additionally, psittacosis is another atypical 
pneumonia caused by Chlamydophila psittaci. This organism is acquired by 
inhalation of feces from infected birds, which serve as the reservoir. 

Diagnosis 

Infection with C. trachomatis can be rapidly diagnosed by detection of the 
bacterial nucleic acid in patient samples from the oropharynx, conjunctiva, 
urethra, or cervix. Other specimens such as the conjunctiva can be cultured 
using McCoy cells in a tissue culture assay. Diagnostic tests for nucleic acid 
detection include PCR amplification or direct DNA hybridization assays, 
measuring for specific 16S ribosomal RNA sequences can be performed on all 
of above specimens including urine. 



CLINICAL CASES 55 

Treatment and Prevention 

Currently, the best method of preventing chlamydial infection is education and 
proper sanitation. Ocular infection of C trachomatis can sometimes but not 
always be prevented by administration of topical tetracycline drops. It is the 
lack of this antibiotic in underdeveloped countries that makes C. trachomatis 
prevalent in these areas. Chlamydia trachomatis, C. psittaci, and C. pneumonia 
are all treated with tetracycline or erythromycin. Azithromycin is effective for 
cervicitis and urethritis. Pelvic inflammatory disease is treated with ceftriax- 
one and 2 weeks of doxycycline. 

COMPREHENSION QUESTIONS 

[5.1] A 32-year-old immigrant woman from Tanzania delivers a healthy 
baby boy. Because this woman had no regular doctor, no preliminary 
tests were performed prior to delivery. Thirteen days after delivery, the 
child develops swelling of both eyes with the presence of a yellow dis- 
charge. The presence of intracytoplasmic inclusion bodies is detected. 
Which antibiotic would be most appropriate in this situation? 

A. Tetracycline 

B. Ceftriaxone 

C. Penicillin G 

D. Doxycycline 

E. Erythromycin 

[5.2] Which diagnostic test is best to identify an infection with C. trachomatis? 

A. Aerobic and anaerobic blood cultures 

B. Stool culture 

C. DNA probe 

D. Urine culture 

E. Culture and darkfield microscopy 

[5.3] A 29-year-old bird collector presents to the local clinic with what he 
describes as flu-like symptoms. He doesn't look ill, and has a slight 
fever, headache, and a dry hacking cough. He denies the production of 
sputum or hemoptysis. There are no crackles heard on auscultation, and 
a radiograph shows small streaks of infiltrate. It is determined that he 
has pneumonia. What is the most likely organism causing his disease? 

A. Streptococcus pneumoniae 

B. Chlamydophila psittaci 

C. Haemophilus influenzae 

D. Staphylococcus aureus 

E. Chlamydophila pneumoniae 



56 CASE FILES: MICROBIOLOGY 

Answers 

[5.1] E. The symptoms described are classic for inclusion conjunctivitis 
caused by C. trachomatis. The infection was most likely passed from 
the mother to child during vaginal delivery. The infection usually 
presents 2 weeks after delivery and is characterized by swollen eyes 
and a yellow discharge. The drug of choice for this infection is eryth- 
romycin eyedrops. Most children are given erythromycin eye drops 
prophylactically postbirth. Tetracyclines are not given to young chil- 
dren due to staining of teeth. 

[5.2] C. The most specific test used to detect a chlamydial infection is a 
DNA probe. C. trachomatis is a gram-negative obligate intracellular 
parasite, and any blood or urine culture would not be helpful for diag- 
nosis. Chlamydia trachomatis is not present in stool. Darkfield 
microscopy is used to view spirochetes, which C. trachomatis is not. 

[5.3] B. Although all of the organisms listed above cause pneumonia, only 
two of them are atypical. Atypical pneumonia is characterized by a dry 
hacking cough, fever, and headache. These include C. psittaci and C. 
pneumoniae. The mention of birds should point you in the direction of 
C. psittaci, because they are the reservoir for the organism that is 
inhaled through dry feces. Typical pneumonias are characterized by 
hemoptysis of pus-laden sputum, and patients appear very sick. 



MICROBIOLOGY PEARLS 

*♦* Chlamydial disease is the most common STD in the United States. 
*♦* Cervical motion tenderness and adnexal tenderness are common 



findings of pelvic inflammatory disease. 

The elementary body is the infectious stage in C. trachomatis. 

Chlamydia trachomatis is the most common preventable cause of 
worldwide blindness (Proper sanitation is important for preven- 
tion and the use of tetracycline or erythromycin is important for 
early treatment.). 



REFERENCES 

Murray PR, Rosenthal KS, Pfaller MA. Medical Microbiology, 5th ed. St. Louis, 
MO: Mosby, 2005:412-420. 

Schneider AS, Szanto PA. Pathology Board Review Series, 2nd ed. Philadelphia, 
PA: Lippincott Williams & Wilkins, 2001:272, 280. 

Boman J, Gaydos C, Quinn T. Minireview: molecular diagnosis of Chlamydia pneu- 
moniae infection. J Clin Microbiol 1999;37:3791-9. 



♦♦♦ CASE 6 

A 52-year-old man presents for the evaluation of diarrhea and abdominal 
pain, which have been worsening over the past week. He is now having 8-10 
watery stools a day and mild cramping pain. He denies vomiting, fever, ill 
contacts, or having had blood in his stool. He has no history of gastrointesti- 
nal diseases. He states that approximately 10 days ago he completed a course 
of amoxicillin/clavulanate for pneumonia. On examination he is mildly ill 
appearing, but his vital signs are normal. His abdomen is soft, has hyperac- 
tive bowel sounds, and is diffusely, mildly tender. A stool sample is negative 
for blood but positive for leukocytes. A stool culture is negative, but a specific 
toxin assay is positive. 

^ What is the most likely etiologic agent of this disease? 

^ Which condition predisposes this organism to cause disease in 
humans? 



58 CASE FILES: MICROBIOLOGY 

ANSWERS TO CASE 6: CLOSTRIDIA 

Summary: A 52-year-old man who recently took oral antibiotics, now has diar- 
rhea. Fecal leukocytes are present in the stool, and a toxin test is positive. 

^ Most likely etiologic agent: Clostridium difficile 

^ Condition predisposing disease in humans: Recent antibiotic 
exposure 

CLINICAL CORRELATION 

There are approximately 90 bacterial species of Clostridium, about 20 of 
which are known to cause disease in humans. They are found widely in soil, 
decaying vegetation, and the intestinal tracts of humans and other vertebrates. 
Infection occurs in patients with predisposing factors including trauma, sur- 
gery, immunosuppression, and prior treatment with antibiotics. Clostridium 
perfringens is the most common Clostridium species isolated from human 
infections and is a cause of wound infections including gas gangrene. Clostridium 
tetani is associated with the toxin mediated disease, tetanus, which occurs in 
unvaccinated persons who come in contact with the organism. The spores of 
the organism survive for long periods of time in the soil and are introduced 
into the person following deep puncture wounds. Tetanus is characterized by 
tonic spasms usually involving the muscles of the neck, jaw (lockjaw), and 
trunk. Clostridium botulinum is the causative agent of botulism. Botulism 
occurs when spores are consumed usually from improperly canned vegetables. 
Symptoms of nausea, blurred vision, and weakness of the upper extremities 
spreading downward occur within 12-36 hours after ingestion of the toxin. 
Infant botulism is associated with consumption of honey. 

Clostridium difficile can be isolated in the stool of fewer than 5 percent of 
healthy adults; however, up to 70 percent of healthy infants have the organism 
in their stool. Most cases of C. difficile colitis occur during or after a course of 
antibiotics. Antibiotics alter the intestinal flora allowing for an overgrowth of 
C. difficile, which either already exists in the intestinal tract or is introduced 
from an environmental source. Disease can range from asymptomatic carriage 
of the organism to mild diarrhea to pseudomembranous colitis, which can be 
further complicated by toxic megacolon and bowel perforation. 

APPROACH TO SUSPECTED Clostridium difficile 
INFECTION 

Objectives 

1. Know the characteristics of the Clostridium species. 

2. Know the virulence factors and diseases associated with Clostridium 
bacteria. 



CLINICAL CASES 59 

Definitions 

Antibiotic-associated diarrhea: Gastroenteritis caused by C. difficile 
Pseudomembranous colitis: Presence of nodules or plaques on erythematous 

(red) colonic mucosa seen by sigmoidoscopy, associated with C. difficile 

colitis 

DISCUSSION 

Characteristics of Clostridium difficile 

Clostridium difficile is an anaerobic, spore-forming, toxigenic gram-positive 

rod. Some strains have a thin capsule and some have fimbriae, although the sig- 
nificance of these is uncertain. Clostridium difficile, so named because of the 
initial difficulty in isolating and culturing the organism, requires a selective 
medium for growth which also inhibits normal stool flora. 

The virulence factors of C. difficile include toxin production as well as 
production of other enzymes, such as hyaluronidase. Toxin A is an entero- 
toxin, and Toxin B, the more biologically active toxin in humans, is a cytotoxin. 
The specific role each component plays in disease in humans is unknown. The 
enterotoxin is chemotactic and initiates the release of cytokines, hypersecretion 
of fluids in the intestinal tract, and hemorrhagic necrosis. Depolarization of actin 
microfilaments occurs, which leads to destruction of the cellular cytoskeleton 
disruption of tight junctions between epithelial cells. A new strain of C. difficile 
has been recently identified, which is more virulent and more likely to cause 
megacolon. This strain has been found to produce larger quantities of Toxins 
A and B in addition to a new toxin, binary toxin. Formation of spores allows the 
organism to survive under stressful situations in the environment for extended 
periods of time. Spore formation also allows the organisms to survive in the hos- 
pital environment and can be transferred from patient to patient on fomites. 

Diagnosis 

Antibiotic-associated diarrhea is the most common cause of diarrhea that 
develops in patients who have been hospitalized for 3 or more days. Clinical 
diagnosis can be made by visualization of the pseudomembrane (fibrin, 
bacteria, cell debris, white blood cells). 

The gold standard for laboratory diagnosis of antibiotic-associated 
diarrhea caused by C. difficile is detection of toxin production in the stool 
using a tissue culture assay, where a specific antibody neutralizes the toxin and 
therefore, the production of cytopathic effect. However, this assay requires 
tissue culture facilities as well as approximately 3 days for completion. Culture 
of C. difficile can be performed on selective media, cycloserine, cefoxitin, and 
fructose agar in an egg yolk agar base (CCFA medium), in an anaerobic envi- 
ronment. After 24-48 hours, incubation colonies will fluoresce chartreuse on 



60 CASE FILES: MICROBIOLOGY 

CCFA and have a barnyard odor. Specific identification can be made using 
commercially available rapid methods that detect fatty acids produced by the 
organism or by gas-liquid chromatography. Growth of the organism would have 
to be followed up by detection of toxin for a specific diagnosis of disease. 

Commercially available membrane or microwell based enzyme immunoas- 
says are available for rapid detection of Toxin A or Toxin A and B in a stool spec- 
imen. For optimal recovery testing of three stools on 3 days is recommended. 

Treatment and Prevention 

The first-line treatment for C. difficile disease is oral metronidazole, with oral van- 
comycin reserved for those who fail first-line treatment. Unfortunately, relapse can 
occur in 20-30 percent of adequately treated patients because of the resistance of 
the spores to treatment. A second round of treatment is usually successful. Failure 
is not usually attributed to resistance of the organism to vancomycin or metron- 
idazole. Prevention of C. difficile in hospitalized patients involves good infection 
control procedures that include isolation of the infected patient. 

COMPREHENSION QUESTIONS 

[6.1] Which organism listed below may cause a life-threatening gastroen- 
teritis as a result of use of a broad spectrum antimicrobial agent? 

A. Bacillus anthracis 

B. Bacillus cereus 

C. Clostridium botulinum 

D. Clostridium difficile 

E. Clostridium tetani 

[6.2] Clostridium difficile, as the causative agent in antibiotic-associated 
diarrhea, can best be detected using which of the following gold stan- 
dard laboratory tests? 

A. Gas-liquid chromatography 

B. Pseudomembranous visualization 

C. Rapid fatty acid detection assays 

D. Tissue culture toxin detection assay 

[6.3] A hospitalized patient developed severe diarrhea and pseudomembra- 
nous colitis within 5 days after antibiotic therapy was initiated. The 
severe diarrhea and pseudomembranous colitis occurred as a result of 
which of the following? 

A. Collagenase 

B. Fibrinolysin 

C. Hyaluronidase 

D. Lecithinase 

E. Mucinase 

F. Toxin A and B 



CLINICAL CASES 61 

Answers 

[6.1] D. The use of broad spectrum antibiotics such as ampicillin and clin- 
damycin has been associated with pseudomembranous colitis. 
Antibiotic administration results in the proliferation of drug-resistant 
C. difficile that produces Toxin A (a potent enterotoxin with cytotoxic 
activity) and Toxin B (a potent cytotoxin). This disease is best treated 
by discontinuing the use of the offending antibiotic and administering 
oral doses of metronidazole or vancomycin. Administration of antibi- 
otics may also lead to a milder form of diarrhea, called antibiotic- 
associated diarrhea. This form is associated with C. difficile about 
25 percent of the time. 

[6.2] D. All of the above tests may be used as detection assays for C. diffi- 
cile. However, only the tissue culture toxin detection assay is the gold 
standard laboratory test. This test involves a specific toxin neutraliz- 
ing antibody that detects toxin (Toxin A and B) production in the 
stool using a tissue culture detection assay. Not all C. difficile strains 
produce toxins, and the tox genes are not carried on either plasmids 
or phages. 

[6.3] F. Clostridium difficile produces two toxins, Toxins A and B. Both 
toxins are present in stool samples. Toxin A is enterotoxic causing the 
severe diarrhea, whereas Toxin B is cytotoxic leading to the destruc- 
tion of enterocytes resulting in pseudomembranous colitis. For addi- 
tional information please refer to the discussions for Questions 6.1 
and 6.2. 



MICROBIOLOGY PEARLS 

*♦* The most common cause of diarrhea in a patient who has been hos- 
pitalized for 3 or more days is C. difficile. 

*♦* The initial treatment for pseudomembranous colitis is metronida- 
zole. Oral vancomycin is used for those who fail to respond to 
metronidazole. 

*♦* Detection of toxins in the stool is the method of choice for diagno- 
sis of C. difficile colitis. 



REFERENCES 

Allen SD, Emery CL, Lyerly DM. In: Murray PR, Baron EJ, Jorgensen JH, et al., 

eds. Manual of Clinical Microbiology, 8th ed. Washington, DC: ASM Press, 

2003:835-56. 
Murray PR, Rosenthal KS, Pfaller MA. Clostridium. In: Murray PR, Rosenthal KS, 

Pfaller MA. Medical Microbiology, 5th ed. St. Louis, MO: Mosby, 

2005:401-20. 



This page intentionally left blank 



♦♦♦ CASE 7 

A 6-year-old girl is brought into the office for evaluation of a sore throat and 
fever, which she has had for approximately 4 days. Her parents have immi- 
grated to the United States from Russia about 6 months ago. She has not had 
much medical care in her life, and her immunization status is unknown. On 
examination the child is anxious, tachypneic, and ill appearing. Her tempera- 
ture is 38.6°C (101.5°F), and her voice is hoarse. Examination of her pharynx 
reveals tonsillar and pharyngeal edema with the presence of a gray membrane 
coating of the tonsil, which extends over the uvula and soft palate. She has 
prominent cervical adenopathy. Her lungs are clear. You immediately transfer 
her to the local children's hospital with the presumptive diagnosis of pharyn- 
geal diphtheria and order confirmatory tests. 

^ What Gram stain characteristics does Corynebacterium diphtheriae 
have on microscopy? 

^ What factor is required for the expression of diphtheria toxin? 



64 CASE FILES: MICROBIOLOGY 



ANSWERS TO CASE 7: Corynebacterium diphtheriae 

Summary: A 6-year-old girl who recently arrived from Russia is diagnosed 
with pharyngeal diphtheria. 

^ Characteristics of C. diphtheriae on Gram stain: The club-shaped 
appearance of the gram-positive bacillus, often characterized as 
"Chinese letters" because of adherence of cells following division. 

^ Factor required for the expression of diphtheria toxin: Lysogenic 
bacteriophage. 



CLINICAL CORRELATION 

Corynebacteria are ubiquitous in nature and are part of the normal flora of the 
human respiratory tract and skin. Although most species of Corynebacterium 
can be opportunistic pathogens, only a few species are commonly associated 
with human disease. One of those species is Corynebacterium jeikeium, which 
is most commonly associated with bacteremia and line-related infection in 
immunocompromised patients. This organism is one of the few species of 
Corynebacterium that tends to be multidrug resistant. Corynebacterium diph- 
theriae, the cause of diphtheria, is one of the most pathogenic of the species. 
Humans are the only known reservoir and transmission is thought to occur by 
contact with aerosolized droplets, respiratory secretions, or infected skin 
lesions. Respiratory diphtheria occurs 2-6 days after inhalation of infected 
droplets. Patients develop nonspecific signs and symptoms of an upper respira- 
tory infection as the organisms multiply locally with in epithelial cells in the 
respiratory tract. Toxin is then produced eliciting systemic symptoms including 
fever. An exudate, containing organisms, fibrin, and white and red blood cells, 
is formed, which is called a pseudomembrane. This grayish membrane covers 
the tonsils, uvula, and palate and can extend as far as the nasopharynx or larynx. 
Complications of membrane formation can be respiratory compromise by aspi- 
ration of the pseudomembrane, which is a common cause of death in this dis- 
ease. Symptoms include fever and cervical lymphadenopathy (bull neck). 

Cutaneous diphtheria, although rare in the United States, occurs from inva- 
sion of the organism from the patient's skin into the subcutaneous tissue. 
A papule develops at the site of contact that later becomes covered by a gray- 
ish membrane. As in respiratory diphtheria, toxin production by the organism 
elicits a systemic response with fever. Diphtheria toxin can also have effects 
on the heart (myocarditis) and nervous system (dysphagia, paralysis). 



CLINICAL CASES 65 

APPROACH TO SUSPECTED DIPHTHERIA 
INFECTION 

Objectives 

1 . Know the characteristics and virulence factors of C. diphtheriae. 

2. Know the factors involved with the expression of and the mechanism 
of action of the C. diphtheriae exotoxin. 

Definitions 

Lysogenic bacteriophage: Virus that infects bacteria. 

Elek Test: An immunodiffusion test to detect the production of diphtheria 

toxin in a strain of C. diphtheria. 
Pseudomembrane: Membrane formed in diphtheria, which consists of 

dead cells, leukocytes, and fibrin. 

DISCUSSION 

Characteristics of Corynebacterium diphtheriae 

Corynebacterium diphtheriae is a nonencapsulated gram-positive bacillus. 
It is nonmotile, non-spore-forming, and club shaped. The cells often remain 
attached after division and form sharp angles, giving a characteristic 
"Chinese letter" appearance on microscopy. 

Corynebacterium diphtheriae is divided into three subtypes — gravis, inter- 
medin, and mitis — based on colony morphology and biochemical testing. 

In the presence of a lysogenic (3-phage, C. diphtheriae can produce a highly 
potent exotoxin. The toxin, which is the major virulence factor of this organ- 
ism, consists of two components. The B segment binds to specific receptors 
on susceptible cells. Following proteolytic cleavage, the A segment is released 
into the host cell, where it can inhibit protein synthesis. The exotoxin targets 
a factor present in mammalian cells but not in bacterial cells, thus causing host 
tissue damage without affecting bacterial replication. Toxin-related tissue 
necrosis causes the characteristic pseudomembrane seen in clinical diphtheria. 

Diagnosis 

The differential diagnosis in the presence of sore throat, fever, and cervical 
lymphadenopathy would include streptococcal pharyngitis and infectious 
mononucleosis. 

Clinical diagnosis of diphtheria can be made by visualization of the 
characteristic pseudomembrane formation. The membrane should not be 
removed because of the tight adherence to the epithelial surface and the 
chance for subsequent bleeding. Cultures should be collected from the throat 



66 CASE FILES: MICROBIOLOGY 

or nasopharynx. A Gram stain would reveal the characteristic gram-positive 
club-shaped bacilli. 

Corynebacterium with the exception of a few lipophilic species will grow 
well on most nonselective media within 24 hours. Colonies are usually 
nonpigmented and small, without hemolysis on blood agar. However, 
C. diphtheriae is more fastidious and specimens should be plated on a selective 
medium such as Tellurite in addition to the nonselective media. Colonies of C. 
diphtheriae will appear black on Tellurite media. Colonies growing on Loeffler 
media can be stained with methylene blue to observe the characteristic 
metachromatic granules. Definitive identification is made by biochemical tests 
usually performed at a reference or state public health laboratory, where the iso- 
late will be further tested for toxin production. The Elek test is an immunodif- 
fusion assay for detection of production of C. diphtheria toxin by the isolate. 

Treatment and Prevention 

Therapy for diphtheria is a combination of antimicrobial therapy (erythro- 
mycin) and antitoxin. The antitoxin must be administered rapidly, before 
the toxin binds to epithelial cells. Diphtheria can be prevented by vaccination 
with diphtheria toxoid (DPT). Infected patients should be isolated from other 
susceptible persons to prevent secondary spread of the disease. Prophylaxis 
with erythromycin can also be given to close contacts who are at risk. 

COMPREHENSION QUESTIONS 

[7.1] The mechanism of action of the exotoxin produced by C. diphtheriae 
can be characterized by which of the following? 

A. Acting as a superantigen that binds to MHC class II protein and 
the T-cell receptor. 

B. Blocking the release of acetylcholine causing anticholinergic 
symptoms. 

C. Blocking the release of glycine (inhibitory neurotransmitter). 

D. Inhibits protein synthesis via EF-2 adenosine diphosphate (ADP) 
ribosylation. 

E. Stimulation of adenylate cyclase by ADP ribosylation of G-protein. 

[7.2] Which of the following most accurately describes the therapy available 
for the prevention and treatment of C. diphtheriae? 

A. Antimicrobial therapy for prophylaxis only 

B. Antimicrobial therapy and prophylaxis, antitoxin, and toxoid (DPT) 

C. Antitoxin only 

D. Diphtheria toxoid (DPT) booster vaccination only 



CLINICAL CASES 67 

Answers 

[7.1] D. Corynebacterium diphtheriae produces a potent exotoxin encoded 
by a lysogenic (3-prophage. Following proteolytic cleavage, the A 
segment is released into the host cell where it inhibits only mam- 
malian protein synthesis (ribosomal function) via ADP ribosylation 
of EF-2. Inhibition of protein synthesis disrupts normal cellular phys- 
iologic functions that are believed to be responsible for the necrotiz- 
ing and neurotoxic effects of diphtheria toxin. An exotoxin with a 
similar mode of action is produced by some Pseudomonas aerugi- 
nosa strains. Staphylococcus aureus is responsible for producing the 
toxic shock syndrome toxin that acts as a superantigen leading to 
T-cell activation. Clostridium tetani blocks the release of glycine, 
leading to "lock-jaw." Clostridium botulinum blocks the release of 
acetylcholine, causing central nervous system (CNS) paralysis and 
anticholinergic symptoms. Finally, the heat-labile toxin produced by 
Escherichia coli causes watery diarrhea by stimulating adenylate 
cyclase. 

[7.2] B. Protection against C. diphtheriae can be established through both 
active and passive immunity. Active immunity consists of a toxoid 
administered in the form of the DPT vaccine. Passive immunity is 
established by administering diphtheria antitoxin (immunoglobulins). 
Antimicrobial therapy (erythromycin) can be used to effectively treat 
patients with clinical diphtheria. 



MICROBIOLOGY PEARLS 

*♦* The club-shaped appearance of the gram-positive bacillus 
C. diphtheriae is often characterized as "Chinese letters" as a 
result of adherence of cells following division. 

*♦* The typical clinical feature of diphtheria is pseudomembrane formation. 

*♦* Diphtheria is preventable by administration of DPT vaccine, which 
provides immunity for diphtheria, pertussis, and tetanus. 



REFERENCES 

Holmes, RK. Biology and molecular epidemiology of diphtheria toxin and the tox 

gene. J Infect Dis 2000;181(suppl l):S156-67. 
Murray PR, Rosenthal KS, Pfaller MA. Corynebacterium and other gram-positive 

bacilli. In: Murray PR, Rosenthal KS, Pfaller MA. Medical Microbiology, 5th ed. 

St. Louis, MO: Mosby, 2005:279-85. 



This page intentionally left blank 



►♦♦ CASE 8 



♦:♦ 



A 72-year-old female nursing home resident is transferred to the hospital 
because of fever and altered mental status. She has advanced Alzheimer dis- 
ease, is bed bound, and has an indwelling Foley catheter as a consequence of 
urinary incontinence. Her baseline mental status is awake and talkative, but 
oriented only to person. In the hospital now, she has a temperature of 38.3°C 
(101°F) and tachycardia (a rapid heart rate). She mumbles incoherently and is 
otherwise nonverbal. Her skin is cool, dry, and without ulceration. Her mucous 
membranes are dry. Her abdomen is soft, has normoactive bowel sounds, and 
is apparently tender in the suprapubic region. Her urinary catheter is draining 
cloudy urine. A urinalysis reveals too numerous to count white blood cells and 
bacteria. Gram stain of the urinary sediment reveals gram-positive cocci. 
Blood and urine cultures also grow gram-positive cocci. 

^ What is the most likely cause of this infection? 

^ How does this organism acquire antibiotic resistance? 



70 CASE FILES: MICROBIOLOGY 

ANSWERS TO CASE 8: Enterococcus faecalis 

Summary: A 72-year-old woman with an indwelling urinary catheter has a 
urinary tract infection and bacteremia. Gram-positive cocci are isolated from 
the urine and blood cultures. 

<%> Most likely etiology of infection: Enterococcus faecalis 

^ Mechanism of development of antibiotic resistance: DNA mutation, 
or plasmid or transposon transfer 

CLINICAL CORRELATION 

Enterococci are normal flora of the gastrointestinal (GI) tract and are therefore 
more likely to cause infections in patients with a history of preceding abdom- 
inal or genital tract procedures. Although a common cause of community 
acquired urinary tract infections (UTIs), enterococci are most often associated 
with nosocomial UTI, particularly in patients with urinary catheters. 
Bacteremia and rarely endocarditis can result as complications of enterococ- 
cal urinary tract or wound infections, with the GI tract the most likely source. 
Patients at higher risk for enterococcal endocarditis are elderly patients and 
those with underlying heart disease, particularly the presence of artificial heart 
valves. Enterococci usually are a cause of subacute left-sided or mitral 
valve endocarditis. 

APPROACH TO SUSPECTED ENTEROCOCCAL UTI 
Objectives 

1 . Know the characteristics of E. faecalis. 

2. Know the nature of the intrinsic and acquired antibiotic resistances of 
E. faecalis. 

Definitions 

Tachycardia: Increased heart rate above 100 beats per minute. 
Transposons: Small pieces of DNA that can replicate and insert randomly 

in the chromosome. 
Leukocyte esterase: An enzyme present in leukocytes, therefore used as an 

indirect marker of their presence. 



CLINICAL CASES 71 

DISCUSSION 

Characteristics of Enterococcus Species 

Enterococcus faecalis is an aerobic gram-positive coccus commonly found 
as normal fecal flora of healthy humans and other animals. It is capable of 
growing in extreme conditions, including a wide range of temperatures, high 
pH, the presence of high concentrations of bile salts, and saline concentrations 
of up to 6.5 percent. Enterococci have also been isolated from soil, food, and 
water. Enterococci are difficult to distinguish morphologically from streptococci, 
and for years were considered a member of the Streptococcus family. They 
possess the group D streptococcal carbohydrate antigen on their cell sur- 
face. Like streptococci, enterococci are often seen singly, in pairs, or short 
chains on microscopy. Little is known about the virulence factors associated 
with E. faecalis. Some strains of Enterococcus produce factors, not totally elu- 
cidated, which allow their adherence to both heart valves and urinary epithelial 
cells. One of its other major virulence factors is an intrinsic resistance to 
multiple antibiotics, including ampicillin, penicillin, and aminoglycosides, 
which are effective against other gram-positive bacteria. There is also evidence 
for acquired antibiotic resistance, either by mutation of native DNA or acqui- 
sition of new DNA from plasmid or transposon transfer. It is capable of 
acquiring resistance both from other enterococci and from other bacterial 
species and has recently been shown to transfer the gene for vancomycin 
resistance to Staphylococcus aureus. 

Diagnosis 

Clinical diagnosis of UTI is made by typical clinical symptoms of urgency and/or 
dysuria followed by a urinalysis and bacterial culture. The presence of white 
blood cells (positive leukocyte esterase) and bacteria in the urine are indicative of 
cystitis. The specific etiologic agent can only be determined by culturing the urine 
in a quantitative manner. The presence of greater than 10 5 colony-forming units 
(CFUs) per milliliter of clean catch urine or 10 4 CFU/mL of catheterized urine is 
considered significant for a UTI. Colonies of Enterococcus appear nonhemolytic 
or, in rare cases, a-hemolytic on blood agar and can be specifically identified 
using a rapid PYR (L-pyrrolidonyl-(3-naphthylamide) test. Conventional, 
overnight identification includes growth in 6.5 percent sodium chloride 
and esculin hydrolysis in the presence of bile. Further identification of 
enterococci to the species level is not commonly done in routine clinical lab- 
oratories. Although most commercially available identification methods can 
speciate enterococci difficulties in accurate speciation occurs without the use 
of DNA sequence analysis. Most clinically significant enterococci are either 
E. faecalis or E. faecium. Enterococcus faecium tends to be more resistant to 
antibiotics particularly ampicillin and vancomycin. 



72 CASE FILES: MICROBIOLOGY 

Treatment and Prevention 

Although enterococci are intrinsically resistant to low concentrations of 
(3-lactam antibiotics, such as ampicillin, these agents are still the first choice 
for uncomplicated enterococcal urinary tract infections in cases in which the 
affecting strain is not highly resistant. For complicated UTI or endocarditis, 
bactericidal therapy is necessary and includes ampicillin or vancomycin plus 
an aminoglycoside, assuming that the infecting strain is susceptible to ampi- 
cillin or vancomycin and high levels of aminoglycosides. An alternative would 
be vancomycin if it is susceptible; or if resistant, alternative agents such as 
linezolid or quinupristin/dalfopristin might be appropriate. 

Although there is no specific prevention for enterococci because they are 
able to survive for extended periods of time on inanimate objects, nosocomial 
outbreaks have been associated with antibiotic-resistant strains of enterococci 
and proper disinfection and infection control measures are necessary to pre- 
vent further spread. 

COMPREHENSION QUESTIONS 

[8.1] Testing of blood culture isolates from a hospitalized patient revealed gram- 
positive cocci, (3-lactamase positive, vancomycin-resistant, PYR -positive, 
and the presence of Lancefield group D antigen. Which of the following is 
the most likely isolate identification? 

A. Enterococcus faecalis 

B. Streptococcus agalactiae 

C. Streptococcus bovis 

D. Streptococcus pneumoniae 

[8.2] Which of the following is the most serious condition that can result as 
complications of enterococcal urinary tract or wound infections? 

A. Cellulitis 

B. Gastroenteritis 

C. Scarlet fever 

D. Subacute endocarditis 

E. Toxic shock syndrome 

[8.3] After an abdominal surgery for removal of ovarian cysts, this 56-year-old 
woman has had low-grade fever for the past 2 weeks. She has a history of 
rheumatic fever as a child. Three of the blood cultures grew gram-positive 
cocci. Which of the following is the most likely etiologic agent? 

A. Group A streptococci 

B. Group B streptococci 

C. Group C streptococci 

D. Group D streptococci 

E. Viridans streptococci 



CLINICAL CASES 73 

Answers 

[8.1] A. All bacteria listed are gram-positive cocci. Streptococcus pneumo- 
niae does not have a Lancefield grouping, whereas 5*. agalactiae has 
a group B classification. Streptococcus bovis is PYR -negative. Thus, 
only E. faecalis fulfills all laboratory test results in the above question. 

[8.2] D. In patients, the most common sites of enterococci infection are the 
urinary tract, wounds, biliary tract, and blood. In neonates, entero- 
cocci can cause bacteremia and meningitis. In adults, enterococci 
may cause endocarditis. Thus, bacteremia and/or endocarditis are 
rare and very serious complications that can result from enterococcal 
UTI. Cellulitis and toxic shock syndrome are typically associated 
with both staphylococci and streptococci, whereas scarlet fever is 
associated only with streptococci. Finally, gastroenteritis can be asso- 
ciated with a number of organisms such as Clostridium difficile. 

[8.3] D. Streptococcus bovis is among the nonenterococcal group D strep- 
tococci. They are part of the enteric flora and have the ability to cause 
endocarditis. 



MICROBIOLOGY PEARLS 

*♦* Enterococci, gram-positive cocci, are normal flora of the human and 

animal GI tract. 
V Enterococci are a common cause of wound infections following 

procedures involving the GI or genitourinary (GU) tracts. 
*♦* Bacteremia and/or endocarditis are rare complications of enterococcal 

UTIs. 
*♦* Enterococcal UTIs are often nosocomial infections, especially in 

elderly patients with urinary catheters. 
*♦* Ampicillin and vancomycin are the principal antibiotics used to treat 

enterococcal infections. 



REFERENCES 

Moellering, RC. Enterococcus species, streptococcus bovis, and leuconostoc species. 

In: Mandell GL, Bennett JE, Dolin R, eds. Principles and Practice of Infectious 

Diseases, 5th ed. Philadelphia, PA: Churchill Livingstone, 2000:2147-56. 
Murray PR, Rosenthal KS, Pfaller MA. Enterococcus and other gram-positive 

cocci. In: Murray PR, Rosenthal KS, Pfaller MA. Medical Microbiology, 5th ed. 

St. Louis, MO: Mosby, 2005:259-63. 



This page intentionally left blank 



♦♦♦ CASE 9 

A 21 -year-old woman presents with a 3-day duration of discomfort with urination 
and increased urinary frequency. She has noted that her urine has a strong odor as 
well. She denies fever, abdominal pain, back pain, vaginal discharge, or skin rash. 
She is sexually active and takes oral contraceptive pills. On examination, she is 
comfortable appearing and afebrile. She has no costovertebral angle tenderness. 
Abdominal exam is notable only for suprapubic tenderness. Microscopic exami- 
nation of the sediment of a centrifuged urine sample reveals 10-15 white blood 
cells per high power field and numerous bacteria. 

^ What type of organism would a Gram stain of the urine most likely 
show? 

^ What is the most common etiologic agent of this infection? 

^ What is the most likely reservoir of this infection? 

^ What is the most likely mechanism by which this organism infects 
the urinary tract? 



76 CASE FILES: MICROBIOLOGY 

ANSWERS TO CASE 9: Escherichia coli 

Summary: A 21 -year-old woman has urinary frequency and dysuria. The uri- 
nalysis shows numerous white blood cells. 

^ Organism most likely to be seen on Gram stain: Gram-negative rod 

^ Most common etiologic agent: Escherichia coli 

Most likely reservoir for the organism: Patient's own gastrointestinal 
(GI) tract 

^ Most likely mechanism of introduction of organism into the urinary 
tract: Urethral contamination by colonic bacteria followed by ascension 
of the infection into the bladder 

CLINICAL CORRELATION 

Escherichia coli is the most commonly found aerobic, gram-negative bacilli in 
the GI tract of humans. Escherichia coli is responsible for over 80 percent of 
all urinary tract infections (UTIs), along with other clinical diseases includ- 
ing gastroenteritis, sepsis, and neonatal meningitis. The E. coli that causes 
diarrhea is usually acquired from the environment, whereas most other 
infections caused by E. coli are acquired endogenously. Much of the diarrhea 
resulting from E. coli is acquired in developing countries particularly in trav- 
elers to these countries. The serotypes that are associated with travelers' 
diarrhea can be grouped based on their method of pathogenesis: enterotoxi- 
genic, enterohemorrhagic, enteroaggregative, and enteroinvasive strains. 
These strains produce toxins, which account for their invasiveness as well as 
decreased absorption in the GI tract. Most of these cause a self-limited diarrhea 
with the exception of enterohemorrhagic E. coli, frequently associated with 
E. coli serotype 0157:H7, causing a bloody diarrhea, which is usually 
acquired from eating poorly cook meat from an infected cow. Complications 
of infection with this organism include hemolytic uremic syndrome (HUS), 
which is a triad of hemolytic anemia, thrombocytopenia, and renal failure. 
HUS is a significant cause of acute renal failure in children. 

UTIs caused by E. coli are associated with organisms from the GI tract or 
vagina ascending up to the bladder. These organisms can colonize the vagina 
and be introduced into the bladder during instrumentation or sexual inter- 
course. Those serotypes that produce adhesions, which mediate adherence of 
the organisms to epithelial cells in the urinary tract are more likely to cause 
infections. The majority of cases of uncomplicated and complicated 
pyelonephritis are caused by E. coli, a complication of a UTI, where the organ- 
isms continue to ascend from the bladder to the kidney. 



CLINICAL CASES 77 

APPROACH TO SUSPECTED Escherichia coli UTI 
Objectives 

1 . Know the structure, characteristics, and virulence factors of E. coli. 

2. Know the pathogenic groups and toxins involved in diarrhea caused by 
E. coli. 

Definitions 

Pyelonephritis: Infection of the kidney. 
Cystitis: Infection of the bladder. 

Hemolytic uremic syndrome (HUS): A syndrome characterized by hemolytic 
anemia, thrombocytopenia (low platelets), and acute renal failure. 

DISCUSSION 

Characteristics of Escherichia coli 

Escherichia coli is a member of the family Enterobacteriaceae (see Table 9-1 
for an abbreviated Ust). All members of this family have in common the fact 
that they ferment glucose, are oxidase negative, and reduce nitrates to 
nitrites. Many members of the family Enterobacteriaceae, like E. coli, are 
normal flora of the human GI tract. 

Escherichia coli produces numerous adhesins, which allow the organism to 
attach to cells in the urinary and gastrointestinal tracts. This prevents the bac- 
teria from being flushed from these organs by the normal passage of urine or 
intestinal motility. Escherichia coli also can produce several exotoxins, 
involved in the pathogenesis of diarrhea, including shiga toxins, heat-stable 
toxins, heat-labile toxins, and hemolysins. Hemolysin HlyA is particularly 

Table 9-1 

ABBREVIATED LISTING OF ENTEROBACTERIACIAE 

Shigella 

Salmonella 

Escherichia 

Enterobacter 

Klebsiela 

Serratia 

Proteus 



78 



CASE FILES: MICROBIOLOGY 



Lipopolysaccharide 
O side chain (O) 




Capsule (K) 



Flagella (H) 



Cell envelope 

(cytoplasmic membrane, peptidoglycan, 

outer membrane) 



Figure 9-1. Structures used for antigenic identification in Enterobacteriaceae. 



important in producing an inflammatory response in the urinary tract, 

whereas most of the other exotoxins are more pathogenic in the GI tract. 

Escherichia coli are divided into serogroups based on the O antigen found 
on the lipopolysaccharide (LPS) of the cell membrane and the H antigen 
found on the flagella (Figure 9-1). 

Diagnosis 

The diagnosis of a UTI is made by urinalysis and urine culture. Complications 
such as pyelonephritis would be indicated by fever and flank pain. On urinalysis, 
the presence of white blood cells or leukocyte esterase and bacteria are sugges- 
tive of a true infection. Definitive diagnosis of the etiology is made by culture of 
the urine. Escherichia coli is easily grown on most culture media. A quantitative 
urine culture from a symptomatic patient should demonstrate greater than 10 s 
colony-forming units (CFUs) bacteria/mL urine to be considered significant. 
Escherichia coli would appear as pink colonies on MacConkey agar indicating 
fermentation of lactose. A rapid spot indole test would give a preliminary iden- 
tification of E. coli, which would be confirmed by biochemical analysis. 



Treatment and Prevention 

Treatment of UTIs is based on the affecting organism and its susceptibility to 
antibiotics. Common antimicrobials chosen include trimethoprim sul- 
famethoxazole, or a fluoroquinolone. Most E. coli are resistant to ampicillin 



CLINICAL CASES 79 

and penicillin. Recurrent UTIs are quite common, particularly in young 
women. Prevention can include consumption of large amounts of liquid and 
attention to totally emptying the bladder during urination. Fluid and electrolyte 
replacement should be administered to patients with E. coli diarrhea; however, 
antimicrobial treatment should not be administered. Escherichia coli diarrhea 
is best prevented by improved hygiene. 

COMPREHENSION QUESTIONS 

[9.1] Escherichia coli can be classified by their characteristic virulence 
properties and different mechanisms that cause disease. To which 
group does the verotoxin producing E. coli 0157:H7 serotype belong? 

A. Enteroaggregative E. coli (EAEC) 

B. Enterohemorrhagic E. coli (EHEC) 

C. Enteroinvasive E. coli (EIEC) 

D. Enteropathogenic E. coli (EPEC) 

E. Enterotoxigenic E. coli (ETEC) 

[9.2] Several children are hospitalized with bloody diarrhea and severe 
hematological abnormalities. A 4-year-old girl dies of kidney failure 
shortly after admittance. An epidemiological investigation establishes 
that all of the patients developed symptoms following consumption of 
hamburgers from the same fast-food restaurant chain. Which of the fol- 
lowing organisms is most likely to be responsible for the outbreak? 

A. Campylobacter jejuni 

B. non-Ol serogroup of Vibrio cholerae 

C. 0157:H7 serotype of Escherichia coli 

D. Shigella dysenteriae 

E. Salmonella typhimurium 

[9.3] A Gram stain of an isolate from voided urine in a patient with a UTI 
reveals the presence of pink rods. Further biochemical analysis reveals 
that these bacteria ferment glucose, reduce nitrates to nitrites and are 
unable to synthesize the following reaction: 

2H+ + 2e- + V 2 2 -> H 2 

Which of the following characterize the above test results? 

A. Escherichia coli 

B. Neisseria gonorrhoeae 

C. Proteus vulgaris 

D. Pseudomonas aeruginosa 

E. Staphylococcus aureus 

F. Streptococcus pyogenes 



80 CASE FILES: MICROBIOLOGY 

[9.4] A 7-year-old child with bloody diarrhea is admitted after laboratory 
results indicating anemia and abnormal kidney function. After testing, 
it is determined that the etiologic agent is an E. coli that is most likely 
to produce which of the following? 

A. Endotoxin 

B. Exotoxin 

C. LT toxin 

D. ST toxin 

E. Vero toxin 

[9.5] Several days after an appendectomy a patient develops a high fever, 
dangerously low blood pressure, and disseminated intravascular coag- 
ulation (DIC). Based on these and other findings, a diagnosis of 
septicemia as a result of an enteric gram-negative rod is made. Which 
of the following cytokines is most likely to be responsible for the fever, 
low blood pressure, and DIC? 

A. IFN-y 

B. IL-2 

C. IL-10 

D. TGF-p 

E. TNF-ot 

Answers 

[9.1] B. All of the above classes of E. coli cause diarrhea. However, only 
EHEC produce a verotoxin that has many properties that are similar 
to Shiga toxin. EHEC has been associated with hemorrhagic colitis, 
a severe form of diarrhea, and with HUS. HUS is a disease resulting 
in acute renal failure, microangiopathic hemolytic anemia, and 
thrombocytopenia. 

[9.2] C. Escherichia coli 157:H7 strains are classically associated with out- 
breaks of diarrhea after ingestion of undercooked hamburger at fast- 
food restaurants. Many cases of hemorrhagic colitis and its associated 
complications can be prevented by thoroughly cooking ground beef. 
The other organisms listed can cause GI disturbances; however, E. coli 
is the classic disturbing pathogen in this case. Shigella dysenteriae pro- 
duces a heat-labile enterotoxin that affects the gut and central nervous 
system and is a human disease that is transmitted via a fecal -oral route. 
Salmonella and Campylobacter are associated with poultry and eggs 
primarily, whereas Vibrio is associated mainly with seafood. 

[9.3] A. The biochemical reaction described above is catalyzed by the 
enzyme oxidase. Thus, E. coli is the only bacteria listed that is a 
gram-negative rod (pink), ferments glucose, converts nitrates to 
nitrites, and is oxidase negative. 



CLINICAL CASES 81 

[9.4] E. The verotoxin produced by E. coli is similar to Shiga toxin, caus- 
ing bloody diarrhea. Please refer to the discussion for Question 9.1. 

[9.5] E. The acute phase response involves the increase in the levels of 
various plasma proteins (C-reactive protein and mannose-binding 
proteins) and is part of innate immunity. These proteins are synthe- 
sized and secreted by the liver in response to certain cytokines such 
as IL-1, IL-6, and TNF-oc (produced after exposure to microorgan- 
isms) as nonspecific responses to microorganisms and other forms of 
tissue injury. Specifically, endotoxin (LPS) from gram-negative 
bacteria has the ability to activate macrophages that, in turn, synthe- 
size TNF-oc. TNF-oc then functions to cause fever and hemorrhagic 
tissue necrosis (inflammatory reaction/immune response). 



MICROBIOLOGY PEARLS 

*♦* Escherichia coli is the most common cause of UTIs in otherwise 

healthy patients. 
*♦* Escherichia coli can easily be identified following growth of a flat 

lactose fermenter on MacConkey agar that is indole positive. 
*♦* Many serotypes of E. coli are associated with traveler's diarrhea. 



REFERENCES 

Murray PR, Rosenthal KS, Pfaller MA. Enterobacteriaceae. In: Murray PR, 
Rosenthal KS, Pfaller MA. Medical Microbiology, 5th ed. St. Louis, MO: 
Mosby, 2005:323-38. 



This page intentionally left blank 



►♦♦ CASE 10 



♦:♦ 



A 48-year-old man presents for the evaluation of a 2-month history of upper 
abdominal pain associated with nausea. It is made worse when he drinks 
coffee, soda, or alcohol. He has taken multiple over-the-counter antacid med- 
ications that provide temporary relief. He admits to a 20-pack-year smoking 
history and drinking one or two alcoholic beverages a week but denies signif- 
icant use of nonsteroidal anti-inflammatory drugs (NSAIDs). His general 
physical examination and vital signs are normal. His abdominal examination 
is notable for epigastric tenderness without the presence of masses, rebound 
tenderness, or guarding. A rectal examination reveals his stool to be heme 
positive. A complete blood count (CBC) shows a mild hypochromic, micro- 
cytic anemia. He is referred to a gastroenterologist for an upper GI endoscopy, 
which shows diffuse gastritis and a gastric ulcer. 

^ What organism is most likely to be visualized on histologic 
evaluation of a gastric biopsy specimen? 

^ Besides microscopic evaluation, what other clinical test may provide 
a rapid detection of this organism? 

^ What two factors facilitate this organism's ability to colonize the 
stomach? 



CASE FILES: MICROBIOLOGY 



ANSWERS TO CASE 10: Helicobacter pylori 

Summary: A 48-year-old man has diffuse gastritis and a gastric ulcer on endo- 
scopic examination. 

^ Organism on histologic evaluation of a gastric biopsy: The organism 
likely to be visualized on biopsy specimen is Helicobacter pylori. 

^ Other clink ;il test for rapid detection of this organism: The urease 
test. 

^ Factors facilitating this organism's ability to colonize the stomach: 

Blockage of acid production by a bacterial acid-inhibitory protein and 
neutralization of acid by ammonia produced by urease activity. 

CLINICAL CORRELATION 

Helicobacter pylori has been implicated in the development of multiple GI 
diseases, including gastritis, ulcers, and gastric cancers. Humans are the 
primary reservoir of the infection and human-to-human transfer, via fecal-oral 
contact, is likely to be an important mode of transfer. Helicobacter pylori is a 
curved gram-negative bacillus with motility facilitated by corkscrew motion 
and polar flagella. Culture of this organism requires a complex medium and 
microaerophilic environment. Helicobacter pylori that colonize the stomach 
produce urease, an enzyme that has many effects. Urease activity produces 
ammonia, which neutralizes gastric acid. This, along with a specific acid- 
inhibitory protein that directly blocks gastric acid production, facilitates the 
colonization of the acidic stomach environment. Urease by-products also 
cause local tissue damage and stimulate an inflammatory response. Urease 
activity is enhanced by the presence of a heat shock protein, HspB, which 
exists on the surface of H. pylori. The identification of urease activity in a 
gastric biopsy sample is highly specific for the presence of an active H. pylori 
infection, making it the basis for a widely used clinical test for the rapid detec- 
tion of H. pylori infections. 

APPROACH TO SUSPECTED Helicobacter pylori 
INFECTION 

Objectives 

1. Be able to describe the characteristics of Helicobacter bacteria. 

2. Understand the role of H. pylori in causing gastric ulcers. 



CLINICAL CASES 85 

Definitions 

Urease: Helicobacter pylori uses this enzyme to convert urea into ammo- 
nia and carbon dioxide. This chemical reaction is the basis of the rapid 
urea breath test for diagnosis of H. pylori. The increased ammonia 
produced by this reaction neutralizes gastric acid, which allows the 
organism to survive the normally harsh gastric environment and dam- 
ages the gastric mucosa. 

Type B gastritis: Type B gastritis is gastritis of the antrum caused by 
H. pylori infection (compare with Type A of the fundus, caused by 
autoimmune disorders). 

Corkscrew motility: Helicobacter pylori is highly motile because of 5-6 
polar flagella. 

Microaerophilic organisms: Organisms that require reduced oxygen 
concentration (5%) to grow optimally (include: H. pylori and 
Campylobacter jejun i) . 

Upper endoscopy: Visual examination of the mucosa of the esophagus, 
stomach, and duodenum using a flexible fiberoptic system introduced 
through the mouth. 



DISCUSSION 

Characteristics of Helicobacter pylori Impacting 
Transmission 

Helicobacter pylori is a curved gram-negative bacilli that requires 
microaerophilic environments to grow. Discovered in 1983, the organism was 
originally classified under the Campylobacter genus, but eventually was reclas- 
sified under a new and separate genus, Helicobacter, as understanding of the 
organism has evolved. Urease production is the most important enzyme that 
distinguishes H. pylori from Campylobacter species and other various 
Helicobacter species, and allows the organism to survive the harsh gastric 
environment. Helicobacter pylori also has oxidase, catalase, mucinase, 
phospholipase enzymes, and vacuolating cytotoxin, which aid in the viru- 
lence and pathogenesis of the organism. Infections of H. pylori are ubiquitous, 
worldwide, and extremely common in developing nations and among lower 
socioeconomic groups. Humans are the primary reservoir, and no animal 
reservoir has been identified at the present time. The primary mode of trans- 
mission is person to person (usually by the fecal-oral route), and the infection 
commonly is clustered in families or among close contacts. Some speculation 
has been made that contaminated water or food sources may be a reservoir, but 
at the present time there are no data to support this. 



86 CASE FILES: MICROBIOLOGY 

Diagnosis 

Helicobacter pylori has been clearly associated with Type B gastritis, gastric 
ulcers, gastric adenocarcinoma of the body and antrum, and gastric mucosa- 
associated lymphoid tissue (MALT) B-cell lymphomas. Diagnosis of H. pylori 
should be considered in patients with symptoms of these diseases. The most 
rapid test to detect H. pylori is the urease test or urea breath test that detects 
by-products of the urease reaction cleaving urea into ammonia and carbon 
dioxide; however, the invasiveness of biopsy or the expense of breath detec- 
tion instruments limits the use of these assays. Microscopy is both extremely 
sensitive and specific for diagnosis of H. pylori in gastric biopsy specimens 
when stained with Warthin-Starry silver stain, hematoxylin-eosin, or Gram 
stain. Antigen detection in stool samples via a commercial polyclonal enzyme 
immunoassay is highly sensitive and specific, while also inexpensive and easy 
to perform. Serology is another preferred diagnostic test, whereby H. pylori 
stimulates a humoral immune response (IgM early in infection; IgG and IgA 
later in infection and persisting), but it cannot distinguish between past and 
present infections. Culture is a more challenging and time-consuming way to 
diagnose H. pylori, because it must be grown in a microaerophilic atmosphere 
on an enriched medium containing charcoal, blood, and hemin. 

Treatment and Prevention 

Because H. pylori is primarily transmitted person to person via fecal-oral 
route, the best prevention is improving hygiene by frequent hand washing, 

especially before meals. In symptomatic patients who are positive for infection 
with H. pylori, combination therapy is needed. This therapy includes (1) acid 
suppression, usually with a proton pump inhibitor, (2) one or more antibiotics, 
and (3) occasional additive therapy with bismuth. 



COMPREHENSION QUESTIONS 

[10.1] A 45-year-old man presents to the hospital vomiting blood. He is diag- 
nosed with a perforated peptic ulcer. The causative agent discovered by 
gastric biopsy is a spiral gram-negative bacillus. What other long-term 
complications could this organism cause if not treated? 

A. Skin ulcers 

B. Esophageal varices 

C. Gastric MALT lymphomas 

D. Colon cancer 



CLINICAL CASES 87 

[10.2] Which of the following is an important distinguishing characteristic of 
H. pylori as compared to Campylobacter species? 

A. Oxidase production 

B. Catalase production 

C. Urease production 

D. Curved shape 

E. Polar flagellum 

[10.3] A 58-year-old man presents to the clinic with decreased appetite, nau- 
sea, vomiting, and upper abdominal pain. If the causative agent is a 
curved gram-negative rod with urease production, what treatment 
should be given to this patient? 

A. Proton pump inhibitor and antibiotic 

B. Proton pump inhibitor, antibiotic, and bismuth 

C. Over-the-counter antacids and antibiotics 

D. Nonsteroidal anti-inflammatory drugs (NSAIDs) 

Answers 

[10.1] C. Helicobacter pylori is the causative agent that can cause Type B 
gastritis, peptic ulcers, gastric adenocarcinoma, and gastric MALT B 
cell lymphomas. 

[10.2] C. Both Campylobacter species and H. pylori have a curved shape, 
are oxidase and catalase positive, with polar flagellum. Urease pro- 
duction is the distinguishing factor of H. pylori, and it is the basis of 
the rapid urease breath test that diagnoses H. pylori infection. 

[10.3] B. The combination therapy of proton pump inhibitor, antibiotic, and 
bismuth is required to eradicate an infection with H. pylori. 



MICROBIOLOGY PEARLS 

*♦* Helicobacter pylori are characterized as being curved gram-nega- 
tive bacilli and microaerophilic, and by having multiple polar fla- 
gella and urease activity. 

*♦* Clinical manifestations include Type B gastritis, gastric ulcers, gas- 
tric adenocarcinoma, gastric MALT lymphomas conjunctivitis, or 
gastroenteritis. 

*♦* It is transmitted via the fecal-oral route. 

*♦* Antimicrobial treatment includes acid suppression, antibiotics, and 
adjunctive treatments (i.e., bismuth). 



CASE FILES: MICROBIOLOGY 



REFERENCES 

Johnson AG, Hawley LB, Lukasewycz OA, Jiegler RJ. Microbiology and Immuno- 

biology, 4th ed. Baltimore, MD: Lippincott Williams & Wilkins, 2002. 
Levinson W, Jawetz E. Medical Microbiology and Immunology, 7th ed. New York: 

McGraw-Hill, 2002. 
Murray PR, Rosenthal KS, Pfaller MA. Medical Microbiology, 5th ed. St. Louis, 

MO: Mosby, 2005:291-295. 
Passaro D, Chosy EJ, Parssonet J. Helicobacter pylori: Consensus and controversy. 

Clin Infect Dis 2002;35:298-304. 
Weingart V, Riissmann H, Koletzko S, et al. Sensitivity of a novel stool antigen test 

for detection of Helicobacter pylori in adult outpatients before and after eradia- 

cation therapy. J Clin Microbiol 2004. 42:1319-21. 
Youngerman-Cole S. Digestive disorders health centre: Helicobacter pylori tests. 

http://www.webmd.com/digestive-disorders/Helicobacter-pylori-Tests. Last 

updated January 12, 2006. 



♦♦♦ CASE 11 

A 19-month-old child is brought to the emergency room following a seizure. 
His mother says that he had a cold for 2 or 3 days with a cough, congestion, 
and low-grade fever, but today he became much worse. He has been fussy and 
inconsolable, he would not eat and has slept most of the morning. He then had 
two grand-mal seizures. He has no history of seizures in the past. His mother 
reports that he has not received all of his immunizations. She is not sure which 
ones he's had, but he's only had two or three shots in his life. On examination 
his temperature is 38.1°C (100. 5°F), his pulse is 1 10 beats per minute, and he 
appears very ill. He does not respond to your voice but does withdraw his 
extremities from painful stimuli. He grimaces when you try to bend his neck. 
His skin is without rash and his HEENT (head, eyes, ear, nose, throat), car- 
diovascular, lung, and abdominal examinations are normal. His white blood 
cell count is elevated, and a CT scan of his head is normal. You perform a lum- 
bar puncture, which reveals numerous small gram-negative coccobacilli. 

^ What organism is the most likely etiology of this illness? 

^ What component of this organism is the target of vaccine-induced 
immunity? 



90 CASE FILES: MICROBIOLOGY 

ANSWERS TO CASE 11: Haemophilus influenzae 

Summary: A 19-month-old boy who has not received many immunizations 
presents with meningitis. The lumbar puncture shows multiple gram-negative 
coccobacilli. 

^ Organism most likely causing this infection: Haemophilus influenzae 

type B. 

^ Component of this organism that is the target of vaccine-induced 
immunity: Purified polyribitol phosphate, a component of the 
H. influenzae type B polysaccharide capsule. 

CLINICAL CORRELATION 

Haemophilus species, particularly H. parainfluenzae and H. influenzae non-type 
B are normal flora in the human upper respiratory tract. These strains can, how- 
ever, be associated with respiratory infections such as otitis media and bronchi- 
tis. Haemophilus influenzae type B was the most common cause of pediatric 
meningitis (ages 2 months to 2 years of age) until the introduction of routine 
childhood immunization against this bacterium. 

Transmission of H. influenzae occurs by close contact with respiratory tract 
secretions from a patient colonized or infected with the organism. Prior viral 
infection promotes colonization of the respiratory tract with H influenzae. 
Invasive infections such as meningitis occur when the colonizing organisms 
invade the bloodstream and subsequently the meninges. Usually as a result of H. 
influenzae type B, the capsule aids in adherence of the organism and evasion of 
phagocytosis. Neurological sequelae can occur in up to 20 percent of cases of 
meningitis. Haemophilus influenzae type B can also be a cause of epiglottitis in 
young children, which can result in respiratory obstruction requiring intubation. 

Haemophilus aphrophilus and H. paraphrophilus are causes of culture neg- 
ative endocarditis named thus because of the fastidious nature and difficulty in 
recovering these organisms from the blood of infected patients. Haemophilus 
ducreyi is a cause of an uncommon sexually transmitted disease chancroid. 
Chancroid is characterized by genital skin lesions and lymphadenopathy, lead- 
ing to abscess formation if remains untreated. 

APPROACH TO SUSPECTED Haemophilus influenza 
MENINGITIS 

Objectives 

1 . Know the structure and physiology of Haemophilus. 

2. Know the significance of the capsule of Haemophilus in the virulence, 
infection, and development of protective immunity. 



CLINICAL CASES 91 

Definitions 

Epiglottitis: Inflammation of the epiglottis usually caused by H. influenzae, 
which presents as sore throat, fever, and difficulty breathing. 

Meningitis: Inflammation of the meninges leads to headache, stiff neck, 
and fever with increase in cells in the cerebrospinal fluid. 

Grand-mal seizure: Seizure that results in loss of consciousness and 
generalized muscle contractions. 

DISCUSSION 

Characteristics of Haemophilus Species 

Haemophilus are small, pleomorphic, gram-negative bacilli or coccobacilli. 

Humans are the only known reservoir. They are facultative anaerobes and 
grow on media that contain growth-stimulating factors known as X factor 
(hematin) and V factor (NAD). Heated sheep blood agar, chocolate agar, 
contains both of these factors and is used to grow Haemophilus. Many strains 
of Haemophilus have a polysaccharide capsule, and specific capsular anti- 
gens are used to identify strains of H. influenzae. Six types, A through F, have 
been identified. The polysaccharide capsule of H. influenzae type B repre- 
sents its major virulence antiphagocytic factor. The capsule contains ribose, 
ribitol, and phosphate, known collectively as polyribitol phosphate (PRP). 
Phagocytosis and complement-mediated activity are stimulated in the pres- 
ence of antibodies directed at the H. influenzae type B capsule. This represents 
the basis for the H. influenzae type B vaccine, which contains purified PRP 
antigens conjugated to specific protein carriers. 

Diagnosis 

Acute meningitis typically involves the rapid onset (over several days) of 
headache, fever, and stiff neck, although in young children only fever and 
irritability may be evident. Rash may also be present in some forms of menin- 
gitis. Without treatment, progression of the disease includes loss of con- 
sciousness and/or seizures and coma. Specific diagnosis is based on culture of 
the etiologic organism from the cerebrospinal fluid (CSF). Prior to culture a 
rapid presumptive diagnosis of bacterial meningitis is based on increased num- 
ber of polymorphonuclear leukocytes (PMNs) in the CSF as well as an ele- 
vated protein and a decreased glucose. Gram stain of the CSF may reveal the 
presence of bacteria if the number of organisms is high enough. In the case of 
H. influenzae meningitis, the presence of tiny gram-negative coccobacilli is 
seen in a Gram-stained smear of the CSF. 

Haemophilus influenzae require both X and V factors for growth; there- 
fore no growth would be seen on blood agar unless growth of Staphylococcus 
aureus on the agar allowed for lysis of the blood and release of the required 



92 CASE FILES: MICROBIOLOGY 

factors into the media. Good growth would be evident on chocolate agar as gray- 
ish colonies after 24 hours incubation at 35°C (95°F) and 5 percent carbon diox- 
ide. Identification of Haemophilus to the species level can be made by 
requirement for X or V for growth. More specifically, a commercially available 
identification system could be used that is based on the presence of preformed 
enzymes and can be made within 4 hours. Haemophilus species other than 
H. influenzae grow much more slowly, particularly H. ducreyi, which may require 
5-7 days of incubation after culture of an infected lymph node or genital abscess. 

Treatment and Prevention 

Up to 50 percent of strains of H. influenzae produce a (3-lactamase, rendering 
them resistant to ampicillin. Treatment for H. influenzae meningitis involves 
the use of a third-generation cephalosporin (cefotaxime, ceftriaxone). 
Respiratory infections caused by H. influenzae may be treated with antibiotics 
such as amoxicillin-clavulanate or a macrolide (such as azithromycin). 
Routine pediatric immunization with the vaccine against H. influenzae type B 
has reduced the incidence of invasive disease by approximately 90 percent and 
has also reduced respiratory colonization. Haemophilus ducreyi is usually 
treated with erythromycin or a newer macrolide antibiotic. An alternative for 
therapy of chancroid includes a fluoroquinolone. 

COMPREHENSION QUESTIONS 

[11.1] A 2-year-old child has high fever, is irritable, and has a stiff neck. 
Gram-stain smear of spinal fluid reveals gram-negative, small pleo- 
morphic coccobacillary organisms. Which of the following is the most 
appropriate procedure to follow to reach an etiologic diagnosis? 

A. Culture the spinal fluid in chocolate blood agar and identify the 
organism by growth factors. 

B. Culture the spinal fluid in mannitol salt agar. 

C. Perform a catalase test of the isolated organism. 

D. Perform a coagulase test with the isolate. 

E. Perform a latex agglutination test to detect the specific antibody in 
the spinal fluid. 

[11.2] Haemophilus influenzae synthesizes immunoglobulin A (IgA) pro- 
tease, which enables the bacterium to penetrate and invade the host's 
respiratory epithelium. This is an example of a bacterium's ability to 
evade which of the host's immune system responses? 

A. Cellular or cell-mediated immunity (CMI) against H. influenzae 

B. Nonspecific humoral immunity 

C. Nonspecific innate immunity 

D. Phagocytic function and intracellular killing of bacteria 

E. Specific humoral immunity against H. influenzae 



CLINICAL CASES 93 

[11.3] An 18-month-old baby girl is suspected to have H. influenzae menin- 
gitis. She has not been immunized with the HIB vaccine. A rapid latex 
agglutination test is performed with the spinal fluid to make a defini- 
tive diagnosis. What chemical component in the spinal fluid are we 
detecting with this assay? 

A. IgG antibody 

B. IgM antibody 

C. Lipopolysaccharide (LPS) 

D. Polypeptide 

E. Polysaccharide capsule 

[1 1.4] The H. influenzae vaccine contains which of the following? 

A. Lipopolysaccharide (LPS) 

B . Live attenuated H. influenzae 

C. Polypeptide antigens containing D-glutamate 

D. Polyribitol phosphate antigens 

E. Teichoic acid 

F. Toxoids 

[11.5] Cerebrospinal fluid from a spinal tap of a patient complaining of a 
severe headache, fever, and nuchal rigidity revealed the presence of 
gram-negative coccobacilli. Further testing revealed growth of the 
organism on growth factor X and V supplemented chocolate agar, and 
no hemolysis when grown on blood agar. Which of the following 
organisms represents the above description? 

A. Bordetella pertussis 

B. Haemophilus ducreyi 

C. Haemophilus haemolyticus 

D. Haemophilus influenzae 

E. Haemophilus parainfluenzae 

Answers 

[11.1] A. The organism in the above description is H. influenzae. This organ- 
ism is differentiated from other related gram-negative bacilli by its 
requirements of a chocolate media supplemented with growth factors, 
such as X and V factors, and by its lack of hemolysis on blood agar. 

[11.2] E. Immunoglobulin A (IgA) is associated with immunological 
protection of the host at the epithelial boundary. An IgA protease has 
the ability to breakdown IgA and thereby act as a virulence mecha- 
nism enabling the bacterium to invade the host through an unpro- 
tected epithelial boundary. Because IgA is an antibody associated 
with the humoral (specific) arm of the immune system, IgA protease 
allows the bacterium the ability to evade the specific humoral immu- 
nity of the host. 



94 CASE FILES: MICROBIOLOGY 

[11.3] E. The latex agglutination test involves the use of latex beads coated 
with specific antibody that become agglutinated in the presence of 
homologous bacteria or antigen. This test is used to determine the 
presence of the capsular polysaccharide antigen of H. influenzae in 
serum or spinal fluid. 

[11.4] D. Encapsulated H. influenzae contains capsular polysaccharides of 
one of six types (A-F). Haemophilus influenzae type B is an important 
human pathogen with its polyribose phosphate capsule being its major 
virulence factor. As a result, active immunity is built using polyribitol 
phosphate antigens (capsular polysaccharide) of H. influenzae. 

[11.5] D. Haemophilus haemolyticus and H. influenzae are the only organ- 
isms listed above that require both growth factors X and V for 
growth; however, they can be distinguished from each other in that H. 
influenzae is hemolysis negative on blood agar, whereas H. 
haemolyticus is hemolysis positive. 



MICROBIOLOGY PEARLS 

*♦* Haemophilus species other than H influenzae type B are still a sig- 
nificant cause of systemic infections. 

*♦* The widespread use of H influenzae vaccines in developed countries 
has decreased the incidence of H. influenzae meningitis. 

*♦* Haemophilus influenzae meningitis is treated with cefotaxime or 



ceftriaxone. 

lemophilus S] 
infections), sinusitis, and epiglottitis in young children. 



*♦* Haemophilus species are a significant cause of otitis (middle ear 



REFERENCES 

Murray PR, Rosenthal KS, Pfaller MA. Pasteurellaceae. In: Murray PR, Rosenthal 
KS, Pfaller MA. Medical Microbiology, 5th ed. St. Louis, MO: Mosby, 
2005:367-76. 

Slack MPE. Gram-Negative Coccobacilli in Infectious Diseases. Philadelphia, PA: 
Mosby, 1999;8(20):1-18. 



♦♦♦ CASE 12 

A 45 -year-old homeless alcoholic man presents to the emergency room with 
fever and cough of 4-day duration. The cough is productive of thick, bloody 
phlegm. He complains of pain in the right side of his chest with coughing or 
taking a deep breath. He denies any other medical history and says he can't 
remember the last time he saw a doctor. He doesn't smoke cigarettes, drinks a 
pint of whiskey whenever he can get it, and denies drug use. On examination, 
he is dirty, disheveled, and appears malnourished. His temperature is 38.9°C 
(102°F), pulse 105 beats per minute, and respiratory rate is 30 breaths per 
minute. The lung examination is notable for decreased breath sounds and 
crackles in the right lower, posterior field. His white blood cell count is ele- 
vated. A chest x-ray reveals a dense right lower lobe infiltrate with evidence of 
a pulmonary abscess. Sputum samples are collected for Gram stain and cul- 
ture, and a blood culture is sent. A bacterial etiology is suspected. 

^ What is the most likely organism to be isolated in the sputum and 
blood cultures? 



^ By what mechanism does this organism commonly develop 
antibiotic resistance? 



96 CASE FILES: MICROBIOLOGY 

ANSWERS TO CASE 12: Klebsiella pneumoniae 

Summary: A 45 -year-old alcoholic man has developed a bacterial cavitary 
pneumonia with evidence of a pulmonary abscess. 

-^ Most likely organism to be isolated: Klebsiella pneumoniae 

^ Mechanism whereby Klebsiella commonly develops antibiotic 
resistance: Plasmid acquisition 

CLINICAL CORRELATION 

Klebsiella causes lower respiratory infections, wound soft tissue infections 
and is a common cause of hospital -acquired urinary tract infections (UTIs). 
Klebsiella pneumoniae is also associated with lobar pneumonia in a person 
with an underlying debilitating condition such as alcoholism or diabetes. 
Pneumonia caused by this organism tends to be necrotic, inflammatory, and 
hemorrhagic and has a high propensity for cavitation or abscess formation. 
Patients often produce thick, bloody sputum. Because of the destructive nature 
of the infection and the underlying debility of the patient, pneumonia caused 
by K. pneumoniae carries a high mortality. 

Community-acquired pneumonia is usually the result of spread of organisms 
that normally reside in the upper respiratory tract into the lower respiratory tract. 
Organisms that have virulence factors that allow them to survive the host response 
in the lung can establish an infection. Patients with disruption of their immune 
defenses are at greater risk of infection. Klebsiella pneumoniae pneumonia is 
therefore considered an opportunistic infection because it is not a common cause 
of pneumonia in normally healthy patients. Symptoms of bacterial pneumonia are 
usually nonspecific and include malaise, anorexia, headache, myalgia, arthralgia, 
and fever. Klebsiella pneumoniae produces a severe acute, necrotic, and hemor- 
rhagic pneumonia, which is evidenced by cavitary lung lesions on chest x-ray, 
pleural effusions, and possible abscess formation or empyema. Because of the 
hemorrhagic nature of the pneumonia, patients tend to have blood-tinged sputum. 

Two uncommon species of Klebsiella are also causes of respiratory disease. 
Klebsiella rhinoscleroma is associated with chronic granulomatous disease of 
the upper respiratory mucosa (predominantly outside the United States), and 
K. ozaenae is associated with chronic atrophic rhinitis. 

APPROACH TO SUSPECTED Klebsiella pneumoniae 
PNEUMONIA 

Objectives 

1 . Know the structure, physiology, and virulence factors of K. pneumoniae. 

2. Know the nature of the native and acquired antibiotic resistance of 
K pneumoniae. 



CLINICAL CASES 97 

Definitions 

Chronic obstructive pulmonary disease (COPD): A progressive lung 
disease that commonly results from heavy smoking and is evident by 
difficulty breathing, wheezing, and a chronic cough. 

Empyema: Accumulation of pus in the pleural space around the lung. 



DISCUSSION 

Characteristics of Klebsiella Species 

The genus Klebsiella, which belongs to the family Enterobacteriaceae includes 
five species, with the most clinically significant being K. pneumoniae. Klebsiella 
pneumoniae is a large, nonmotile, gram-negative rod with a prominent poly- 
saccharide capsule. The capsule is antiphagocytic and retards leukocyte 
migration into an infected area. 

Another virulence factor of K. pneumoniae is its propensity to develop 
resistance to multiple antibiotics. All strains of K pneumoniae are innately 
resistant to ampicillin, because of the production of [3-lactamase. 
Acquisition of resistance to other antibiotics usually occurs by transfer of 
plasmids from other organisms. Recently strains of nosocomially acquired K. 
pneumoniae have been isolated that produce an extended spectrum (3- 
lactamase and therefore are resistant to all (3-lactam antibiotics. 

Diagnosis 

Diagnosis of community-acquired pneumonia is made clinically based on 
symptoms of cough, especially with blood, and chest x-ray indicating infil- 
trates, cavitary lesions, or pleural effusions. Specific diagnosis of pneumonia 
is made by culture of expectorated sputum. Sputum samples must be of good 
quality (many white blood cells and rare squamous epithelial cells) represent 
the flora of the lower respiratory tract and not mouth flora. In a small percent- 
age of cases of community-acquired pneumonia blood cultures will also be 
positive for the affecting organism. 

Klebsiella pneumoniae will grow rapidly producing large mucoid 
colonies on routine laboratory media. Colonies are often extremely mucoid 
and will tend to drip into the lid of the plate while incubating in an inverted 
position. Pink colonies will be evident on MacConkey agar indicating their 
fermentation of lactose. Confirmatory identification is made for other 
members of the family Enterobacteriaceae by commercially available identi- 
fication systems using a combination of sugar fermentation and enzyme 
production. Both K. oxytoca and especially K. rhinoscleromatis are slower 
growing than K. pneumoniae and the other Enterobacteriaceae. All Klebsiella 



98 CASE FILES: MICROBIOLOGY 

species are very closely related with nearly identical biochemical reactions, 
except for the fact that K. pneumoniae is indole negative, and K. oxytoca is 
indole positive. Commercial identification systems have a difficult time dif- 
ferentiating these species. 

Treatment and Prevention 

Treatment of K. pneumoniae pneumonia would be based on the susceptibility of 
the isolate. Treatment can be complicated by the presence of multidrug-resistant 
strains. Most strains are susceptible to extended spectrum cephalosporins such 
as cefepime as well as fluoroquinolones such as moxifloxacin. In cases of 
strains that produce an extended spectrum (3-lactamase, the treatment of choice 
would be imipenem or meropenem. 

Prevention of community-acquired pneumonia would involve avoidance of 
high-risk activities such as smoking or drinking in excess. Prevention of 
spread in the hospital would involve appropriate infection control procedures 
to isolate patients with multidrug-resistant organisms. Klebsiella oxytoca has 
similar susceptibility patterns as K. pneumoniae and can also produce 
extended spectrum (3-lactamases. 



COMPREHENSION QUESTIONS 

[12.1] The most common mechanism by which K. pneumoniae attains its 
antibiotic resistance is by plasmid acquisition. Which of the following 
best describes the direct transfer of a plasmid between two bacteria? 

A. Competence 

B. Conjugation 

C. Recombination 

D. Transduction 

E. Transformation 

[12.2] A specimen of thick, bloody sputum from a hospitalized 80-year-old 
patient with diabetes mellitus and difficulty in breathing is sent for lab- 
oratory analyses. The tests yield heavy growth of a lactose-positive, 
nonmotile, gram-negative rod with a large capsule. Which of the fol- 
lowing bacteria is most likely to be the cause of the pulmonary 
problems? 

A. Enterobacter aerogenes 

B. Escherichia coli 

C. Klebsiella pneumoniae 

D. Pseudomonas aeruginosa 

E. Yersinia pseudotuberculosis 



CLINICAL CASES 99 

[12.3] A 65-year-old diabetic man presents to the emergency room with a 
severe productive cough producing thick bloody sputum resembling a 
"currant-jelly" like appearance. Culture using MacConkey agar reveals 
pink colonies, with large mucoid colonies on routine laboratory media. 
Which of the following organisms is most likely responsible for this 
patient's pneumonia? 

A. Enterobacter cloacae 

B. Escherichia coli 

C. Klebsiella pneumoniae 

D. Pseudomonas aeruginosa 

E. Serratia marcescens 

[12.4] The O antigens that are used to help characterize members of the 
Enterobacteriaceae family are found on which of the following? 

A. Capsules 

B. Endotoxins 

C. Exotoxins 

D. Fimbriae 

E. Flagella 

Answers 

[12.1] B. The three important processes by which DNA is transferred 
between bacteria are via transformation, transduction, and conjuga- 
tion. Transformation is defined as the uptake of soluble DNA by a 
recipient cell. Transduction refers to the transfer of DNA by a virus 
from one cell to another. Conjugation refers to the direct transfer of 
soluble DNA (plasmids) between cells. Examples of such plasmids 
are the sex factors and the resistance (R) factors. 

[12.2] C. Whereas all of the above listed organisms are gram-negative rods, 
only K. pneumoniae fulfill all of the laboratory criteria listed in the 
question, such as the presence of a very large capsule, which gives a 
striking mucoid appearance to its colonies. 

[12.3] C. Patients with K. pneumoniae infections usually have predisposing 
conditions such as alcoholism, advanced age, chronic respiratory dis- 
ease, and diabetes. The "currant-jelly" sputum distinguishes K. pneu- 
moniae from the other organisms. Infections can lead to necrosis and 
abscess formation. Please refer to the discussion for Question 12.2. 

[12.4] B. There are three surface antigens associated with several members 
of the Enterobacteriaceae. The cell wall antigen (somatic or O anti- 
gen) is the outer polysaccharide portion of the lipopolysaccharide 
(LPS/endotoxin). The H antigen is on the flagellar proteins 
(Escherichia and Salmonella). The capsular or K polysaccharide 
antigen is particularly prominent in heavily encapsulated organisms 
such as Klebsiella. 



700 CASE FILES: MICROBIOLOGY 



MICROBIOLOGY PEARLS 



*♦* Klebsiella pneumoniae are a common cause of hospital-acquired 

UTIs. 
*♦* Patients with diabetes or COPD or who drink alcohol excessively 

are predisposed to pneumonia with K. pneumoniae. 
*♦* Klebsiella pneumoniae produces a large mucoid colonies on agar 

plates as a result of the presence of a polysaccharide capsule that 

also acts to allow the organism to avoid phagocytosis. 



REFERENCES 

Baldwin DR, MacFarlane JT. Community Acquired Pneumonia in Infectious 

Diseases. Philadelphia, PA: Mosby, 1999;2(27):1-10. 
Eisenstein BI, Zaleznik DF. Enterobacteriaceae. In: Mandell GL, Bennett JE, Dolin 

R, eds. Principles and Practice of Infectious Diseases, 5th ed. Philadelphia, PA: 

Churchill Livingstone, 2000:2294-310. 
Murray PR, Rosenthal KS, Pfaller MA. Enterobacteriaceae. In: Murray PR, 

Rosenthal KS, Pfaller MA. Medical Microbiology, 5th ed. St. Louis, MO: Mosby, 

2002:223-38. 



♦♦♦ CASE 13 

An 18-day-old female infant is brought to the pediatric emergency room by 
her panicked mother. The child has developed a fever and has been crying 
nonstop for the past 4 hours. She has fed only once today and vomited all of 
the ingested formula. The baby was born by vaginal delivery after an uncom- 
plicated, full-term pregnancy to a healthy 22-year-old gravida, para : (one 
pregnancy, one delivery) woman. The mother has no history of any infectious 
diseases and tested negative for group B Streptococcus prior to delivery. The 
immediate postpartum course was routine. The baby had a routine check-up in 
the pediatrician's office 3 days ago, and no problems were identified. On 
examination, the child has a temperature of 38.3°C (100.9°F), pulse of 140 beats 
per minute, and respiratory rate of 32 breaths per minute. She is not crying at 
the moment. She has poor muscle tone, will not regard your face or respond to 
loud stimuli. Her anterior fontanelle is bulging. Her mucous membranes are 
moist, and her skin is without rash. Her heart is tachycardic but regular, and 
her lungs are clear. Her white blood count is elevated, a urinalysis is normal, 
and a chest x-ray is clear. A Gram stain of her cerebrospinal fluid (CSF) from 
a lumbar puncture shows gram-positive coccobacilli. 

^ What organism is responsible for this infection? 

^ How does this organism avoid antibody-mediated host defenses? 



702 CASE FILES: MICROBIOLOGY 



ANSWERS TO CASE 13: Listeria monocytogenes 

Summary: An 18-day-old infant presents with meningitis and Gram stain of 
the CSF reveals gram-positive coccobacilli. 

^ Organism responsible for this infection: Listeria monocytogenes 

^ Mechanism of avoidance of antibody-mediated defenses: Intracellular 
replication and spread from cell to cell by phagocytosis 



CLINICAL CORRELATION 

Listeria is found in the environment but is not normal fecal flora in humans. 
Infection with Listeria is more common in the summer months. Disease is 
often the result of ingestion of the organism from infected foods such as milk, 
soft cheese, undercooked meat, or unwashed vegetables. Listeria monocyto- 
genes causes asymptomatic or mild gastrointestinal infections in persons with 
intact immune systems and severe disease, most commonly meningitis, in 
those with impaired cellular immunity, such as pregnant women, neonates, 
AIDS patients, and posttransplant patients on immunosuppressive medica- 
tions. Two types of neonatal disease have been described. Early-onset disease, 
which occurs with in the first 2 days of life, is the result of transplacental infec- 
tion. Initial signs and symptoms include difficulty breathing and pneumonia. 
This infection is also called granulomatosis infantiseptica, because severe dis- 
ease can be associated with a granulomatous rash with abscesses. Late-onset 
disease, which usually occurs 2-3 weeks after birth, is thought to result from 
exposure to Listeria either during or shortly after delivery. This infection most 
commonly presents as meningitis. Clinically these syndromes can be difficult 
to distinguish from that seen with group B streptococci. 

APPROACH TO SUSPECTED Listeria monocytogenes 
Objectives 

1 . Know the structure and physiology of L. monocytogenes. 

2. Know the life cycle, virulence factors, and diseases associated with 
L. monocytogenes. 

Definitions 

Cold enrichment: Used to enhance growth of Listeria, particularly from 

food. 
Granulomatosis infantiseptica: Severe form of Listeria infection of 

neonates in which granulomatous skin lesions are evident. 



CLINICAL CASES 103 

Anterior fontanelle: An opening of the neonatal skull between the sutures. 
Gravida: Number of total pregnancies. 

Para: Number of deliveries (usually after 20-week gestation); a pregnancy 
that ends prior to 20-week gestation is an abortus. 

DISCUSSION 

Characteristics of Listeria 

Listeria monocytogenes is a small, facultative anaerobic, gram-positive 
bacillus. It may appear as coccobacilli in pairs or chains, so it can be mistaken 
for Streptococcus pneumoniae or Enterococcus on Gram stain. Listeria mono- 
cytogenes is an intracellular pathogen, which allows it to avoid antibody- 
mediated defenses of the host. It initially enters host cells via the action of a 
protein, internalin, which induces phagocytosis of the bacteria. Listeria 
produces a toxin, which then lyses the phagosome, releasing the bacteria into 
the host cell's cytoplasm. It replicates in the host cytoplasm and moves to the 
host membrane. By pushing against the membrane, a protrusion, known as a 
filopod, is produced, which can be phagocytized by adjacent cells. 

This cycle is then repeated in the new host cell, allowing Listeria to spread 
without being exposed to antibodies or other humoral immunity factors. For 
this reason, host cellular immunity factors protect against infection and those 
with impaired cellular immunity are vulnerable. 

Diagnosis 

Clinical diagnosis is difficult based on the nonspecific signs and symptoms. 
Clinically L. meningitis in neonates resembles group B streptococci, both are 
a significant cause of bacteria meningitis in that age group. Definitive diagno- 
sis of Listeria is made by culture of the CSF and/or blood. Gram stain of the 
CSF would demonstrate small gram-positive bacilli, appearing similar to 
corynebacteria or S. pneumoniae. 

Listeria will grow on routine agar media within 24-48 hours. On blood agar 
media Listeria demonstrate P-hemolysis, which differentiates it from 
Corynebacterium, but adds to the difficulty in distinguishing them from strep- 
tococci. Specific identification is made in part by observation of characteristic 
tumbling motility on a wet preparation after room temperature incubation. A 
reaction of catalase positive helps to distinguish Listeria from streptococci. 

Culture of Listeria from food may require cold enrichment, which would 
enhance the growth of Listeria. Food samples would be sent to a public health 
laboratory where some of the food would be enriched in a selective broth 
media at room temperature or lower. 



104 CASE FILES: MICROBIOLOGY 

Treatment and Prevention 

Treatment of Listeria septicemia or meningitis is with ampicillin plus or 
minus gentamicin. Of significance is the inherent resistance of Listeria to 
cephalosporins, which are commonly chosen as empiric therapy for meningi- 
tis in adults and would be appropriate for treatment of streptococcal meningi- 
tis in children. Prevention involves the avoidance of the consumption of 
under-cooked foods, especially in high-risk patients. 



COMPREHENSION QUESTIONS 

[13.1] A 22-year-old medical student suffers diarrhea for more than a week 
since his return from a short vacation in Mexico. While in Mexico, he 
consumed a large quantity of raw cheeses almost every day. Which of 
the following is the most likely organism causing his diarrhea? 

A. Bacillus cereus 

B. Escherichia coli 

C. Listeria monocytogenes 

D. Salmonella enteritidis 

E. Shigella dysenteriae 

[13.2] A premature neonate suffers pneumonia and sepsis. Sputum culture on 
blood agar plate yields pinpointed P-hemolytic colonies. Which of the 
following is a simple test to determine whether the organism is 
S. agalactiae or L. monocytogenes (these two organisms are important 
neonatal pathogens)? 

A. Bacitracin test 

B. Catalase test 

C. Coagulase test 

D. Polymerase chain reaction 

E. Sugar fermentation test 

[13.3] The most frequent source of infection with L. monocytogenes is 
through which of the following? 

A. Human feces 

B. Livestock 

C. Raw milk 

D. Soil 

E. Ticks 



CLINICAL CASES 705 

Answers 

[13.1] C. Outbreaks of gastroenteritis, as a consequence of L. monocyto- 
genes, are related to the ingestion of unpasteurized milk products, for 
example, cheese. Bacillus cereus, causing food poisoning, is associ- 
ated with spore survival and germination when rice is held at warm 
temperatures. Escherichia coli is usually associated with the entero- 
hemorrhagic E. coli (EHEC) form (verotoxin) that causes a bloody 
diarrhea and is associated with improperly cooked hamburger. 
Salmonella causes a diarrhea associated with contaminated chicken 
consumption. Finally, Shigella is usually associated with enterocolitis 
outbreaks among children in mental institutions and day care centers. 

[13.2] B. Streptococcus agalactiae (group B streptococci) is the leading 
cause of neonatal sepsis and meningitis. All streptococci (including 
S. agalactiae) are catalase-negative, whereas staphylococci are catalase- 
positive. Listeria monocytogenes is also catalase-positive. 

[13.3] C. Unpasteurized milk is a common vector for transmission of 
Listeria. See answer to Question 13.1. 



MICROBIOLOGY PEARLS 

Listeria meningitis clinically resembles group B streptococcal 
meningitis and needs to be distinguished because of the resist- 
ance of Listeria to cephalosporins. 

*♦* Listeria grown on blood agar media will be [3-hemolytic and resem- 
ble streptococci; however, by Gram stain Listeria are small bacilli 
and not chains of cocci. 

*♦* Listeria infection is commonly associated with consumption of 
undercooked food or unpasteurized milk or cheese products. 



REFERENCES 

Bortolussi R, Schlech WF. Listeriosis. In: Infectious Diseases of the Fetus and 
Newborn infant, 4th ed. Philadelphia, PA: W.B. Saunders, 1995:1055-73. 

Murray PR, Rosenthal KS, Pfaller MA. Listeria and erysipelothrix. In: Murray PR, 
Rosenthal KS, Pfaller MA. Medical Microbiology, 5th ed. St. Louis, MO: 
Mosby, 2005:273-78. 



This page intentionally left blank 



♦♦♦ CASE 14 

A 50-year-old man, a recent immigrant from Vietnam, is brought to the 
emergency room with a cough productive of bloody sputum. He first noticed a 
cough approximately 2 months ago, but there was not much sputum. In the past 
several days his sputum production has increased and become mixed with 
blood. He reports having lost approximately 15 lb in this time frame as well. He 
also notes that he's had drenching night sweats 2 or 3 nights a week for the past 
month. He has a 50-pack-year smoking history but no other medical history. He 
came to the United States from Vietnam 7 months ago. On examination, he is a 
thin, frail-appearing male. His vital signs are normal. His head and neck exam 
is normal. He has no palpable adenopathy in his neck or axilla. His lung exam 
is notable only for decreased breath sounds diffusely. A chest x-ray shows a 
cavitary infiltrate of the left upper lobe. 

^ What type of organism is likely to be seen on Gram stain of a 
sputum sample? 

^ What technique of staining is most commonly used to identify this 
organism? 

^ What is the histologic characteristic of Langhans cells? 



108 CASE FILES: MICROBIOLOGY 

ANSWERS TO CASE 14: Mycobacterium 

Summary: A 50-year-old Vietnamese man presents with chronic bloody 
sputum, weight loss, and a cavitary lesion on chest radiograph, all consistent 
with tuberculosis. 

^ Likely Gram stain findings of sputum sample: Mycobacterium 
tuberculosis appear as colorless ("ghost") cells 

Most commonly used staining technique for M. tuberculosis: Acid-fast 
staining 

^ Histologic characteristics of Langhans cells: Multinucleated giant 
cells of fused macrophages 

CLINICAL CORRELATION 

It is estimated that M. tuberculosis infects approximately one-third of the 
world's population. Mycobacterium tuberculosis is spread from person to 
person via aerosolized respiratory droplets that travel to the terminal 
airways. The bacteria are phagocytized by alveolar macrophages but inhibit 
destruction by the phago-some and proceed to replicate. Circulating 
macrophages are attracted to the site of infection and create multinucleated 
giant cells, composed of fused macrophages called Langhans cells. 
Extrapulmonary sites are infected through the spread of infected macrophages 
via lymphatic or hematogenous dissemination. Because of the intracellular 
nature of M. tuberculosis infections, antibody mediated defenses are 
relatively ineffective. Persons with conditions of reduced cellular immunity, 
such as AIDS, alcoholism, or drug abuse, or persons living in crowded, close 
quarters, such as prisoners, are at increased risk for infection. Organisms can 
remain dormant in granulomas for many years and reactivate following 
immunosuppression at a later date. At that time the organisms can infect extra- 
pulmonary sites. 

Mycobacterium avium-intracellulare is found in the environment and is an 
opportunistic pathogen that causes disease in AIDS patients. Disease can 
range from pneumonia to gastroenteritis to disseminated disease. 

Mycobacterium kansasii can clinically mimic pulmonary tuberculosis, but 
it is most often seen in middle-aged men with prior lung damage such as 
silicosis or asbestosis. Mycobacterium leprae is acquired by contact with the 
nine-banded armadillo. Most infections are seen in the southern United 
States including Texas and Louisiana. Thought to be lepers (skin lesions and 
deformation of the features of the face), these patients were contained in san- 
itariums and left to die. Infection can be either of the lepromatous or tubercu- 
lous type. Most infections caused by rapidly growing Mycobacteria are 
chronic wound infections, because this organism is found in the soil. 



CLINICAL CASES 709 

APPROACH TO SUSPECTED MYCOBACTERIAL 
INFECTION 

Objectives 

1 . Know the structure and physiology of Mycobacterium. 

2. Know the reservoirs, transmission, and diseases caused by Mycobacterium. 

3. Know the mechanisms of host defenses and treatments for mycobacterial 
diseases. 

Definitions 

Langhans cells: Multinucleated giant cells composed of fused macrophages. 
Granuloma: Chronic inflammatory response to either Mycobacterium or 

fungi, composed of macrophages and multinucleated giant cells. 
PPD: Purified protein derivative, prepared from M. tuberculosis antigens 

inoculated intradermally and a positive reaction is indicative of exposure 

to M. tuberculosis. 

DISCUSSION 

Characteristics of Mycobacteria Species 

Mycobacterium are small rod-shaped bacilli that stain as ghost cells with 
Gram stain, but because of the presence of mycolic acids in their cell wall 

stain with an "acid-fast stain" such as Kinyon or Ziehl-Neelsen. This complex, 
lipid-rich cell wall also makes the organisms resistant to many commonly used 
laboratory stains and is responsible for the resistance of this organism to many 
common detergents and antibiotics. 

In general, Mycobacterium species are slow-growing organisms, with a 
generation time of 15-20 hours, compared to about 1 hour for most bacteria. 
Mycobacterium can be divided into groups as characterized by Runyon, based 
on their growth characteristics, particularly pigment production. The photo- 
chromogens, which are pigmented only in the presence of light, include 
M. kansasii, as well as other saprophytic Mycobacterium. The scotochro- 
mogens, which are pigmented even without the presence of light, include M. 
szulgai, as well as the nonpathogenic M. gordonae, which has an orange pig- 
ment. The nonchromogens are not pigmented in the light or dark including 
M. avium-intracellulare, as well as M. haemophilum. The fourth runyon group 
is composed of the rapidly growing Mycobacteria such as M. fortuitum, M. 
chelonae, and M. abscessus. The M. tuberculosis complex includes M. tuber- 
culosis, M. africanum, M. ulcerans, and M. bovis, as well as other rarely iden- 
tified Mycobacterium. These colonies appear buff or tan color and are dry when 
growing on Lowenstein-Jensen agar. Mycobacterium leprae is not considered 
in that classification because it cannot be cultured in the laboratory. 



110 CASE FILES: MICROBIOLOGY 

One of the virulence factors of M. tuberculosis is cord factor. This can be 
visualized microscopically as organisms grown in broth culture will demonstrate 
a ropelike pattern indicating cording. The rapid growing Mycobacterium 
include M. fortuitum complex, M. chelonae complex, and M. abscessus, as well 
as other uncommonly isolated nonpathogenic Mycobacterium. These organisms 
by definition will grow within seven days of subculture onto routine microbio- 
logical media such as a blood agar plate. 

Diagnosis 

Diagnosis of tuberculosis is initially made based on a history (exposure to 
patient with tuberculosis, immigration, a stay in a jail or homeless shelter) and 
physical exam in patients with a productive cough, night sweats, and fever. A 
positive PPD test would indicate exposure to M. tuberculosis and warrant fur- 
ther testing with chest x-ray. Patients with the characteristic upper lobe cavi- 
tary lesion would have sputum collected and cultured for Mycobacterium. The 
classic chest radiograph findings include lower lobe consolidation in 
active infection, and apical lobe scarring with reactivation. 

A fluorescent, direct smear, of the respiratory specimen after deconta- 
mination to remove bacterial flora is reported within 24 hours of receipt of the 
respiratory specimen in the laboratory. Several first early morning deep cough 
specimens should be collected. Growth of M. tuberculosis on Lowenstein- 
Jensen (LJ) agar can take 3-8 weeks because of the slow dividing time of the 
organism; however, with the use of broth medium growth time has been 
decreased to as short as 1 week. Newly designed automated broth systems read 
bottles for growth based on carbon dioxide production of the organisms on a 
daily basis for up to 8 weeks. Bottles which are determined to be positive are 
stained by Kinyoun stain to visualize the presence of Mycobacterium. 
Positive bottles can be tested directly for M. tuberculosis, M. kansasii, 
M. avium-intracellulare, or M. gordonae using DNA probes. Other 
Mycobacterium species are identified either by routine biochemical tests, 
which require several weeks, or by high-pressure liquid chromatography, 
which can speciate Mycobacterium based on mycolic acids extracted from 
their cell surface. Optimal growth temperature for Mycobacterium species is 
35 °C (95 °F); however, the Mycobacterium that infect the skin such as M. 
haemophilum grow best at lower temperatures, and organisms such as M. szul- 
gai prefer 42°C (107.6°F). Growth on solid media is also enhanced in the pres- 
ence of 5-10 percent carbon dioxide. 

Treatment and Prevention 

Prophylaxis for tuberculosis consists of oral isoniazid for 6-9 months and 
is given to all patients with a recent conversion of their PPD to positive and 
a negative chest x-ray. Treatment for tuberculosis based on culture of 



CLINICAL CASES 111 

M. tuberculosis from any patient specimen is initially (first 2 months) with a 
multiagent regimen based on likely resistance patterns; one such combina- 
tion is isoniazid, rifampin, ethambutol, and pyrazinamide. Once the results 
indicate susceptibility to all of the four firstline drugs, treatment can continue 
with two drugs (usually isoniazid and rifampin) for the remaining 4-6 months. 
Because of the interaction of rifampin with other drugs, particularly HIV drugs 
and antifungals, this therapy may need to be individualized. 

Prevention of tuberculosis besides prophylactic isoniazid includes isolation 
of patients in the hospital to prevent spread. Patients with a positive acid-fast 
smear must remain in isolation until a diagnosis of tuberculosis is ruled out, 
until they leave the hospital, or following several weeks of appropriate anti- 
tuberculous therapy with obvious clinical improvement. All known close con- 
tacts (family members) of the index case should have a PPD test to determine 
if they should be given therapy and/or worked up for disease. 

Vaccination with BCG, (an attenuated strain of M. bovis) is not routinely 
performed in the United States because of the comparatively low incidence of 
tuberculosis. Protection from tuberculosis is not 100 percent with the vaccine 
and can confuse the results of the PPD for screening of recent converters. 

Treatment of the other atypical mycobacteria varies based on the species. 
Mycobacterium avium-intracellulare is usually treated with clarithromycin or 
azithromycin and ethambutol plus or minus amikacin. Current treatment for 
leprosy is dapsone and rifampin for at least 6 months. 



COMPREHENSION QUESTIONS 

[14.1] An emaciated prisoner in a New York prison began coughing up spu- 
tum streaked with blood. Examination of the sputum revealed the pres- 
ence of acid-fast bacilli. Which of the following would confirm a 
diagnosis of tuberculosis? 

A. Inclusion bodies of the nuclei of macrophages 

B. Presence of gram-positive pleomorphic organisms 

C. Rough, nonpigmented colonies 

D. Rapid growth on Lowenstein-Jensen medium 

[14.2] A 45-year-old traveler discovers that he has converted from negative to 
positive on the tuberculin (PPD) skin test. This indicates which of the 
following? 

A. He has active tuberculosis. 

B. He has delayed-type hypersensitivity against M. tuberculosis. 

C. He is most likely to be infected with an "atypical" Mycobacterium. 

D. He needs to be immediately isolated to prevent spread of 
M. tuberculosis. 

E. He will eventually develop tuberculosis. 



112 CASE FILES: MICROBIOLOGY 

[14.3] A 60-year-old man with a chronic cough, bloody sputum, and marked 
weight loss is diagnosed as having tuberculosis. A "serpentine-like" 
colonial morphology is noted on Lowenstein-Jensen agar. This latter 
finding is caused by which of the following factors? 

A. A large "slimy" capsule 

B. An endotoxin 

C. Coagulase 

D. Cord factor 

E. WaxD 

[14.4] A 25-year-old man known to have AIDS experiences a gradual onset 
of malaise and anorexia, proceeding within a few weeks to photopho- 
bia, impaired consciousness, and oculofacial palsy. An acid-fast bac- 
terium with trehalose-6, 6'-dimycolate is isolated. The identity of this 
organism is which of the following? 

A. Mycobacterium fortuitum-chelonei 

B. Mycobacterium kansasii 

C. Mycobacterium marinum 

D. Mycobacterium scrofulaceum 

E. Mycobacterium tuberculosis 

Answers 

[14.1] C. To mount a protective immune response against a specific 
microorganism requires that the appropriate population of cells play 
a role in the response. A lipoprotein of M. tuberculosis stimulates a 
specific "toll-like receptor" on the macrophage. Activated 
macrophages then synthesize IL-12, which causes differentiation of 
naive helper T cells into the Th-1 type of helper T cells that partici- 
pates in the cell-mediated (delayed hypersensitivity) response. In 
addition, delayed hypersensitivity (not humoral) reactions are produced 
against antigens of intracellular pathogens such as M. tuberculosis. 
Thus, humoral immunity is not protective against M. tuberculosis, 
and the patient will suffer severe tuberculosis if cell-mediated immu- 
nity is not functional. Therefore, an agglutination test for antibodies 
is useless as M. tuberculosis is an intracellular pathogen and will not 
elicit a humoral (antibody-formation) immune response that will pro- 
tect the patient against M. tuberculosis. The growth is usually slow, 
and the colonies are rough and nonpigmented. 

[14.2] B. The purified protein derivative (PPD) skin test, or tuberculin skin 
test, contains several proteins from M. tuberculosis, which when 
combined with waxes elicits a delayed hypersensitivity. It does not 
assess for the status of infection, but only speaks about prior expo- 
sure. The clinical presentation and/or chest radiograph would be the 
next steps in evaluation. 



CLINICAL CASES 113 

[14.3] D. Virulent strains of M. tuberculosis grow in a characteristic "ser- 
pentine" cord-like pattern, whereas avirulent strains do not. Virulence 
of the organism is correlated with cord factor (trehalose dimycolate). 

[14.4] E. Mycobacterium tuberculosis commonly has the trehalose dimyco- 
late factor (see answer to Question 14.3). 



MICROBIOLOGY PEARLS 



*♦* Mycobacterium tuberculosis is a slow growing organism that causes 

pulmonary infection after close contact with an infected individual. 

*♦* A positive skin test (PPD) indicates exposure to the organism and 



not necessarily disease. 
*♦* Mycobacterium species stain positive with an acid-fast stain because 
of components in their cell wall. 
Initial therapy for tuberculosis requires multiple agents to avoid the 
development of resistance, and culture susceptibilities will dic- 
tate which agents should be continued. 



REFERENCES 

Haas DW. Mycobacterium tuberculosis. In: Mandell GL, Bennett JE, Dolin R, eds. 

Principles and Practice of Infectious Diseases, 5th ed. Philadelphia, PA: 

Churchill Livingstone, 2000:2576-607. 
Murray PR, Rosenthal KS, Pfaller MA. Mycobacterium. In: Rosenthal KS, Kobayashi 

GS, Pfaller MA. Medical Microbiology, 5th ed. St. Louis, MO: Mosby, 

2005:297-310. 



This page intentionally left blank 



♦♦♦ CASE 15 

A 15-year-old teenager is brought to the office for evaluation of a cough and 
fever. His illness began several days ago with low-grade fever, headache, 
myalgias, and fatigue, and has slowly worsened. He now has a persistent 
cough. He has tried multiple over-the-counter cold and cough medications 
without relief. He has no significant medical or family history. No family 
members have been ill recently, but one of his good friends missed several 
days of school approximately 2 weeks ago with "walking pneumonia." On 
examination he is coughing frequently but is not particularly ill-appearing. His 
temperature is 38.1°C (100. 5°F), pulse is 90 beats per minute, and respiratory 
rate is 22 breaths per minute. His pharynx is injected (red and inflamed); oth- 
erwise, a head and neck exam is normal. His lung exam is notable only for 
some scattered rhonchi. The remainder of his examination is normal. A chest 
x-ray shows some patchy infiltration. A sputum Gram stain shows white blood 
cells but no organisms. 

^ What is the most likely etiology of this infection? 

^ What is the explanation for no organisms being seen on Gram 
stain? 

^ What rapid, although nonspecific, blood test can provide 
presumptive evidence of infection by this organism? 



776 CASE FILES: MICROBIOLOGY 



ANSWERS TO CASE 15: Mycoplasma 

Summary: A 15 -year-old adolescent presents with a persistent cough, patchy 
infiltrate on chest x-ray, and exposure to a friend with "walking pneumonia." 

•^ Most likely infectious agent: Mycoplasma pneumoniae 

^ Reasons no organisms are seen on Gram stain: Mycoplasma 
pneumoniae does not stain because it does not have a cell wall. 

^ Rapid blood test for presumptive evidence of M. pneumoniae: Cold 

agglutinins 

CLINICAL CORRELATION 

Mycoplasma pneumoniae is transmitted from person to person by aerosolized 
respiratory droplets and is most common in children and adolescents. Disease 
caused by M. pneumonia usually has an insidious onset and can progress to 
tracheobronchitis or pneumonia, which is often patchy or diffuse, as 
opposed to lobar. Because of the inability to diagnose this on microscopy, and 
the difficulty and length of time required for culture, serologic testing is often 
used to identify this organism. 

APPROACH TO SUSPECTED Mycoplasma pneumoniae 
PATIENT 

Objectives 

1. Know the structure and physiology of M. pneumoniae and other 
Mycoplasma organisms. 

2. Know the clinical diseases associated with and tests for identification 
of M. pneumoniae. 

Definitions 

Tracheobronchitis: Inflammation of the trachea in addition to the bronchi; 

causing swelling & narrowing of the airways. 
Rhonchi: A coarse rattling sound heard on auscultation of the lungs of a 

patient with partially obstructed airways. 
Pruritus: Itching; can have many causes: food allergy, drug reation, kidney/ 

liver disease, aging or dry skin, cancers, infectious agents or other 

unknown causes. 



CLINICAL CASES 117 

DISCUSSION 

Characteristics of Mycoplasma pneumoniae That Impact 
Transmission 

Mycoplasma pneumoniae is a short, strictly aerobic rod. It has a trilamellar, 
sterol-containing cell membrane but no cell wall, therefore it is not identifi- 
able with Gram or other stains. The lack of a cell wall also confers resist- 
ance against ^-lactams and other antibiotics that act on the cell walls of 
bacteria. It is the smallest free-living bacterium, even during infection, it 
remains extracellular. It divides by binary fission and has a doubling time of 
approximately 6 hours, much slower than most bacteria. This contributes to 
the difficulty in isolating this organism by culture, as up to 6 weeks of incu- 
bation is required. Mycoplasma has the adherence protein PI at one end, 
which is responsible for its attachment to a protein on target cells and may 
confer its preference for respiratory epithelium. When attached to ciliated res- 
piratory epithelial cells, first the cilia and then the cell is destroyed. This inter- 
feres with normal mucociliary clearance and allows the lower airways to be 
irritated and contaminated with infectious agents. 

Mycoplasma pneumoniae is transmitted from person to person by 
aerosolized respiratory droplets and secretions, and close association with an 
index case is usually required. No seasonal peak is observed. There is usually a 
1-3 week incubation period before the onset of clinical disease. Although it can 
infect those of all ages, disease more commonly occurs in children and young 
adults. Mycoplasma pneumoniae is responsible for 15-20 percent of community- 
acquired pneumonias. Clinical presentation consists of a low-grade fever, 
headache, malaise, and later a nonproductive cough, with a slow resolution. 

Diagnosis 

Diagnosis is primarily made from clinical presentation. Because of the inabil- 
ity to diagnose the infection with microscopy and the difficulty and length of 
time required for culture, serologic testing is often used to confirm a clinical 
diagnosis. Antibody-directed enzyme immunoassays and immunofluorescence 
tests or complement fixation tests are used in diagnosis. Another useful test is 
to analyze the titer of cold agglutinins. Mycoplasma pneumoniae infection 
often results in the stimulation of an IgM antibody against the I-antigen on 
erythrocytes. This antigen-antibody complex binds at 4°C (39.2°F) causing 
the clumping of erythrocytes. Although this response can be triggered by other 
organisms, titers of these antibodies of 1:128 or greater, or a fourfold increase 
with the presence of an appropriated clinical presentation are considered pre- 
sumptive evidence of M. pneumoniae disease. 

Another Mycoplasma, M. hominis, causes pelvic inflammatory disease 
(PID), nongonococcal urethritis (NGU), pyelonephritis, and postpartum 



118 CASE FILES: MICROBIOLOGY 

fever. Another cause of NGU and an organism that is detected with cold agglu- 
tinins is Ureaplasma urealyticum, a facultative anaerobic rod. Although this 
organism can also be a commensal, it can also lead to the sexually transmitted 
disease NGU and infertility. It is diagnosed via serology, by both cold agglu- 
tinins and specific serology with complement fixation and ELISA (enzyme- 
linked immunosorbent assay) for IgM. Like Mycoplasma, culture is not 
reliable and takes many weeks. PCR probes are also used for diagnosis. The 
clinical picture of NGU consists of urethral discharge, pruritus, and dysuria. 
Typically, systemic symptoms are absent. The onset of symptoms in NGU 
can often be subacute. There are 3 million new cases of NGU (including 
M. hominis, U. urealyticum, Chlamydia trachomatis, and Trichomonas vagi- 
nalis) a year, and 10-40 percent of women suffer PID as a result, compared to 
only 1-2 percent of males, with morbidity from NGU because of stricture or 
stenosis. NGU occurs equally in men and women, though can be asympto- 
matic in 50 percent of women. 

Treatment and Prevention 

Mycoplasma pneumoniae -related pneumonia, as well as other Mycoplasma 
infections resulting in NGU, can be effectively treated with tetracycline and 
macrolides. Tetracyclines can be used to treat most mycoplasmas, as well as 
Chlamydia; whereas macrolides can be used to treat Ureaplasma infections, 
which are resistant to tetracycline. Mycoplasma pneumoniae infections are dif- 
ficult to prevent because patients are infectious for extended periods of time, 
even during treatment. Several attempts have been made to produce inactivated 
and attenuated live vaccines without success. 

COMPREHENSION QUESTIONS 

[15.1] A 33-year-old woman is diagnosed with "walking pneumonia" caused 
by Mycoplasma. Which of the following best describes the character- 
istics of the etiologic organism? 

A. Absence of a cell wall 

B. Belonging to the class of Eukaryotes 

C. Often evoke an IgM autoantibody response leading to human ery- 
throcyte agglutination 

D. Typically colonize the gastrointestinal tract 

[15.2] Which of the following antibiotics is the best treatment for the above 
patient? 

A. Ampicillin 

B. Ceftriaxone 

C. Erythromycin 

D. Gentamicin 

E. Vancomycin 



CLINICAL CASES 119 

[15.3] Mycoplasma organisms may also cause disease in nonpulmonary sites. 
Which of the following is the most commonly affected nonpulmonary 
site? 

A. Meningitis 

B. Prosthetic heart valve 

C. Septic arthritis 

D. Urethritis 

[15.4] A 20-year-old man presents to the clinic with a history of fever and 
nonproductive cough. The patient's chest x-ray shows consolidation of 
the right lower lobe. An infection with M. pneumoniae is considered as 
the cause of the patient's pneumonia. Which of the following methods 
would confirm this diagnosis? 

A. Culture of sputum specimen on solid medium 

B. Detection of organism by microscopy 

C. Complement fixation test of acute and convalescent sera 

D. PCR amplification of patient's sputum specimen 

E. Enzyme immunoassay to detect cell wall antigens 

Answers 

[15.1] A. Mycoplasma are the smallest living organisms, and they do not 
have cell walls but rather have cell membranes. Thus, they are typi- 
cally resistant to antibiotics that interfere with cell wall synthesis. 
Also, because of their absence of a cell wall, they are not usually 
detected on Gram stain. They have a propensity for attaching to res- 
piratory, urethral, or genital tract epithelium. 

[15.2] C. Erythromycin, clarithromycin, or azithromycin (macrolides) are 
effective against mycoplasma species. 

[15.3] D. Mycoplasma and Ureaplasma species are commonly isolated from 
the lower genital tract. They are likely the most common cause of 
nonchlamydial nongonococcal urethritis. 

[15.4] C. Diagnosis is primarily made from serologic testing (enzyme 
immunoassays, immunofluorescence, cold agglutinins) Answers A, 
B, D, and E are all incorrect: (A) culturing M. pneumoniae is difficult 
and slow and is not used for diagnosis; (B) Mycoplasmas lack a 
cell wall making microscopy inappropriate; (D) PCR amplification 
of a sputum specimen is not an appropriate method of diagnosis; 
(E) M. pneumoniae lacks a cell wall and thus, cell wall antigens. 



120 CASE FILES: MICROBIOLOGY 



MICROBIOLOGY PEARLS 

*♦* Mycoplasmas are small free-living microorganisms that lack a cell 

wall. 
*♦* Mycoplasma pneumoniae is a common cause of atypical pneumonia 

in children and adolescents. 
*♦* Symptoms include nonproductive cough, fever, headache, and 

"walking pneumonia." 
*♦* Effective treatment is with erythromycin or tetracycline. 



REFERENCES 

Baseman JB, Tully JG. Mycoplasmas: sophisticated, reemerging, and burdened by 
their notoriety. Emerg Infect Dis 1997;3:21. www.cdc.gov/ncidod/EID/vol3nol/ 
baseman.htm. 

Murray PR, Rosenthal KS, Pfaller MA. Medical Microbiology, 5th ed. St. Louis, 
MO: Mosby, 2005:395-9. 

Loens K, Ursi D, Goossens H, et al. Molecular diagnosis of Mycoplasma pneumo- 
niae respiratory tract infections. J Clin Microbiol 41:4915-23. 2003. 

Ryan JR, Ray CG. Sherris Medical Microbiology, 4th ed. New York: McGraw-Hill, 
2004:409-11. 



♦> CASE 16 



A 19-year-old woman presents to the physician's office for the evaluation of a 
swollen knee. She states that for the past 1-2 week she has had some achiness 
in several of her joints and a low-grade fever, but it seemed to localize to her 
left knee approximately 3 days ago. It has been red, hot, and swollen. She has 
had no injury to the area and has never had anything like this before. Her past 
medical history is significant for having been treated for Chlamydia at the age 
of 17. She takes oral contraceptive pills regularly. She is sexually active, has 
been with her most recent boyfriend for about a month, and has had 5 partners 
in her lifetime. On examination, her vital signs are normal, but you notice that 
she walks with a limp. Her general examination is normal, and her skin is 
without rash. Her left knee is erythematous and warm to the touch. There is a 
visible effusion. Movement is limited because of pain and stiffness from the 
swelling. She refuses a pelvic examination because she doesn't see what that 
has to do with her sore knee. However, she does allow you to perform a joint 
aspiration. 

^ What are the most likely Gram stain findings of the aspirated joint 
fluid? 

^ What cell surface factors facilitate attachment and penetration 
of this organism into the host cell? 



722 CASE FILES: MICROBIOLOGY 

ANSWERS TO CASE 16: Neisseria 

Summary: A 19-year-old woman presents with septic arthritis. She has had an 
infection previously with Chlamydia. 

^ Likely findings on Gram stain of the joint fluid aspirate: Multiple 
polymorphonuclear leukocytes with intracellular gram-negative 
diplococci. 

^ Cell surface factors facilitating attachment and penetration into 
the host cell: Pili, which attach to epithelial cells, and Opa protein, 
which promotes firm attachment and cell penetration. 

CLINICAL CORRELATION 

Humans are the only known reservoir of Neisseria species. Neisseria gonor- 
rhoeae is transferred from person to person by sexual contact. Approximately 
half of infected women have an asymptomatic carrier state. This is much less 
common in men. Neisseria gonorrhoeae causes urethritis in men and cervicitis 
in women. Complications of genital infections include pelvic inflammatory dis- 
ease. The organism can also infect the rectum and oropharynx. Newborns pass- 
ing through an infected birth canal may develop conjunctivitis by direct contact, 
a disease called ophthalmia neonatorum. Disseminated disease, including bac- 
teremia with resultant joint and/or skin infections is more common in patients 
with complement deficiencies. Septic arthritis as a complication of disseminated 
disease may present in two forms, either as a systemic disease with fever, chills, 
and polyarticular syndrome, or as a monoarticular suppurative infection of a sin- 
gle joint without skin lesions or systemic symptoms. Most cases of disseminated 
gonococcal disease occur in persons with an asymptomatic genital infection. 

Neisseria meningitidis is carried as normal upper respiratory flora in 
approximately 10 percent of the population. The polysaccharide capsule 
allows the organism to avoid phagocytosis and under unknown circumstances 
enter the blood and in some cases the central nervous system. The subsequent 
inflammatory response induced by the organism causes shock and dissemi- 
nated intravascular coagulation. This is evidenced by skin lesions, which can 
mimic those seen in disseminated gonococcal infection. Bacteremia with or 
without meningitis usually occurs in teenage children. If untreated, the disease 
has a high mortality rate. 

APPROACH TO SUSPECTED GONORRHOEAE PATIENT 
Objectives 

1. Know the structure and characteristics of Neisseria species. 

2. Know the factors associated with the development on Neisseria infec- 
tions and diseases. 



CLINICAL CASES 123 

Definitions 

Disseminated intravascular coagulation (DIC): A complication of septic 
shock usually caused by endotoxin produced by the affecting organism. 

Ophthalmia neonatorum: Conjunctivitis in the first month of life usually 
as a result of N. gonorrhoeae or Chlamydia trachomatis. 

DISCUSSION 

Characteristics of Neisseria Species 

Neisseria species are aerobic, nonmotile, nonspore-forming, gram-negative 

cocci. They usually are arranged in pairs (diplococci) with adjacent sides flat- 
tened, resembling kidney beans. Neisseria are fastidious organisms that 
require a complex medium and an atmosphere supplemented with carbon 
dioxide for optimal growth. Neisseria gonorrhoeae has specific cell surface 
components related to its adherence, cellular penetration, toxicity, and evasion 
of host defenses. Cellular adherence is conferred by the presence of pili, which 
attach to host epithelial cells and also provide resistance to killing by host neu- 
trophils. The outer membrane also contains the Opa proteins (opacity pro- 
teins), which promote tight attachment and migration of the bacteria into the 
host. Then Por proteins (porin), which form channels (pores) in the outer 
membrane, prevent phagolysosome fusion, allowing intracellular survival. 
Rmp proteins (reduction-modifiable proteins) stimulate antibodies, which 
inhibit host bactericidal antibodies, protecting the other surface antigens from 
host attack. 

Plasmid acquisition and transfer appear to play significant roles in the 
development of antibiotic resistance by N. gonorrhoeae. Multiple plasmids, 
which confer ^-lactamase, have been identified. A conjugative plasmid that 
causes high-level tetracycline resistance has also been identified. These plas- 
mids are becoming more common, resulting in more antibiotic-resistant gono- 
coccal disease. LOS (lipooligosaccharide), also present in the cell wall, 
produces the inflammatory response responsible for most of the symptoms 
associated with gonococcal disease by its release of tumor necrosis factor-a. 

Neisseria meningitidis appear the same as N. gonorrhoeae on Gram stain. 
They also produce a polysaccharide capsule that prevents phagocytosis. 
Neisseria meningitidis is divided into 13 serogroups, the most common of 
which are A,C,Y, W135, and B. 

Diagnosis 

Septic arthritis must be differentiated from other noninfectious forms of 
arthritis such as rheumatoid arthritis and gout. Definitive diagnosis is made by 
analysis of cells and Gram stain from an aspirate of the joint. Gram stain 
would reveal intracellular gram-negative diplococci. A presumptive diagnosis 



724 CASE FILES: MICROBIOLOGY 

of gonorrhea can be made from a smear from a male urethra; otherwise, cul- 
ture is required for diagnosis. 

Neisseria species are fastidious organisms in that they require carbon dix- 
ide atmosphere, and N. gonorrhoeae also require chocolate agar. Neisseria 
gonorrhoeae also may require at least 48 hours for production of small grey 
colonies. Selective media such as Thayer Martin or Martin Lewis is usually 
needed to isolate N. gonorrhoeae from nonsterile sites such as the cervix or 
urethra. Neisseria gonorrhoeae are quite sensitive to drying, so plates must be 
placed in a warm environment quickly to maintain viability. If a delay in tran- 
sit to the laboratory is expected to be longer than several hours, a transport 
media such as Jembec is required. Rapid identification can be made from gram- 
negative diplococci, growing on selective media that are oxidase positive. 
Isolates are specifically identified by acid production from select sugars. 
Neisseria gonorrhoeae ferments glucose only, and N. meningitidis ferments 
both glucose and maltose. Because of the fastidious nature of N. gonorrhoeae, 
genital infections are identified using DNA probes, which detect both N. gon- 
orrhoeae and C. trachomatis, which commonly occur together and don't 
require live organisms for detection. 

Treatment and Prevention 

Penicillin is the treatment of choice for meningococcemia. Approximately 
30 percent of TV. gonorrhoeae produce ^-lactamase and are therefore resist- 
ant to penicillin. Treatment with ceftriaxone or a quinolones is usually rec- 
ommended, although increase in resistance to quinolones has been 
demonstrated in some geographic locations. Prevention of meningococcal 
disease is hy vaccination. A recent CDC recommendation advises vaccinat- 
ing all adolescents for meningococcus at the age of 11-12 years. Other sus- 
ceptible persons, such as military personnel, college students who will be 
living in dormitories and asplenic patients, should be vaccinated as well. 
Prophylaxis of close contacts is also recommended to prevent spread of the 
disease. Prevention of N. gonorrhoeae includes practicing safe sex and use of 
a condom, as well as screening sexually active persons. Screening of pregnant 
women for congenitally transmitted infections with appropriate treatment 
would prevent infection of the neonate with N. gonorrhoeae, as well as other 
congenitally transmitted infections. 



CLINICAL CASES 725 

COMPREHENSION QUESTIONS 

[16.1] The source of N. meningitidis is the nasopharynx of human carriers 
who exhibit no symptoms. The ability of this bacterium to colonize the 
respiratory mucosa is associated with its ability to synthesize which of 
the following? 

A. Coagulase 

B. Collagenase 

C. Hyaluronidase 

D. Lipases 

E. Pili 

[16.2] Several Neisseria species are a part of the normal flora (commensals) 
of the human upper respiratory tract. Which of the following state- 
ments accurately describes the significance of these bacteria? 

A. As a part of the normal flora, Neisseriae provide a natural immu- 
nity in local host defense. 

B. As a part of the respiratory flora, they are the most common cause 
of acute bronchitis and pneumonia. 

C. Commensal bacteria stimulate a cell-mediated immunity (CMI). 

D. Commensal Neisseriae in the upper respiratory tract impede 
phagocytosis by means of lipoteichoic acid. 

E. Normal flora such as nonpathogenic Neisseriae provide effective 
nonspecific B -cell-mediated humoral immunity. 

[16.3] A 22-year-old man presents to the STD clinic with a 5-day history of 
burning on urination and a 3-day history of a nonpurulent urethral dis- 
charge. He is sexually active with many female partners and does not 
use condoms. There is no history of prior sexually transmitted diseases. 
Laboratory findings from endourethral exudate are most likely to show 
which of the following? 

A. A negative gonorrhea culture 

B. Abundant intracellular diplococci in neutrophils 

C. Immunofluorescence using monoclonal antibodies to serotypes A-C 

D. Intracellular elementary bodies 

[16.4] The two pathogenic Neisseria species, N. meningitidis and N. gonor- 
rhoeae, differ from the nonpathogenic Neisseria species in what way? 

A. The former are less resistant to certain antibiotics than the non- 
pathogenic species. 

B. The pathogenic species are oxidase positive. 

C. The pathogenic species grow well in enriched chocolate agar. 

D. The pathogenic species do not grow well at room temperature. 



126 CASE FILES: MICROBIOLOGY 

Answers 

[16.1] E. Both N. gonorrhoeae and N. meningitidis adhere to the mucous 
membrane tissues by means of pili (short protein appendages from 
the membrane through the cell wall). Coagulase and lipase are prod- 
ucts of streptococci, whereas collagenase and hyaluronidase are 
enzyme products of streptococci. 

[16.2] A. The normal or usual flora seldom cause disease in humans, except 
the several species that may be opportunistic in the right circumstances. 
One mechanism that has been suggested as to how the normal flora 
help to protect humans from pathogenic strains of bacteria is to stimu- 
late the immune system to produce antibodies (or CMI) that would rec- 
ognize related pathogens and inhibit their growth. An unexplained 
component of this mechanism is how the normal flora continue to exist 
as part of the body flora in spite of these immune mechanisms. 

[16.3] B. This presentation is classic for gonorrhea infection and symptoms. 
Abundant gram-negative diplococci will be found both intracellularly 
and outside of the phagocytic cells. Interestingly, gonococci may 
even divide within the phagocytic cell. This evidence (Gram stain of 
the exudate) is presumptive evidence of gonococcal infection, and 
treatment should be made immediately. Such a specimen should be 
positive for culture with the correct medium (e.g., Thayer-Martin) 
and incubation conditions (37°C [98.6°F], increased carbon dioxide 
atmosphere). Serotypes A-C refers to N. meningitidis, and elemen- 
tary bodies would indicate Chlamydia microorganisms. 

[16.4] D. Neisseria gonorrhoeae and N. meningitidis are true human 
pathogens, surviving best in the human host. They are more fastidi- 
ous in their nutritional requirement, requiring an enriched selective 
medium for growth. All Neisseria are oxidase positive. Normal flora 
(nonpathogenic) Neisseria will grow at room temperature on simple 
medium. Because of p-lactamase production, sensitivities should be 
done to ensure proper antimicrobial selection for treatment. 



MICROBIOLOGY PEARLS 



*♦* Neisseria meningitidis is a highly contagious organism, which can 

cause meningitis in otherwise healthy young people. 
*♦* Neisseria meningitidis can be successfully prevented with the use of 

the vaccine for high-risk individuals. 
*♦* Neisseria gonorrhoeae is a treatable sexually transmitted disease 

and should be ruled out in high-risk patients to prevent further 

complications including disseminated disease. 



CLINICAL CASES 727 

REFERENCES 

Apicella, MA. Neisseria meningitidis. In: Mandell GL, Bennett JE, Dolin R, eds. 

Principles and Practice of Infectious Diseases, 5th ed. Philadelphia, PA: 

Churchill Livingstone, 2000:2228-41. 
Murray PR, Rosenthal KS, Pfaller MA. Neisseria. In: Murray PR, Rosenthal KS, 

Pfaller MA. Medical Microbiology, 5th ed. St. Louis, MO: Mosby, 

2005:311-21. 
Sparling, PF, Handsfield, HH. Neisseria gonorrhoeae. In: Mandell GL, Bennett JE, 

Dolin R, eds. Principles and Practice of Infectious Diseases, 5th ed. 

Philadelphia, PA: Churchill Livingstone, 2000:2242-58. 



This page intentionally left blank 



♦> CASE 17 

A 35 -year-old woman presents to the emergency department with right flank 
pain. She reports that she had a few days of urinary urgency, frequency, and 
burning which she tried to treat herself by drinking cranberry juice. Earlier 
today she started having a severe, colicky pain on her whole right side. She has 
had a fever, and when she urinated this morning she noticed that it appeared to 
have blood. She's had a few urinary tract infections (UTIs) in the past but noth- 
ing like this. She is on no medications regularly and has no other significant 
medical history. On examination, she has a temperature of 37.5°C (99.5°F), her 
other vital signs are normal, and she appears to be in pain. Notable on exami- 
nation is some tenderness on the right flank but no masses, rebound tenderness, 
or guarding on palpation of her abdomen. She has costovertebral angle tender- 
ness on the right side but not the left. Her peripheral white blood cell count is 
elevated. A urinalysis shows the presence of leukocyte esterase, blood, and a 
high pH. An abdominal CT scan reveals an obstructing stone in the right ureter 
causing hydronephrosis of the right kidney. 

^ What organism is likely to be responsible for this infection? 

^ What is the cause of the high pH of this patient's urine? 



130 CASE FILES: MICROBIOLOGY 

ANSWERS TO CASE 17: Proteus mirabilis 

Summary: A 35-year-old woman presents with a UTI and nephrolithiasis (kidney 
stones). The urine has a high pH. 

^ Organism most likely to be responsible for this infection: Proteus 
mirabilis. 

^ Mechanism of high pH in urine: Proteus produces urease, which 
splits urea into carbon dioxide and ammonia, raising the urinary pH. 

CLINICAL CORRELATION 

Proteus species are normal flora of the gastrointestinal tract and predomi- 
nantly associated with hospital-acquired UTIs as well as bacteremia, 
osteomyelitis, empyema, and neonatal encephalitis. Proteus causes UTIs after 
urethral contamination with fecal bacteria followed by ascension into the blad- 
der. Most infections occur in patients with structural abnormalities or long- 
term catheters of the urinary tract. Proteus infections can result in significant 
renal damage by several mechanisms. Proteus produces large amounts of ure- 
ase, which breaks down urea into carbon dioxide and ammonia and results in 
elevated urinary pH levels. High urinary pH can contribute to direct renal 
toxicity and also can result in increased urinary stone formation. Urinary 
stones can result in further renal damage by obstructing urine flow and serv- 
ing as a focus of ongoing infection. Crystalline material tends to build up 
inside of a long-term catheter leading to biofilm formation. This can essen- 
tially block flow through the catheter. Proteus is also among the most common 
causes of bacteremia in the family Enterobacteriaceae often associated with 
underlying disease such as diabetes, malignancy, or heart or lung disease. 
Bacteremia is usually secondary to a primary UTI. Pediatric meningitis with 
Proteus species, especially in the first week of life, has a high mortality rate 
and a predilection for abscess formation when the organism gains access to the 
brain. It is hypothesized that the organisms gain entry into the blood through 
the umbilicus and from the blood they disseminated into the brain. 

APPROACH TO Proteus mirabilis INFECTION 
Objectives 

1 . Know the structure and characteristics of P. mirabilis. 

2. Know the mechanisms by which P. mirabilis produces renal damage. 



CLINICAL CASES 131 

Definitions 

Nephrolithiasis: The presence of calculi (solid, crystalline) that develop in 

the kidney and pass through the genitourinary tract. 
Hydronephrosis: Enlargement of the kidney because of an abnormality 

such as the presence of stones. 

DISCUSSION 

Characteristics of Proteus Species 

The genus Proteus includes five species, the most common of which are 
P. mirabilis and P. vulgaris. Proteus species are commonly found in the envi- 
ronment and as normal flora in the intestinal tract of humans and other animals. 
Proteus mirabilis, like other members of the Enterobacteriaceae family, is a 
nonspore-forming, facultative anaerobic, gram-negative bacillus. Proteus 
has fimbriae, which facilitate attachment to uroepithelium, and flagellae, 
which provide the motility required for ascending infection. Proteus also has 
the ability to transform from a single cell form to a multicell elongated 
(swarmer) form. The swarmer cells are more likely to be associated with cel- 
lular adherence in the kidney as demonstrated in an animal model of infection. 
Hemolysin, which induces cell damage by forming pores, may also play a role 
in establishment of pyelonephritis. 

Diagnosis 

Diagnosis of a UTI is initially by the urinalysis followed by a culture. The 
presence of leukocyte esterase, which is an indicator of the presence of white 
blood cells, in this patient indicated a presumptive UTI. The increased urinary 
pH as well as the evidence of a stone by CT indicated an obstructive process. 
Gram stain of the urine may be helpful if a significantly large number of organ- 
isms are present in the urine (greater than 10 5 colonies per milliliter). Culture 
of urine would likely be diagnostic after 24 hours. Members of the family 
Enterobacteriaceae, the most common cause of UTIs in an otherwise healthy 
young person, should grow rapidly on blood as well as MacConkey agars. The 
presence of greater than or equal to 10 5 CFU/mL in the urine of a single organ- 
ism would indicate a significant pathogen. Proteus is easily identified on a 
MacConkey agar plate as a clear colony (nonlactose fermenter). The obvi- 
ous swarm seen on blood agar would indicate a Proteus species. Definitive 
confirmation of P. mirabilis would be made by biochemical tests included in 
most commercially available identification systems. A quick test to differenti- 
ate P. mirabilis and P. vulgaris would be indole positivity in the latter. 



132 CASE FILES: MICROBIOLOGY 

Treatment and Prevention 

Proteus species are usually among the most susceptible genera of all of the 
Enterobacteriaceae, most susceptible to penicillin, although it is not uncom- 
mon for them to be resistant to tetracyclines. Proteus vulgaris, however, tends 
to be resistant to more antimicrobials than P. mirabilis. As is the case with most 
bacteria, new resistance mechanisms are being seen in otherwise susceptible 
organisms. 

COMPREHENSION QUESTIONS 

[17.1] A 78-year-old patient with an episode of acute urinary retention was 
catheterized. Three days later, he developed fever and suprapubic pain. 
Culture of the urine revealed a thin film of bacterial growth over the 
entire blood agar plate, and the urease test was positive. Which of the 
following is the most likely organism to cause this infection? 

A. Escherichia coli 

B. Helicobacter pylori 

C. Morganella morganii 

D. Proteus mirabilis 

E. Enterococcus faecalis 

[17.2] A urinary tract infection as a result of P. mirabilis facilitates the for- 
mation of kidney stones because the organism does which of the 
following? 

A. Destroys blood vessels in the kidney 

B. Exhibits "swarming" motility 

C. Ferments many sugars 

D. Produces a potent urease 

E. Secretes many exotoxins 

[17.3] A 55 -year-old woman is noted to have pyelonephritis with shaking 
chills and fever. Blood cultures are obtained, and the Gram stain is read 
preliminarily as consistent with Proteus species. Which of the following 
bacteria also may be the etiology? 

A. Escherichia coli 

B. Group B Streptococcus 

C. Staphylococcus aureus 

D. Streptococcus pyogenes 



CLINICAL CASES 133 

Answers 

[17.1] D. Proteus species produce infections in humans only when the bac- 
teria leave the intestinal tract. They are found in UTIs and produce 
bacteremia, pneumonia, and focal lesions in debilitated patients or 
those receiving intravenous infusions. Proteus mirabilis is a common 
cause of UTI. Proteus species produce urease, making urine alkaline 
and promoting stone formation. The rapid motility of these organisms 
is evidenced by "swarming," a thin film of organisms over the entire 
agar plate. 

[17.2] D. Proteus species produce a urease, which hydrolyzes urea leading 
to ammonia, which alkalinizes the urine (leading to a higher pH). 

[17.3] A. Both proteus and E. coli are gram-negative rod bacilli. Escherichia 
coli is the most common isolate in UTIs. 



MICROBIOLOGY PEARLS 

The ability of Proteus species to swarm is an important virulence 

mechanism in causing UTIs. 
The swarming nature of Proteus also is an early diagnostic indicator 
as to the organism's identity. 
*♦* Proteus species have the ability to produce obstructive stones 

because of the presence of the urease enzyme. 
*♦* Proteus mirabilis is the most common cause of Proteus infections. 



A. 



REFERENCES 

Mobley HLT, Belas R. Swarming and pathogenicity of proteus mirabilis in the uri- 
nary tract. Trends Microbiol 1995;3:280-84. 

Murray PR, Rosenthal KS, Pfaller MA. Enterobacteriaceae. In: Murray PR, 
Rosenthal KS, Pfaller MA. Medical Microbiology, 5th ed. St. Louis, MO: 
Mosby, 2005:323-38. 

O'Hara CM, Brenner FW, Miller JM. Classification, identification, and clinical sig- 
nificance of proteus, providencia and morganella. Clin. Microbiol. Rev 
2000:13:534-46. 



This page intentionally left blank 



♦> CASE 18 



A 73-year-old man with a history of hypertension and type II diabetes melli- 
tus presents to the physician's office with excruciating left ear pain for the past 
3 days. He also has noticed today that his speech seems a bit slurred, and his 
wife says that the left side of his face looks "droopy." He has had so much pain 
and swelling that he hasn't been able to put his hearing aid in for several days. 
He has had "swimmer's ear" in the past, which responded to treatment with 
ear drops, but has not had any ear problems in several years. He cleans his ears 
daily with cotton swabs prior to putting in his hearing aid. He denies having a 
fever, but says that his blood sugars have been higher than usual for the past 
2 days. On examination, his vital signs are normal, and he is in obvious pain. 
He has a prominent left-sided facial droop. His left ear is diffusely swollen, 
and he is tender on the pinna, the entire periauricular area, and mastoid. There 
is purulent drainage from the ear canal. You are unable to insert a speculum into 
the canal because of the swelling and pain. He has evidence of facial nerve 
palsy on the left side. Blood tests show an elevated white blood cell count and 
a markedly elevated erythrocyte sedimentation rate (ESR). Your diagnosis is 
malignant external otitis infection. 

^ What organism is the most likely cause of this infection? 

^ Which two toxins contribute to most of the systemic toxicity of this 
organism? 



136 CASE FILES: MICROBIOLOGY 

ANSWERS TO CASE 18: Pseudomonas aeruginosa 

Summary: A 73-year-old man is diagnosed with malignant otitis externa. 

^ Organism most likely to cause this infection: Pseudomonas 
aeruginosa 

^ Which two toxins contribute to most of the systemic signs 
of infection: Lipopolysaccharide endotoxin and exotoxin A 

CLINICAL CORRELATION 

Pseudomonas causes numerous types of infections, with the common factor 
being that they are usually in a debilitated host. Pseudomonas aeruginosa is 
an opportunistic pathogen that is most commonly associated with nosocomial 
pneumonia. Pulmonary infections primarily occur in patients with underlying 
disease such as cystic fibrosis or chronic lung or heart disease, who have 
immune suppression, or who are on ventilators. Skin infections occur in 
patients whose skin has been disrupted either by burn or trauma. Skin lesions 
can also be a secondary effect of disseminated disease and are known as 
ecthyma gangrenosum. Other common infections include urinary tract infec- 
tion in catheterized patients and chronic otitis. Malignant otitis externa, a 
severe external ear infection, which can potentially invade through the cranial 
bones and nerves, is seen primarily in the elderly and in diabetics. Other com- 
plications are uncommonly a result of Pseudomonas bacteremia, such as endo- 
carditis, meningitis, and bone and joint infections. 

APPROACH TO SUSPECTED Pseudomonas PATIENT 
Objectives 

1 . Know the structural and physiologic characteristics of P. aeruginosa. 

2. Know the virulence factors associated with P. aeruginosa. 

Definitions 

Periauricular: Around the external ear. 

Erythrocyte sedimentation rate (ESR): A measure of the time it takes for 

red blood cells to settle, which is a nonspecific measure of inflammation. 
Ecthyma gangrenosum: Pustular skin lesions that later become necrotic 

ulcers and can lead to gangrene. 



CLINICAL CASES 137 

DISCUSSION 

Characteristics of Pseudomonas Species 

Pseudomonas species is a ubiquitous, aerobic gram-negative bacillus. At 

least ten species are included in the genus that can cause disease in humans. 
Some of the closely related organisms have been transferred to their own 
genus, such as Burkholderia cepacia, formerly Pseudomonas cepacia. 
Pseudomonas aeruginosa is the most common cause of human infections. 
It is motile as a result of the presence of polar flagellae. It is found commonly 
in the environment and has a predilection for moist areas. Reservoirs in nature 
include soil, vegetation, and water. Reservoirs in a hospital include sinks, 
toilets, mops, respiratory therapy, and dialysis equipment. It exhibits 
intrinsic resistance to many antibiotics and disinfectants. It has minimal 
growth requirements and can be easily cultured on many media in a wide 
range of temperatures. It has multiple virulence factors. Pseudomonas adheres 
to host cells by pili and nonpili adhesins. It produces a polysaccharide cap- 
sule that allows the organism to adhere to epithelial cells, inhibits phagocyto- 
sis, and confers protection against antibiotic activity. Patients with cystic 
fibrosis are more likely to be infected with a strain whose colony appears 
mucoid because of the presence of the capsule. 

Pseudomonas produces multiple toxins and enzymes, which contribute to 
its virulence. 

Its lipopoh saccharide endotoxin and exotoxin A appear to cause most of 
the systemic manifestations of Pseudomonas disease. Exotoxin A blocks pro- 
tein synthesis in host cells, causing direct cytotoxicity. It mediates systemic 
toxic effects as well. It is similar in function to diphtheria toxin but is struc- 
turally and immunologically distinct. Endotoxin contributes to the develop- 
ment of many of the symptoms and signs of sepsis, including fever, 
leukocytosis, and hypotension. 

Antibiotic resistance is another important aspect of its virulence. It is 
intrinsically resistant to numerous antibiotics and has acquired resistance to 
others through various means. The polysaccharide capsule prevents the pen- 
etration of many antibiotics into Pseudomonas. Penetration of antibiotic into 
the Pseudomonas cell is usually through pores in the outer membrane. 
Mutation of these porin proteins appears to be a primary mechanism of its 
antibiotic resistance. Multidrug efflux pumps and ^-lactamase production 
also contribute to the antibiotic resistance that so frequently complicates the 
treatment of Pseudomonas infections. 

Some P. aeruginosa strains produce a diffusable pigment: pyocyanin, 
which gives the colonies a blue color; fluorescein, which gives them a yellow 
color; or pyorubin, which gives them a red-brown color. Pyocyanin also 
seems to aid in the virulence of the organism by stimulating an inflammatory 
response and by producing toxic oxygen radicals. 



138 CASE FILES: MICROBIOLOGY 

Diagnosis 

Diagnosis of malignant otitis externa is by the common clinical features of otor- 
rhea, painful edematous ear canal with a purulent discharge. Culture of the dis- 
charge from the internal ear grows P. aeruginosa in most cases. Pseudomonas 
aeruginosa grows readily on routine laboratory media. Preliminary identifica- 
tion can be made by colony morphology particularly if typical green pigment 
is produced. Pseudomonas aeruginosa appears as a clear to dark colony on 
MacConkey agar, indicating that it does not ferment lactose. Colonies of 
P. aeruginosa are (3-hemolytic and dark colored, as a result of pigment produc- 
tion and blood agar media. The organisms are motile, and therefore colonies 
appear spread. It does not ferment glucose, is oxidase positive, and is therefore 
not a part of the Enterobacteriaceae family, but is considered a nonfermenter. 
Pseudomonas aeruginosa can be distinguished from some of the other closely 
related species by its ability to grow at a wide range of temperatures, up to as 
high as 42°C (107.6°F). The colonies also have a distinct odor, sometimes con- 
sidered a grape-like odor. Confirmatory identification can be made by numer- 
ous commercially available identification systems. 

Treatment and Prevention 

Treatment of malignant otitis externa includes surgery to remove necrotic 
tissue and pus and appropriate antibiotics. Treatment with two antibiotics 

to which the organism is susceptible is optimal. Pseudomonas aeruginosa is 
usually inherently resistant to multiple antibiotics. Most are susceptible to the 
antipseudomonal penicillins, such as piperacillin and ticarcillin and to the 
newer fluoroquinolones as well as the aminoglycosides. Imipenem is often 
reserved for treatment of infections caused by drug-resistant strains. 

COMPREHENSION QUESTIONS 

[18.1] A severely burned firefighter develops a rapidly disseminating bacterial 
infection while hospitalized. "Green pus" is noted in the burned tissue 
and cultures of both the tissue and blood yield small oxidase-positive 
gram-negative rods. Which of the following statements most accu- 
rately conveys information about this organism? 

A. Endotoxin is the only virulence factor known to be produced by 
these bacteria. 

B. Humans are the only known reservoir hosts for these bacteria. 

C. The bacteria are difficult to culture because they have numerous 
growth requirements. 

D. These are among the most antibiotic resistant of all clinically relevant 
bacteria. 

E. These highly motile bacteria can "swarm" over the surface of cul- 
ture media. 



CLINICAL CASES 139 

[18.2] The fluoroquinolone resistance seen with increasing frequency in P. 
aeruginosa infections is best explained by which of the following 
mechanisms? 

A. Changes in the structure or composition of the cell envelope that 
make it more difficult for the antibiotic to gain entrance 

B. Enzymatic cleavage of the antibiotic molecule 

C. Inactivation of the antibiotic by enzymatic acetylation 

D. Overproduction of the cellular target that the antibiotic attacks 

E. Removal of the antibiotic from the cell interior by a membrane 
pump 

[18.3] An aerobic, oxidase positive organism is isolated from the sputum of a 
12-year-old cystic fibrosis patient with pneumonia and lung abscesses. 
On culture the organisms have a "fruity" odor and form greenish 
colonies. The etiologic agent of the respiratory tract infection is most 
likely to be which of the following? 

A. Chlamydia pneumoniae 

B. Klebsiella pneumoniae 

C. Pseudomonas aeruginosa 

D. Serratia marcescens 

E. Streptococcus pneumoniae 

Answers 

[18.1] D. Pseudomonas aeruginosa is an obligate aerobe that grows on 
many types of culture media, sometimes producing a sweet or grape- 
like odor. It often produces a nonfluorescent bluish pigment (pyocyanin) 
which diffuses into agar or pus fluids. Many strains also produce a 
fluorescent pigment (pyoverdin), which gives a greenish color. One 
of the most significant problems with Pseudomonas infections is the 
high level of natural resistance to many antimicrobials that this wide- 
spread environmental opportunist exhibits. 

[18.2] E. Clinically significant infections with P. aeruginosa should not be 
treated with single-drug therapy, because the bacteria can develop 
resistance when single drugs are employed. The newer quinolones, 
including ciprofloxacin, are active against Pseudomonas. Quinolones 
inhibit bacterial DNA synthesis by blocking DNA gyrase. The fluo- 
rinated forms of ciprofloxacin and norfloxacin have low toxicity and 
greater antibacterial activity than the earlier forms. Plasmids code for 
enzymes that determine the active transport of various antimicrobials 
across the cell membrane. 

[18.3] C. All of the options are potential etiologic agents for pneumonias in 
humans. The laboratory descriptions of the organism best fits P. aerug- 
inosa (also see the answer for Question 18.1). 



140 CASE FILES: MICROBIOLOGY 



MICROBIOLOGY PEARLS 



*♦* Pseudomonas aeruginosa is most commonly treated with a (3-lactam 

such as piperacillin or cefepime plus an aminoglycoside such as 

tobramycin. 
*♦* Pseudomonas aeruginosa is an opportunistic pathogen that is most 

often found in hospital environments as a source of nosocomial 

infection. 
*♦* Pseudomonas aeruginosa is a nonfermentative gram-negative bacilli 



that is oxidase positive. 



REFERENCES 

Murray PR, Rosenthal KS, Pfaller MA. Pseudomonas and related organisms. In: 

Murray PR, Rosenthal KS, Pfaller MA. Medical Microbiology, 5th ed. St. Louis, 

MO: Mosby, 2005:587-615. 
Sparling PF, Handsfield HH. Pseudomonas aeruginosa. In: Mandell GL, Bennett JE, 

Dolin R, eds. Principles and Practice of Infectious Diseases, 5th ed. 

Philadelphia, PA: Churchill Livingstone, 2000:2310-27. 



t* CASE 19 



♦> 



A 48-year-old man presents to the emergency room with 2 days of crampy 
abdominal pain, nausea, vomiting, diarrhea, and fever. He has not had any 
blood in his stool. He denies contact with anyone with similar symptoms 
recently. He has not eaten any raw or unprocessed foods recently. The only 
food that he did not prepare himself in the past week was a breakfast of eggs 
"sunny-side up" and bacon that he had at a diner the day before his symptoms 
started. On examination, he is tired appearing; his temperature is 37.7°C 
(99.9°F); and his heart rate is 95 beats per minute while he is lying down, but 
it increases to 120 beats per minute when he sits up. His blood pressure is 
145/85 mm Hg while lying down and 1 10/60 mm Hg when sitting. The physi- 
cian interprets this as a positive "tilt test," indicating significant volume deple- 
tion. His mucous membranes appear dry. His abdominal exam is notable for 
diffuse tenderness but no palpable masses, rebound, or guarding. A rectal 
exam reveals only heme-negative watery stool. 

^ What is the most likely etiologic agent of this infection? 

^ What are the most common sources of human infections with this 
organism? 



742 CASE FILES: MICROBIOLOGY 



ANSWERS TO CASE 19: Salmonella AND Shigella 

Summary: A 48-year-old man with acute gastroenteritis has fever, a positive 
tilt test, abdominal pain, and diarrhea after eating eggs a day before. 

^ Most likely etiology of infection: Salmonella 

^ Most common sources of infection: Undercooked poultry, eggs, dairy 
products, or foods prepared on contaminated work surfaces 



CLINICAL CORRELATION 

This individual has the acute onset of diarrhea and vomiting. The first priorities 
as with any patient are the ABC's: airway, breathing, circulation. Circulatory 
status is assessed by monitoring the pulse rate and blood pressure, which may be 
normal at rest, but abnormal on changing of position. This patient had a rise in 
10 beats per minute heart rate from the lying to the sitting position, and a fall of 
10 mm Hg of blood pressure. This constitutes a positive tilt test and may indi- 
cate a volume depletion of 10-25 percent. Thus, the first therapeutic goal would 
be volume repletion, such as with intravenous normal saline. 

In humans, most cases of nontyphoidal Salmonella result from ingesting 
contaminated food products. Poultry, eggs, dairy products, or other foods pre- 
pared on contaminated work surfaces are the most common sources. Fecal-oral 
spread is common among children. Live animals, especially exotic pets such as 
reptiles, have also been identified as sources of infection. Host gastric acid is a 
primary defense against the organism, and conditions or medications that 
reduce gastric acidity may predispose to infection. The primary site of invasion 
of Salmonella is the M (microfold) cells in the Peyer's patches of the distal 
ileum. M cells internalize and transfer foreign antigens from the intestinal 
lumen to macrophages and leukocyte. The infection can then spread to adjacent 
cells and lymphoid tissue. Host inflammatory responses usually limit the infec- 
tion to the gastrointestinal (GI) tract, but bacteremia can occur. Bacteremia is 
more common in the children, elderly patients, or those with immune deficien- 
cies, such as AIDS. Gastroenteritis is the most common clinical manifestation 
of Salmonella infection. Nausea, vomiting, nonbloody diarrhea, fever, and 
abdominal cramps starting 8-48 hours after ingestion of contaminated food are 
typical. The illness is generally self-limited and will last from 2 to 7 days. 

Enteric fever, or typhoid fever, is a more severe form of gastroenteritis with 
systemic symptoms that are caused by either Salmonella typhi or Salmonella 
paratyphi. Symptoms include chills, headache, anorexia, weakness, and mus- 
cle aches; and later fever, lymphadenopathy, and hepatosplenomegaly; and in 
a third of patients a maculopapular rash (rose spots). Symptoms persist for a 
longer period of time than nontyphoidal gastroenteritis as does the carrier state 
in a small percentage of infected patients. 



CLINICAL CASES 143 

Gastroenteritis caused by Salmonella can mimic the signs and symptoms of 
other forms of infections such as Shigella. Infection with Shigella produces 
predominantly diarrhea, sometimes grossly bloody as a result of invasion of 
the mucosa. The infection is also usually self-limited; however, dehydration 
can occur if diarrhea is severe. 

APPROACH TO SUSPECTED Salmonella AND Shigella 
INFECTION 

Objectives 

1. Know the structure, characteristics, and clinical diseases associated 
with salmonella. 

2. Know the virulence, epidemiology, and pathogenesis of salmonella 
infections. 

3. Know the structure, characteristics, and clinical diseases associated 
with shigella. 

4. Know the virulence, epidemiology, and pathogenesis of shigella infections. 

Definitions 

Rose spots: Papular rash usually on the lower trunk leaving a darkening of 

the skin, characteristic of typhoid fever. 
Fecal leukocytes: White blood cells found in the stool, nonspecific finding 

of an invasive process. 
Hepatosplenomegaly: Enlargement of both the liver and the spleen which 

can be a feature of many diseases, including typhoid fever. 

DISCUSSION 

Characteristics of Salmonella and Shigella 

Salmonella are motile, facultative anaerobic, nonspore-forming, gram- 
negative bacilli that are part of the family Enterobacteriaceae. The genus 
Salmonella consists of more than 2400 serotypes capable of infecting almost 
all animal species. However, S. typhi and S. paratyphi only colonize humans. 
Salmonella is protected from phagocytic destruction by two mechanisms: 
an acid tolerance response gene, which protects it both from gastric acid and 
from the acidic pH of the phagosome, and Salmonella-secreted invasion pro- 
teins (Sips or Ssps). These rearrange M-cell actin, resulting in membranes 
that surround and engulf the Salmonella and enable intracellular replication of 
the pathogen with subsequent host cell death. 

Shigella is a nonmotile gram-negative bacilli that is also part of the fam- 
ily Enterobacteriaceae. There are 40 serotypes of Shigella that are divided into 
four groups or species, based on biochemical reactivity. Shigella dysenteriae 



144 CASE FILES: MICROBIOLOGY 

is group A, Shigella flexneri is group B, Shigella boydii is group C, and 
Shigella sonnei is group D. Virulence mechanisms of Shigella include their 
ability to invade the intestinal mucosa and production of shiga toxin, which 
acts to destroy the intestinal mucosa once the organism has invaded the tissue. 
Some strains of Escherichia coli are closely related to Shigella species and are 
also capable of producing shiga toxin. 

Diagnosis 

The diagnosis of gastroenteritis is based on the patient's age, risk factors, 
exposures, and symptoms. Collection of stool and blood cultures, if fever 
and other systemic symptoms are present, is necessary for the definitive diag- 
nosis. A direct exam for fecal leukocytes and occult blood may initially help 
narrow down the differential diagnosis. For example, blood in the stools usu- 
ally indicates invasive bacterial infection. In cases of bacterial gastroenteritis, 
final diagnosis is made by culture of the stool for enteric pathogens such as 
Campylobacter, Shigella, and Salmonella. Although culture of Campylobacter 
requires specialized media and incubation conditions, both Salmonella and 
Shigella grow rapidly on routine microbiologic media. Because of the fact that 
stool contains many organisms that are normal flora, stools are also cultured 
onto selective media to aid in more rapid diagnosis. 

Both Salmonella and Shigella are nonlactose fermenters that appear as clear 
colonies on MacConkey agar. The use of a medium that contains an indicator 
for production of H 2 S helps differentiate the two genera. Shigella does not 
produce H 2 S and appears as clear or green colonies on a media such as 
Hektoen enteric (HE) agar, whereas Salmonella appears black as a result of 
production of H 2 S. This is only presumptive and further biochemical testing 
needs to be performed because other organisms also produce black colonies on 
HE agar. The diagnosis of Shigella can also be made by testing for Shiga 
toxin directly in the stool. This cannot differentiate Shigella from the entero- 
hemorrhagic E. coli that also produce shiga toxin and are associated with 
hemolytic uremic syndrome. 

Treatment and Prevention 

Nontyphoid Salmonella gastroenteritis is usually not treated because it is a 
self-limited disease, and antibiotics have not been shown to alter the course of 
the infection. Primary treatment should be supportive including fluid replace- 
ment if necessary. Antibiotic treatment is recommended for treatment of 
bacteremia, long-term carriers, or typhoid fever. Amoxicillin, sul- 
famethoxazole and trimethoprim (SMX-TMP), or, in areas where antibiotic 
resistant strains are prevalent (India, Asia and Africa), quinolones can be used. 
The specific choice of antimicrobial agent should be based on susceptibility 
testing of the patient's isolate. 



CLINICAL CASES 745 

In the case of infection caused by Shigella, antibiotic therapy has been 

shown to be useful, especially in the prevention of person-to-person spread of 
the disease. Quinolones can also be used to treat, although Shigella therapy 
should be based on antimicrobial susceptibility testing of the isolate. 

Prevention of disease caused by enteric pathogens is based on control of the 
contaminated source in the environment and good personal hygiene. The thor- 
ough cooking of poultry and cooking eggs until the yolk is hard can kill 
Salmonella and prevent infection. A vaccine does exist for prevention of 
typhoid fever, which is useful for travelers to endemic areas of the world. The 
efficacy of the vaccine is thought to be between 50 and 80 percent. 

COMPREHENSION QUESTIONS 

[19.1] In which of the following sites is S. typhi most likely to be found dur- 
ing the carrier state? 

A. Blood 

B. Gallbladder 

C. Kidney 

D. Liver 

E. Spleen 

[19.2] A 4-year-old has fever and diarrhea. Blood culture grows a gram- 
negative rod. This is most likely to be which of the following? 

A. Group B Streptococcus 

B. Listeria species 

C. Salmonella species 

D. Shigella species 

[19.3] Which of the following is a frequent cause of osteomyelitis in patients 
with sickle cell anemia? 

A. Group A Streptococcus 

B. Group B Streptococcus 

C. Salmonella species 

D. Streptococcus pneumoniae 

[19.4] Which of the following is mismatched? 

A. Ecthyma gangrenosum - Pseudomonas aeruginosa 

B. Halophilic - Salmonella typhi 

C. Kl antigen - neonatal meningitis caused by Escherichia coli 

D. Red pigment - Serratia marcescens 

E. Severe dehydration - Vibrio cholerae 



146 CASE FILES: MICROBIOLOGY 

Answers 

[19.1] B. The feces of persons who have unsuspected subclinical disease or 
are carriers is a more important source of contamination than frank 
clinical cases that are promptly isolated. The high incidence of 
Salmonellae in commercially prepared chickens has been widely 
publicized, possibly related to the use of animal feeds containing 
antimicrobial drugs that favor the proliferation of drug-resistant 
Salmonellae and their potential transmission to humans. Permanent 
carriers usually harbor the organisms in the gallbladder or biliary 
tract and, rarely, in the intestine or urinary tract. 

[19.2] C. Enterocolitis is the most common manifestation of Salmonella 
infection. In the United States, S. typhimurium and S. enteritidis are 
prominent, but enterocolitis may be caused by more than 1400 strains 
of Salmonella. Bacteremia is rare {2-A percent) except in immunod- 
eficient persons. Stool cultures may remain positive for Salmonella 
weeks after clinical recovery. Streptococci and Listeria stain gram- 
positive, and Shigella organisms rarely, if ever, enter the blood stream 
from the intestines. 

[19.3] C. Hematogenous infections account for about 20 percent of cases of 
osteomyelitis and primarily affect children, in whom the long bones 
are infected. More than 95 percent of these cases are caused by a sin- 
gle organism, with Staphylococcus aureus accounting for 50 percent 
of the isolates. Group B streptococci and E. coli are common during 
the newborn period and group A streptococci and Haemophilus 
influenzae in early childhood. Salmonella species and S. aureus are 
major causes of long-bone osteomyelitis complicating sickle cell ane- 
mia and other hemoglobinopathies. Septic arthritis may be encoun- 
tered in sickle cell disease with multiple joints infected. Joint 
infection may result from spread of contiguous osteomyelitis areas. 
Salmonella infection is seen more often in osteomyelitis than in sep- 
tic arthritis. 

[19.4] B. Organisms requiring high salt concentrations are called halophilic. 
Usually, this refers to microorganisms that are capable of living or 
surviving in an ocean or saltwater area. Vibrios are especially well 
known for this ability. Salmonella typhi (typhoid fever) multiply in 
intestinal lymphoid tissue and are excreted in stools. They are hardy 
survivors in water sources, but they do not survive in halophilic con- 
ditions as well as Vibrios. 



CLINICAL CASES 747 



_ 



MICROBIOLOGY PEARLS 

Shigella is a common cause of gastroenteritis, which can be bloody 
as a result of the ability of the organism to invade the mucosa. 
Because of the low inoculum required for infection, person-to- 
person transmission may occur in close contacts. 

Salmonella nontyphi is associated with a self-limited diarrhea asso- 
ciated with ingestion of contaminated food products such as 
undercooked eggs. 

Salmonella and Shigella are nonlactose fermenters that are differen- 
tiated in the laboratory by production of H 2 S; the appearance of 
black colonies on Hektoen enteric agar result from Salmonella 
species. 



REFERENCES 

Dupont HL. Shigella species (bacillary dysentery) In: Mandell GL, Bennett JE, 

Dolin R, eds. Principles and Practice of Infectious Diseases, 5th ed. 

Philadelphia, PA: Churchill Livingstone, 2000:2363-69. 
Mandell GL, Bennett, JE, Dolin, R eds. Principles and Practice of Infectious 

Disease. 5th ed. Philadelphia, PA: Churchill Livingstone, 2000:2344-63. 
Miller SI, Pegues DA. Salmonella species, including salmonella typhi. In: 

Mandell GL, Bennett JE, Dolin R, eds. Principles and Practice of Infectious 

Diseases, 5th ed. Philadelphia, PA: Churchill Livingstone, 2000. 
Murray PR, Rosenthal KS, Pfaller MA. Enterobacteriaceae. In: Murray PR, 

Rosenthal KS, Pfaller MA. Medical Microbiology, 5th ed. St. Louis, MO: 

Mosby, 2005:266-80. 



This page intentionally left blank 



♦> CASE 20 



A 59-year-old man with emphysema secondary to a 50-pack-year smoking 
history presents with a fever, chills, chest pain, and cough. He had a "cold" 
with mild cough and congestion for approximately 3 days but then had the 
abrupt onset of more severe symptoms. His temperature has been as high as 
39.4°C (103°F), and he's had shaking chills. His cough is productive of spu- 
tum that looks like "rust." When he coughs or takes a deep breath, he gets a 
sharp, stabbing pain in his left lower chest. He has been taking numerous over- 
the-counter cold medications without relief and has had to use his ipratropium 
inhaler more often than usual. On examination, he is quite ill appearing. His 
temperature is 38.8°C (101.9°F), pulse is 110 beats per minute, blood pressure 
1 10/60 mm Hg, and respiratory rate is 28 breaths per minute. His pulmonary 
examination is significant for the presence of crackles and rhonchi in the left 
lower fields and expiratory wheezing heard in all other fields. His heart is 
tachycardic but otherwise normal on auscultation. The remainder of his exam- 
ination is normal. His white blood cell count is markedly elevated. An elec- 
trocardiogram is normal. A chest x-ray shows a dense infiltration of the left 
lower lobe along with a pleural effusion on the left side. 

^ What would you expect to see on Gram stain of a sputum sample? 

^ What is the likely reservoir from which this patient's pneumonia 
occurred? 



750 CASE FILES: MICROBIOLOGY 

ANSWERS TO CASE 20: Streptococcus 

Summary: A 59-year-old male complains of fever and cough, with "rust" colored 
sputum. A chest x-ray shows a dense infiltration of the left lower lobe and a left 
pleural effusion. 

^ Most likely Gram stain findings: multiple polymorphonuclear 
leukocytes (PMNs) and encapsulated gram-positive cocci in pairs 
and short chains. 

^ Likely reservoir of this infection: colonization of the upper airway 
(naso- or oropharynx) and aspiration into the lower airways. 

CLINICAL CORRELATION 

Streptococci cause a wide range of diseases from localized skin and soft tissue 
infections to systemic infections such as necrotizing fasciitis, endocarditis, and 
arthritis. Streptococcus pyogenes is commonly associated with pharyngitis and 
its sequelae of rheumatic fever and glomerulonephritis, in addition to the skin 
and soft-tissue infections previously mentioned. Streptococcus agalactiae is 
most well known for its association with neonatal meningitis following vagi- 
nal colonization of the pregnant women. 

Streptococcus pneumoniae is a cause of otitis media, sinusitis, bronchitis, 
pneumonia, and meningitis. Streptococcus pneumoniae (pneumococcus) is the 
most frequent cause of bacterial pneumonia, otitis, and meningitis. It commonly 
colonizes the upper airways in humans, more frequently in children than adults. 
Pneumococcal diseases occur when organisms spread from the site of coloniza- 
tion to a distant, susceptible site. Pneumonia occurs when pneumococcus is aspi- 
rated into the distal airways and multiplies in the alveoli. Pneumococcal 
pneumonia typically follows a milder upper respiratory infection. Symptoms of 
pneumococcal pneumonia include cough, fever, chills, and shortness of breath. 
Patients may also have increased white blood cells and anemia. A common com- 
plication of pneumococcal pneumonia is pleural effusion, which occurs in up to 
40 percent of patients. Meningitis either follows sinusitis or otitis or occurs as a 
result of bacteremic spread of the organisms. Patients that are immunocompro- 
mised, elderly, or have underlying heart or lung disease, as well as those that are 
asplenic, are at higher risk than normal for developing serious disease with S. 
pneumoniae. 



CLINICAL CASES 757 

APPROACH TO SUSPECTED PNEUMOCOCCUS 
INFECTION 

Objectives 

1. Know the structure and physiologic features common to the genus 
Streptococcus. 

2. Know the virulence factors, epidemiology, and diseases associated 
with specific Streptococcus species. 

Definitions 

Rhonchi: A vibration of the chest wall that can be felt with the hand and 
sounds like a dull roaring or murmuring. 

Cytokines: Proteins produced by leukocytes that act as mediators of a fur- 
ther inflammatory response. 

DISCUSSION 

Characteristics of Streptococcus 

The genus Streptococcus contains multiple species that are differentiated either 
by their cell wall carbohydrate group antigen, their hemolysis on blood 
agar, or their biochemical reactivity. Not all streptococci, including S. pneu- 
moniae, possess a carbohydrate cell wall antigen. Streptococci are facultative 
anaerobes that require carbon dioxide for growth. Streptococci are gram- 
positive cocci that form either pairs or chains, whereas S. pneumoniae are 
elongated, lancet shaped, gram-positive cocci, and are usually in pairs or 
short chains. 

Virulent strains of pneumococcus are encapsulated by a polysaccharide 
capsule. Strains that are unencapsulated are easily cleared by host defenses. 
Colonization is facilitated by binding of the pneumococcus to epithelial cells by 
surface protein adhesins, producing secretory IgA protease, which prevents 
host immunoglobulin A from binding to it and producing pneumolysin, which 
destroys phagocytic and ciliated epithelial cells by creating pores in their cell 
membranes. Phagocytosis is limited by the antiphagocytic nature of the poly- 
saccharide capsule and by the pneumolysin's inhibition of the oxidative burst 
required for intracellular killing. Much of the tissue damage caused by pneu- 
mococcal infections is mediated by the inflammatory response of host defense 
systems. The complement system is activated by teichoic acid, peptidoglycan 
fragments, and pneumolysin. Cytokine production is stimulated, causing more 
inflammatory cells to migrate to the site of infection. Hydrogen peroxide is 
produced by pneumococcus, which causes tissue damage via reactive oxygen 
intermediates. 



752 CASE FILES: MICROBIOLOGY 

Antibiotic resistance in pneumococcus is an increasing problem. 
Penicillin resistance has developed, primarily via mutations in penicillin- 
binding proteins in the cell wall. This is a consequence of mutations in the cel- 
lular DNA and from acquisition of DNA from both other pneumococci and 
other bacteria with which pneumococcus comes in contact. Efflux pumps also 
confer some degree of resistance to antibiotics. 

Diagnosis 

Diagnosis of pneumococcal pneumonia is made based on clinical signs and 
symptoms, chest x-ray demonstrating infiltration of a single lobe, and sputum 
Gram stain with many PMNs and gram-positive cocci in pairs and chains. 
Confirmation of the diagnosis can be made by culturing the organisms from 
the sputum and/or blood. Streptococcus pneumoniae grows rapidly on routine 
laboratory media including blood and chocolate agar. Colonies on blood agar 
demonstrate P-hemolysis (green color) and may be slightly to extremely 
mucoid because of their polysaccharide capsule. Colonies are differentiated 
from viridans streptococci by sensitivity to optochin and bile solubility. 
Although optochin susceptibility is considered definitive, the addition of bile 
to a colony will identify the organism as S. pneumoniae if the colony lyses and 
disappears within a few minutes. 

More rapid diagnosis of pneumococcal pneumonia can be made using the 
urinary antigen test. 

Treatment and Prevention 

Treatment of uncomplicated pneumonia is usually with either a quinolone or a 
macrolide such as azithromycin. Complicated or disseminated pneumococcal 
disease is usually treated with penicillin or cefotaxime depending on suscepti- 
bility of the isolate to penicillin. Treatment of the other streptococcal species is 
usually with penicillin, but in serious infections should be based on the individ- 
ual isolate susceptibility. Adult and pediatric vaccines directed against pneu- 
mococcal capsular antigens are available, and current guidelines recommend 
universal vaccination of children, persons over the age of 65, and others at high 
risk for pneumonia, such as persons with diabetes or chronic lung disease. 

COMPREHENSION QUESTIONS 

[20.1] A newborn has a temperature of 39.4°C (103°F). Blood culture grows 
gram-positive cocci in chains. This is most likely to be which of the 
following? 

A. Group A Streptococus (Streptococcus pyogenes) 

B. Group B Streptococcus (Streptococcus agalactiae) 

C. Salmonella species 

D. Streptococcus pneumoniae 



CLINICAL CASES 753 

[20.2] A 3-year-old is diagnosed with bacterial meningitis. Cerebrospinal 
fluid grows out gram-positive cocci in short chains and diplococci. 
This is most likely to be which of the following? 

A. Group B Streptococcus 

B. Salmonella 

C. Staphylococcus aureus 

D. Streptococcus pneumoniae 

[20.3] Which of the following is the primary virulence factor of S. pneumoniae? 

A. Bile solubility 

B. Optochin production 

C. Pili 

D. Polypeptide capsule 

E. Polysaccharide capsule 

[20.4] Which of the following is true regarding meningitis with S. pneumoniae? 

A. Cephalosporins are always effective. 

B. One desires a concentration of antibiotics in the cerebral spinal 
fluid 10 times the minimal inhibition concentration. 

C. Penicillin is always effective. 

D. Resistance is not increasing in S. pneumoniae. 

Answers 

[20.1] B. Most human infections caused by streptococci involve the group A 
organisms (S. pyogenes). The group B streptococci are members of the 
female genital tract and are important causes of neonatal sepsis and 
meningitis. They are usually (3-hemolytic (similar to group A), hydrolyze hip- 
purate and give a positive response in the so-called CAMP test (Christie, 
Atkins, Munch-Peterson). Detection of the infection and prompt 
antimicrobial treatment is necessary because the infections may become 
life-threatening. Streptococcus pneumoniae organisms are important 
in meningitis cases in young children, but are more frequently seen as 
diplococci forms rather than long chains. 

[20.2] D. Streptococcus pneumoniae is responsible for 10-20 percent of 
meningitis cases in children ages 1 month to 15 years. Neisseria 
meningitidis range from 25 to 40 percent, whereas Haemophilus 
influenzae may be involved in 40-60 percent. Group A and B strep- 
tococci appear to be involved only 2-A percent of the time. Under the 
conditions described above, S. pneumoniae would be the most likely 
etiologic agent. 

[20.3] E. Bile solubility and optochin sensitivity are presumptive identifica- 
tion tests that identify S. pneumoniae from other a-hemolytic strep- 
tococci. The polysaccharide capsule occurs in dozens of antigenic 
types, but types 1-8 are responsible for approximately 75 percent of 



154 CASE FILES: MICROBIOLOGY 

the cases of pneumococcal pneumonia. Vaccines are available that 
give approximately 90 percent protection and usually contain 23 
types of carbohydrates for the United States-licensed preparation. 

[20.4] B. Because pneumococci are sensitive to many antimicrobial drugs, 
early treatment usually results in rapid recovery. Antibody response 
(host's active immunity) seems to play a diminished role today. 
Penicillin G is the drug of choice, but 5-10 percent of the isolates in 
the United States are penicillin resistant (MIC >2 ji,g/mL), and 20 
percent are moderately resistant (0.1-1 (i.g/mL). Resistance to 
cephalosporins, tetracycline, and erythromycin has been demon- 
strated, although pneumococci remain susceptible to vancomycin. In 
reference to penicillin therapy, one rule of thumb is to aim for a con- 
centration of 10 times the MIC in the CSF. 



MICROBIOLOGY PEARLS 



*♦* Streptococcus pneumoniae is a common cause of otitis media and 
meningitis. 

*♦* Because of the increasing incidence of penicillin resistance of 
S. pneumoniae empiric therapy of disseminated disease is with 
ceftriaxone. 
Streptococcus pneumoniae is an a-hemolytic streptococci suscepti- 
ble to optochin. 



REFERENCES 

Murray PR, Rosenthal KS, Pfaller MA. Streptococcus. In: Murray PR, Rosenthal 

KS, Pfaller MA. Medical Microbiology, 5th ed. St. Louis, MO: Mosby, 

2005:237-58. 
Musher, DM. Streptococcus pneumoniae. In: Mandell GL, Bennett JE, Dolin R, 

eds. Principles and Practice of Infectious Diseases, 6th ed. Philadelphia, PA: 

Churchill Livingstone, 2005:2392-2411. 



♦ CASE 21 



While on call on a Saturday night in July, you receive a call from the mother 
of a 15 -year-old man who developed the acute onset of nausea, vomiting, and 
diarrhea shortly after returning from an outdoor party that was held at the 
home of a friend. At the party, a picnic lunch of hamburgers, hot dogs, potato 
salad, baked beans, and lemonade was served. The food was served on an out- 
door picnic table, and the guests were free to eat at any time during the party. 
None of the food tasted spoiled or tainted. His symptoms started abruptly 
about an hour after he returned home, which was approximately 4 hours after 
he had eaten. He currently is unable to keep down anything. He does not have 
a fever and has not passed any blood in his stool or vomitus. Prior to calling 
you, your patient's mother spoke with the hostess of the party, who said that 
she had heard from three other guests who became ill with similar symptoms. 

^ What organism is the most likely cause of this patient's illness? 

^ Your patient's mother requests that you call in a prescription for an 
antibiotic to treat the infection. What is your response? 



156 CASE FILES: MICROBIOLOGY 

ANSWERS TO CASE 21: STAPHYLOCOCCI 

Summary: A 15-year-old male with gastroenteritis after eating food at an out- 
door picnic. Several other participants also developed similar symptoms. 

^ Most likely organism causing this infection: Staphylococcus aureus. 

^ Response to request to treat with antibiotic: No, the gastroenteritis 
is caused by a preformed toxin, not by the ingested staphylococci, 
therefore antibiotic therapy would be of no help. 

CLINICAL CORRELATION 

Staphylococcus aureus is a common colonizer of the human nasopharynx and 
skin. Infection occurs when the normal skin barrier is disrupted by either sur- 
gery or trauma. 

Staphylococcus aureus causes numerous infections, ranging from simple 
localized skin and soft tissue infections to disseminated disease, such as bac- 
teremia, endocarditis, osteomyelitis, and septic arthritis. Many of the infec- 
tions caused by S. aureus are toxin mediated, such as toxic shock syndrome, 
scalded skin syndrome, and gastroenteritis. 

Previously thought to be a nosocomial infection seen in hospitalized 
patients, community acquired MRSA has become a significant public health 
concern. The majority of these strains produce a toxin called Panton- 
ValentineLeukocidin (PVL) that is associated with more severe disease, 
including skin and soft tissue infections and necrotizing pneumonia. 

Staphylococcal food poisoning, the second most reported cause of food 
poisoning in the United States, is a result of the presence of enterotoxin. Food 
is contaminated by a human carrier, with processed meats, custard-filled baked 
goods, potato salad, or ice-cream being common vectors. The toxin rapidly 
produces nausea, vomiting, and diarrhea, usually within 2-6 hours of inges- 
tion. Further toxin is not produced by the ingested S. aureus, and the disease 
also rapidly resolves, usually within 12-24 hours. 

Staphylococcal species also frequently colonize human skin but can cause 
disease in certain situations. Although there are more than 20 other species, the 
majority of the species isolated are Staphylococcus epidermidis. The most 
common predisposing factor for disease with staphylococci not (S. aureus) is 
the presence of artificial devices in the patient such as catheters and replace- 
ment joints. Staphylococcus epidermidis produces a slime that allows it to 
adhere to plastics and form a biofilm that makes it very difficult for antibiotics 
to penetrate. 



CLINICAL CASES 757 

APPROACH TO SUSPECTED STAPHYLOCOCCAL 
INFECTION 

Objectives 

1 . Know the structure, physiology, and virulence factors associated with 
S. aureus and the coagulase-negative staphylococci. 

2. Know the diseases caused by staphylococci and the mechanisms by 
which staphylococci develop antibiotic resistance. 

Definitions 

Biofilm: Bacteria grow on an artificial surface and form a conglomerate 

with secreted polysaccharides and glycopeptides. 
Superantigens: Antigens, most often bacterial toxins, that recruit a large 

number of T lymphocytes to an area. 
Enterotoxins: Substances produced by bacteria that are toxic to the GI 

tract that cause diarrhea and/or vomiting. 

DISCUSSION 

Characteristics of Staphylococcus 

Staphylococci belong to the family Micrococcaceae, which includes the genus 
Micrococcus in addition to Staphylococcus. Staphylococci grow rapidly on 
multiple culture media, in a wide range of environments, including up to 10 
percent sodium chloride, and in a broad range of temperatures. Staphylococcus 
aureus is a nonmotile, nonspore forming, facultative anaerobic gram- 
positive coccus that commonly colonizes healthy humans and is a frequent 
cause of disease. It is frequently identified as growing in clusters or clumps. 
This is a result of the effect of bound coagulase ("clumping factor"), which 
binds fibrinogen, converts it to insoluble fibrin, and results in aggregation. 
Staphylococcus aureus is the only Staphylococcus found in humans which 
produces coagulase; other staphylococcal species are commonly identified as 
coagulase-negative staphylocci. 

Staphylococcus aureus produces at least five cytolytic toxins, two exfo- 
liative toxins, eight enterotoxins, and toxic shock syndrome toxin. Some of 
these toxins act as superantigens, which recruit host defense cells that liberate 
cytokines and, therefore, produce systemic effects. Heating will kill the S. aureus 
organisms, but not inactivate the enterotoxins, because they are stable to 
heating at 100°C (112°F) for 30 minutes and are resistant to breakdown by 
gastric acids. 

Of growing public health concern is the rapid spread of antibiotic resist- 
ance within S. aureus isolates. Almost all S. aureus produces penicillinase, a 
P-lactamase specific for penicillin. Many isolates have also acquired a gene 



158 CASE FILES: MICROBIOLOGY 

that codes for an altered penicillin binding protein, PBP2, providing antibi- 
otic resistance to semisynthetic penicillins and cephalosporins as well, includ- 
ing methicillin and nafcillin. Some of these genes will also be associated with 
resistance to non-p-lactamase antibiotics, such as quinolones and macrolides. 
Some S. aureus isolates have been identified recently with reduced sensitivity 
to vancomycin. The mechanism of this resistance is unknown. Genes that con- 
fer resistance can be transferred between organisms by plasmid transfer, trans- 
duction and cell-to-cell contact. 

Diagnosis 

The initial diagnosis of staphylococcal infection may be difficult because 
many of the skin and soft tissue infections mimic those of streptococci. 
Definitive diagnosis is made by Gram stain and culture of the infected site as 
well as blood. Staphylococci are large gram-positive cocci grouped in clus- 
ters (Figure 21-1). Staphylococci grow rapidly on routine laboratory media. 




Figure 21-1. Gram stain of Staphylococcus aureus showing gram-positive 
cocci in clusters. (Reproduced, with permission, from Brooks G, ButelJ, Morse 
S. Jawetz, Melnick, and Aldelburg 's Medical Microbiology, 23rd ed. New York: 
McGraw-Hill, 2004:224.) 



CLINICAL CASES 159 

Their colony morphology is different from streptococci in that the colonies are 
larger, white or yellow instead of grey. They also can be differentiated from 
streptococci by a positive catalase test (reactivity with hydrogen peroxide). 
Staphylococcus aureus is f}-hemolytic on blood agar medium and is differ- 
entiated from the other Staphylococcus species by production of coagulase or 
positive latex agglutination for Staphylococcus protein A. Further confirma- 
tion of the identification of S. aureus is not necessary; however, many com- 
mercially available identification systems can identify the organism based on 
biochemical reactivity. A selective media such as mannitol salts agar, which 
also differentiates S. aureus from other staphylococcal species is available, but 
not often used in clinical laboratories. 

Staphylococcal gastroenteritis is usually self-limited with symptoms dis- 
appearing within 12 hours, and therefore diagnosis is made clinically based on 
incubation period and history of others eating similar foods with same symp- 
toms. Staphylococcus saprophyticus is the only other staphylococcal species 
that is identified as a consequence of its association with urinary tract infec- 
tions in young women. Staphylococcus saprophyticus is differentiated from 
the other coagulase negative staphylococci by its susceptibility to novobiocin, 
which is tested by disk diffusion. Staphylococcus lugdunensis has recently 
gained attention as a significant cause of bacteremia and endocarditis. These 
organisms look morphologically like Staphylococcus epidermidis, but clini- 
cally resemble Staphylococcus auereus. Their distinguishing feature is that 
they are PYR positive. 

Treatment and Prevention 

Treatment of local wound infections without systemic symptoms does not usu- 
ally require treatment with antibiotics; however, in the cases of more compli- 
cated infections or presence of fever, antimicrobial therapy is usually 
warranted. Although nafcillin is the drug of choice for staphylococcal infec- 
tions, because of the high percentage of strains that are resistant to methicillin 
and nafcillin, initial treatment is usually with vancomycin until the suscepti- 
bility results are available. Oral antibiotics, such as dicloxacillin, rifampin and 
sulfamethoxazole and trimethoprim (SMX-TMP) or clindamycin can also be 
used dependent on the susceptibility of the isolate. Treatment of 
Staphylococcus non-aureus is with vancomycin, because the majority of iso- 
lates are resistant to nafcillin. 

Control of S. aureus involves strict adherence to hand washing policies, 
particularly in the hospital setting. The organism can easily be spread from 
person to person. Colonization with S. aureus is usually transient; however, an 
attempt can be made in some situations to decolonize the nares by using 
intranasal mupirocin and/or the skin by using oral anti-staphylococcal antibi- 
otics in combination with topical agents. 



760 CASE FILES: MICROBIOLOGY 

COMPREHENSION QUESTIONS 

[21.1] A 12-year-old girl was playing soccer when she began to limp. She has 
pain in her right leg and right upper thigh. Her temperature is 38.9°C 
(102°F). X-ray of the femur reveals that the periosteum is eroded, sug- 
gestive of osteomyelitis. Blood culture yields gram-positive bacteria. 
The most likely etiologic agent is which of the following? 

A. Listeria monocytogenes 

B. Salmonella enteritidis 

C. Staphylococcus aureus 

D. Staphylococcus saprophyticus 

E. Streptococcus pneumoniae 

[21.2] An outbreak of staphylococcal infection involving umbilical cords of 
seven newborn babies was reported in the nursery. Bacteriologic sur- 
vey reveals that two nurses have a large number of S. aureus in the 
nasopharynx. What test should be performed to determine whether 
these nurses may have been responsible for the outbreak? 

A. Bacteriophage typing 

B. Coagulase testing 

C. Nasopharyngeal culture on mannitol salt agar 

D. Protein A typing 

E. Serologic typing 

[21.3] Virulence factors of S. aureus include all of the following except one. 
Which one is this exception? 

A. [3-lactamases 

B. Coagulase 

C. Enterotoxins 

D. M Protein 

E. Protein A 

[21.4] Short incubation food poisoning, caused by ingestion of preformed 
en tero toxin, is caused by which bacteria listed below? 

A. Staphylococcus aureus 

B. Staphylococcus epidermidis 

C. Enterococcus faecalis 

D. Streptococcus pneumoniae 

E. Streptococcus pyogenes 



CLINICAL CASES 161 

Answers 

[21.1] C. Staphylococci, especially S. epidermidis, are normal flora of the 
human skin and respiratory and gastrointestinal tracts. Nasal carriage 
of S. aureus, the pathogen, occurs in 20-50 percent of humans. 
Abscesses are the typical lesion of S. aureus. From any one focus, 
organisms may enter the bloodstream and lymphatics to spread to 
other parts of the body. In osteomyelitis, the primary focus is gener- 
ally in a terminal blood vessel of the metaphysis of a long bone, 
which may lead to necrosis of bone and chronic suppuration. 
Staphylococcus saprophyticus is usually a nonpathogenic normal 
flora organism. Listeria is usually transmitted in unpasteurized dairy 
products, whereas Salmonella enteritidis is primarily intestinal. 
Streptococcus pneumoniae is primarily a respiratory pathogen, although 
it is an important central nervous system pathogen in children. 

[21.2] A. Bacterial viruses (bacteriophages or phages) can attach to separate 
receptors on the cell walls of various strains of S. aureus. Different 
specific receptors have been identified and used as the basis of epi- 
demiologic typing of pathogenic S. aureus strains. Typical cultures 
from the outbreak and strains obtained from personnel can be sub- 
jected to a standardized procedure using a series of bacteriophages 
that attack S. aureus strains. This procedure can readily identify the 
source of the outbreak organism if it came from a medical care 
worker. 

[21.3] D. M proteins are virulence factors of group A streptococci (S. pyo- 
genes). All of the other listed virulence factors may be found rou- 
tinely in S. aureus bacteria. 

[21.4] A. Of the options given the best answer is S. aureus, as a result of 
enterotoxin production in food. None of the other strains listed pro- 
duce enterotoxins that result in short-term gastroenteritis. 



MICROBIOLOGY PEARLS 

*♦* Staphylococcus aureus is a common cause of community-acquired 

and nosocomial wound infections. 
*♦* Treatment of S. aureus is with nafcillin if the isolate is susceptible, 

or alternatively with vancomycin. 
*♦* Staphylococcus aureus is differentiated from the other staphylococ- 



cal species by production of coagulase. 



162 CASE FILES: MICROBIOLOGY 



REFERENCES 

Murray PR, Rosenthal KS, Pfaller MA. Staphylococcus and related organisms In: 
Murray PR, Rosenthal KS, Pfaller MA. Medical Microbiology, 5th ed. St. Louis, 
MO: Mosby, 2005:202-16. 

Moreillon P, Que Y, Glauser MP. Staphylococcus aureus (including staphylococ- 
cal toxic shock). In: Mandell GL, Bennett JE, Dolin R, eds. Principles and 
Practice of Infectious Diseases, 6th ed. Philadelphia, PA: Churchill Livingstone, 
2005:321-51. 



♦ CASE 22 



A 20-year-old man presents for evaluation of a rash that he thinks is an aller- 
gic reaction. For the past 4 or 5 days he has had the "flu," with fever, chills 
headache, and body aches. He has been taking an over-the-counter flu med- 
ication without any symptomatic relief. Yesterday he developed a diffuse rash 
made up of red, slightly raised bumps. It covers his whole body, and he says 
that it must be an allergic reaction to the flu medication. He has no history of 
allergies and takes no other medications, and his only medical problem in the 
past was being treated for gonorrhea approximately 2 years ago. On further 
questioning, he denies dysuria or penile discharge. He denies any genital 
lesions now, but says that he had a "sore" on his penis a few months ago that 
never really hurt and went away on its own after a few weeks so he didn't think 
much about it. On exam, his vital signs are all normal. He has palpable cervi- 
cal, axillary, and inguinal adenopathy. His skin has an erythematous, macu- 
lopapular eruption covering his whole body including his palms and soles of 
his feet. No vesicles are noted. His genital examination is normal. 

^ What organism is the likely etiology of this disease? 

^ What disease and stage does this patient have? 

^ What microscopic examination could confirm this diagnosis? 

^ Which serologic tests could assist in his diagnosis? 



164 CASE FILES: MICROBIOLOGY 

ANSWERS TO CASE 22: Treponema pallidum 

Summary: A 20-year-old man has adenopathy and a macular papular rash 
affecting his soles and palms. He had a painless penile "sore" that sponta- 
neously resolved. 

^ Most likely causative organism: Treponema pallidum. 

^ Disease and stage: The patient has syphilis, more specifically 
secondary syphilis. 

^ Microscopic examination to confirm the diagnosis: Examination by 
darkfield microscopy of exudates from skin lesions could confirm the 
diagnosis of T. pallidum infection and secondary syphilis. 

^ Serologic tests to assist in the diagnosis: The following serologic 
examinations are useful in diagnosis: Venereal Disease Research 
Laboratory (VDRL) and rapid plasmin reagin (RPR) tests are used for 
initial screening, while the fluorescent treponemal antibody-absorption 
test (FTA-ABS ), and the microhemagglutination test for T. pallidum 
(MHA-TP), are the more specific diagnostic tests. 

CLINICAL CORRELATION 

Treponema pallidum is a gram-negative, microaerophilic spirochete that 
causes venereal syphilis, the third most common bacterial sexually transmitted 
disease in the United States. It is transmitted by contact with fluid from an 
ulcer containing the infectious agent either through sexual contact by pene- 
trating intact mucous membranes or through nonsexual contact with the agent 
with skin that is broken or abraded. Studies estimate that transmission occurs 
in over half of sexual encounters where a lesion is present. Treponema pal- 
lidum infection results in multiple disease phases with distinctive clinical man- 
ifestations. Primary syphilis usually involves the formation of a painless ulcer 
at the site of entry of the organism, called a chancre. Chancres are highly con- 
tagious by contact and can spontaneously heal after a few weeks to a few 
months. Secondary syphilis develops 2-12 weeks after the primary stage and 
is characterized by a flu-like illness, followed by a rash that typically starts on 
the trunk but can spread to any skin or mucous membrane surface. Without 
treatment, the symptoms generally resolve in 3-12 weeks. This is followed by 
a relatively asymptomatic period known as latency, which can last for years. 
Some infected persons have no further symptoms; however, some progress to 
tertiary syphilis, a diffuse disease with many effects on the dermatologic, mus- 
culoskeletal, cardiovascular, and central nervous systems. Currently the popu- 
lation most at risk is heterosexual African Americans living in urban areas. 



CLINICAL CASES 765 

APPROACH TO SUSPECTED Treponema pallidum 
INFECTION 

Objectives 

1. Know the natural history of syphilis infection. 

2. Know the methods of diagnosis and treatment of syphilis. 

Definitions 

Macule: Flat lesion that is not palpable, of a different color from sur- 
rounding skin and smaller than 1 cm. 

Microaerophilic: Organisms that can tolerate small amounts of oxygen 
because they contain superoxide dismutase. They use fermentation in 
the absence of oxygen. 

Tabes dorsalis: A condition characterized by diminished vibratory, propri- 
oceptive, pain, and temperature senses, as well as the loss of reflexes. 

Argyll Robertson pupil: Constricts during accommodation but does not 
react to light. 

DISCUSSION 

Characteristics of Treponema pallidum That Impact 
Transmission 

Treponema pallidum is a thin spirochete and an obligate human pathogen. 

It consists of three subspecies, each of which causes disease in humans. 
Treponema pallidum is labile, unable to survive exposure to drying, and is very 
difficult to grow in culture. Treponema pallidum does not have a capsule and 
usually contains six axial filaments, located between the outer membrane and 
the peptidoglycan layer. It produces no toxins that have been currently identi- 
fied. Treponema pallidum is too thin to be seen with standard microscopy with 
Gram stain but can be seen with darkfield microscopy or by staining with 
antitreponemal antibodies labeled with fluorescent dyes. Treponema pallidum 
is transmitted by direct contact with an infectious lesion, transfusion of 
infected blood, or congenital transfer. It attaches by one or both ends to host 
cells, although it rarely penetrates the cell. The resultant disease of syphilis 
occurring primarily because of the host immune response to the treponemal 
infection, with both humoral and cell-mediated immune systems playing a 
role. 

Syphilis disease presents in three different stages with characteristics spe- 
cific to each stage. Primary syphilis presents with a hard, painless, broad- 
based chancre. The chancre has a punched-out base and rolled-up edges, 
sometimes expelling a serous exudate. This primary lesion presents 3-6 weeks 



166 



CASE FILES: MICROBIOLOGY 



after the initial contact with the infectious agent. It typically resolves in 4-6 
weeks and does not leave scar tissue. Secondary syphilis presents with a sym- 
metrical widely distributed macular rash. The rash can infect the mucous 
membranes including the cervix, throat, and mouth. It may also appear on the 
palms and soles of the patient, an important clinical finding because there are 
few diseases that present with a rash on palms and soles. Patchy hair loss is 
also seen, typically causing the eyebrows to fall out. There is usually a low- 
grade fever, weight loss, and general malaise. Condyloma latum is a painless, 
wart-like lesion on the scrotum or vulva that may also be present during this 
stage (Figure 22-1). Secondary syphilis occurs several weeks after the lesion 
of primary syphilis has healed. It is during the secondary stage where syphilis 
is considered to be most infectious. 

After secondary syphilis, there is a latent period where the disease is not 
infectious, although the patient is still seropositive. This stage can range from 
2 years to several decades. Tertiary syphilis can present with personality 




Figure 22-1. Genital condylomata lata of secondary syphilis. (Reproduced, 
with permission, from Cunningham FG et al. William's Obstetrics, 21st ed. 
New York: McGraw-Hill, 2001:1487.) 



CLINICAL CASES 767 

changes, blindness, paresis, gummas, Argyll Robertson pupils and tabes 
dorsalis. Gummas are granulomatous lesions of the skin and bone which are 
necrotic and fibrotic. Tabes dorsalis is characterized by diminished vibratory, 
proprioceptive, pain, and temperature sense, as well as the loss of deep tendon 
reflexes. It is the damage to the dorsal roots and ganglia which cause the loss 
of reflexes, pain, and temperature sense, while the loss of proprioception and 
vibratory sense are because of the posterior column involvement. The Argyll 
Robertson pupil constricts during accommodation but does not react to light. 

Diagnosis 

The diagnosis of syphilis can be made by identification of spirochetes by 

darkfield microscopy of a chancre or skin lesion sample of the primary and 
secondary stages, respectively; however, most syphilis is diagnosed by sero- 
logic studies. 

There are several serologic laboratory tests that may be used to detect 
syphilis. The VDRL and RPR are nonspecific tests of host production of anti- 
cardiolipin antibody. These will be positive in approximately 80 percent of 
cases of primary and all secondary stages of syphilis. False-positive VDRLs 
may be encountered in patients with lupus, infectious mononucleosis, hepati- 
tis A, the antiphospholipid antibody syndrome, leprosy, malaria, and occa- 
sionally pregnancy. False-negative RPR and VDRLs may be obtained early in 
the disease. The more specific treponemal tests, the FTA-ABS and the TP-PA, 
are used for confirmation of infection. They detect the presence of antibodies 
specific to T. pallidum. 

Treatment and Prevention 

The drug of choice for the treatment of syphilis is benzathine penicillin. One 
injection is given when the infection is less than 1 year duration, whereas 
injections each week for 3 weeks is administered for infection longer than 1 year. 
Patients who are allergic to penicillin are treated with erythromycin and doxy- 
cycline. However, doxycycline is contraindicated in patients who are pregnant, 
because it can cross the placenta and is toxic to the fetus. Universal precau- 
tions used in the clinical setting are adequate to prevent the transmission of 
syphilis. Outside of the clinical setting, safe sex should be practiced to prevent 
the transmission through sexual contact. Currently, there is no vaccine avail- 
able for the prevention of T. pallidum infection. 



168 CASE FILES: MICROBIOLOGY 

COMPREHENSION QUESTIONS 

[22.1] A 21 -year-old Asian woman visits her obstetrician and is later diag- 
nosed with secondary syphilis. On further questioning, it is determined 
she is allergic to penicillin. Because T. pallidum is known to cross the 
placenta, treatment is started immediately. Which antibiotic would be 
most appropriate in this situation? 

A. Tetracycline 

B. Ceftriaxone 

C. Penicillin G 

D. Doxycycline 

E. Erythromycin 

[22.2] A 27-year-old white man presents to his family doctor complaining of 
being tired all the time and having a slight fever for the past 2 weeks. 
He recently returned from a trip to Las Vegas, where he indulged in 
some of the infamous nightlife. His physical exam is unremarkable 
except for a macular rash over his trunk and on the palms of his hands. 
There are no lesions or ulcers on the penis. What organism is causing 
this man's illness? 

A. Chlamydia trachomatis 

B. Neisseria gonorrhea 

C. Treponema pallidum 

D. Borrelia burgdorferi 

E. Rickettsia rickettsii 

[22.3] A sample is taken from a vulvar ulcer in a 25-year-old sexually active 
African American female. The organism is a weakly staining gram- 
negative, microaerophilic organism. When attempting to view a smear 
under a microscope, no organisms are seen. Which method of visuali- 
zation is most appropriate in this setting? 

A. Ziehl-Neelsen stain 

B. India ink preparation 

C. Congo red stain 

D. Darkfield microscopy 

E. Giemsa stain 

[22.4] A third-year medical student is on his first rotation in internal medi- 
cine. His attending physician points out that there are several tests that 
are used to diagnose syphilis. Which test is most specific for the detec- 
tion of syphilis? 

A. Rapid plasmin reagin (RPR) 

B. Fluorescent treponemal antibody-absorption (FTA-ABS) 

C. Venereal Disease Research Laboratory (VDRL) 

D. Ziehl-Neelsen stain 

E. Aerobic and anaerobic blood cultures 



CLINICAL CASES 169 

[22.5] A 28-year-old sexually active woman presents for her annual well- 
woman exam. She at times has a low-grade fever and lately has noticed 
a rash on her face, mainly on the cheeks. She is saddened to learn she 
has a positive VDRL test for syphilis. However, she is asymptomatic 
for syphilis and is in a monogamous relationship with her husband who 
has not had any other sexual contacts. Which of the following is the 
most likely reason for the positive syphilis test? 

A. She has secondary syphilis. 

B. She has HIV, altering her immune reaction. 

C. She had exposure to syphilis earlier this week. 

D. She has systemic lupus erythematosus (SLE). 

E. She has Chlamydia. 

Answers 

[22. 1] E. Erythromycin and doxycycline are used when allergy to penicillin 
is present. Doxycycline is contraindicated in pregnant women 
because it crosses the placenta and is toxic to the fetus. Ceftriaxone 
and tetracycline are not used for the treatment of syphilis. Penicillin 
is contraindicated because of the patient's allergies. 

[22.2] C. Treponema pallidum is usually transmitted through unprotected 
sexual activity with an infected individual. This man presents with 
the symptoms of secondary syphilis, which includes malaise, mild 
fever, and rash on the palms or soles of the feet. The primary lesion 
(chancre) may go unnoticed because it is painless and subsides in a 
few weeks. Neisseria gonorrhea is associated with a serous exudate. 
Chlamydia is associated with painful urination. Rickettsia rickettsii 
and Borrelia burgdorferi are associated with arthropod vectors. 

[22.3] D. The organism present is T. pallidum, a spirochete. No organisms are 
seen under light microscopy because spirochetes are too small to be 
visualized by this technique. Use of darkfield microscope allows for 
visualization of the corkscrew morphology. The Ziehl-Neelsen stain is 
used to detect acid-fast bacteria such as mycobacteria. India ink prepa- 
rations are used to visualize a capsule that is present in Cryptococcus 
neoformans but that spirochetes do not have. Giemsa stain is used to 
detect Borrelia, Plasmodium, Trypanosomes, and Chlamydia species. 

[22.4] B. There are two classes of test used to detect the presence of an 
infection. The nontreponemal tests detect the presence of antibodies 
against lipids present on the organism. The nontreponemal tests 
include RPR and VDRL. Specific tests that detect antibodies against 
the organism itself, include TP-PA and FTA-ABS. Aerobic and 
anaerobic cultures are not specific tests used to identify syphilis. The 
Ziehl-Neelsen stain is used to identify acid fast bacteria. 



770 CASE FILES: MICROBIOLOGY 

[22.5] D. In the presence of a woman with no known contacts with syphilis 
and a low-grade fever and rash, it is most likely that she had a false- 
positive reaction to the VDRL test because of lupus (SLE). This is 
often a common finding in lupus patients, and may be the first sign 
that they have lupus. In contrast, the VDRL test would be positive in 
secondary syphilis, often in high titer (greater than 1:32). Her being 
positive for HIV, while she may also have a false-positive reaction to 
the VDRL test if HIV positive, is not the most likely answer choice 
for this patient. The presence of the malar rash makes SLE more 
likely. Recent exposure to syphilis would lead to a false-negative test 
result; antibodies form between 4 and 8 weeks from exposure. 
Chlamydia trachomatis infection would not lead to a positive test 
result for syphilis. 



MICROBIOLOGY PEARLS 

*♦* Some of the nontreponemal nonspecific tests are the VDRL and 

RPR. 
*♦* The specific treponemal tests include the FTA-ABS and TP-PA. 
*♦* Primary syphilis generally consists of a painless chancre. 
*♦* Secondary syphilis consists of a generalized macular popular rash 

especially affecting the palms or soles, or condyloma latum. 
*♦* Tertiary syphilis is typified by gummas, neurosyphilis, tabes dor- 

salis, Argyll Robertson pupil. 
*♦* The best treatment for syphilis is penicillin. 



REFERENCES 

Murray PR, Rosenthal KS, Pfaller MA. Medical Microbiology, 5th ed. St. Louis, 

MO: Mosby, 2005:339-45. 
Ryan JR, Ray CG. Sherris Medical Microbiology, 4th ed. New York: McGraw-Hill, 

2004:422-30. 
Schneider AS, Szanto PA. Pathology: Board Review Sseries, 2nd ed. Philadelphia, 

PA: Lippincott Williams & Wilkins, 2001:272, 278, 281. 



♦> CASE 23 



A 35-year-old woman presents to the emergency room with a 2-day history of 
severe diarrhea and vomiting. Her symptoms started shortly after returning 
from a mission trip that she took with her church to a rural area in central 
Africa. She recalls eating shrimp that seemed undercooked. Her symptoms 
started abruptly, with watery diarrhea followed by vomiting. She has not had 
a fever and denies abdominal pain. On examination, her temperature is 37.2°C 
(98.9°F), pulse is 115 beats per minute, and blood pressure is 80/50 mm Hg. 
Her mucous membranes are dry, and her eyes appear sunken. Her skin is dry 
and tents when lightly pinched. Her abdomen has hyperactive bowel sounds 
but is soft and nontender. Her stool is watery and tests negative for blood. 
A complete blood count shows an elevated white blood cell count and an ele- 
vated hematocrit. A metabolic panel shows hypokalemia, low serum bicar- 
bonate, and prerenal azotemia. You assess this patient to be in hypovolemic 
shock and metabolic acidosis, and institute appropriate therapy. 

^ What organism is most likely to be identified on stool culture? 

^ What is the cause of this patient's diarrhea? 



172 CASE FILES: MICROBIOLOGY 

ANSWERS TO CASE 23: Vibrio cholerae 

Summary: A 35-year-old woman recently traveled to Africa and developed 
diarrhea causing hypovolemic shock and metabolic acidosis. She remembers 
eating undercooked shrimp. 

^ Most likely etiologic agent: Vibrio cholerae 

^ Cause of the diarrhea: Hypersecretion of water and electrolytes into 
the intestinal lumen caused by cholera toxin 

CLINICAL CORRELATION 

The first priorities as with any patient are the ABCs: airway, breathing, circu- 
lation. This patient is in hypovolemic shock, meaning insufficient circulation 
to maintain tissue perfusion needs. The most important step in intervention is 
volume repletion, usually with intravenous isotonic saline solution. A likely 
therapy would be 3 L of normal saline intravenously. 

Vibrio species are found in saltwater and infections usually occur in the 
spring and summer. Transmission is by either consumption of contaminated 
shellfish or traumatic injury associated with infected water. The disease 
cholera is caused by toxigenic strains of V. cholerae (01 and 0139 serotypes). 
Vibrio cholerae is spread by ingestion of contaminated water or food. The 
organism is sensitive to gastric acid; therefore, the dose required to cause an 
infection is high. Conditions that reduce gastric acid, such as antacid medica- 
tions or achlorhydria, increase the risk of infection. 

The hallmark of cholera is severe watery diarrhea with mild to severe 
dehydration because of production of toxin by the organism. In cases of severe 
dehydration, patients have a nonpalpable pulse and very low blood pressure. 
Fever is usually not present. Patients may become obtunded with sunken 
eyes and dry mucous membranes. 

Vibrio parahaemolyticus is associated with gastroenteritis that is self- 
limited even though patients present with explosive watery diarrhea, with 
abdominal pain and fever. The disease rarely progressed to the severity of 
dehydration of V. cholerae. Vibrio vulnificus is more often associated with 
wound infections, that is, cellulitis, rather than gastroenteritis. In alcoholic 
patients or those with other underlying liver disease, the organism can become 
disseminated and be associated with a high mortality rate. 

APPROACH TO SUSPECTED Vibrio INFECTION 
Objectives 

1 . Know the structure, physiology, and virulence factors of V cholerae. 

2. Know the reservoirs and mechanisms of spread of V. cholerae and the 
mechanism of action of the cholera toxin in causing disease. 



CLINICAL CASES 173 

Definitions 

Azotemia: Buildup in the blood of nitrogenous end-products of protein 

metabolism. 
Obtunded: Loss or dulling of sensations. 

DISCUSSION 

Characteristics of Vibrio 

Vibrio species are motile, curved, gram-negative bacilli with a single polar 
flagellum. They are facultative anaerobic organisms. Their natural environ- 
ment is saltwater, where they can multiply freely, and it has been found in 
shellfish and plankton. The major human pathogens are V. parahaemolyticus, 
V. vulnificus, and Vibrio cholerae. 

Over 200 serotypes of V. cholerae have been identified, based on their O 
antigen serogroup. Serotype Ol has been responsible for the major cholera 
pandemics of the past 200 years, but serotype 0139 has been identified as con- 
tributing to disease since 1992. 

The major virulence of this organism is its enterotoxin. The toxin consists 
of five B subunits, which bind to mucosal cell receptors and allow for release 
of the single A subunit into the cell. The A subunit activates adenyl cyclase, 
resulting in the hypersecretion of water, sodium, potassium, chloride, and 
bicarbonate into the intestinal lumen. 

Bacteria that survive transit through the stomach can colonize the upper small 
intestine. Colonization pili facilitate attachment to the intestinal mucosa. The 
volume of the secreted fluid and electrolytes can overwhelm the gastrointestinal 
tract's ability to reabsorb them, resulting in large volumes of watery diarrhea. 
The loss of an isotonic, bicarbonate-containing fluid results in dehydration, 
hypovolemia, metabolic acidosis, hemoconcentration, and hypokalemia. 

Diagnosis 

The presumptive diagnosis of Vibrio disease can be made after history of asso- 
ciation with saltwater, either involving trauma or consumption of raw shell- 
fish. The watery diarrhea associated with V parahaemolyticus cannot be easily 
distinguished clinically from other forms of bacterial gastroenteritis. Cellulitis 
caused by V vulnificus should be diagnosed rapidly to avoid mortality. History 
of recent exposure to seawater is helpful in making a presumptive diagnosis. 
The diagnosis of cholera should be suspected in those with severe diarrheal 
illness who live in or have traveled to an endemic area. Diagnosis of Vibrio 
infection can be confirmed by culturing stool or wound samples. Gram stain 
of wound or blood cultures may demonstrate a characteristic curved 
appearance to the gram-negative bacilli. 



174 CASE FILES: MICROBIOLOGY 

Most of the Vibrio species require salt for growth and therefore specialized 
media, such as thiosulfate citrate bile salts sucrose (TCBS) agar. Most of the 
Vibrio species will grow on blood agar and may appear (3-hemolytic, but poor 
growth is seen on MacConkey agar. Vibrio cholerae appear as yellow colonies, 
and V. parahaemolyticus and V. vulnificus appear as green colonies on 
TCBS agar. 

Treatment and Prevention 

The treatment of cholera involves volume replacement with isotonic, 
bicarbonate-containing fluids, either using oral rehydration solutions in mild 
to moderate dehydration or IV fluids, such as Ringer lactate, in the profoundly 
dehydrated or those unable to tolerate oral intake. Oral antibiotics can be 
given to kill the bacteria and decrease the duration of the illness, but do not 
take away the need for appropriate rehydration therapy. Most commonly 
administered antimicrobial is doxycycline. 

Treatment with antimicrobials is not usually needed for gastroenteritis 
caused by V. parahaemolyticus. Wound infections or bacteremia caused by 
V. vulnificus require rapid administration of antimicrobials such as tetra- 
cycline or a quinolones. Prevention of cholera includes improvement of 
hygienic practices including treatment of the potable water supply with either 
heat or chlorine and ensuring thorough cooking of seafood. Research is ongo- 
ing to perfect a vaccine to prevent cholera. 

COMPREHENSION QUESTIONS 

[23.1] An individual experiences diarrhea after eating raw shellfish in San 
Francisco. What is the most probable cause of the problem? 

A. Campylobacter jejuni 

B. Salmonella choleraesuis 

C. Shigella dysenteriae 

D. Vibrio parahaemolyticus 

E. Yersinia enterocolitica 

[23.2] Which of the following statements is true of cholera enterotoxin? 

A. Appears to produce its effect by stimulating adenyl-cyclase activity 
in mucosal cells 

B. Causes destruction of the intestinal mucosa allowing for invasive 
infection 

C. Causes a net efflux of ions and water from tissue into the lumen of 
the large intestine 

D. Is a protein with a molecular weight of approximately 284,000 
daltons 



CLINICAL CASES 775 

[23.3] Fever, leukopenia, disseminated intravascular coagulation, and 
hypotension caused by members of the Enterobacteriaceae family are 
most strongly associated with which of the following structures? 

A. H antigens 

B. K antigens 

C. Lipid A 

D. Polysaccharides 

E. R antigens 

[23.4] A 50-year-old man recently visited India and developed diarrhea 
before returning to the United States. Vibrio cholerae Ol (El Tor, 
Ogawa) was isolated from his stool. Which of the following is the bio- 
type of the V. cholerae strain? 

A. Classical 

B. El Tor 

C. 10 

D. Ogawa 

Answers 

[23.1] D. Vibrio parahaemolyticus is a halophilic bacterium that causes 
acute gastroenteritis following ingestion of contaminated seafood 
such as raw fish or shellfish. After 12-24 hours, the patient develops 
nausea and vomiting, abdominal cramps, fever, and watery to bloody 
diarrhea. It is usually self-limited in 1-4 days, requiring only restora- 
tion of water and electrolytes. All other answer options could produce 
episodes of gastroenteritis, but the halophilic nature of V. para- 
haemolyticus and seafood is recognized as a classic combination. 

[23.2] A. The clinical correlation section of this case study summarizes the 
action of the V. cholerae enterotoxin quite well. It can be the cause of 
20-30 L/day diarrheal output, resulting in dehydration, shock, acido- 
sis, and death. It is antigenically related to the LT of Escherichia coli, 
has a molecular weight of approximately 84,000 daltons, does not 
damage the mucosa, and affects the small intestine. 

[23.3] C. The lipopolysaccharide (LPS) of gram-negative cell walls consists 
of a complex lipid, lipid A, to which is attached a polysaccharide 
made up of a core and a terminal series of repeat units. LPS is 
attached to the outer membrane by hydrophobic bonds and is 
required for the function of many outer membrane proteins. LPS is 
also called endotoxin. All the toxicity resides in the lipid A compo- 
nent. Endotoxin (lipid A) can activate complement, resulting in 
inflammation and the clinical features referred to in the question. 



176 CASE FILES: MICROBIOLOGY 

[23.4] B. The O antigen of Vibrio species has been given numbers to indi- 
cate biotype, a form of subdivision for strains of cholera organism. 
Vibrio cholerae serogroups Ol and 0139 have long been recognized 
as strains responsible for epidemic and pandemic cholera. There have 
been six pandemics from 1817 to 1923, most likely caused by the Ol 
subtype. A new pandemic caused by the El Tor biotype started in Asia 
in 1961 and spread to Central and South America by 1991. The dis- 
ease and biotype is rare in North America, but it does have an 
endemic focus on the Gulf of Mexico coastal areas (Louisiana and 
Texas). 






MICROBIOLOGY PEARLS 

The A subunit of the Vibrio enterotoxin activates adenyl cyclase, 
resulting in the hypersecretion of water, sodium, potassium, chlo- 
ride, and bicarbonate into the intestinal lumen. 

The predominant clinical presentation of Vibrio gastroenteritis is 
watery diarrhea. 

The Vibrio organism appears as gram-negative, curved, motile 
bacilli. Vibrio gastroenteritis or cholera is associated with con- 
sumption of contaminated seafood or water. 

Vibrio vulnificus is associated with cellulitis caused by trauma 
incurred in a seawater environment and carries a high mortality 
rate if not treated rapidly. 

Vibrio species require salt for growth and can be differentiated from 
other organisms by growth on TCBS agar. 



REFERENCES 

Murray PR, Rosenthal KS, Pfaller MA. Vibrio, Aeromonas, and Plesiomonas. In: 

Murray PR, Rosenthal KS, Pfaller MA. Medical Microbiology, 5th ed. St. Louis, 

MO: Mosby, 2005:339-45. 
Neill MA, Carpenter CCJ. Other pathogenic vibrios. In: Mandell GL, Bennett JE, 

Dolin R, eds. Principles and Practice of Infectious Diseases, 6th ed. 

Philadelphia, PA: Churchill Livingstone, 2005:2544-48. 
Seas C, Gotuzzo E. Vibrio cholerae. In: Mandell GL, Bennett JE, Dolin R, eds. 

Principles and Practice of Infectious Diseases, 6th ed. Philadelphia, PA: 

Churchill Livingstone, 2005:2553-44. 



♦> CASE 24 



A 5-year-old girl is brought to the physician's office because of "pink eye." 
She was sent home from kindergarten yesterday by the school nurse because 
her left eye was red. When she awakened this morning, the right eye was red 
as well. She has had watery drainage but no purulent discharge. She's had a 
mild head cold with a runny nose and a mild sore throat but no fever. When 
her mother called the school this morning, she was told that five of her daugh- 
ter's classmates were out with pink eye today. On examination, the child has 
injected conjunctiva bilaterally with clear drainage. No crusting of the lashes 
is noted, and the corneas are clear. She has mildly tender preauricular 
adenopathy. The remainder of her examination is unremarkable. 

^ What organism is the most likely cause of this infection? 

^ How does this organism gain entry into host cells? 



775 CASE FILES: MICROBIOLOGY 

ANSWERS TO CASE 24: ADENOVIRUS 

Summary: A 5-year-old girl with conjunctivitis of both eyes, with nonpurulent 
drainage associated with an upper respiratory infection. 

^ Most likely organism causing the infection: Adenovirus. 

^ Method that the organism gains entry into host cells: Adenoviruses 
gain entry into host cells by binding to the coxsackie adenovirus 
receptor (CAR) followed by receptor-mediated endocytosis. 

CLINICAL CORRELATION 

Conjunctivitis is a normal feature of many childhood infections. However, the 
most common cause of conjunctivitis is related to infection with adenoviruses. 
In addition to being the most common cause of viral conjunctivitis, aden- 
oviruses also commonly cause upper respiratory infections and gastrointestinal 
infections. Most adenoviral diseases are mild and self-limiting in immune- 
competent persons. Children are infected more frequently than adults. 

APPROACH TO SUSPECTED ADENOVIRAL INFECTION 
Objectives 

1. Be able to describe the characteristics of adenovirus. 

2. Understand how adenovirus causes infection. 

3. Know the strategies of prevention. 

Definitions 

Conjunctivitis: Inflammation of the eye tissue. 

Lymphadenopathy: Enlargement of a lymph node occurring singly or in 

multiple nodes. 
Preauricular adenopathy: Enlargement of a lymph node occurring singly 

or in multiple nodes anterior to the ear. 

DISCUSSION 

Characteristics of Adenoviruses That Impact Transmission 

Adenoviruses are nonenveloped viruses that contain linear, double- 
stranded DNA with a terminal protein attached to both 5' ends of the genome. 
The viral capsid is composed of an icosadeltahedral structure that contains a 
penton base and fiber at each vertex. The fibers contain viral attachment pro- 
teins that determine the target cell specificity among viral serotypes. The fiber 



CLINICAL CASES 779 

also serves as a hemagglutinin. Over 100 different serotypes have been rec- 
ognized, more than 49 of which are known to infect humans. 

To gain entry into the host cell, the viral fiber proteins bind to the cox- 
sackie adenovirus receptor on host cell surfaces and become internalized 
by receptor-mediated endocytosis. The virus then lyses the endosome, and 
the viral DNA is delivered to the host nucleus still within the viral capsid. Viral 
DNA replication occurs in the nucleus, via a virally encoded DNA poly- 
merase, and the viral capsid proteins are made in the cytoplasm and then trans- 
ported to the nucleus for viral assembly. Adenoviral genes are transcribed from 
both strands of the DNA genome in either direction at different times during 
viral replication. A single viral replication cycle takes approximately 32-36 
hours and produces around 10,000 new virions. Figure 24-1 shows the repli- 
cation cycle. However, errors in assembly and replication are common, result- 
ing in a much lower number of infectious viral particles. 

Adenoviruses infect epithelial cells of the respiratory tract, conjunctiva, 
and enteric organs. Infections are spread from person to person by 
aerosolized respiratory droplets, close contact, or a fecal-oral route. Fomite 
transmission is also common because adenoviruses are nonenveloped, making 
them more resistant to detergents and desiccation. They can cause lytic infec- 
tions in epithelial cells and tend to cause latent infections in lymphoid tissue. 
Persistence in lymphoid tissues involves integration of viral DNA into the host 
genome. Reactivation of virus can occur with stress. Viremia may occur and 
cause spread to distant organs such as the kidney, bladder, liver, and lymphoid 
tissue. Viremia is especially common in immunosuppressed patients. 

Diagnosis 

Adenoviruses primarily infect children under 3 years and appear clinically 
with a variety of symptoms including fever, cough, nonstreptococcal exuda- 
tive pharyngitis, cervical adenitis, conjunctivitis, or gastroenteritis. 

Symptoms can last from 3 to 5 days. More severe respiratory diseases include 
laryngitis, bronchiolitis, and pneumonia. Reactivated viral disease occurs pri- 
marily in immune compromised individuals. Adenoviral follicular conjunc- 
tivitis, or "pink eye," outbreaks in children often involve swimming pools as 
a common source of infection. Gastroenteritis is also a major clinical mani- 
festation of adenoviral infection. Adenoviral types 40, 41, and 42 have been 
shown to be associated with gastrointestinal disease in infants and hospital- 
ized patients. 

In addition to clinical presentation of infection, laboratory diagnostic tests, 
including cell culture, ELISA (enzyme-linked immunosorbent assay), PCR, 
and DNA-probe analysis are available and can be used to detect the viral 
type in clinical samples and tissue culture. However, their primary use is for 
epidemiological studies, and they are not used widely in clinical practice for 
diagnostic purposes. Typically, diagnosis is made by clinical presentation and 
patient history. 



180 



CASE FILES: MICROBIOLOGY 



Absorption 





^ 


Viral DNA 


TT 


Early proteins 


w\ 


Viral mRNA 


o 


Viral caps id 


V 


Viral protein 


o 


Ribosomes 



Penetration /^^^\ 

— vQ' 



s 

Assembly 



Nucleus 
Cytoplasm 



Uncoating 




Transfer of 
■x\capsid proteins 
v to nucleus 



Viral DNA 
in the nucleus 1 - 



Synthesis of 

early mRNA 



"&; 




Synthesis of 

late proteins 

(capsid proteins 



4 

Early 

iTranscription 



5 

Early 

Translation 



7 

Late 

Translation 



6 

DNA 

Synthesis 

and 

Late 

Transcription 



Figure 24-1. Adenovirus replication cycle. 



Treatment and Prevention 

Currently, there is no treatment for adenoviral infection. Live oral vaccines have 
been developed for adenovirus types 4 and 7, which cause acute respiratory tract 
infections, and have been used primarily in military settings. However, because 
some adenoviruses are oncogenic, such vaccines have not been made available 
to the general population. Thus, prevention is the most important aspect involv- 
ing careful hygiene, handwashing, and isolation of infected individuals. 



CLINICAL CASES 181 

COMPREHENSION QUESTIONS 

[24.1] An 11-year-old boy attending summer Boy Scout camp develops 
symptoms of sore throat, headache, fatigue, and conjunctivitis. He is 
seen by the camp medical staff and on examination is found to have a 
slight fever of 39.8°C (103.6°F), but no rash. Within the next 1-2 days, 
several of the other campers develop similar symptoms, which last for 
5-7 days. The larger number of campers with similar symptoms indi- 
cates that a common source of infection is causing the outbreak. Which 
of the following activities is the most likely source of the campers' 
infection? 

A. Hiking in wooded areas with tall grass 

B. Sharing water canteens with other campers 

C. Sleeping outdoors without protective netting 

D. Swimming in the camp pond 

E. Walking barefoot in the bath house 

[24.2] The causative agent in the question above was determined to be an ade- 
noviral infection. Which of the following best describes this viral 
agent? 

A. Nonenveloped, double-stranded DNA virus with fibers at its vertices 

B. Nonenveloped, double-stranded, circular DNA virus 

C. Enveloped, single-stranded, negative-sense RNA virus 

D. Enveloped, double-stranded, linear DNA virus with glycoprotein 
spikes 

E. Enveloped, double-stranded, circular DNA virus 

[24.3] A 2-year-old child attending day care develops diarrhea and gastroen- 
teritis as a result of an adenoviral infection. Which of the following ade- 
noviral serotypes would most likely be responsible for this girl's illness? 

A. Type 4 

B. Type 7 

C. Type 19 

D. Type 37 

E. Type 41 



182 CASE FILES: MICROBIOLOGY 

Answers 

[24.1] D. The campers' symptoms are consistent with adenoviral conjunc- 
tivitis, which is commonly spread through contaminated swimming 
pools or ponds; answers A, B, C, and E are incorrect. 

[24.2] A. Adenoviruses are nonenveloped, double-stranded linear DNA 
viruses with fiber structures projecting from their vertices or penton 
bases; answers B, C, D, and E are incorrect: (B) describes polyoma 
viruses such as human papillomavirus; (C) describes viruses such as 
rhabdoviruses, orthomyxoviruses, paramyxoviruses, and the like; 
(D) describes herpesviruses; (E) describes hepatitis B virus. 

[24.3] E. Adenoviral types 40, 41, and 42 have been shown to be associated 
with gastrointestinal disease in infants; answers A, B, C, and D are 
incorrect: Adenoviral types 4 and 7, commonly cause upper respira- 
tory infections in military recruits; adenoviral types 19 and 37 have 
been implicated in causing epidemic keratoconjunctivitis. 



MICROBIOLOGY PEARLS 



*♦* Adenoviruses commonly cause conjunctivitis, in combination with 

pharyngitis, and upper respiratory infections. 
*♦* Children under 3 years and immunocompromised adults are at 



particular risk. 



*♦* Clinical manifestations are fever, cough, nonstreptococcal exudative 

pharyngitis, cervical adenitis, conjunctivitis, or gastroenteritis. 
*♦* No treatment or vaccination is available for the general public. 



REFERENCES 

Brooks GF, Butel JS, Morse SA. Jawetz, Melnick, & Adelberg's Medical 
Microbiology, 23rd ed. New York: McGraw-Hill, 2004:420-8. 

Ryan JR, Ray CG. Sherris Medical Microbiology, 4th ed. New York: McGraw-Hill, 
2004:507-10. 

Shenk TE. Adenoviridae: the viruses and their replication. Fields Virology, 4th ed. 
Philadelphia, PA:Lippincott Williams & Wilkins, 2001:2111-35. 



i* CASE 25 



♦> 



You are called to examine a 1 -day-old male because the nurse is concerned that 
he is jaundiced. He was born by spontaneous vaginal delivery to a 19-year-old 
gravidaj paraj after a full-term, uncomplicated pregnancy. The mother had no 
illnesses during her pregnancy; she did not use tobacco, alcohol, or drugs; and 
the only medication that she took was prenatal vitamins. She denied any sig- 
nificant medical history, and there is no family history of genetic syndromes or 
illnesses among children. The infant is mildly jaundiced but has a notable 
abnormally small head circumference (microcephaly). His cardiovascular 
examination is normal. His liver and spleen are enlarged on palpation of the 
abdomen. Neurologic exam is notable for the lack of a startle response to a loud 
noise. CT scan of his head reveals intracerebral calcifications. The pediatrician 
explains to the child's mother that the virus involved is the most commonly 
transmitted transplacental viral infection in the United States. 

^ What is the most likely cause of this infant's condition? 

How did he likely acquire this? 

^ What is the test of choice to confirm the diagnosis? 



184 CASE FILES: MICROBIOLOGY 

ANSWERS TO CASE 25: CYTOMEGALOVIRUS 

Summary: A 1 -day-old male with microcephaly, jaundice, hepatosplenomegaly, 
and deafness caused by a viral infection. 

^ Most likely cause of this infant's condition: The infant is most likely 
suffering from a congenital infection with cytomegalovirus (CMV). 

^ Likely acquisition of this infection: Transplacental spread of the virus 
during a primary CMV infection of the pregnant mother. 

^ Test of choice to confirm the diagnosis: The definitive diagnostic test 
to confirm CMV infection in this patient is to demonstrate the presence 
of CMV in the infant's urine. 

CLINICAL CORRELATION 

Human cytomegalovirus (CMV) is the largest member of the human 
Herpesviridae family. It is lymphotrophic and commonly produces asympto- 
matic infections in immune competent hosts. However, it can cause serious 
primary and recurrent infections in immunosuppressed individuals and 
neonates. CMV is the most common transplacentally transmitted infection in 
the United States. 

APPROACH TO SUSPECTED CMV INFECTION 
Objectives 

1. Be aware of the genomic characteristics of CMV. 

2. Be able to describe how CMV causes infection, including transplacen- 
tal infection. 

Definitions 

Lymphotrophic: Having a specific affinity for lymph cells or their precursors. 
Microcephaly: Abnormally smaller sized head, which may be associated 

with mental retardation. 
Hepatosplenomegaly: Enlargement of the liver and spleen. 
Subclinical infection: Without the presence of noticeable clinical disease. 

DISCUSSION 

Characteristics of CMV That Impact Transmission 

CMV belongs to the Betaherpesvirinae subunit of the Herpesviridae family 
and is the largest known virus to infect humans. The genome of linear, 
double-stranded DNA is housed in an icosadeltahedral capsid. Between 



CLINICAL CASES 185 

the envelope and the capsid is a layer called the tegument, a phosphoprotein- 
containing matrix that plays a role in initiating replication. CMV, like other 
herpesviruses, has a lipid envelope that contains glycoproteins that facilitate 
attachment and entry into host cells. The virus often establishes latent infec- 
tion in lymphocytes, leukocytes, and organs like the kidney, heart, and 
lung. Cell-mediated immunity is required for the control of CMV infections. 
Suppression of the immune system by medications or infection, such as AIDS, 
can result in reactivation of the virus and severe, symptomatic disease. 

CMV has a ubiquitous distribution and approximately 10-15 percent of chil- 
dren are infected before the age of 5. Most CMV infections in immune com- 
petent hosts are asymptomatic, although occasionally a mononucleosis-like 
syndrome can occur. Yet, even in subclinical infections, CMV can be isolated 
from saliva, cervical secretions, semen, urine, and white blood cells for months 
to years following infection. Although CMV is found in many host secretions, 
its major routes of transmission are via contact with blood, oral secretions, 
sexual contact, organ transplant, or congenital infection. CMV is the most 
common viral cause of congenital disease and infection, and its spread is 
thought to occur via transplacental transfer. The risk to the fetus is particularly 
high when the mother has a primary infection during her pregnancy. 

Diagnosis 

Although most CMV infections during childhood and in adults are asympto- 
matic, infants and immunocompromised patients can develop severe clinical 
symptoms from either primary infection or reactivation. Of the 1 percent of 
infants infected in utero or during delivery, 90 percent will develop asympto- 
matic infections, while the other 10 percent will develop symptomatic infec- 
tions with congenital defects or disorders. Nearly all of the infants with 
symptomatic infections are born of mothers with primary infections dur- 
ing their pregnancies. Congenital CMV can cause a devastating syndrome that 
includes microcephaly, intracerebral calcifications, hepatosplenomegaly, 
thrombocytopenia, chorioretinitis, deafness, mental retardation, jaun- 
dice, and rash. Many of the infants with severe CMV congenital syndrome 
die within a short time, and those who survive have been shown to have per- 
sistent neurologic deficits. Reactivation of a latent infection during pregnancy 
confers a much lower risk, as the fetus is protected by the maternal immune 
response. 

In addition to assessing the clinical symptoms, more definitive approaches 
to diagnosing CMV infection include direct detection of CMV antigen or 
DNA in tissues or fluids via immunoassays or quantitative PCR. Diagnosis 
of CMV infection can be confirmed by identification of the virus in the 
infant's urine during the first week of life. Histologically, CMV infection can 
also be detected by its ability to produce characteristic enlarged cytomegaly 
of infected cells with pronuclear inclusions, or "owl's eyes." 



186 CASE FILES: MICROBIOLOGY 

Treatment and Prevention 

CMV infections are primarily treated with ganciclovir, immune globulin 
plus ganciclovir, or foscarnet. Treatment with ganciclovir has been used to 
prevent CMV disease in AIDS patients and transplant recipients. Use of this 
agent also reduces the severity of CMV syndromes such as retinitis and gas- 
trointestinal disease. Treatment with both immune globulin and ganciclovir 
has been used to reduce the high mortality of CMV pneumonia in bone mar- 
row transplant patients. 

Unfortunately, congenital and perinatal transmission of CMV cannot be pre- 
vented once acquired by the pregnant woman. Thus, hygiene and handwashing 
may play a role in prevention. Isolation of infants with CMV infections can pre- 
vent spread to other infants. Prevention of transplantation-acquired CMV infec- 
tion can be obtained by transplanting organs and blood products from 
seronegative donors into seronegative recipients. In situations where it is not 
possible to use organ or blood products from seronegative donors, prophylactic 
treatment of all transplant patients or preemptive therapy of those patients with 
evidence of active CMV infection should be used. Such therapies include the 
use of hyperimmune CMV globulin, anti-CMV agents or a combination of 
both. Additionally, safe sex practices also reduce transmission of new CMV 
infections. CMV vaccines are currently under development; however, none are 
currently available. 

COMPREHENSION QUESTIONS 

[25.1] An 18-year-old female presents to her physician with a 1-week history 
of fever, sore throat, fatigue, and myalgia. Physical examination reveals 
enlarged tonsils and exudative pharyngitis. Based on her clinical presen- 
tation, her physician diagnoses her with infectious mononucleosis. 
Because there are multiple causes of infectious mononucleosis-like ill- 
nesses, which of the following diagnostic assays would rule out CMV 
as the causative agent of this patient's infection? 

A. A negative Gram stain of a throat swab 

B. A lack of atypical lymphocytes in the patient's blood 

C. A positive histological finding of cytomegaly 

D. A positive Monospot test 

[25.2] A previously healthy 8-year-old boy develops a classic childhood ill- 
ness as a result of a primary viral infection. Which of the following agents 
would most likely produce symptomatic disease in a boy of this age? 

A. Cytomegalovirus 

B. Epstein-Barr virus (EBV) 

C. Herpes simplex virus 2 (HSV-2) 

D. Polio virus 

E. Varicella-zoster virus (VZV) 



CLINICAL CASES 187 

[25.3] A 32-year-old HIV infected male is noted to have acute CMV infection 
causing acute gastrointestinal symptoms. The treating physician has 
ordered that antiviral therapy be administered. Which of the following 
is most likely to be targeted by the antiviral agent? 

A. Protease cleavage 

B. Nuclear transport of virus 

C. Synthesis of viral DNA 

D. Transcription of viral proteins 

E. Viral-cell fusion 

Answers 

[25.1] D. Both CMV and EBV infections can cause infectious mononucle- 
osis disease; however, only EBV produces heterophile antibodies that 
would result in a positive Monospot test; answers A, B, and C, incor- 
rect: A negative Gram stain of the patient's throat culture would rule 
out group A Streptococcus; atypical lymphocytes are commonly 
present in EBV infection but not CMV-related infections; and 
cytomegaly is typically present in CMV infections. 

[25.2] E. VZV is a classic childhood disease that produces symptomatic 
primary infections; answers A, B, C, and D are incorrect: Most pri- 
mary CMV, EBV, and poliovirus infections are asymptomatic, 
whereas HSV-2 infections would rarely occur in a child of this age. 

[25.3] C. Ganciclovir has been used primarily to treat severe CMV infec- 
tions, and its method of action involves inhibition of DNA synthesis; 
answers A, B, D, and E are incorrect methods of antiviral therapy for 
CMV infection. 



MICROBIOLOGY PEARLS 

*♦* CMV is the most common viral cause of congenital infection in the 

United States with the mechanism being primarily transplacental 

infection. 
*♦* CMV establishes latent infection in lymphocytes and leukocytes. 
*♦* CMV can be excreted in saliva, semen, urine, blood, and cervical 

secretions for months to years following infection. 
*♦* Clinical manifestations include a mononucleosis-like disease in 

immune competent individuals; microcephaly, hepatosplenomegaly, 

deafness, neurological deficits, and jaundice in congenital 

infections. 
*♦* CMV cytopathology involves cytomegaly or enlargement of 

infected cells with pronuclear inclusions, or "owl's eyes." 



188 CASE FILES: MICROBIOLOGY 

REFERENCES 

Murray PR, Rosenthal KS, Pfaller MA. Medical Microbiology, 5th ed. St. Louis, 

MO: Mosby, 2005:558-62. 
Ryan JR, Ray CG. Sherris Medical Microbiology, 4th ed. New York: McGraw-Hill, 

2004:566-9. 
Sia IG, Patel R. New strategies for the prevention and therapy of Cytomegalovirus 

infection and disease in solid-organ transplant recipients. Clin Microbiol Rev 

2000:13:83-121. 



♦ CASE 26 



A 17-year-old female is brought to the physician's office for evaluation of a 
sore throat and fever. Her symptoms started approximately 1 week ago and 
have been worsening. She has been extremely fatigued and has spent most of 
the last 3 days in bed. She denies any ill contacts. She has no significant med- 
ical history, takes no medications, and has no allergies. On examination, she is 
tired and ill appearing. Her temperature is 38.5°C (101.3°F). Examination of 
her pharynx shows her tonsils to be markedly enlarged, almost touching in the 
midline. They are erythematous and covered with white exudates. She has 
prominent cervical adenopathy, which is mildly tender. A cardiovascular 
examination is normal, and her abdomen is soft, nontender, and without pal- 
pable organomegaly. A rapid streptococcal antigen test in the office is nega- 
tive. You send a throat culture and decide to start amoxicillin for strep 
pharyngitis, assuming that the office test was a false-negative. Two days later, 
you get a call from her mother stating that she has had an allergic reaction to 
the amoxicillin, and she now has a red rash from head to toe. 

^ What is the most likely diagnosis of this patient? 

^ What is the most likely cause of her infection? 

^ In what human cells can this virus replicate? In what cells can it 
cause latent infection? 



790 CASE FILES: MICROBIOLOGY 

ANSWERS TO CASE 26: EPSTEIN-BARR VIRUS 

Summary: A 17-year-old female with fever, exudative pharyngitis, and adenopa- 
thy develops a prominent macular-papular rash after ampicillin is instituted. 

^ Most likely diagnosis: Infectious mononucleosis. 

^ Most likely etiology: Epstein-Barr virus (EBV). 

^ In the human cells this virus can replicate, and in the cells it can 
cause latent infection: EBV preferentially replicates in epithelial cells 
and B cells, and is known to cause latent infections in B cells. 

CLINICAL CORRELATION 

Epstein-Barr virus (EBV) is a member of the human herpesvirus family, and 
more specifically a member of the Gammaherpesvirinae subfamily. Humans 
are the only known natural host for these viruses. EBV infections are most com- 
monly known for causing infectious mononucleosis in adolescents and young 
adults, and it is often referred to as the "kissing disease." Viral transmission 
occurs via repeated close intimate contact or through the sharing of items con- 
taminated with saliva, because virus is intermittently shed in the saliva of most 
seropositive individuals. Secondary attack rates with family and household con- 
tacts tend to be low because 90-95 percent of adults have previously been 
exposed to EBV. Most primary infections are asymptomatic, whereas sympto- 
matic infections are marked with fever, fatigue, pharyngitis, tender lym- 
phadenitis, and possible hepatosplenopathy. Infections with these symptoms 
can be mistakenly diagnosed as streptococcal pharyngitis, and the resulting 
inappropriate treatment with amoxicillin can produce an allergic rash. 

APPROACH TO SUSPECTED EBV INFECTION 
Objectives 

1. Be aware of the genomic and other characteristics of EBV. 

2. Be able to describe the clinical disease caused by EBV, mode of trans- 
mission, and strategies for treatment. 

Definitions 

Lymphocytosis: A larger than normal number of T lymphocytes. 

Atypical lymphocytes: Enlarged T lymphocytes, also referred to as 
"Downey cells," with eccentric nuclei and a vacuolated cytoplasm. 

Heterophile antibodies: Nonspecific antibodies, including an IgM anti- 
body, that recognizes the Paul-Bunnell antigen on sheep, horse, and 
bovine erythrocytes. 



CLINICAL CASES 



191 



DISCUSSION 

Characteristics of EBV That Impact Transmission 

Similar to other members of the Herpesviridae (Table 26-1), EBV is an 
enveloped virus with a double-stranded linear DNA genome that is approx- 
imately 172 kb in size and encodes more than 70 viral proteins. The DNA core 
is surrounded by an icosadeltahedral nucleocapsid, with a protein tegu- 
ment located between the capsid and viral envelope, containing viral enzymes 
and proteins necessary for replication. The outer membrane of EBV contains 
virally encoded glycoprotein spikes, important for host cell attachment to 
human B cells and epithelial cells of the oro- and nasopharynx via the recep- 
tor for the C3d component of the complement system. As an enveloped virus, 
EBV is easily disrupted by acids, detergents, and desiccation and, thus, is 
effectively transmitted via intimate contact and saliva. 

There are two infectious subtypes of EBV, EBV-1, and EBV-2, which are 
closely related except for differences in their nuclear antigens. The various 
EBV antigens are expressed in different phases of productive viral replication or 
in latent infection and can be used in diagnoses. Early EBV antigens, such as 
early antigens (EAs) and nuclear antigens (NAs), are nonstructural proteins 



Table 26-1 

PROPERTIES OF HERPES VIRUSES 



VIRUS 


TARGET CELL 


SITE OF 
LATENCY 


TRANSMISSION 


Herpes simplex 
virus type 1 


Epithelial cells of 
the mucosa 


Neurons 


Close contact 


Herpes simplex 
virus type 2 


Epithelial cells of 
the mucosa 


Neurons 


Close contact 
(sexually transmitted) 


Varicella-zoster 
virus 


Epithelial cells of 
the mucosa 


Neurons 


Respiratory and 
close contact 


Epstein-Barr 
virus 


B cells and 
epithelial cells 


Bcell 


Person to person 
via saliva 


Kaposi 

sarcoma- related 
virus 


Lymphocytes 


Unknown 


Close contact 
(sexually transmitted) 


Cytomegalovirus 


Monocytes, 
lymphocytes, 
epithelial cells 


Monocytes 
lymphocytes 


Close contact, 
blood, and tissue 
transplantation, and 
congential 



192 CASE FILES: MICROBIOLOGY 

expressed at the onset of lytic viral infection and are followed by the expres- 
sion of late viral antigens, including the structural components of the viral cap- 
sid (VCA) and membrane (MA). Latent phase antigens are expressed in 
latently infected B cells and include Epstein-Barr nuclear antigens (EBNAs), 
latent proteins (LPs), and latent membrane proteins (LMPs). 

EBV was first discovered in association with African Burkitt lymphoma, 
a common malignancy of young children in sub-Saharan Africa. The highest 
occurrence of Burkitt lymphoma appears to occur in regions with high inci- 
dence of malaria, indicating malaria as a possible disease cofactor. Other 
EBV-related diseases include nasopharyngeal carcinoma and, in immuno- 
compromised patient populations, B-cell lymphomas, interstitial lympho- 
cytic pneumonia, and hairy leukoplakia of the tongue. 

EBV can cause lytic infections of epithelial cells and latent infection or 
immortalization of B cells. The lytic infection of epithelial and B cells promotes 
virus shedding into the saliva of the host, allowing for viral transmission to other 
hosts and spread within the host. In B cells, EBV promotes cell growth and pre- 
vents apoptosis. The proliferating B cells produce an IgM antibody to the Paul- 
Bunnell antigen, called a heterophile antibody, which serves as a diagnostic 
indicator of infectious mononucleosis. In this stage of infection, antibody is pro- 
duced against several EBV antigens, and a T-cell response is mounted. This 
response contributes to the symptoms and signs of mononucleosis, such as lym- 
phadenopathy, splenomegaly, and atypical lymphocytosis. Latent infection of B 
cells may occur after the resolution of the acute infection, with periodic reacti- 
vation and shedding of the virus in the saliva for months, years, or even lifetime. 
Persons with inadequate T cell immunity may not be able to suppress EBV 
infection and may progress to lymphoproliferative disease, B-cell lymphomas, 
or Hodgkin disease. Nasopharyngeal carcinoma, seen primarily in Asian and 
Aleutian populations, is thought to be associated with EBV infection in con- 
junction with some other genetic or environmental component. 

Diagnosis 

EBV-related infectious mononucleosis is clinically recognized by high fever, 
malaise, lymphadenopathy, pharyngitis, and occasional hepatosplenomegaly. 

Young children, who have a less active immune response, tend to have milder or 
subclinical infections. Symptoms from infection can last for days to weeks and 
then tend to resolve slowly on their own. Some of the rarer, but serious, compli- 
cations of EBV infection include laryngeal obstruction, meningitis, encephalitis, 
hemolytic anemia, thrombocytopenia, and splenic rupture. A macular papular 
rash often erupts when amoxicillin is taken by an EBV infected patients. 

Definitive diagnosis of EBV infections involves the finding of lymphocy- 
tosis with the presence of atypical lymphocytes, heterophile-positive anti- 
bodies, and antibody to EBV antigens. Atypical lymphocytes appear with the 
onset of infection, whereas a positive heterophile antibody response can be 
detected approximately 1 week after the onset of symptoms and remain present 



CLINICAL CASES 193 

for several months. The Monospot test and ELISA (enzyme-linked immunosor- 
bent assay) are widely used for detection of heterophile antibody. 

Treatment and Prevention 

Currently, there is no effective treatment for EB V infection, nor is a viral vac- 
cine available. However, because EBV infections in children tend to be less 
severe, and the immunity developed is lifelong, it is speculated by some clini- 
cians that early exposure to EBV may be a means of preventing more severe 
infections and symptomatic disease. 

COMPREHENSION QUESTIONS 

[26.1] Which of the following statement regarding the serologic diagnosis of 
infectious mononucleosis is correct? 

A. A heterophile antibody is formed that reacts with the membrane 
protein of EBV. 

B. A heterophile antibody is formed that agglutinates sheep or horse 
red blood cells. 

C. A heterophile antigen occurs that cross-reacts with atypical 
lymphocytes. 

D. A heterophile antigen occurs following infection with both EBV 
and CMV 

[26.2] A transplant patient taking high levels of immunosuppressive drugs 
becomes infected with EBV and develops a lymphoma. The dosage of 
immunosuppressive drugs given to the patient is subsequently 
decreased, and the tumor regresses. Which of the following properties 
of EBV infection is related to the patient's tumor development? 

A. Immortalization of B cells 

B. Increased white blood cell count 

C. Presence of atypical lymphocytes 

D. Production of heterophile antibodies 

[26.3] A 21 -year-old man visits the student health center suffering from a sore 
throat, swollen glands, fatigue, and a temperature of 39.4°C (103°F). 
Examination of the patient's peripheral blood smear shows 10 percent 
atypical lymphocytes, an elevated white blood cell count, and a positive 
heterophile antibody test. The patient asks for antimicrobial therapy. 
Which of the following statements would best dictate the clinician's 
response? 

A. OC-Interferon is helpful in EBV infections but has multiple side effects. 

B. Ribavirin is effective in patients over 60 years. 

C. Attenuated-viral vaccine has been developed but not effective in 
this case because the infection has already occurred. 

D. There is no effective treatment. 



194 CASE FILES: MICROBIOLOGY 

Answers 

[26.1] B. A nonspecific heterophile agglutination test (Monospot test) is 
commercially available and can be used to diagnose EBV infec- 
tious mononucleosis within a week to months of infection. Infectious 
mononucleosis-like infection caused by CMV is heterophile negative. 

[26.2] A. Aggressive monoclonal B-cell lymphomas can develop in patients 
with reduced T-cell function. The immortalization of B cells in the 
absence of functional T cell immunity can give rise to lymphoprolif- 
erative disease such as Hodgkin lymphoma, Burkitt lymphoma, and 
nasopharyngeal carcinoma. 

[26.3] D. Currently, there are no effective treatments or vaccines available for 
EBV infection. Ribavirin is useful in treating respiratory syncytial virus- 
related and hepatitis C virus-related infections, while a-interferon has 
been used in treating the following viral infections: condyloma acumi- 
natum, chronic hepatitis B and C, and Kaposi sarcoma. 



MICROBIOLOGY PEARLS 

*♦* EBV-related infectious mononucleosis is heterophile antibody- 
positive. 

*♦* Clinical manifestations are fever, malaise, lymphadenopathy, pharyn- 
gitis, with presence of lymphocytosis and atypical lymphocytes. 

*♦* No current treatment or vaccines are available. 



REFERENCES 

Cohen JL. Epstein-Barr virus infection. N Engl J Med 2000;343:481-92. 

Murray PR, Rosenthal KS, Pfaller MA. Medical Microbiology, 5th ed. St. Louis, 

MO: Mosby, 2005:553-8. 
Ryan JR, Ray CG. Sherris Medical Microbiology, 4th ed. New York: McGraw-Hill, 

2004:567-73. 



i* CASE 27 



♦> 



A 62-year-old male presents to your office for follow-up of some abnormal 
blood test results. You had seen him 2 weeks ago as a new patient for a routine 
physical examination. You ordered blood tests and found that his liver 
enzymes were elevated by approximately three times the upper limits of nor- 
mal. The patient says that to his knowledge he's never had abnormal liver tests 
before, although he has not been to a doctor in several years. He denies alco- 
hol or drug use and is not taking any medications. He gives no history of jaun- 
dice. His past medical history is significant only for hospitalization at the age 
of 45 for a bleeding stomach ulcer. He required surgery and had transfusion of 
4 units of blood. He recovered from this episode without further complication 
and has had no recurrences. Your complete physical examination 2 weeks ago 
was normal, and a focused physical examination today shows no signs of jaun- 
dice, no hepatosplenomegaly, and no physical exam findings suggestive of 
portal hypertension. You diagnose an infectious etiology for the laboratory 
findings (elevated liver enzymes). 

^ What is the most likely infectious cause of his abnormal liver 
function tests? 

^ How did he most likely acquire this infection? 



196 CASE FILES: MICROBIOLOGY 

ANSWERS TO CASE 27: HEPATITIS VIRUSES 

Summary: A 62-year-old man with abnormal liver function tests who had a 
blood transfusion previously, likely caused by an infectious etiology. 

^ Most likely infectious etiology: Hepatitis C virus 

^ Most likely route of transmission: Blood transfusion 

CLINICAL CORRELATION 

Hepatitis C virus (HVC) is transmitted parenterally by blood transfusions or 
intravenous drug use and rarely by sexual contact. It is uncommonly diag- 
nosed as a cause of acute hepatitis, often producing subclinical infection, but 
it is frequently diagnosed later as a cause of chronic hepatitis. The natural his- 
tory of infection is not completely understood, but 50-85 percent of patients 
with hepatitis C will develop chronic infection. 

On initial infection, approximately 15 percent of persons will develop an 
acute hepatitis syndrome and recover completely. However, most infected 
individuals will appear asymptomatic yet progress to chronic infection. This 
chronic infection may progress to cirrhosis, liver failure, or hepatocellular car- 
cinoma. HCV is transmitted from person to person, primarily via contact with 
infected blood or sexual contact. Routine screening of the donated blood sup- 
ply for HCV was started in 1992. Prior to this, HCV was the primary cause of 
posttransfusion hepatitis. The high percentage of infections that are asympto- 
matic contributes to the spread of the virus in the population. Diagnosis is 
made by the presence of circulating antibody to HCV. HCV-polymerase chain 
reaction is also used to quantitate the amount of circulating virus present in an 
infected person. This serves as a measure of disease activity and as a monitor 
of response to therapy. Recombinant interferon, which induces host antiviral 
and antiproliferative activity, is the most widely used therapy for HCV. 

APPROACH TO VIRAL HEPATITIS 
Objectives 

1 . Know the structure and characteristics of the viruses that cause hepatitis. 

2. Know the specific diseases associated with and routes of transmission 
of the hepatitis viruses. 

3. Understand the mechanisms of development of acute and chronic hep- 
atitis infections. 



CLINICAL CASES 197 

Definitions 

Hepatitis: Inflammation of the liver; viral causative agents include hepati- 
tis viruses A, B, C, D, E, and G. The clinical presentation can include 
fever, nausea or vomiting, jaundice, dark urine, pale feces, and elevated 
liver enzymes (AST and ALT). 

Dane particle: A 42-nm particle that is the hepatitis B virion. 

Fulminant hepatitis: Severe acute hepatitis that causes rapid destruction 
of the liver. 

DISCUSSION 

Characteristics of Hepatitis Viruses 

Because of its rich vascular supply, the liver may be involved in any systemic 
blood-borne infection, but the most common and clinically significant infec- 
tions are those with one of five hepatotropic viruses: Hepatitis A, B, C, D, or E. 
They can produce virtually indistinguishable clinical syndromes. Affected indi- 
viduals often complain of a prodrome of nonspecific constitutional symptoms 
including fever, nausea, fatigue, arthralgias, myalgias, headache, and some- 
times pharyngitis and coryza. This is followed by the onset of visible jaundice 
as a result of hyperbilirubinemia, with tenderness and enlargement of the liver, 
and dark urine caused by bilirubinuria. The clinical course, outcomes, and pos- 
sible complications then vary depending on the type of virus causing the hepa- 
titis. A comparison of features of these five viruses is shown in Table 27-1. 

Hepatitis A 

Hepatitis A and E are both very contagious and transmitted by fecal-oral 
route, usually by contaminated food or water where sanitation is poor and in 
day care by children. Hepatitis A virus (HAV) is found worldwide, and is the 
most common cause of acute viral hepatitis in the United States. Hepatitis 

E is much less common, and it is found in Asia, Africa, and Central America. 
Both hepatitis A and E infections usually lead to self-limited illnesses and gen- 
erally resolve within weeks. Almost all patients with hepatitis A recover com- 
pletely and have no long-term complications. 

HAV is a member of the Picornaviridae family. It is a nonenveloped, lin- 
ear, positive-sense, single-stranded RNA virus with only one serotype. Its 
average incubation period is approximately 30 days, and it results in 25,000 
symptomatic cases in the United States annually. However, nearly 90 percent 
of HAV infections in children and 30-50 percent in adults are asympto- 
matic. Clinical symptoms vary according to the age of the patient; infections 
in children are mostly asymptomatic or present with nonspecific symptoms, 
whereas adults generally have a more severe clinical course. Only 1-4 percent 
of patients develop fulminant liver failure, and there is 1 percent mortality 



Table 27-1 
COMPARISON OF HEPATITIS VIRUS 





VIRUS 
TYPE 


TRANSMISSION 


INCUBATION 


MARKERS 
SEROLOGIC 


CARRIER 
STATE 


CHRONIC 
HEPATITIS 


PREVENTION 


Hepatitis A 


RNA 


Enteral (fecal-oral) 


15^45 days 


Anti-HAV IgM 


No 


No 


Vaccine, avoid 
contaminated food 
and water 


Hepatitis B 


DNA 


Sexual, parenteral 


30-180 days 
(mean 60-90 days) 


+HBsAg, 
anti-HBcIgM 

(acute) 


Yes 


Yes 


Vaccine, avoid 
blood and fluids 


Hepatitis C 


RNA 


Parenteral 


15-160 days 
(means 50 days) 


Anti-HCV 
HCV RNA 
or ELISA 


Yes 


Yes 


No vaccine, 
avoidance 


Hepatitis D 


Defective 
RNA 


Parenteral 


Same as HBV 


Anti-HDV 
IgM 


Yes 


Yes 


Vaccine for 
HBV, avoidance 


Hepatitis E 


RNA 


Enteral (fecal-oral) 


14-60 days 
(mean 40 days) 


Anti-HEV 


No 


No 


Avoid contaminated 
food and water 



CLINICAL CASES 799 

from HAV infection. It is not known to cause chronic infection. The virus is 
contagious before symptoms appear. Serologically, HAV infection can be 
diagnosed as ALT level rise initially with the appearance of symptoms. Then, 
anti-HAV IgM antibodies are produced and can be detected by enzyme- 
linked immunoassay. Then 1-3 weeks later, anti-HAV IgG antibodies are 
made, providing lifelong immunity to the host. 

Proper hand washing, avoidance of contaminated food and water, and the 
administration of a vaccine for travelers are all methods for prevention of 
HAV infection. The CDC now recommends universal vaccination against 
HAV for all children 1 year and older. Alternatively, exposed persons can be 
treated with HAV immunoglobulin intramuscularly within 14 days of expo- 
sure. Additional treatment for infected patients is supportive. 

Hepatitis B 

Hepatitis B virus (HBV) is a member of the Hepadnavirus family and has 
a DNA genome, making it unique among the hepatitis viruses. It is an 
enveloped virus with a circular and partially double-stranded DNA genome. 
The HBV virion, known as the Dane particle, consists of the genome, a viral 
DNA polymerase, and P protein, which is attached to the genome. The DNA 
polymerase also contains reverse transcriptase and ribonuclease H activity, 
allowing HBV to use an RNA intermediate during replication. 

HBV has a specific affinity for liver cells, with attachment to these cells 
being mediated by viral glycoproteins. Once attached, HBV is taken up by the 
hepatocytes and the genome is converted into fully double-stranded DNA 
which is then delivered to the nucleus. The host cell's transcription and trans- 
lation machinery are then used to make new HBV virions. These virions are 
then released from the hepatocyte via exocytosis. 

Several viral proteins can be detected during HBV infection and are useful in 
diagnosis and monitoring of disease. The virion is surrounded by a core protein 
antigen (HBcAg), and the presence of HBcAg in a patient's serum indicates that 
the patient has been exposed to HBV. Other HBV antigens include surface anti- 
gen (HBsAg) and the "e" antigen (HBeAg). HBsAg can be detected when live 
virions are present in an infection, and HBeAg is a glycoprotein cleavage prod- 
uct of the core which is shed into the serum. The presence of HBeAg and 
HBsAg correlate with active HBV infection and thus, active disease. 
Antibodies to these viral antigens can help to determine whether infection is 
recent or not. IgM anti-HBc indicates a new infection, whereas IgG anti-HBc 
indicates past infection. Figure 27-1 shows a hepatitis B serology diagram. 

HBV is the second most common type of viral hepatitis in the United 
States, and it is usually sexually transmitted. It may also be acquired parenter- 
ally, such as from intravenous drug use, or during birth, from chronically infected 
mothers. The outcome then depends on the age at which the infection was 
acquired. Up to 90 percent of infected newborns develop chronic hepatitis B 
infection, which places the affected infant at significant risk of hepatocellular 



200 



CASE FILES: MICROBIOLOGY 




100 



Weeks after exposure 
Figure 27-1. Clinical and laboratory features of acute viral hepatitis B infection. 



carcinoma later in adulthood. For those individuals infected later in life, 
approximately 95 percent of patients will recover completely without sequelae. 
Between 5 and 10 percent of patients will develop chronic hepatitis, which 
may progress to cirrhosis. Also, a chronic carrier state may be seen in which 
the virus continues to replicate but does not cause hepatic damage in the host. 
A vaccine consisting of recombinant HBsAg is available and is a scheduled 
immunization given to all infants and adolescents, as well as to persons with 
increased risk of exposure (i.e., health-care workers and IV drug users). The inci- 
dence of HB V infection has decreased with the onset of the HB V vaccine and the 
screening of pregnant women prior to delivery. Yet, HB V remains in high rates in 
Southeast Asia and the Mediterranean areas. Nonimmunized persons exposed to 
HBV can be treated with immunoglobulin within 1 week of exposure. HBV 
infection can be treated with reverse transcriptase inhibitors or a-interferon. 

Hepatitis C 

Hepatitis C virus (HCV) is a member of the Flaviviridae family. It is a 

lipoprotein-enveloped virus with a positive-sense RNA genome. There are 
hundreds of HCV genotypes as a result of a hypervariable region in the enve- 
lope region. The virus is more or less virulent depending on the hypervariable 
region, thus making it very difficult to produce an effective vaccine against 
HCV. The virus enters cells through endocytosis after binding to the CD8 1 sur- 
face receptor. The acidity of the endosome causes fusion of the viral envelope 
with the endosomal membrane and results in release of the viral RNA into the 
host cytoplasm. The viral RNA acts as messenger RNA, directing the production 
of the viral polyprotein. The polyprotein anchors to the host cell endoplasmic 
reticulum and the virus remains cell-associated. The HCV proteins inhibit 
apoptosis and the action of interferon-a. By remaining associated with the cell 
and inhibiting apoptosis, HCV can cause chronic infection and persistent liver 



CLINICAL CASES 207 

disease. The incubation period for infection can vary from 2 to 26 weeks, with 
an average of 6-7 weeks. 

Most initial HCV infections are asymptomatic or result in mild nonspe- 
cific symptoms such as malaise or abdominal pain. On initial infection, approx- 
imately 15 percent of persons will develop an acute hepatitis syndrome and 
recover completely. More than 70 percent of infected patients will be asymp- 
tomatic, yet many will progress to chronic hepatitis. Yet, another 1 5 percent 
of infected patients rapidly develop cirrhosis. Chronic infection can also 
progress to more serious disease including: cirrhosis, liver failure, or hepato- 
cellular carcinoma. The high percentage of infections that are asymptomatic 
also contributes to the spread of the virus in the population. 

HCV infection is diagnosed by demonstrating the presence of circulating 
IgG antibodies to HCV antigens through enzyme immunoassay. Unfortunately, 
these antibodies may not be detected until up to 4 months postinfection, making 
it difficult to diagnose an acute HCV infection. Additionally, such tests cannot 
distinguish between acute, chronic, or resolved HCV infections. Alternatively, 
reverse transcriptase polymerase chain reaction (RT-PCR) testing can be 
used to diagnose infection and to quantitate the amount of circulating HCV RNA 
in an infected person. This assay serves as both a measure of HCV disease activ- 
ity and as a monitor of response to therapy. 

Numerous antiviral agents are used as therapy for HCV infection, including 
recombinant interferon-a, which helps to induce host antiviral and antipro- 
liferative activity. End-stage chronic HCV hepatitis may require liver trans- 
plantation; however, the risk of graft reinfection is 50 percent for HCV. 
Currently, there is no effective vaccine to prevent HCV infection. However, the 
establishment of routine screening of donated blood and organs has reduced 
the spread of HCV via these modes of transfer. 

Hepatitis D 

Hepatitis D virus (HDV) is a defective RNA virus that requires the presence 

of the hepatitis B virus to replicate. Specifically, it lacks genes for envelope 
proteins, and thus to replicate it requires infection with HBV It then consists of 
an envelope (provided by HBV) with HBsAg, delta antigen, and single-stranded, 
circular RNA. If infection with HDV occurs during a superinfection of preex- 
isting HBV, there is a higher risk of chronic liver infection and chronic HDV 
infection. This type of superinfection is also more likely to lead to fulminant 
hepatitis and has a 5-15 percent mortality rate. However, when HDV coinfects 
a person simultaneously with HBV, it typically presents as severe acute disease 
with a low risk of developing chronic liver infection or mortality. HDV is spread 
similarly to HBV, via percutaneously, mucosally, or through sexual contact. 

Laboratory diagnosis of HDV is made by detection of the RNA genome 
(via RT-PCR) or the delta antigen (via enzyme-linked immunosorbent assay, 
ELISA) from blood samples. Additionally, anti-HDV antibodies can also be 
detected by ELISA; however, antibodies are present only transiently. To prevent 
coinfection with HBV, prophylaxis to HBV can be administered. To prevent a 



202 CASE FILES: MICROBIOLOGY 

HDV superinfection, it is important to educate HBV-positive patients about 
reducing risk factors for infection. The only treatment available for HDV 
infection is a-interferon, which lessens clinical symptoms. 

Hepatitis E 

Hepatitis E virus (HEV), also called "enteric non-A, non-B hepatitis," is a 

member of the Caliciviridae family. It is nonenveloped, and its genome con- 
sists of linear, positive-sense, single-stranded RNA. HEV is similar in many 
ways to HAV Both are transmitted by the fecal-oral route, most frequently 
through contaminated water sources. It is not endemic to the United States and 
is therefore seen most often in travelers. The average incubation period is 40 
days. HEV infection is most often diagnosed by exclusion, because laboratory 
testing is not available. 

Like HAV, it has a low mortality rate (1-2 percent), except for infection 
in pregnancy where a 15-25 percent mortality rate is noted, and there is no 
chronic stage. Like all of the hepatitis viruses, the clinical severity of infec- 
tion increases with the age of the patient. The immunological response is also 
similar to that of HAV. To protect from HEV infection while traveling to 
endemic areas, travelers are advised not to drink the water (or ice) and not to eat 
unpeeled fruits or vegetables. There is no vaccine available, and immunoglobu- 
lin does not prevent infection with HEV. 

Hepatitis G 

The hepatitis G virus (HGV), has been more recently identified. As a mem- 
ber of the Flavivirus family it resembles HCV in its viral structure (positive- 
sense RNA genome), transmission (blood-borne) and in its high production of 
chronic hepatitis disease. Lab diagnosis involves identifying the HGV genome 
via RNA detection methods (i.e., RT-PCR). 

Hepatitis Serologies 

Clinical presentation does not reliably establish the viral etiology, so serologic 
studies are used to establish a diagnosis. Antihepatitis A IgM establishes an 
acute hepatitis A infection. If Antihepatitis C antibody is present, an acute hep- 
atitis C is diagnosed, but it may be negative for several months. The hepatitis C 
PCR assay, which becomes positive earlier in the disease course, often aids in the 
diagnosis. Acute hepatitis B infection is diagnosed by the presence of hepatitis B 
surface antigen (HBsAg) in the clinical context of elevated serum transaminase 
levels and jaundice. HBsAg later disappears when the antibody (anti-HBs) is 
produced. 

There is often an interval of a few weeks between the disappearance of 
HBsAg and the appearance of anti-HBsAb, which is referred to as the "window 
period." During this interval, the presence of antihepatitis B core antigen IgM 



CLINICAL CASES 203 

(anti-HBc IgM ), will prove indicate an acute hepatitis B infection. Hepatitis B 
precore antigen (HBeAg) represents a high level of viral replication. It is almost 
always present during acute infection, but its persistence after 6 weeks of ill- 
ness is a sign of chronic infection and high infectivity. Persistence of HBsAg or 
HBeAg are markers for chronic hepatitis or a chronic carrier state; elevated or 
normal serum transaminase levels distinguish between these two entities, 
respectively. 

COMPREHENSION QUESTIONS 

[27 . 1 ] A 33-year-old nurse suffered a needle stick injury. The patient used illicit 
intravenous drugs. One month later, the nurse develops jaundice. Which 
of the following findings would implicate hepatitis B as the etiology? 

A. Positive antihepatitis B surface antibody 

B. Positive antihepatitis B-core antibody 

C. Positive hepatitis B surface antigen 

D. Positive antihepatitis A antibody 

[27.2] A 25-year-old male tests positive for a hepatitis C infection. Which of 
the following is the most likely method of transmission? 

A. Fecal-oral 

B. Fomite 

C. Intravenous drug (needles) 

D. Sexual transmission 

[27.3] A 12-year-old teenager is brought into the emergency room with skin 
"turning yellow" and abdominal discomfort. The liver function tests 
reveal serum transaminase levels in the 2000 IU/L range. Which of the 
following is the most accurate statement about probable complications? 

A. Significant likelihood of hepatocellular carcinoma 

B. Almost no chance of long-term sequelae 

C. About a 10 percent chance of a chronic carrier state 

D. Long-term complications usually respond to a-interferon therapy 

[27.4] A 28-year-old woman presents with symptoms of jaundice, right upper 
quadrant pain, and vomiting. She also has elevated ALT. It is deter- 
mined that she acquired hepatitis A from a church picnic where several 
other adults also became infected. What should be done to protect the 
family members? 

A. One dose of HAV immunoglobulin should be administered intra- 
muscularly. 

B. No treatment is necessary. 

C. A series of three vaccinations should be administered at 0, 1, and 6 
months. 

D. a-Interferon should be administered. 

E. Household contacts should be quarantined and observed. 



204 CASE FILES: MICROBIOLOGY 

Answers 

[27.1] C. The presence of hepatitis B surface antigen means actively repli- 
cating virus, and in the context of the recent needle stick injury, this 
likely represents a hepatitis B infection. The presence of HBeAg is a 
marker of active disease and infectivity. For example, pregnant moth- 
ers infected with HBV who have the absence of serum HBeAg, there 
is a greater than 10 percent transmission rate to the fetus, whereas in 
pregnant mothers with HBeAg in their serum, there is a greater than 
90 percent transmission rate to the fetus. 

[27.2] C. Intravenous drug use is the primary method of transmission of 
hepatitis C virus. 

[27.3] B. This is most likely hepatitis A infection, which carries a very low 
chance of long-term sequelae. 

[27.4] A. HAV immunoglobulin should be given to household contacts in 
one IM dose. This must be done within 14 days of exposure to the 
index patient as prophylaxis against hepatitis A. Answer (C), the series 
of three vaccinations at time 0, 1, and 6 months, refers to the immu- 
nization schedule for hepatitis B, not hepatitis A. This would not be 
protective for those exposed to HAV. (D) a-Interferon is used to treat 
symptomatic patients with HBV and HCV, not prophylaxis of family 
members of patients with HAV. (E) Quarantining the household con- 
tacts of the patient is not the appropriate treatment. 



MICROBIOLOGY PEARLS 

*♦* HCV is an enveloped virus with a positive-sense RNA genome. 
*♦* HCV transmission occurs primarily via infected blood/parenteral 

transmission. 
*♦* Although most acute infections are asymptomatic, HCV produces 

high rates of chronic infection and mortality. 
*♦* Treatment of HCV infection includes recombinant interferon-a. 
*♦* No vaccine is available for HCV. 



CLINICAL CASES 205 



REFERENCES 

Brooks GF, Butel JS, Morse SA. Jawetz, Melnick, & Adelberg's Medical 

Microbiology, 23rd ed. New York: McGraw-Hill, 2004:466-86. 
Howley PM, Knipe DM. Fields Virology, 4th ed. Philadelphia, PA:Lippincott 

Williams &Wilkins, 2001. 
Murray PR, Rosenthal KS, Pfaller MA. Medical Microbiology, 5th ed. St. Louis, 

MO: Mosby, 2005:675-90. 
Ryan JR, Ray CG. Sherris Medical Microbiology, 4th ed. New York: McGraw-Hill, 

2004:541-53. 
National Institute of Allergy and Infectious Disease. Hepatitis, www.niaid.nih.gov/ 

publications/hepatitis .htm 



This page intentionally left blank 



♦♦♦ CASE 28 



The mother of a 3-year-old girl brings the child in for the evaluation of a 
"wart" on her thumb. It has been present for 3 or 4 days and seems to cause 
some pain. The week prior, the child had a "head cold" and "cold sores" 
around her mouth, all of which have resolved. She has never had warts, and 
the mother says that the child is otherwise healthy. On examination, you see a 
well appearing child who is sitting in her mother's lap and sucking her thumb. 
Her head and neck exam is normal. On her left thumb, just proximal to the 
base of the thumbnail, is the lesion about which the mother is concerned. It is 
a cluster of small vesicles with a faint area of surrounding erythema. The 
remainder of the child's examination is normal. 

^ What virus is the most likely cause of this skin lesion? 

^ How was it transmitted to this patient's thumb? 



208 CASE FILES: MICROBIOLOGY 

ANSWERS TO CASE 28: HERPES SIMPLEX VIRUSES 

Summary: A 3-year-old girl had "cold sores" previously and now a cluster of 
small vesicles with a faint area of surrounding erythema on the thumb, con- 
sistent with herpetic whitlow. 

^ Most likely viral cause of this skin lesion: The most likely cause of 
the girl's skin lesion is herpes simplex type 1 (HSV-1). 

^ How was it transmitted to this patient's thumb: The patient most 

likely acquired the infection at this secondary site via self-inoculation of 
the skin by sucking her thumb. 

CLINICAL CORRELATION 

There are two serotypes of herpes simplex viruses, types 1 and 2 (HSV-1 and 
HSV-2), which both cause vesicular lesions via infection of mucosal mem- 
branes and or compromised epithelial cells. Both HSV-1 and HSV-2 are 
known to replicate in the basal epithelium of these vesicular lesions and then 
establish latent and recurring infections within the innervating neurons of 
these cells. HSV primarily cause clinical symptoms at the site of inoculation 
of the virus. Although there is some overlap, HSV-1 tends to cause disease 
above the waist, and HSV-2, which is more commonly transmitted via sexual 
contact, causes disease below the waist. 

APPROACH TO SUSPECTED HSV INFECTION 
Objectives 

1. Know the structure and characteristics of HSV. 

2. Know the clinical disease caused by HSV, mode of transmission, and 
strategies for treatment. 

Definitions 

Vesicular lesions: Small, blister-like lesions filled with clear fluid. 

Syncytia: Fusion of neighboring cells infected with virus, resulting in 
multinucleated giant cells. 

Gingivostomatitis: Localized inflammation and or ulcerative lesions in the 
mucous membranes of the oral cavity. 

Prodrome: Early symptoms of HSV infection, including itching and tin- 
gling of skin 12-24 hours prior to lesion formation. 



CLINICAL CASES 209 

DISCUSSION 

Characteristics of HSV That Impact Transmission 

The herpes simplex viruses (HSV) are members of the Alphavirinae sub- 
family of human herpesviruses. As with other herpesviruses, they are large, 
enveloped viruses containing linear, double-stranded DNA genomes sur- 
rounded by an icosadeltahedral nucleocapsid, with a protein tegument (space 
containing viral enzymes and transcription factors) located between the capsid 
and viral envelope. The structures of the HSV-1 and HSV-2 genomes are sim- 
ilar and share approximately 50 percent homology. They can infect many cell 
types in humans and in other animals. They tend to cause lytic infections in 
fibroblast and epithelial cells and latent infections in neurons. HSV enters host 
cells via fusion at the cell membrane and releases gene transcription proteins, 
protein kinases, and proteins that are cytotoxic to the host cell. Once inside the 
host, the viral genome is delivered to the nucleus of the cell. HSV uses the host 
transcription/translation machinery; yet it is a viral-encoded DNA polymerase 
that replicates the DNA genome. Newly formed enveloped virions are released 
from the cell by either exocytosis or cell lysis. 

The virus enters through mucosal membranes or breaks in the skin. It 
replicates in cells at the infection site and then establishes latent infection of 
the neuron that innervates the primarily infection site via retrograde transport. 
HSV avoids antibody-mediated defenses by cell-to-cell spread by the forma- 
tion of syncytia. Cell-mediated immunity is necessary for control of HSV 
infections, and persons with impaired cellular immunity can get more severe 
and diffuse disease. The latent infection of neurons also helps the virus to 
avoid host defenses and provides the potential for recurrent disease. 
Recurrences can be triggered by many events, including stress and other ill- 
nesses. Recurrent HSV disease is usually less severe than primary disease 
because of the memory response of the host immune system. HSV-1 tends to 
be transmitted via contact with saliva or direct contact with skin or mucous 
membrane lesions. It causes gingivostomatitis, cold sores, and pharyngitis. 
Herpetic whitlow, an infection of the finger with HSV-1, results from direct 
contact with herpes lesions and is most commonly seen in children who suck 
their fingers or in health-care workers who care for infected patients. 

Diagnosis 

Clinical signs of HSV-1 and HSV-2 infections include (1) oropharyngeal dis- 
ease, with symptoms of fever, sore throat, gingivostomatitis, and submandibu- 
lar lymphadenopathy; (2) keratoconjunctivitis, with recurrent lesions of the 
eye and eyelid; (3) cutaneous infections, with vesicular lesions of the mouth, 
fingers, and genital tract (Figure 28-1); and (4) encephalitis. Neonatal infec- 
tions occur most commonly during vaginal delivery in pregnant mothers expe- 
riencing primary or recurrent genital lesions. HSV neonatal infections are 



210 



CASE FILES: MICROBIOLOGY 




Figure 28-1. First episode primary genital herpes simplex virus infection. 
{Reproduced, with permission, from Cunningham FG, et al. William's 
Obstetrics, 21st ed. New York: McGraw-Hill, 2001:1495.) 



nearly always symptomatic and have high mortality rates if not promptly diag- 
nosed and treated. Signs of infection include localized vesicular lesions of the 
skin, eye or mouth, encephalitis, and/or disseminated disease. 

Cytopathologically, HSV can be diagnosed by visualizing multinucleated 
giant cells on direct examination of cells from the base of a vesicular lesion, 
referred to as a Tzanck smear. However, this assay lacks both sensitivity and 
specificity, because it does not distinguish among HSV-1, HSV-2, and 
varicella-zoster virus (VZV) infections. Isolation of virus from herpetic 
lesions, cerebral spinal fluid, or stool specimens remains the definitive diag- 
nostic approach, where the appearance of characteristic HSV cytopathic 
effects on cells can be observed. Other rapid diagnostic tests include: detect- 
ing viral antigen (via immunofluorescence) or DNA (via PCR) in tissue sam- 
ples or vesicle fluid. HSV-1 and HSV-2 serotyping can be performed by 
several biochemical, nucleic acid, or immunologic methods, with DNA probe 
analysis being the most widely used in current clinical practice. 



CLINICAL CASES 211 

Treatment and Prevention 

Several antiviral drugs have been developed to treat HSV infections, including 
acyclovir, valacyclovir, and famciclovir. All of these drugs function as 
inhibitors of viral DNA synthesis and are capable of shortening the duration of 
clinical symptoms and suppressing viral reactivation. 

Prevention of HSV infection relies on the avoidance of direct contact with 
the virus or viral lesions. Asymptomatic shedding of virus can occur in infected 
persons in saliva, urethral, and cervical sources, and because only about one- 
fourth of individuals infected with HSV know that they are infected, safe sex 
practices are highly recommended to avoid spread. The vast majority of HSV 
infections in newborns can be prevented by cesarean delivery of neonates in 
women experiencing primary HSV-2 infection or recurrent genital lesions. This 
practice has significantly decreased the rate of neonatal infection and mortality. 
Additionally, experimental recombinant HSV-2 vaccines are currently being 
developed and tested. One prospective trial has shown efficacy in preventing 
genital herpes infections in HSV-1 and HSV-2 seronegative women. 

COMPREHENSION QUESTIONS 

[28.1] Which of the following cell types are specific to a latent genital infec- 
tion with HSV-2? 

A. Trigeminal ganglia 

B. Sacral ganglia 

C. Vagal nerve ganglia 

D. Neural sensory ganglia 

[28.2] Which of the following viruses, in addition to HSV-1 and HSV-2, pro- 
duces the cytopathologic findings of multinucleated giant cells? 

A. Adenovirus 

B. Cytomegalovirus 

C. Epstein-Barr virus 

D. Human papillomavirus 

E. Varicella-zoster virus 

[28.3] Which of the following statements most accurately describes HSV 
infections? 

A. HSV establishes lytic infection in neural ganglion cells. 

B. Latent HSV infections can be prevented in persons with functional 
cell-mediated immunity. 

C. Primary and recurrent HSV infections are treated with drugs that 
inhibit the viral DNA polymerase. 

D. HSV infection is transmitted via direct contact with symptomatic 
shedding of viral particles in active lesions. 

E. Severe neonatal herpes infections are commonly associated with in 
utero transmission. 



212 CASE FILES: MICROBIOLOGY 

Answers 

[28.1] B. Latent infection by HSV-2 has been shown to occur primarily in 
the sacral ganglia, whereas HSV-1 latency has been demonstrated in 
trigeminal, superior cervical, and vagal nerve ganglia. Varicella- 
zoster virus remains latent in neural sensory ganglia. 

[28.2] E. A Tzanck smear assay can be used to identify the characteristic 
cytopathologic effects of multinucleated giant cells in herpetic skin 
lesions; however, this assay cannot distinguish among HSV-1, HSV-2, 
and VZV infections. 

[28.3] C. Most antiviral therapies for HSV are nucleoside analogues or 
other inhibitors of the viral DNA polymerase; answers A, B, D, and 
E are incorrect: HSV establishes lytic infections in fibroblast and 
epithelial cells and latent infections in neurons; infection with HSV 
results in lifelong latent infection even in persons with functional 
cell-mediated immunity; HSV is transmitted most commonly from 
direct contact with active lesions (however, virus may be shed asymp- 
tomatically in saliva and urethral and cervical fluids); although in 
utero transmission of HSV is possible, it is very uncommon, and most 
neonatal HSV infections occur via vaginal delivery in mothers with 
primary genital infections. 



MICROBIOLOGY PEARLS 



*♦* Clinical manifestations include painful vesicular lesions of the 



mouth, fingers, and genital tract. 

*♦* Characteristic viral cytopathology includes syncytia, cells with "bal- 
looning" cytoplasm, and Cowdry A type inclusion bodies. 

*♦* Diagnosis of infection is made by viral isolation; HSV-1 and HSV-2 
serotyping is performed by nucleic acid restriction mapping or 



♦♦♦ 



DNA probe analysis. 
Effective treatment of primary and recurrent infections is with 
viral DNA polymerase inhibitors: acyclovir, valacyclovir, and 
famciclovir. 



REFERENCES 

Murray PR, Rosenthal KS, Pfaller MA. Medical Microbiology, 5th ed. St. Louis, 

MO: Mosby, 2005:543-50. 
Ryan JR, Ray CG. Sherris Medical Microbiology, 4th ed. New York: McGraw-Hill, 

2004:555-62. 
Whitley, RJ, Kimberlin, DW, Roizman, B. Herpes simplex viruses. J Clin Infect Dis 

1998:26:541-55. 



♦> CASE 29 



A 28-year-old man presents to the physician's office for evaluation of a rash 
on his chest. He started with one oval-shaped purplish area that he thought was 
a bruise but has subsequently developed multiple new lesions. The growths 
don't hurt, itch, or bleed, but he continues to get new ones, and the existing 
ones are getting larger. He has never had anything like this before, has no his- 
tory of allergies and denies exposure to any new medications, foods, lotions, 
or soaps. His past medical and family histories are unremarkable. His review 
of systems is significant for a 15-lb weight loss in the past 2 months, approx- 
imately 6 weeks of diarrhea, and a 3-week history of a sore throat. On exami- 
nation, he is a thin but generally well appearing male. His vital signs are 
normal. Examination of his pharynx shows thick white plaques on the poste- 
rior pharynx and soft palate. On the skin of his chest are multiple oval-shaped 
purple or brown macules. They are firm on palpation and vary in size from 0.5 
to 4 cm in length. Several of them appear to be growing together into larger, 
confluent plaques. You perform a punch biopsy of one of the lesions. In 5 days 
you get the pathology report with the diagnosis of Kaposi sarcoma. 

^ With what virus is this patient likely infected? 

^ What specific cell types are most commonly infected with this 

virus? What cell surface receptor is the binding site of this virus? 

^ What serologic testing is most frequently performed to make this 
diagnosis? 



214 CASE FILES: MICROBIOLOGY 

ANSWERS TO CASE 29: HUMAN 
IMMUNODEFICIENCY VIRUS 

Summary: A 29-year-old man has weight loss, white plaques on the pharynx, 
and purple lesions on the abdomen, which on biopsy reveals Kaposi sarcoma. 

^ Virus with which this patient is most likely infected: Human 
immunodeficiency virus (HIV) 

^ Specific cells infected by and binding site of HIV: CD4 surface 
receptor protein on macrophages and T lymphocytes 

^ Serologic testing to confirm diagnosis: HIV enzyme-linked 

immunosorbent assay (ELISA) and Western-blot analysis, or PCR 

CLINICAL CORRELATION 

The HIV is a human retrovirus in the Lentivirinae subfamily. It is a spherical, 
enveloped RNA virus with a cone-shaped capsid that contains two copies of a 
positive-strand RNA genome. HIV infects cells of macrophage lineage and 
helper T cells by binding to the CD4 surface receptor protein on these target 
cells, resulting in fusion of the viral envelope with the cellular plasma mem- 
brane to gain entry. On entry into the host cell cytoplasm, an RNA-dependent 
DNA polymerase enzyme (reverse transcriptase), which is present in the viral 
capsid, uses the viral RNA to synthesize viral DNA. The viral DNA is trans- 
ported to the host nucleus, where it is spliced into the host genome. The inte- 
grated viral DNA acts as a host cellular gene and is transcribed by host RNA 
polymerase II to produce new copies of viral RNA and proteins, which assem- 
ble into new HIV virions. HIV initially infects cells of macrophage lineage, but 
quickly reaches the lymph nodes where CD4 T cells are infected. The immuno- 
suppression caused by HIV is primarily caused by a reduction in the helper and 
delayed type hypersensitivity responses mediated by CD4 T cells. Infected 
macrophages probably serve as reservoirs and means of distribution of HIV. 
HIV avoids the host immune system in several ways. Infection of macrophages 
and helper T cells inactivates central components of the host immune system. 
Also, HIV has an intrinsic genetic instability as a result of errors caused by 
reverse transcriptase which may contribute to an antigenic drift in the virus, 
resulting in reduced host immune system recognition. Symptomatic disease 
caused by HIV is proportionate to the loss of CD4 T cells and the resulting 
immune dysfunction. Acquired Immune Deficiency Syndrome (AIDS) is 
defined by the presence of HIV, a reduction of CD4 T cells, and the acquisition 
of characteristic opportunistic infections. Serologic diagnosis of HIV infection 
is primarily made by ELISA testing and, when this is positive, confirmation by 
Western blot analysis. Current HIV treatment involves using medications, indi- 
vidually or in combinations, which interfere with the actions of reverse tran- 
scriptase and block the proteases that activate the virion. 



CLINICAL CASES 215 

APPROACH TO HUMAN IMMUNODEFICIENCY VIRUS 
Objectives 

1. Know the structure and characteristics of the human immunodefi- 
ciency virus. 

2. Know the mechanism by which HIV is transmitted, infects target cells, 
replicates, and causes immune deficiency. 

3. Know the diagnosis, treatment, and opportunistic infections associated 
with AIDS. 

Definitions 

Retrovirus: An RNA virus. Retroviruses have an enzyme called reverse 
transcriptase that gives them the unique property of transcribing RNA 
into DNA. The retroviral DNA can then integrate into the chromosomal 
DNA of the host cell to be expressed there. 

HAART: Highly active antiretroviral therapy. It is the treatment for human 
immunodeficiency virus (HIV) infection that uses a combination of sev- 
eral antiretroviral drugs. The drugs inhibit the ability of the virus to mul- 
tiply in the body, and they slow down the development of AIDS. 

AIDS: Acquired immunodeficiency syndrome. A cell-mediated immunity 
caused by infection by the HIV leading to depletion of CD4 helper T 
lymphocyte cells. 

DISCUSSION 

Characteristics of Human Immunodeficiency Virus 

HIV appears to have been derived from primate (chimpanzee, especially) 
lentiviruses and are the etiologic cause of AIDS. AIDS was described in 1981, 
and the virus was isolated in 1983. AIDS is one of the most significant public 
health problems worldwide at the current time. 

HIV is a retrovirus (reverse transcriptase or RNA-dependent DNA poly- 
merase) in the lentivirus subgroup. It is a medium-sized virus (approximately 
100 nm) with two copies of a positive-sense (same as messenger RNA 
[mRNA]) single-stranded RNA genome. This genome is the most complex of 
all retroviruses. The lipid envelope contains glycoproteins that undergo anti- 
genic variation, making vaccine development difficult, if not impossible, at 
the present time. Protease enzymes are coded for by the viral genome, and 
these are required for the production of infectious viruses. The reverse tran- 
scriptase makes a double-stranded DNA copy (provirus) of the viral genomic 
RNA, which is incorporated into a host chromosome. The proviral DNA later 
serves as a template for viral mRNA's and new virion genomes. Virions bud 
from the plasma membrane of the host cell. Heterogeneous populations of 



216 CASE FILES: MICROBIOLOGY 

viral genomes are found in an infected individual, especially the env gene, 
which codes for envelope glycoproteins. The gpl20 viral receptor contains 
binding domains responsible for viral attachment to the CD4 molecule (host 
receptor) and coreceptors and detennines cell tropisms (lymphocytes versus 
macrophages). These glycoproteins cause antibodies to be formed by the host 
and are only weakly neutralizing to the virus. The gp41 product contains a 
transmembrane domain that anchors the glycoprotein in the viral envelope 
and a fusion domain that facilitates viral entry into the target (host) cells. The 
virus is inactivated by treatment at room temperature for 10 minutes by any of 
the following: 10 percent bleach, 50 percent ethanol, 35 percent isopropanol, 
0.5 percent paraformaldehyde, or 0.3 percent hydrogen peroxide. HIV in 
blood in a needle or syringe, however, requires exposure to undiluted bleach 
for 30-60 seconds for inactivation. Heating at 56°C (132. 8°F) for 10 minutes 
(same as for complement inactivation) will inactivate HIV in 10 percent 
serum, but HIV in dried protein-containing mixtures is protected. Lyophilized 
blood products need to be heated to 68°C (154.4°F) for 72 hours to ensure 
inactivation of contaminating viruses. 

Diagnosis 

HIV infection can be diagnosed by virus isolation, detection of antiviral anti- 
bodies, or measurement of viral nucleic acid or antigens. HIV may be cul- 
tured from lymphocytes in peripheral blood primarily. Virus numbers vary 
greatly in an individual. The magnitude of plasma viremia is an excellent cor- 
relate of the clinical stage of HIV infection compared to the presence of anti- 
bodies. The most sensitive viral isolation technique requires cocultivation of 
the test sample with uninfected mitogen-stimulated peripheral blood mononu- 
clear cells. Virus growth is usually detected in 7-14 days by measuring viral 
reverse transcriptase activity or virus-specific antigens. Virus isolation of HIV 
is usually considered a research technique, and most medical center viral diag- 
nostic laboratories will not offer this service. 

Antibody detection is the most common way to diagnose HIV infection. 
Seroconversion in HIV infection is generally found to occur in approximately 
4 weeks. Most individuals are seropositive within 6-12 weeks after infec- 
tion, and essentially all will be antibody positive in 6 months. Commercially 
available enzyme-linked immunoassays (EIA, ELISA) are routinely used as 
screening tests. If done properly, the reported sensitivity and specificity are at 
least 98 percent. Two separate EIA tests need to be positive for antibodies in 
the usual screening situation, and a confirmation test (Western blot usually) 
will be done to rule out EIA false-positives. Western blot tests (also com- 
mercially available) will usually detect antibodies to viral core protein p24 or 
envelope glycoproteins gp41, gpl20, or gp 160. An oral swab test is com- 
monly used. 

Amplification assays (RT-PCR, DNA PCR. or bDNA tests) are used to 
detect viral RNA in clinical specimens. These tests may be quantitative when 



CLINICAL CASES 217 

reference standards are used in each test. These molecular-based tests are very 
sensitive and form the basis for plasma viral load measurements. HIV RNA 
levels are important predictive markers of disease progression and monitors of 
the effectiveness of antiviral therapies. 

Treatment and Prevention 

Treatment of HIV infection uses classes of drugs that inhibit the virally-coded 
reverse transcriptase and inhibitors of the viral protease enzymes. 

Unfortunately, current treatments are biostatic, not virucidal. Therapy with 
combinations of antiretroviral drugs is called highly active antiretroviral 
therapy (HAART). It appears to lower viral replication below the limits of 
laboratory detection but is not curative. The virus persists in reservoirs of long- 
lived, latently infected cells. When HAART is discontinued, viral production 
rebounds. Monotherapy usually results in the rapid emergence of drug- 
resistant mutants of HIV. HAART therapy has turned HIV infection into a 
chronic, treatable disease. Unfortunately, large numbers of HIV-infected per- 
sons worldwide do not have access to the drugs. 

A safe and effective vaccine would be the best hope for controlling HIV 
infection. Currently, many candidate vaccines are under development and in 
clinical trials. We have seen that viral vaccines are best when used in a pre- 
ventative manner. Uninfected individuals are given the vaccine and develop 
antibodies that prevent infection or disease if the wild-type virus is encoun- 
tered. HIV vaccine development is difficult because HIV mutates so rapidly. 
There appears to be so much variation in immune responses in HIV infections 
that no vaccine has been able to be protective to all individuals in a population. 
Nothing being currently developed appears to be close to approval in this area, 
although many organizations are working to produce an effective vaccine. A 
big hurdle for this, in part, is the lack of an appropriate and cost-effective lab- 
oratory animal model for HIV. The SIV-macaque model of simian AIDS is 
only partially useful for the development of a human HIV vaccine. 

COMPREHENSION QUESTIONS 

[29.1] During a medical check-up for a new insurance policy, a 60-year-old 
grandmother is found to be positive in the ELISA screening test for 
antibodies against HIV-1. She has no known risk factors for exposure 
to the virus. Which of the following is the most appropriate next step? 

A. Immediately begin therapy with azidothymidine. 

B. Perform the screening test a second time. 

C. Request that a blood culture be done by the laboratory. 

D. Tell the patient that she is likely to develop AIDS. 

E. Test the patient for Pneumocystis carinii infection. 



218 CASE FILES: MICROBIOLOGY 

[29.2] In a person with HIV-1 infection, which of the following is the most 
predictive of the patient's prognosis? 

A. CD4+ cell count 

B. CD4:CD8 cell ratio 

C. Degree of lymphadenopathy 

D. Level of HIV-1 RNA in plasma 

E. Rate of decline in anti-HIV antibody 

[29.3] Highly active antiretroviral therapy against HIV infection includes one 
or more nucleoside analogue reverse transcriptase inhibitors in combi- 
nation with representatives of which class of antiretroviral agents? 

A. Inhibitors of viral binding 

B. Inhibitors of viral protein processing 

C. Inhibitors of viral release 

D. Inhibitors of viral uncoating 

E. Nonnucleoside antiretroviral agents 

[29.4] Which of the following is the pathogen responsible for blindness in 
advanced HIV infections? 

A. Cytomegalovirus 

B. Epstein-Barr virus 

C. Fungus 

D. Toxoplasma 

Answers 

[29.1] B. Because HIV cannot be safely isolated and grown in the standard 
medical center diagnostic laboratory, diagnosis of HIV infections 
relies on detection of antibodies against the virus. The standard 
screening test is done by ELISA (enzyme-linked immunosorbent 
assay). ELISA test formats are quite reliable and accurate and can be 
used for antibody or antigen detection. By definition, however, 
screening tests are not 100 percent accurate for sensitivity and speci- 
ficity. HIV infection, especially, is a tragic infection that requires 
utmost accuracy in laboratory diagnosis results to aid the physician in 
counseling the involved patient and family. Under the conditions 
described in Question 29. 1, no known risk factors for HIV contact are 
claimed or identified. For this situation and any other requiring diag- 
nostic laboratory testing for HIV infection, extra effort is taken to 
ensure accuracy and correct results. Because it is widely accepted 
that HIV ELISA screening is not 100 percent sensitive and specific 
(approximately 98 percent accurate, however), a second blood sam- 
ple is collected for retesting by ELISA. If both ELISA results are pos- 
itive, a second confirming test is done. This is usually a Western blot 
technique. If the Western blot test is positive, then HIV infection is 
confirmed and related to the patient. 



CLINICAL CASES 219 

[29.2] D. Amplification assays (RT-PCR, DNA PCR, and b DNA tests) are 
routinely used to detect viral RNA in clinical specimens. The tests 
can be quantitative when reference standards are used, and appropri- 
ate positive and negative controls must be included in each test. 
Because these molecular based tests are very sensitive, they form the 
basis for plasma viral load determinations. It is generally agreed that 
the amount of HIV in the blood (viral load) is of significant prognos- 
tic value. There are continual rounds of viral replication and cell 
killing in each patient, and the steady-state level of virus in the blood 
varies with individuals. A single measurement of plasma viral load 
approximately 6 months after infection can predict the risk of devel- 
opment of AIDS in men several years later. In women, viral load 
appears to be less predictive. The plasma viral load appears to be the 
best predictor of long-term clinical outcome, whereas CD4 lympho- 
cyte counts are the best predictor of short-term risk of developing an 
opportunistic disease. Plasma viral load measurements are a critical 
element in assessing the effectiveness of antiretroviral drug therapy. 

[29.3] B. A growing number of drugs have been approved for treatment of 
HIV infections. It must be remembered that all HIV drug treatments 
are only virostatic and not virucidal at this point in time. Classes of 
drugs include nucleoside and nonnucleoside inhibitors of the viral 
reverse transcriptase and inhibitors of the viral protease enzyme. The 
protease inhibitors are significant because protease activity is 
absolutely essential for production of infectious virus, and the viral 
enzyme is distinct from human cell proteases. These inhibitors 
(approved in 2003) block virus entry into host cells. 

[29.4] A. The predominant causes of morbidity and mortality among 
patients with late-stage HIV infection are opportunistic infections. 
These are defined as severe infections induced by agents that rarely 
cause disease in immune-competent individuals. Opportunistic infec- 
tions usually do not occur until CD4 T cell counts drop from normal 
(1000 cells per microliter) to less than 200 cells per microliter. The 
common opportunistic infections in untreated AIDS patients are 
caused by protozoa, fungi, bacteria, and other viruses. Coinfection 
with DNA viruses are reported to lead to enhanced expression of HIV 
in cells in vitro. Herpesvirus infections are common in AIDS patients, 
and cytomegalovirus (CMV) has been shown to produce a protein 
that acts as a chemokine receptor and is able to help HIV infect cells. 
CMV retinitis is the most common severe ocular complications of 
AIDS. 



220 CASE FILES: MICROBIOLOGY 



MICROBIOLOGY PEARLS 

*♦* HIV is a retrovirus and requires reverse transcriptase to make a 

double-stranded DNA copy of the viral genomic RNA. 
*♦* Antibody detection is the most common method of diagnosing an 



infection, 
le treatmer 
transcriptase and protease enzymes. 



*♦* The treatment of HIV largely depends on targeting viral reverse 



REFERENCES 

Brooks GF, Butel JS, Morse SA, Jawetz, Melnick, & Adelberg's Medical 

Microbiology, 23rd ed. New York: McGraw-Hill, 2004:605-22. 
Knipe DM, Howley PM. Fields Virology, 4th ed. Philadelphia, PA:Lippincott 

Williams andWilkins, 2001:1971-2094. 
Murray PR, Rosenthal KS, Pfaller MA. Medical Microbiology, 5th ed. St. Louis, 

MO: Mosby, 2005:657-74. 
Ryan JR, Ray CG. Sherris Medical Microbiology, 4th ed. New York: McGraw-Hill, 

2004:601-16. 



♦> CASE 30 



A 42-year-old woman presents to the physician's office for a routine gyneco- 
logic exam. She is feeling well and has no specific complaints at this visit. 
While reviewing your records, you see that she has not come in for a 
Papanicolaou (Pap) smear in approximately 5 years. She admits that she has 
not come in because she has been feeling fine and didn't think it was really 
necessary. She has a history of three pregnancies resulting in three full-term 
vaginal deliveries of healthy children. She was treated at the age of 22 for 
Chlamydia. She has never had an abnormal Pap smear. Her social history is 
notable for a one-pack per day smoking history for the past 25 years. She is 
divorced from her first husband and is sexually active with a live-in boyfriend 
for the past 3 years. She has had 7 sexual partners in her lifetime. Her exami- 
nation today is normal. You perform a Pap smear as part of the examination. 
The report arrives 10 days later with the diagnosis "high-grade squamous 
intraepithelial lesion." 

^ What is the most likely infectious etiology of this lesion? 

^ What specific virus types confer a high-risk of cervical neoplasia? 

^ Where on a cellular level does this organism tend to replicate in 
benign diseases? In malignancies? 



222 CASE FILES: MICROBIOLOGY 

ANSWERS TO CASE 30: HUMAN PAPILLOMAVIRUS 

Summary: A 42-year-old woman has high-grade squamous intraepithelial neo- 
plasia on a Pap smear. 

^ Most likely infectious etiology of this lesion: Human papillomavirus 
(HPV)-related infection. 

^ Specific virus types confer a high risk of cervical neoplasia: HPV types 
16 and 18 are most commonly associated with anogenital neoplasias. 

^ Location of replication in benign diseases and malignancies: The site 
of replication in benign HPV infections occurs in the host neoplasm where 
the viral DNA remains extrachromosomal. However, in HPV-related 
malignancies the viral DNA is integrated into the host genome. 

CLINICAL CORRELATION 

Human papillomavirus (HPV) preferentially infects the squamous epithelium 
of skin and mucous membranes causing epithelial proliferation and the devel- 
opment of cutaneous warts and genital, oral, and conjunctival papillomas. 
Although most HPV infections are benign, and most warts or lesions regress 
spontaneously with time, some HPV viral types have been shown to be linked 
to cervical and anogenital carcinomas (Table 30-1). Major risk factors for 
infection and progression to carcinomas include: multiple sexual partners, 
smoking, and immunosuppression. 



Table 30-1 

CLINICAL SYNDROMES AND THEIR ASSOCIATED HPV TYPES 

CLINICAL SYNDROME ASSOCIATED HPV TYPES 

Cutaneous warts: 

Plantar wart 1 

Common wart 2, 4 

Flat wart 3, 10 

Benign head & neck tumors: 

Laryngeal papilloma 6, 1 1 

Oral papilloma 6, 1 1 
Conjuncrival papilloma 1 1 

Anogenital warts: 

Condyloma acuminatum 6, 1 1 

Cervical intraepithelial neoplasia 16, 18 



CLINICAL CASES 223 



APPROACH TO SUSPECTED HPV INFECTION 
Objectives 

1 . Know the characteristics of the HPV. 

2. Know the mechanism of infection and strategies for prevention and 
treatment. 

Definitions 

Koilocytes: Enlarged keratinocytes with shrunken nuclei. 
Poikilocytosis: Presence of perinuclear cytoplasmic vacuolization and 

nuclear enlargement of epithelial cells. 
Papillomas: An epithelial neoplasm producing finger-like projections from 

the epithelial surface. 
Condylomas: Epithelial neoplasm and hyperplasia of the skin, resulting in 

the formation of a large cauliflower-like mass. 

DISCUSSION 

Characteristics of HPV That Impact Transmission 

HPV is a member of the Papillomaviridae family. Over 100 distinct types of 
HPV have been identified based on DNA sequence studies. It has circular, 
double-stranded DNA genome contained within a small, nonenveloped cap- 
sid. HPV has a predilection for infecting the squamous epithelium of skin and 
mucous membranes. HPV is transmitted from person to person by direct 
contact, sexual intercourse, or via delivery through an infected birth 
canal. As a nonenveloped virus, HPV is more environmentally resistant to 
acids, detergents, and desiccation, which allows for transmission via contami- 
nated fomites. 

HPV gains entry through breaks in the skin and replicates in the basal cell 
layer of the epithelium. HPV DNA is replicated, and the viral particles are 
assembled in the nucleus of epithelial cells with late viral gene expression 
occurring in the upper layers of differentiated keratinocytes. In benign lesions, 
such as common skin warts, the viral DNA remains extrachromosomal in 
the nucleus of the infected epithelial cell. However, more commonly in car- 
cinomas or high-grade intraepithelial lesions viral DNA becomes inte- 
grated into the host genome. The viral genome encodes transforming genes, 
which have been shown to cause the inactivation of proteins that inhibit cellu- 
lar growth, making infected cells more susceptible to mutation or other factors 
that may lead to the development of dysplasia and cancer. 

HPV DNA, primarily types 16 and 18, has been shown to be present in 
more than 95 percent of cervical carcinoma specimens. Because of their high 
occurrence in cervical cancers, these HPV types are considered to be high-risk, 



224 



CASE FILES: MICROBIOLOGY 



whereas HPV types 6 and 11 are considered low-risk and many other HPV 
types are considered benign. Yet, because many HPV-related infections (even 
those with types 16 and 18) are benign with lesions that can regress sponta- 
neously, the utility of characterizing specific HPV types in clinical specimens 
remains to be determined. 

Diagnosis 

HPV infection presents clinically with the growth of a variety of cutaneous 
warts and papillomas. Warts result from HPV replication stimulating excessive 
growth of the epidermal layers above the basal layer (Figure 30-1). The 
presence of koilocytotic squamous epithelial cells can be detected in a Pap 
smear. While the use of DNA probes and PCR can be used in diagnosis and in 
viral serotyping. 

Different types of warts (flat, plantar, or common), genital condylomas, and 
laryngeal papillomas can develop depending on the infecting viral type and the 
site of infection. Laryngeal papillomas can occur in infants born to mothers 
with active HPV genital lesions. While rare, these papillomas often require 
repeated surgical removal. Anogenital warts occur on the squamous epithelium 
of the external genitalia and anorectum and are most commonly caused by 
HPV types 6 or 1 1 , however, these lesions rarely undergo malignant transfor- 
mation. HPV types 16 and 18 are responsible for most cases of cervical 
intraepithelial neoplasia and cancer. Cervical cancer usually develops after a 
progression of cellular changes from cellular atypia to low-grade intraepithe- 
lial lesion, high-grade intraepithelial lesion and subsequently to carcinoma. 
Although the mechanisms of host defenses against HPV are not well understood, 



Normal Cell 



Papilloma 



Epidermal Cell 
Differentiation Pathway 

Stratum corneum 
(horny layer) 



Epidermis 



Stratum granulosu 
(granular layer) 

Stratum spinosum 
(prickle cell layer) 



Basal cell layer 




Viral Life 
Cycle 



•Capsid proteins 
•Virus particles 
•Viral release 

•Replicating 
viral DNA (high 
copy number) 
•Expression of 
early genes 

•Viral DNA 
(low copy 
number) 



Figure 30-1. Schematic representation of a papilloma wart. HPV is incorpo- 
rated in the basal layer and affects the maturing cells (left) and results in the 
skin wart or papilloma (right). 



CLINICAL CASES 225 

the immune system, especially cellular immunity, are important in the control 
of HPV infections. HPV diseases occur more frequently and tend to be more 
severe in immunocompromised hosts. 

Treatment and Prevention 

Although many HPV infections are benign with the resulting warts or lesions 
regressing spontaneously with time, because of the strong association of HPV 
with cervical carcinomas and transmission via vaginal delivery, physical treat- 
ment and removal of lesions is often performed. Physical treatment of warts 
and other lesions involves local cellular destruction by means of cryotherapy, 
acid application or electrocautery. Alternatively, immune stimulant therapy is 
used to promote immunologic clearance of the abnormal cells using either the 
injection of interferon or topical applications of imiquimod. Cervical cancer 
usually develops after a progression of cellular changes from cellular atypia to 
low-grade intraepithelial lesion, high-grade intraepithelial lesion, and subse- 
quently to carcinoma. The introduction of routine screening of women for cer- 
vical cancer with Pap smears has resulted in finding more abnormalities in 
earlier, more treatable stages, and a marked reduction in the death rate from cer- 
vical cancer. Most deaths from cervical cancer now occur in women who have 
not had adequate Pap smear screening. A new qaudrivalent HPV vaccine, 
Gardasil, was FDA approved in 2006. The vaccine is composed of major cap- 
sid proteins assembled into virus-like particles and provides protection against 
infection with HPV types 6, 1 1, 16, and 18, which together cause 70 percent of 
cervical cancers (HPV 16 & 18) and 90 percent of genital warts (HPV 6 & 11). 
Additionally, infection with HPV can be prevented by avoiding direct contact 
with infected skin lesions, and by safe sex practices. 

COMPREHENSION QUESTIONS 

[30.1] Which of the following types of cancers is HPV most commonly asso- 
ciated with? 



A. 


Anogenital 


B. 


Breast 


C. 


Lung 


D. 


Oral 


E. 


Prostate 



[30.2] HPV-related cervical intraepithelial neoplasia can be diagnosed by the 
presence of which of the following histologic features? 

A. Central, basophilic intranuclear cellular inclusions 

B. Cowdry type A intranuclear cellular inclusions 

C. Enlarged multinucleated cells 

D. Cytoplasmic vacuolization and nuclear enlargement of cells 

E. Numerous atypical lymphocytes 



226 CASE FILES: MICROBIOLOGY 

[30.3] Which of the following viral families is known to be causally associ- 
ated with tumor formation in healthy appearing human adults? 

A. Flaviviruses 

B. Papovaviruses 

C. Paramyxoviruses 

D. Polyoma viruses 

Answers 

[30.1] A. Of the types of cancers listed, HPV is most commonly associated 
with anogenital carcinomas, which includes cervical carcinomas. 

[30.2] D. HPV produces characteristic cytoplasmic vacuolization and 
nuclear enlargement of squamous epithelial cells, referred to as koilo- 
cytosis; answers A, B, C, and E are incorrect: Both Cowdry type A 
intranuclear inclusions and enlarged multinucleated cells can be seen 
with herpes simplex virus (HSV) and varicella-zoster virus (VZV) 
infections; central, basophilic intranuclear inclusion bodies are seen 
in CMV infections, whereas the presence of atypical lymphocytes is 
seen specifically in Epstein-Barr virus (EBV) infections. 

[30.3] B. HPV is a member of the Papillomaviridae family and is causally 
associated with cervical cancer in otherwise healthy individuals; 
answers A, C, D, and E are incorrect: Hepatitis C virus is a member 
of the flaviviruses family and causes chronic hepatitis and in severe 
cases is a factor in liver cancer development; paramyxoviruses 
include agents such as respiratory syncytial virus and measles virus 
and are not associated with carcinomas; human polyoma viruses 
include BK and JC viruses, which have been associated with 
immunocompromised patients, and their role in formation of human 
tumors is still under investigation. 



MICROBIOLOGY PEARLS 



*♦* HPV has a tropism for squamous epithelium of skin and mucous 

membranes. 
*♦* HPV types 16 and 18 are most commonly associated with cervical 

carcinomas. 
*♦* Treatment of HPV-related lesions include immunologic agents, 

cryotherapy, acid application, and electrocautery. 



CLINICAL CASES 227 

REFERENCES 

Bosch FX, Lorincz A, Meijer CJ, et al. The causal relationship between human 

papillomavirus and cervical cancer. J Clin Pathol 2002;55:244. 
Murray PR, Rosenthal KS, Pfaller MA. Medical Microbiology, 5th ed. St. Louis, 

MO: Mosby, 2005:523-4. 
Ryan JR, Ray CG. Sherris Medical Microbiology, 4th ed. New York: McGraw-Hill, 

2004:617-20. 
National Institute of Allergy and Infectious Disease. Human papillomavirus. 

www.niaid.nih.gov/factsheets/stdhpv.htm 
National Cancer Institute. About HPV vaccines, www.cancer.gov/cancertopics/ 

hpv-vaccines 



This page intentionally left blank 



♦:♦ CASE 31 



A 4-year-old girl is brought in by her mother for the evaluation of multiple 
skin growths on her neck and upper chest. They have been present for a month 
or two. They are not pruritic or painful. The mother thinks that they are "pim- 
ples" because she squeezed a couple of them and some white material was 
expressed. She has been treating the lesions with an over-the-counter acne 
preparation, but it hasn't helped. The child has no significant medical history, 
takes no medications, and has no allergies. On examination you see multiple dis- 
crete, isolated 1-2 -mm papules on her upper chest and lower neck. They are flesh 
colored, have a central umbilication, and feel firm on palpation. The remainder of 
her examination is normal. You suspect Molluscum contagiosum. 

^ How did the girl most likely acquire this infection? 

^ What would you see microscopically on a stained slide of material 
expressed from the core of one of these lesions that would confirm 
your diagnosis? 



230 CASE FILES: MICROBIOLOGY 

ANSWERS TO CASE 31: MOLLUSCUM 
CONTAGIOSUM 

Summary: A 4-year-old girl with multiple discrete 1-2-mm papules on her 
upper chest and lower neck that are flesh colored, have a central umbilication, 
consistent with Molluscum contagiosum. 

^ Most likely mechanism of acquisition of infection: Direct contact 
with the virus or via contact with contaminated fomites. 

^ Microscopic findings of expressed material confirming diagnosis: 

Microscopic observation of the core material would show eosinophilic 
cytoplasmic inclusions, also known as molluscum bodies. 

CLINICAL CORRELATION 

Molluscum contagiosum is a poxvirus that causes benign cutaneous disease 
worldwide. It is transmitted via direct contact with infected cells or with 
objects contaminated with virus particles. It causes small wart-like growth on 
infected skin, and occurs more frequently in children than adults. 

APPROACH TO SUSPECTED MOLLUSCUM 
CONTAGIOSUM INFECTION 

Objectives 

1 . Be aware of the characteristics of the poxvirus. 

2. Be able to describe the mechanism of infection and strategies for pre- 
vention and treatment. 

Definitions 

Molluscum bodies: Eosinophilic cytoplasmic inclusions seen in epidermal 

cells infected with Molluscum contagiosum. 
Umbilicated lesions: Lesions with cup-shaped crater and a white core. 
Papules: Lesions that are raised and well circumscribed. 

DISCUSSION 

Characteristics of Molluscum Contagiosum 
That Impact Transmission 

Poxviruses are among the largest, most complex viruses known. They have 
a linear, double-stranded DNA genome that is fused at both ends. The virion 
binds to a cell surface receptor and enters the target cell by fusion of the outer 



CLINICAL CASES 231 

envelope with the cell membrane. Replication of poxviruses occurs entirely in 
the host cytoplasm, making them unique among DNA viruses and requiring 
them to provide all enzymes necessary for viral replication. For example, 
poxviruses encode proteins for mRNA synthesis, DNA synthesis, nucleotide 
scavenging, and immune escape mechanisms. Viral DNA replication and 
virion assembly occurs in cytoplasmic inclusions called Guarnieri inclu- 
sions. The newly assembled virions are released on cell lysis. 

Because the worldwide eradication of smallpox has been documented, 
Molluscum contagiosum is the only poxvirus specific for humans. Viral 
inoculation occurs through small skin abrasions, either from direct contact 
with infectious particles or via contaminated fomites. The incubation period 
for Molluscum contagiosum infection is approximately 2 weeks to 6 months. 
Some documented forms of viral spread include direct contact with lesions 
during wrestling matches, in swimming pools, sharing of towels, and sexual 
contact. Molluscum contagiosum infection is more common in children than 
in adults, and in adults it is most often transmitted by sexual contact. 
Immunocompromised persons who are infected with Molluscum contagiosum 
may develop hundreds of lesions. Patients who are at greatest risk for this pres- 
entation are those with late-stage AIDS, with a CD4 count <200 ceils/|j,L. 

Diagnosis 

Molluscum contagiosum clinically causes discrete, flesh-colored papules 
with a central umbilication. These nodular lesions most commonly form in 
groups of 5-25 occurring on the trunk, genitalia, and extremities. They are also 
known to cause "kissing lesions" via direct contact of a lesion with an unin- 
fected area of skin on the same host; such as a lesion on the lateral chest may 
cause a "kissing lesion" on the inner arm. The semisolid core of these lesions 
can be expressed and examined microscopically for the presence of large, 
eosinophilic inclusions, known as molluscum bodies. 

Treatment and Prevention 

The lesions of Molluscum contagiosum generally develop within 2-3 months 
of contact and usually resolve within 1-2 years. Cell-mediated and humoral 
immunity both appear to be important for disease resolution. When indicated, 
the lesions can be removed by curettage, electrocautery, or liquid nitrogen 

applications. 



232 CASE FILES: MICROBIOLOGY 

COMPREHENSION QUESTIONS 

[31.1] The incidence of Molluscum contagiosum as a sexually transmitted 
disease is increasing in young adults and results in the formation of 
small wart-like lesions in the genital region. Which of the following 
viruses might also be suspected in such a case of sexually acquired 
lesions? 

A. Cytomegalovirus 

B. Varicella-zoster virus 

C. Human papillomavirus 

D. Human immunodeficiency virus 

[31.2] Which of the following statements best describes the viral characteristics 
of Molluscum contagiosum? 

A. A large and complex virus containing single-stranded linear RNA 

B. A double-stranded DNA virus encoding a DNA-dependent RNA 
polymerase 

C. A double-stranded linear DNA virus that integrates into the chro- 
mosome 

D. A single-stranded DNA virus that replicates in the host cytoplasm 

E. A double-stranded circular DNA virus 

[31.3] A sexually active 17-year-old man presents to the local free clinic to 
check some small papules that appeared on his penis. The papules are 
small and white and contain a central depression in their center. There 
is no penile discharge, nor is there pain on urination. To what group is 
the organism most likely associated with? 

A. Poxviridae 

B. Papovaviridae 

C. Adenoviridae 

D. Parvoviridae 

E. Arenaviridae 

Answers 

[31.1] C. Human papillomavirus; answers A, B, and D are incorrect; 
cytomegalovirus is known as a sexually transmitted disease but does 
not characteristically form lesions in or around the genitalia; 
varicella-zoster virus does form vesicular lesions but appears clini- 
cally as a vesicular rash of the head, trunk, or extremities in a der- 
matomal pattern, not as a genital infection; human immunodeficiency 
virus is also known to be transmitted via sexual contact, yet it mani- 
fests clinically primarily through suppression of the host immune 
response, not through the formation of genital lesions. 



CLINICAL CASES 233 

[31.2] B. Molluscum contagiosum is a poxvirus and therefore is a double- 
stranded DNA virus encoding a DNA-dependent RNA polymerase; 
answers A, C, D, and E are incorrect: (A) describes the characteris- 
tics of rabies virus; (C) more appropriately describes herpes simplex 
viruses; (D) poxviruses do replicate in the host cytoplasm, and 
Molluscum contagiosum is a double-stranded DNA virus; (E) more 
appropriately describes human papillomaviruses. 

[31.3] A. The disease in question is Molluscum contagiosum, which 
belongs to the Poxviridae and is characterized by small white papules 
with a central umbilication usually found in the genital region; 
answers B, C, D, and E are incorrect: (B) Papovaviridae include 
human papillomavirus and BK, JC polyomavirus, and although HPV 
causes genital warts, they do not have the central umbilication pres- 
ent in Molluscum contagiosum; (C) Adenoviridae include a variety of 
viral serotypes which cause respiratory, ocular, and gastrointestinal 
diseases', (D) Parvoviridae include erythema infectiosum character- 
ized by the slapped cheek appearance; (E) Arenaviridae include lym- 
phocytic choriomeningitis and Lassa virus, which are not described 
in the question stem. 



MICROBIOLOGY PEARLS 

*♦* Molluscum contagiosum is a poxvirus transmitted via direct contact 
with infected cells, sharing of towels, or via sexual contact. 

*♦* Clinical manifestations are small flesh colored umbilicated lesions 
occurring on the trunk, extremities, or genitalia. 

*♦* Lesions will resolve spontaneously over time, or can be removed via 
scraping or treatment with liquid nitrogen. 



REFERENCES 

Brooks GF, Butel JS, Morse SA. Jawetz, Melnick, & Adelberg's Medical 
Microbiology, 23rd ed. New York: McGraw-Hill, 2004:463^1. 

Murray PR, Rosenthal KS, Pfaller MA. Medical Microbiology, 5th ed. St. Louis, 
MO: Mosby, 2005:523-4. 

Ryan JR, Ray CG. Sherris Medical Microbiology, 4th ed. New York: McGraw-Hill, 
2004:528-9. 



This page intentionally left blank 



i* CASE 32 



♦> 



A 6-year-old boy is brought to your office for evaluation of fever, ear pain, and 
swollen cheeks. His mother reports that he's had 3 or 4 days of low-grade fever 
and seemed tired. Yesterday he developed the sudden onset of ear pain and 
swelling of the cheeks along with a higher fever. He is an only child, and nei- 
ther of the parents has been ill recently. He has had no significant medical ill- 
nesses in his life, but his parents decided not to give him the measles, mumps, 
rubella (MMR) vaccine because they read that it could cause autism. On exam- 
ination, his temperature is 38.6°C (101. 5°F), and his pulse is 105 beats per 
minute. He has swollen parotid glands bilaterally to the point that his earlobes 
are pushed up, and the angle of his mandible is indistinct. His tympanic mem- 
branes appear normal. Opening his mouth causes pain, but the posterior phar- 
ynx appears normal. You do note some erythema and swelling of Stensen duct. 
He has bilateral cervical adenopathy. 

^ What is the cause of this child's illness? 

^ What factor has reduced the incidence of this disease by over 
99 percent in the United States? 



236 CASE FILES: MICROBIOLOGY 

ANSWERS TO CASE 32: MUMPS 

Summary: A 6-year-old boy has tender inflammation of the parotid glands 
(parotitis) and fever. 

^ Most likely cause of this child's disease: Mumps virus 

^ Factor decreasing disease incidence by over 99 percent in the 

United States: Routine vaccination with live, attenuated mumps virus 

CLINICAL CORRELATION 

The mumps virus is primarily a childhood disease that causes acute, painful 
swelling of the parotids and other glands. It is a highly communicable disease that 
has one known serotype and infects only humans. Mumps is endemic around the 
world, with approximately 90 percent of children being infected by the age of 15. 
It is now an uncommon illness in countries such as the United States, where a live 
attenuated vaccine is widely used. The MMR vaccine, a combination vaccine of 
measles, mumps, and rubella, has resulted in a greater than 99 percent reduction 
in the incidence of mumps. Almost all cases of mumps now seen are in the unvac- 
cinated or in persons with depressed cellular immunity. 

APPROACH TO SUSPECTED MUMPS VIRUS INFECTION 
Objectives 

1. Be able to describe the characteristics of the mumps virus. 

2. Be able to describe the strategies for prevention and treatment of the 
infection. 

Definitions 

Parotitis: Inflammation of the parotids; large salivary glands located on 
each side of the face below and in front of the ear. 

Hemagglutinin-neuraminidase protein: A viral capsid glycoprotein 
involved with viral attachment, fusion, and enzymatic hydrolysis of var- 
ious proteins; also produces nonspecific agglutination of red blood cells 
used for diagnostic assay. 

Orchitis: Inflammation of the testes. 

Oophoritis: Inflammation of one or both ovaries. 



CLINICAL CASES 237 



DISCUSSION 

Characteristics of the Mumps Virus That Impact Transmission 

The mumps virus is a member of the family Paramyxoviridae. As a paramyx- 
ovirus, it is an enveloped, virus with a single-stranded, negative-sense RNA 

genome. The viral envelope contains two glycoproteins: a hemagglutinin- 
neuraminidase protein involved in attachment and a membrane fusion pro- 
tein. The mumps virus is transmitted to epithelial cells of the mouth or nose 
via direct contact with contaminated respiratory droplets or saliva or via 
fomites. The virus then fuses with the host cell membrane via the specific viral 
attachment and surface fusion proteins, which results in binding to sialic acid 
on the target cell membrane. Transcription, replication, protein synthesis, and 
assembly occur in the cytoplasm of the host cell. Newly formed virions 
acquire their outer envelope by budding through the host cell membrane and 
are released to infect other host cells. After initial infection and replication in 
the nasal or upper respiratory tract, viral infection spreads to the salivary 
glands. Virus infects the parotids or other salivary glands either by ascend- 
ing infection into the gland through Stensen duct or by viremia. Viral particles 
are also transmitted to distant organs, such as the kidneys, testes, ovaries, and 
central nervous system (CNS) through viremic spread. 

The symptoms of mumps are often the result of the inflammatory response 
of the host immune system. Many mumps infections are subclinical, and this, 
along with the fact that infected persons are contagious even 1-2 weeks prior 
to developing symptoms, promotes person-to-person spread of the disease. 
The cell-mediated immune system is responsible for defense against this 
infection and acquired immunity is lifelong. Passive immunity is transferred 
from mothers to newborns, and thus, mumps is rarely seen in infants less than 
6 months old. 



Diagnosis 

Cases of mumps are now relatively uncommon, but can be diagnosed primarily 
by clinical presentation along with a patient history that lacks mumps virus 
immunization. Clinical symptoms include acute onset of fever and malaise, 
followed with painful bilateral or unilateral swelling of the parotid or other 
salivary glands. Ten to twenty percent of cases may progress to more severe 
infections with CNS involvement, resulting in aseptic meningitis or menin- 
goencephalitis. In adolescent children and adults, additional complications may 
occur including: orchitis, oophoritis, and pancreatitis. These more severe symp- 
toms are rarer and occur primarily in immunocompromised hosts. 

Laboratory diagnosis is not typically required; however, rapid confirmation 
of mumps infection can be obtained through direct viral antigen detection via 
immunofluorescence analysis. Appropriate clinical samples for analysis 
include saliva, CSF, and urine. Alternately, serology can be used to detect a 



238 CASE FILES: MICROBIOLOGY 

fourfold rise in mumps-specific IgM or IgG antibody in clinical samples. Also, 
clinical specimens can be cultured in cells for observation of cytopathic effects 
such as cell rounding and syncytia formation. 

Treatment and Prevention 

Though mumps is usually self-limiting, it is treated with supportive care: flu- 
ids, rest, anti-inflammatories. Patients with mumps or suspected mumps 
should be isolated for up to a week after symptoms begin or until infection is 
ruled out. There is no specific antiviral therapy for mumps. However, immu- 
nization with the live attenuated mumps virus vaccine provides effective pro- 
tection against infection. 

COMPREHENSION QUESTIONS 

[32. 1] A 6-year-old child presents to their pediatrician with symptoms of fever, 
fatigue, and swollen glands. Which of the following patient information 
would confirm a diagnosis of infection with the mumps virus? 

A. A history of exposure to mumps 

B. Clinical evidence of orchitis 

C. Detection of mumps-specific IgM antibody 

D. Resolution of fever followed by signs of encephalitis 

[32.2] Which of the following statement regarding infection with the mumps 
virus is correct? 

A. After initial replication, viremic spread can occur to various organs. 

B. Diagnosis is made solely on symptoms, as virus cannot be cultured. 

C. Passive immunization is the only means of preventing infection. 

D. Reinfection is possible, because of the presence of two viral serotypes. 

E. Virus is transmitted via the fecal-oral route. 

[32.3] Which of the following organs would most commonly exhibit signs of 
mumps infection? 

A. CNS 

B. Ovaries 

C. Pancreas 

D. Parotids 

E. Testes 



CLINICAL CASES 239 

Answers 

[32.1] C. The detection of mumps-specific IgM antibody indicates active 
mumps virus infection; answers A, B, D, and E are incorrect; (A) 
exposure to mumps does not necessarily cause infection, partic- 
ularly if the child has been immunized; (B) symptoms of orchitis 
because of mumps infection occurs only in adolescent males; (D) 
encephalitis is a more rare complication of mumps infection and is 
not specific to the mumps virus. 

[32.2] A. After initial replication in the upper respiratory tract and salivary 
glands, viral particles are also transmitted to distant organs such as 
the kidneys, testes, ovaries, and CNS through viremic spread; answers 
B, C, D and E are incorrect. 

[32.3] D. Swollen parotid glands are a common symptom during infection 
with the mumps virus; answers A, B, C, and E are possible compli- 
cations of infection with the mumps virus, but are less commonly 
occurring. 



MICROBIOLOGY PEARLS 

*♦* Nearly all cases of mumps are seen in unvaccinated children or per- 
sons with depressed cellular immunity. 

*♦* Clinical manifestations: acute fever and painful swelling of the 
parotids and other glands. 

*♦* Immunization with a live attenuated mumps virus vaccine has resulted 
in nearly 100 percent reduction in the incidence of infection. 



REFERENCES 

Brooks GF, Butel JS, Morse SA. Jawetz, Melnick, & Adelberg's Medical 
Microbiology, 23rd ed. New York: McGraw-Hill, 2004:560-2. 

Ryan JR, Ray CG. Sherris Medical Microbiology, 4th ed. New York: McGraw-Hill, 
2004:513. 

Centers For Disease Control and Prevention. Vaccines and preventable diseases: 
mumps vaccines, http://www.cdc.gov/vaccines/vpd-vac/mumps/default.htm. 



This page intentionally left blank 



♦♦♦ CASE 33 



An 8-year-old boy is brought in to the physician's office with a 3-day history 
of fever and a rash. He has also had a mild sore throat and felt somewhat 
fatigued. His mother is concerned that he could have "scarlet fever." The rash 
started on his face and then spread to his arms and legs. He has only been given 
acetaminophen for the fever. He takes no other medications, has no known 
allergies, has no significant medical history, and has had no contact with any- 
one known to be ill. On examination, his temperature is 37.7°C (99.8°F), and 
his other vital signs are normal. His cheeks are notably red, almost as if they 
had been slapped. His pharynx is normal appearing, and the remainder of his 
head and neck exam is normal. On his extremities there is a fine, erythema- 
tous, maculopapular rash but no vesicles or petechiae. A rapid group A strep- 
tococcal antigen test done in the office is negative. 

^ What virus is the likely cause of this illness? 

^ In which human cells does this virus cause lytic infections? 



242 CASE FILES: MICROBIOLOGY 

ANSWERS TO CASE 33: PARVOVIRUSES 

Summary: An 8-year-old boy with fever and "slapped cheek" appearance has 
erythema infectiosum or fifth disease. 

•^ Most likely viral cause of this illness: Infection with parvovirus B19 

^ Human cells in which the virus causes lytic infections: Mitotically 
active erythroid precursor cells 

CLINICAL CORRELATION 

Parvovirus B19 is the only parvovirus known to cause disease in humans. 
Infection occurs typically in school-age children resulting in a mild febrile 
upper respiratory illness followed by an exanthematous rash on the face or 
"slapped cheek" appearance which later spreads to the extremities. It is classi- 
cally described as fifth disease because it was the fifth childhood exanthem to 
be described after varicella, rubella, roseola, and measles. Adults are less com- 
monly infected, and primarily present with polyarthritis of the hands, knees, and 
ankles, occurring with or without rash. Chronic infection occurs in immunode- 
ficient patients with more serious consequences, such as severe anemia and 
aplastic crisis. Additionally, infection occurring in pregnant seronegative mothers 
can lead to serious infection and fetal death. 

APPROACH TO SUSPECTED PARVOVIRAL INFECTION 
Objectives 

1. Be able to describe the characteristics of the virus. 

2. Be able to describe the strategies for prevention and treatment of the 
infection. 

Definitions 

Exanthem: An eruptive disease or eruptive fever appearing on the skin. 
Enanthem: An eruptive disease appearing on mucous membranes. 
Petechiae: Tiny reddish or purplish spots containing blood and appearing 

on the skin or mucous membranes. 
Hydrops fetalis: Serious edema of the fetus which can result in congestive 

heart failure. 
Erythroid progenitor cells: Precursors to erythrocytes. 



CLINICAL CASES 243 

DISCUSSION 

Characteristics of Parvovirus That Impact Transmission 

Parvoviruses are small, nonenveloped viruses with a linear, single- 
stranded DNA genome. They are the smallest of the DNA animal viruses, 

and their virions contain either a positive- or negative-sense copy of the viral 
genome. Parvovirus B19 belongs to the Parvoviridae family and is the only 
parvovirus known to cause human disease. It is spread from person to person 
by respiratory and oral secretions, replicating first in the nasopharynx, then 
spreading by viremia to the bone marrow. It binds to the erythrocyte blood 
group P antigen on erythroid precursor cells and is internalized through 
coated pits. After internalization, the viral DNA is uncoated and transported 
to the nucleus, where a complementary DNA strand is created by the host 
DNA polymerase. Inverted repeat sequences on the 5' and 3' ends of the viral 
DNA genome fold back and hybridize to provide a primer for the initiation of 
DNA replication. The resultant double-stranded viral DNA is then further 
replicated and transcribed via host cell machinery, and newly formed virions 
are assembled in the nucleus. Additionally, other factors only available in the 
S phase of mitosis are required for parvoviral replication. The newly assem- 
bled infectious particles are then released by lysis of the host nuclear and cyto- 
plasmic membranes, resulting in cell death. The major site for parvoviral 
replication is in adult bone marrow and fetal liver cells. Because replication in 
these cells results in cell lysis and death, there is a disruption of red cell pro- 
duction and resultant anemia that occurs with viral infection. 

Clinical symptoms caused by parvovirus B19 are related to the immune 
system response to the infection. The most common clinical illness associated 
with parvovirus B19 is erythema infectiosum, or fifth disease. It is com- 
monly seen in children and usually causes a biphasic infection with mild upper 
respiratory symptoms, low-grade or no fever, and a rash. The initial stage lasts 
for approximately 1 week and involves the infection and killing of erythroid 
cells followed by viremic spread. This stage is the infectious stage and pro- 
duces flu-like symptoms with mild fever and upper-respiratory symptoms. The 
second stage of infection is immune-mediated, with the formation of host 
antibody-virus immune complexes, a reduction in viremia, and the emergence 
of a spreading lacy skin rash and/or anthralgias. The rash usually starts on the 
face and is classically described as causing a "slapped cheek" appearance. A 
maculopapular rash will then frequently develop on the extremities. Adults 
may get a rash, but more often develop arthralgias or frank arthritis. The pro- 
duction of rash, arthralgias, and arthritis are all results of circulating antibody- 
virus immune complexes. These symptoms are usually self-limited. Yet, 
persistent infections can occur in immune-compromised hosts who fail to pro- 
duce virus-neutralizing antibodies, because host antibody-mediated immunity 
is necessary for defense against the infection and prevention of reinfections. 



244 CASE FILES: MICROBIOLOGY 

More rare but potentially life-threatening complications of parvovirus B19 
infection can occur. Aplastic crisis can occur in hosts with a chronic 
hemolytic anemia, such as sickle cell disease or other acquired hemolytic ane- 
mias. In this setting, the combination of viral replication in red cell precursors 
along with the reduced circulatory life span of existing red cells may result in 
a profound anemia. Additionally, parvoviral B19 infection in immunodeficient 
patients can result in persistent infections with chronic bone marrow suppres- 
sion and anemia. There is also an increased risk of fetal loss because of ane- 
mia caused by transplacental infection of the fetus when a seronegative 
mother becomes infected during pregnancy. Fetal parvoviral infection can 
result in hydrops fetalis, severe anemia, and often, fetal death before the third 
trimester. The fetus of a seropositive mother is protected from infection by 
maternal circulating antibodies. 

Diagnosis 

Diagnosis is primarily made from clinical presentation. While definitive diag- 
nosis of parvovirus B19 infection relies on the detection of viral DNA via 
PCR or DNA hybridization assays using patient serum, blood or tissue sam- 
ples. Additionally, serologic detection of viral IgM or IgG antibodies via 

ELISA (enzyme-linked immunosorbent assay) can be used for diagnoses. 

Treatment and Prevention 

At present, there is no specific treatment for parvovirus B19 infections. 
Infection control measures are used in hospitals to avoid parvoviral spread, 
such as rigorous hand washing and isolation of infected patients. Although 
vaccines are available for dogs and cats, there is currently no parvoviral B19 
vaccine available for humans. 

COMPREHENSION QUESTIONS 

[33.1] Which of the following statements is most accurate regarding infection 
with parvovirus B19? 

A. Parvovirus B 19 replicates in the host cell nucleus utilizing a virally 
encoded DNA polymerase to create a double-stranded DNA inter- 
mediate. 

B. Parvovirus B19 causes severe anemia because it preferentially 
infects erythrocyte precursors. 

C. Parvovirus B19 can be diagnosed by detection of viral RNA using 
PCR or hybridization assays. 

D. Parvovirus B19 can cause hydrops fetalis via transplacental infec- 
tion of a fetus in a seropositive mother. 

E. Parvovirus B19 is transmitted primarily by fecal-oral transmission 
and is highly prevalent in infants and young children. 



CLINICAL CASES 245 

[33.2] A normally healthy 7-year-old girl is sent home from school with a 
suspected case of fifth disease as a result of her presenting with the ini- 
tial symptoms of the infection. After being at home for a few days, her 
symptoms change indicating her transition into the second phase of the 
illness. Which of the following symptoms is the girl most likely expe- 
riencing during the second phase of parvoviral infection? 

A. Aplastic crisis 

B. Diarrhea 

C. High fever 

D. Skin rash 

E. Swollen glands 

[33.3] Which of the following conditions would put an individual at increased risk 
for serious chronic illness following an infection with parvovirus B19? 

A. Immunization with a live measles vaccine 

B. Having sickle cell disease 

C. Caring for a pet with canine parvoviral infection 

D. Coinfection with influenza A virus 

E. Not being immunized for poliovirus 

Answers 

[33.1] B. Parvovirus B19 binds preferentially to the erythrocyte blood group 
P antigen on erythroid precursor cells; answers A, B, C, and E are 
incorrect: (A) after entry the single-stranded viral DNA genome is 
transported to the cell nucleus where the host DNA polymerase syn- 
thesizes the complimentary DNA strand; (C) Parvovirus B19 infection 
can be diagnosed by direct detection of viral DNA not RNA; (D) the 
fetus of a seropositive mother is protected from infection by maternal 
circulating antibodies; (E) Parvovirus B19 is transmitted primarily by 
respiratory secretions and is prevalent in school-age children. 

[33.2] D. The second stage of parvoviral B 19 infection is immune-mediated, 
and results in formation of a lacy skin rash occurring first on the face 
and then moves to the extremities; answers A, B, C, and E are incor- 
rect: (A) aplastic crisis occurs in hosts with sickle cell disease or 
other acquired hemolytic anemias, not in normally healthy individu- 
als; (B) parvoviral infection does not cause gastrointestinal symp- 
toms; (C) mild fever, not high fever, is a symptom that occurs in the 
initial stage of fifth disease; (E) parvoviral infection does not result in 
swollen glands. 

[33.3] B. More serious complications of parvoviral B19 infection, such as 
aplastic crisis, can occur in hosts with sickle cell disease or other 
chronic hemolytic anemias; answers A, C, D, and E are incorrect as 
they are not associated with serious complications of parvovirus B19 
infection. 



246 CASE FILES: MICROBIOLOGY 



MICROBIOLOGY PEARLS 

*♦* Parvovirus B19 is the smallest human DNA virus and replicates in 

mitotically active erythroid progenitor cells. 
*♦* Clinical manifestations in children: mild fever followed by rash 

with "slapped cheek" appearance; fetal infections: hydrops 

fetalis and death. 
*♦* Clinical manifestations in adults: polyarthritis of hands, knees, and 

ankles with or without rash; chronic infection may result in 

chronic anemia or aplastic crisis. 
*♦* No specific treatment or vaccination. 



REFERENCES 

Brooks GF, Butel JS, Morse SA. Jawetz, Melnick, & Adelberg's Medical 
Microbiology, 23rd ed. New York: McGraw-Hill, 2004:414-9. 

Heegaard ED, Hornsleth A. Parvovirus: the expanding spectrum of disease. Acta 
Paediatr 1995;84:109-17. 

Ryan JR, Ray CG. Sherris Medical Microbiology, 4th ed. New York: McGraw-Hill, 
2004:522-3. 



i* CASE 34 



♦> 



A 62-year-old man presents to the emergency room after suddenly losing the 
use of his right leg. He reports that he had a few days of headache, fever, and 
sore throat, which was treated with oral antibiotics and resolved approximately 
4 days ago. He was feeling fine until this morning, when he could not lift his 
right leg to get out of bed. All of his other limbs are functioning normally, and 
he has normal sensation in them. He has a medical history significant for lung 
cancer for which he is receiving chemotherapy, with his most recent cycle hav- 
ing been completed a few days prior to the onset of his febrile illness. He 
denies having any recent falls, injuries, current headache, or neurologic symp- 
toms other than in the right leg. He has not traveled outside of the United States. 
His only current medication is amoxicillin/clavulanic acid, which was pre- 
scribed for his recent febrile illness. He lives with his son, daughter-in-law, 
and two young grandchildren. The children are healthy, and each had their rou- 
tine well-child checkups and vaccinations about a month ago, including an 
oral vaccine. On examination, he is anxious appearing but has normal vital 
signs and has unremarkable head and neck, cardiovascular, pulmonary, and 
abdominal examinations. He has flaccid paralysis with normal sensation of the 
right leg, with normal movement and strength in all other extremities and a 
normal cranial nerve examination. A head CT scan and lumbar magnetic res- 
onance imaging (MRI) are also normal. 

^ What is the most likely infectious cause of this man's flaccid 
paralysis? 

^ Assuming that he was infected at home, what is the most likely 
source of his infection? 



248 CASE FILES: MICROBIOLOGY 

ANSWERS TO CASE 34: POLIOVIRUS 

Summary: A 62-year-old man with flaccid paralysis of the right leg. He lives 
with his grandchildren, who were recently given an oral vaccine. 

^ Most likely infectious cause of this man's flaccid paralysis: 

Poliomyelitis, caused by poliovirus. 

^ Most likely source of his infection: Fecal-oral transmission of viral 
particles shed from one of his grandchildren recently vaccinated with 
live attenuated poliovirus vaccine. 

CLINICAL CORRELATION 

Poliovirus is an exclusive human pathogen, which causes an acute infectious 
disease that can result in flaccid paralysis from the destruction of motor neu- 
rons in the spinal cord. Although most childhood infections tend to be subclin- 
ical, the risk of more serious paralytic disease increases with age. Infections are 
spread through fecal-oral transfer and poor sanitation and crowded conditions 
help to promote viral spread. Both attenuated live (orally administered) and 
inactivated poliomyelitis (injected) viral vaccines have been available for over 
40 years, and most industrialized countries have been free from wild poliovirus 
infections since the late 1990s or early 2000s. Use of the killed-virus vaccine 
for childhood immunizations is currently recommended in the United States 
because of safety issues with the live-attenuated vaccine, including possible 
transfer of live virus to close contacts. Efforts are being continued to globally 
eradicate poliovirus from residual areas such as Africa and India. 

APPROACH TO SUSPECTED POLIOVIRUS INFECTION 
Objectives 

1. Be able to describe the characteristics of the virus. 

2. Be able to describe the strategies for prevention and treatment of the 
infection. 

Definitions 

Poliomyelitis: Inflammation and destruction of the gray matter of the 

spinal cord, which can result in paralysis. 
Attenuated live poliovirus vaccine or oral polio vaccine (OPV): A viral 

vaccine consisting of a less virulent form of poliovirus, obtained through 

multiple passages of three types of poliovirus through tissue culture cells. 
Inactivated poliomyelitis vaccine (IPV): A viral vaccine consisting of a 

large dose of viral antigen that will elicit a protective antibody response 

without risk of spreading the infection. 



CLINICAL CASES 249 

DISCUSSION 

Characteristics of Poliovirus That Impact Transmission 

Poliovirus is a member of the enterovirus genus of the Picornaviridae fam- 
ily. It is a small, nonenveloped virus with a single-stranded, positive-sense 
RNA genome that resembles cellular mRNA. It is contained within an icosa- 
hedral capsid composed of four polypeptides (VP1-VP4) that are necessary for 
maintaining virion structure, attachment to specific host cell receptors, and 
entry into cells. The viral genome contains a small protein at the 5' end, termed 
VPg, and is polyadenylated at the 3' end. The genome is transcribed into a single 
polyprotein that is proteolytically cleaved to produce all of the virally encoded 
proteins. One of these proteins is a viral protease, which specifically degrades 
the 5' cap proteins from cellular mRNAs and thus preferentially inhibits trans- 
lation of host mRNA. The 5' viral VPg protein promotes cap-independent 
association of the poliovirus genome with host cell ribosomes and allows 
translation of viral proteins to occur. Polioviruses are cytolytic and cause 
direct damage to infected cells. 

As with other enteroviruses, poliovirus is transmitted primarily by the 
fecal-oral route. Viral particles enter through the mouth and primary replica- 
tion is thought to occur in the oropharynx, tonsils, and lymph nodes or in the 
intestinal epithelium and adjacent lymphoid tissue. The virus is resistant to a 
wide range of pH levels, allowing it to survive the acidity of the stomach. 
Depending on the host immune response and the ability of the virus to spread, 
infection with poliovirus can result in one of four different types of infection: 
asymptomatic illness, abortive poliomyelitis, nonparalytic poliomyelitis, or 
paralytic poliomyelitis. After initial viral replication, an immune competent 
host will make specific antibodies to the virus, and if the infection is limited 
to this stage the infection remains asymptomatic. Host antibodies provide the 
major immune response to poliovirus infections. However, if infection is not 
contained by the host antibody response, there may be a "minor" viremic 
spread to cells containing a specific receptor recognized by the capsid VP pro- 
teins. The specificity of poliovirus infection via these receptors restricts the 
tropism for poliovirus to cells such as the anterior horn cells of the spinal 
cord, dorsal root ganglia, motor neurons, skeletal muscle cells, and lym- 
phoid cells. 

After binding to the receptor, the RNA genome is inserted into the host 
cytoplasm through a channel created in the cell membrane. Viral transcription 
and replication occur in the cytoplasm, and new virions are released by cell 
lysis. Replication in these cells can then lead to a "major" viremia that, when 
controlled by host antibody response, produces the "minor" illness of abortive 
poliomyelitis. Abortive poliomyelitis causes nonspecific symptoms that include 
fever, sore throat, and headache. In a small percentage of infected people, the 
virus may continue to spread to involve the central nervous system (CNS) or 
the meninges. This can occur either as a result of viremic dissemination or 



250 CASE FILES: MICROBIOLOGY 

ascending infection through peripheral nerves into the CNS. This can then 
result in nonparalytic poliomyelitis, aseptic meningitis, or, when anterior horn 
cells of the spinal cord or motor cortex are involved, paralytic poliomyelitis. 
Paralytic poliomyelitis is the least common complication of poliovirus infec- 
tion and appears less than a week following initial symptoms of abortive 
poliomyelitis. Paralytic disease is caused by cytolytic damage caused by the 
virus, not by the immune response. 

Diagnosis 

In addition to the presentation of the above clinical findings, a suspected 
poliovirus infection can be diagnosed by the recovery and culture of the 
virus from clinical samples. The best clinical specimens include throat swabs 
if collected shortly after the onset of infection or rectal swabs and stool spec- 
imens collected up to 30 days post onset. Cells inoculated with poliovirus will 
show cytopathic effects of viral infection in less than a week of culture. Even 
when there is CNS and meningeal involvement, poliovirus is rarely recovered 
from CNS fluid. RT-PCR can also be used to detect RNA sequences in tissues 
and body fluids, increasing the sensitivity and speed of diagnosis. 

Treatment and Prevention 

Universal vaccination has eliminated wild-type polio from the western hemi- 
sphere and has greatly reduced the incidence of the disease worldwide. Two 
vaccine types exist — a live, attenuated virus vaccine given orally and an 
inactivated vaccine given by injection. The live, attenuated virus vaccine has 
the advantages of creating a secretory antibody in the gastrointestinal (GI) 
tract and is easily administered. However, viral shedding in the stool of the 
vaccinated person does occur and has been a source of polio infections during 
the era of widespread vaccination. In very rare cases, the polio vaccine caused 
disease either in the vaccinated individual or a close, usually immunocompro- 
mised, contact. Because of this, the current recommendation in the United 
States is to give only the inactivated vaccine, which induces humoral antibod- 
ies, but does not carry the risk of vaccine-induced disease. A primary series of 
four inoculations is recommended within a 1-2 year period, with periodic 
boosters administered as necessary later in life. However, the inactivated vac- 
cine does not induce local intestinal immunity, allowing poliovirus to still 
replicate in the GI tract. 



CLINICAL CASES 251 

COMPREHENSION QUESTIONS 

[34. 1] Which of the following statements best describes an advantage of the oral 
polio vaccine when compared to the inactivated poliomyelitis vaccine? 

A. It can be administered to immunocompromised patients. 

B. It is not associated with vaccine-related cases of poliomyelitis. 

C. It induces local intestinal immunity. 

D. It is easily administered as a series of multiple injections. 

E. It can be given to young children with other scheduled immunizations. 

[34.2] The primary pathologic effect of polioviral infection is a result of 
which of the following? 

A. Destruction of infected cells 

B. Paralysis of muscle cells 

C. Immune complex formation 

D. Aseptic meningitis 

E. Persistent viremia 

[34.3] The majority of nonimmunized patients infected with poliovirus would 
be expected to experience which of the following symptoms? 

A. Flu-like illness 

B. Aseptic meningitis 

C. Muscle spasms and pain 

D. Flaccid paralysis of one or more extremities 

E. Asymptomatic infection 

Answers 

[34.1] C. The oral polio vaccine or "live" vaccine produces not only IgM 
and IgG antibodies in the blood but also secretory IgA antibodies in the 
intestine, resulting in intestinal immunity; the inactivated poliomyelitis 
vaccine produces humoral immunity, but not localized intestinal 
immunity. Answers A, B, D, and E are incorrect: (A) only the inac- 
tivated poliomyelitis vaccine is administered to immunocompro- 
mised patients; (B) the oral polio vaccine has been associated with 
transfer of live poliovirus to close contacts of immunized patients, 
and therefore, use of the inactivated poliomyelitis vaccine is cur- 
rently recommended in the United States for childhood immunizations; 

(D) it is easily administered in multiple oral doses, not injections; 

(E) both the oral polio vaccine and the inactivated poliomyelitis vaccine 
can be given to young children with other scheduled immunizations. 



252 CASE FILES: MICROBIOLOGY 

[34.2] A. Polioviruses are cytolytic and cause direct damage to infected 
cells; answers B, C, D, and E are incorrect: (B) paralysis results in 
less than 2 percent of patients infected with poliovirus and is a direct 
result of the destruction of infected neurons in the spinal cord and 
brain; (C) paralytic disease is caused by cytolytic damage because of 
the virus, not by the immune response; (D) aseptic meningitis is a 
result of poliovirus infection which occurs in less than 1-2 percent of 
patients infected, and is a result of the destruction of infected cells; 
(E) if not contained by the host antibody response, polioviral infec- 
tion may result in "minor" and "major" viremic spread within the 
patient, however, the primary pathologic effect of poliovirus is still 
the cell lysis of infected cells. 

[34.3] E. Greater than 90 percent of infections with poliovirus result in 
asymptomatic infections; answers A, B, C, and D are incorrect; all are 
potential outcomes of polioviral infections that remain uncontrolled 
by a host immune response, but are much less common outcomes of 
poliovirus infection. 



MICROBIOLOGY PEARLS 

*♦* Poliovirus is a small, nonenveloped virus with a single-stranded, 
positive-sense RNA genome. 

S* More than 90 percent of poliovirus infections are asymptomatic. 

V* Clinical manifestations: "minor" illness of abortive poliomyelitis 
includes fever, sore throat, and headache; "major" illness of non- 
paralytic poliomyelitis or paralytic poliomyelitis also includes 
back pain, muscle spasm, aseptic meningitis, and spinal paralysis 
of one or more limbs. 
Vaccines available: attenuated live poliovirus vaccine or oral polio 
vaccine (OPV) and inactivated poliomyelitis vaccine (IPV). 



* 



* 






REFERENCES 

Brooks GF, Butel JS, Morse SA. Jawetz, Melnick, & Adelberg's Medical 

Microbiology, 23rd ed. New York: McGraw-Hill, 2004:491-2. 
Ryan JR, Ray CG. Sherris Medical Microbiology, 4th ed. New York: McGraw-Hill, 

2004:532-7. 
Plotkin SA, Vidor E. Poliovirus vaccine-Inactive. In Plotkin SA, Orenstein WA. 

Vaccines, 4th ed. Philadelphia, PA: WB Saunders, 2004. 
Sutter RW, et al. Poliovirus-Live. In Plotkin SA, Orenstein WA. Vaccines, 4th ed. 

Philadelphia, RAWB Saunders, 2004. 



♦> CASE 35 



A 3-year-old male infant is brought to the emergency room in the middle of 
January with fever, vomiting, and diarrhea for the past day. He has not been 
able to keep anything down by mouth and has had profuse, very watery stools. 
He attends day care, and several of his classmates have been out sick recently 
as well. No adult members of the household have been ill. He has no significant 
past medical history. On examination, his temperature is 37.9°C (100. 2°F), and 
he has tachycardia. His mucous membranes are dry, and eyes appear somewhat 
sunken. His abdomen has active bowel sounds and is nontender. His stool is 
watery and pale. The stool tests negative for blood and fecal leukocytes. 

^ What is the most likely cause of this child's illness? 

How is this virus activated to form an infectious particle? 



254 CASE FILES: MICROBIOLOGY 

ANSWERS TO CASE 35: ROTAVIRUS 

Summary: A 3-year-old boy who attends day care develops gastroenteritis in 
the winter. 

+ Most likely cause of this child's illness: Rotavirus. 

^ How is this virus activated to form an infectious particle: Activation of 
rotavirus occurs when the outer capsid layer is lysed by gastrointestinal 
(GI) proteases to create an infectious subviral particle (ISVP). 

CLINICAL CORRELATION 

Rotaviruses are ubiquitous worldwide and are estimated to cause more than 
50 percent of gastroenteritis cases occurring in children less 2-3 years of age, 
resulting in approximately 4 billion annual cases. Infections typically occur in 
the cooler months and result in abrupt onset of vomiting followed with fre- 
quent watery diarrhea. Illness is typically self-limiting; however, severe infec- 
tion can result in immunocompromised or malnourished children and may be 
fatal. Outbreaks are common in day care, preschool, and hospital settings. 
Adults may also become infected but usually have few if any symptoms. 

APPROACH TO SUSPECTED ROTAVIRUS INFECTION 
Objectives 

1. Be able to describe the characteristics of the virus. 

2. Be able to describe the strategies for prevention and treatment of the 
infection. 

Definitions 

Tachycardia: An increased heart rate. 

Reassortment: The formation of new virions with hybrid genomes assem- 
bled in cells with mixed viral infections, which occurs among viruses 
containing segmented genomes (i.e., influenza viruses and reoviruses), 
resulting in high genetic variation. 

Intussusception: Blockage of the intestines as a result of the bowel tele- 
scoping into itself. 



CLINICAL CASES 255 

DISCUSSION 

Characteristics of Rotavirus That Impact Transmission 

Rotavirus is one of the four genera of the family Reoviridae and is a com- 
mon cause of childhood gastroenteritis around the world. The virus consists of 
a double-layered protein capsid that contains a genome made of 11 seg- 
ments of double-stranded, negative-sense RNA. The double capsid looks 

like a wheel with short spokes connecting the outer capsid to the inner cap- 
sid and core, thus the name Rotavirus. As a nonenveloped virus, it retains its 
infectivity in a wide range of pH and temperatures and is resistant to many 
common detergents as well. Rotavirus is spread through fecal-oral contact, 
and because of its stability, fomite transmission can also occur. The virus 
would be inactivated by the pH of a normal, empty stomach but can survive in 
a buffered stomach or in the gastric environment following a meal. The outer 
capsid of the virus is partially digested by GI proteolytic enzymes, creating an 
infectious subviral particle (ISVP). A surface protein of the virus, VP4, is also 
cleaved by GI proteases, allowing it to bind to the surface of intestinal epithe- 
lial cells and allow the ISVP to enter by direct penetration. The RNA genome 
remains in the viral core and is transcribed into mRNA by a viral polymerase. 
The mRNA is then transported out of the core to the cell cytoplasm, where it 
is translated and assembled into new virions. The initial assembly of rotaviral 
virions is similar to enveloped virions. As the newly formed rotaviral virions 
bud through the endoplasmic reticulum (ER), they acquire the membrane as an 
"envelope." However, this "envelope" is soon lost as the virions continue 
through the ER, and the infectious rotaviral particles are then released from the 
host cell by cell lysis. 

Rotaviruses have been classified into at least three different major sub- 
groups and nine different serotypes based on antigenic epitopes of the inner 
capsid protein VP6. There are primarily four serotypes that are important in 
causing human disease. Because of the segmented nature of the genome, 
rotaviruses are capable of producing virions with high genetic variation as a 
result of the reassortment of genome sequences in mixed infections. This high 
genetic variability results in increased numbers of serotypes for this viral 
group and allows for reinfection of persons previously exposed to one rotavi- 
ral serotype. Reinfections are common, yet successive infections appear to 
cause less severe symptoms. 

The mechanism by which rotaviral infection causes diarrhea is not entirely 
understood. Rotaviral particles infect the cells of the small intestinal villi and 
multiply in the cytoplasm of enterocytes. Damaged cells are sloughed off, 
releasing large numbers of viral particles into the stool. Virus can be excreted 
for days to weeks after infection. The infection prevents absorption of water, 
sodium, and glucose, resulting in a loss of water and electrolytes. A virally 
encoded nonstructural protein also acts as an enterotoxin, similar to those of 
Escherichia coli and Vibrio cholerae. Typical symptoms of rotaviral infection 



256 CASE FILES: MICROBIOLOGY 

include fever, vomiting, abdominal pain, and watery diarrhea without blood or 
mucus. The net result is a profuse watery diarrhea that can cause dehydration 
without appropriate fluid and electrolyte replacement. Symptoms may last for 
approximately 1 week, with viral excretion lasting weeks longer. Severe and 
prolonged illness can occur in immunodeficient and malnourished children 
and without supportive therapy infection can be fatal. Infection with rotavirus 
stimulates a humoral response; however, protection against reinfection is tem- 
porary and incomplete. The presence of high levels of rotavirus IgA in the 
lumen of the intestine confers relative protection. 

Diagnosis 

Because the symptoms of rotaviral infection resemble those of other viral diarrhea 
producing agents, the definitive diagnoses of rotaviral infection requires the detec- 
tion of viral antigens in stool samples. Enzyme immunoassay and latex aggluti- 
nation are two easy, rapid assays used to confirm rotaviral infection. Additionally, 
PCR can be used for genotyping viral nucleic acid in stool specimens. Viral cul- 
ture is both difficult and unreliable and therefore is not used for diagnoses. 

Treatment and Prevention 

Treatment of rotaviral infection is supportive, including the replacement of 
fluids and electrolytes to restore physiologic balance and prevent dehydration. 
Both oral and intravenous rehydration therapy are effective, and which one is 
used depends on the severity of dehydration. Because rotaviruses can retain 
infectivity over a wide range of pH and temperatures and are resistant to many 
common detergents, strict hand washing and use of gloves is necessary to limit 
nosocomial spread. 

An attenuated recombinant Rotavirus vaccine was developed and used 
in children for several years. However, its approval was withdrawn, and its 
use stopped because of concerns with the development of intussusception 
among vaccine users. A new rotavirus vaccine was approved by the FDA and 
released in 2006; to date, it has not been shown to be associated with intus- 
susception. Another difficulty with the production of such a vaccine is that a 
single vaccine may not protect against all Rotavirus serotypes. 



CLINICAL CASES 257 

COMPREHENSION QUESTIONS 

[35. 1] You isolate a virus from the stool of a 1 -year-old infant with signs of fever, 
vomiting, and diarrhea. Laboratory results show that the viral genome is 
composed of multiple segments of double-stranded RNA, which leads you 
to suspect that rotavirus is the causative agent of infection. Which of the 
following statements is true regarding rotavirus replication? 

A. The viral genome integrates into the host chromosome. 

B . The virus uses the host RNA polymerase for replication of its genome. 

C. The segmented genome contributes to the antigenic variation of the 
virus. 

D. The viral agent has a single antigenic type. 

E. The newly assembled viral particles are released via budding through 
the host cell membrane. 

[35.2] Similar to rotavirus, which of the following viral agents is also a 
nonenveloped RNA virus known to cause gastroenteritis diarrhea in 
young children? 

A. Calicivirus 

B. Paramyxovirus 

C. Parainfluenza virus 

D. Coxsackie virus 

E. None of the above 

Answers 

[35.1] C. The segmented genome of rotaviruses, allows for the assembly of 
new virions with mixed genomes in cells multiply infected as a result 
of reassortment; answers A, B, D, and E are incorrect: (A) the rotavi- 
ral genome consists of double-stranded RNA and replicates in the 
cytoplasm and thus does not integrate into the host chromosome; 
(B) as an RNA virus that replicates in the cytoplasm, the ro to viral 
genome is replicated by a viral RNA polymerase; (D) the high genetic 
variability of rotaviruses because of reassortment results in multiple 
viral serotypes, at least nine different serotypes have currently been 
classified in human illness; (E) newly assembled nonenveloped 
rotaviral particles are released by cell lysis. 

[35.2] A. Like rotaviruses, caliciviruses are nonenveloped RNA viruses that 
cause watery diarrhea, especially in children; answers B, C, D, and E are 
incorrect: (B) Paramyxoviruses are enveloped RNA viruses that cause 
childhood respiratory and exanthemous infections; (C) Parainfluenza 
viruses are enveloped RNA viruses which cause respiratory infections 
such as croup, bronchiolitis, and pneumonia in children; (D) Coxsackie 
viruses are nonenveloped RNA viruses that cause nonspecific respira- 
tory tract infections, febrile rashes, and meningitis. 



258 CASE FILES: MICROBIOLOGY 



MICROBIOLOGY PEARLS 

*♦* Rotaviruses are ubiquitous, causing greater than 50 percent of gas- 
troenteritis cases in children under 2-3 years. 

*♦* Rotaviruses are composed of a double-layered protein capsid and a 
segmented double-stranded RNA genome, allowing the new virions 
to have high genetic variation as a result of reassortment. 

*♦* Clinical manifestations: abrupt onset of fever, vomiting, abdominal 
pain, and watery diarrhea without blood or mucus. 

*♦* Only supportive treatment of infection including fluid and electrolyte 
replacement. 



REFERENCES 

Brooks GF, Butel JS, Morse SA. Jawetz, Melnick, & Adelberg's Medical Microbiology, 

23rd ed. New York: McGraw-Hill, 2004:505-8. 
Centers for Disease Control and Prevention. Intussusception among recipients of 

rotavirus vaccine — United States, 1998-1999. Morb Mortal Wkly Rep 1999; 

48:577-81. 
Ryan JR, Ray CG. Sherris Medical Microbiology, 4th ed. New York: McGraw-Hill, 

2004:577-81. 



♦♦♦ CASE 36 



A 10-month-old female is brought to the pediatric emergency room in late 
December with a cough and fever. She started getting sick with a mild cough 
and runny nose approximately 3 days ago, but has progressively worsened. 
She is now coughing frequently and has vomited after coughing. She has no 
history of asthma or other respiratory illness. She was born after an uncompli- 
cated, full-term pregnancy and has no significant medical history. She attends 
day care 3 days a week. On examination, her temperature is 38.3°C (100. 9°F), 
pulse is 1 10 beats per minute, respiratory rate is 30 breaths per minute, and her 
oxygen saturation is low at 9 1 percent by pulse oximetry. Her head and neck 
examination shows her to have a right otitis media but is otherwise normal. 
Her cardiac exam is notable only for tachycardia. Her pulmonary examination 
shows her to be in moderate respiratory distress. She has prominent nasal flar- 
ing and subcostal retractions on inspiration. She has loud expiratory wheezes 
in all lung fields. The remainder of her examination is normal. A chest x-ray 
shows hyperaeration but no infiltrates. 

^ What is the likely infectious cause of her respiratory illness? 

^ Following resolution of this illness, her mother asks whether she is 
protected from getting this disease again. How do you respond? 



260 CASE FILES: MICROBIOLOGY 

ANSWERS TO CASE 36: RESPIRATORY 
SYNCYTIAL VIRUS 

Summary: A 10-month-old female presents with bronchiolitis. A chest x-ray 
shows hyperaeration but no infiltrates. 

^ Likely infectious cause of her respiratory illness: Respiratory 
syncytial virus (RSV). 

^ Is she protected from getting this disease again: The immunity 
developed with an RSV infection is incomplete, and reinfections are 
common. However, the severity of disease with repeat infections 
appears to be reduced, especially in older children and adults. 

CLINICAL CORRELATION 

RSV is a ubiquitous and highly contagious viral infection and is the single 
most common cause of fatal respiratory tract infections in infants under 12 months 
of age. It accounts for approximately 25 percent of pediatric hospitalizations 
of this age group, resulting in severe respiratory illnesses such as bronchioli- 
tis, pneumonia, and respiratory failure. It is also highly prevalent in childcare 
settings, with 70-95 percent of children attending day care being infected by 
3-4 years of age. Less severe illness occurs in older children and adults and 
may present as a common cold. 

APPROACH TO SUSPECTED RSV INFECTION 
Objectives 

1. Be able to describe the characteristics of the virus. 

2. Be able to describe the strategies for prevention and treatment of the 
infection. 

Definitions 

Bronchiolitis: Inflammation of the bronchioles or thin-walled branches of 

the lungs. 
Right otitis media: Inflammation of the right middle ear marked with pain, 

fever, dizziness, and abnormal hearing. 



CLINICAL CASES 267 

DISCUSSION 

Characteristics of RSV That Impact Transmission 

RSV belongs to the Pneumovirus genus of the family Paramyxoviridae. It 

is a common cause of upper and lower respiratory tract infections in all age 
groups, but tends to cause more severe, lower respiratory disease in infants 
and young children. RSV is an enveloped virus with a single-stranded, 
negativesense RNA genome. It is transmitted by the inhalation of aerosolized 
respiratory droplets. It can survive on nonporous surfaces, such as countertops, 
for 3-30 hours but is inactivated by many detergents and does not tolerate 
changes in temperature or pH well. RSV infections primarily remain localized 
in the respiratory tract. The virus infects target respiratory epithelial cells by 
fusion of its envelope with the host cytoplasmic membrane via the action of 
two viral envelope glycoproteins. However, unlike the related influenza and 
parainfluenza viruses, RSV envelope glycoproteins do not possess hemagglu- 
tinin or neuraminidase activities. RNA transcription, protein synthesis, repli- 
cation, and assembly all occur in the cytoplasm and newly formed virions are 
released by budding from the host cell. RSV is also capable of promoting cell- 
cell fusion, resulting in multinucleated giant cells known as syncytia, an abil- 
ity for which it derives its name. 

RSV is initially transmitted to the nasopharynx through contact with 
infected secretions and fomites, resulting in localized infections of respira- 
tory epithelium. Although viremia is rare, progressive infections can extend to 
the middle and lower airways. Disease caused by RSV is primarily the result 
of the host immune system mediating damage to infected respiratory epithelial 
cells. In adults and older children, mild upper respiratory tract symptoms such 
as a runny nose or mild cough usually develop with clinical symptoms lasting 
for 1-2 weeks. In infants or younger children, more serious illness such as 
bronchiolitis can occur. This occurs when there is inflammation and plugging 
of the bronchi and bronchioles with mucous and necrotic tissue from immune- 
mediated cellular damage. The smaller airways of infants and young children 
are especially susceptible and may result in cough, tachypnea, respiratory distress, 
wheezing, and hypoxia. 

Mortality is high in infants with underlying disease or reduced immune 
function, and causes of death often include respiratory failure, cor pulmonale 
(right-sided heart failure), or bacterial superinfection. The immune response to 
RSV is not entirely understood, but both humoral and cell-mediated systems 
appear to play a role. The immunity developed with an infection does not 
appear to be complete. Repeat infections with RSV are common, but symp- 
toms tend to be less severe with subsequent infections. Although outbreaks of 
RSV infection can occur in elderly patients resulting in severe illness, partic- 
ularly in those residing in long-term care facilities. 



262 CASE FILES: MICROBIOLOGY 

Diagnosis 

In addition to the presenting clinical symptoms, RSV can be diagnosed more 
definitively through viral genome or antigen detection. Direct identification 
of RSV antigens is performed via immunofluorescence analysis on exfoliated 
epithelial cells or with ELISA (enzyme-linked immunosorbent assay) testing 
on nasal secretions. Large amounts of viral particles are present in nasal 
washings, particularly from infected children, making it a good clinical spec- 
imen for viral genome detection via RT-PCR. When attempting to isolate the 
virus in culture, clinical samples should be inoculated immediately into cell 
cultures, because of the labile nature of RSV. The presence of RSV can be rec- 
ognized by the formation of giant cells or syncytia formation in inoculated cul- 
tures in 1-2 weeks. 

Treatment and Prevention 

Treatment of RSV infections relies mainly on supportive care including oxy- 
genation, ventilatory support, IV fluids, and nebulized cold steam. These 
modalities are used in an effort to remove or reduce mucus secretions in the 
airways and allow for adequate oxygen exchange. The antiviral agent rib- 
avirin has been approved for use via aerosolization in high-risk infants 
exposed to RSV and in severe lower respiratory tract illnesses caused by RSV 
infection. Close observation of severe cases is critical. Currently there is no 
vaccine approved for RSV. However, passive immunization with anti-RSV 
immunoglobulin is available for premature infants, using monoglonal anti- 
bodies, Synagis. Premature infants especially with bronchopulmonary dyspla- 
sia and congenital heart disease should be vaccinated. 

Additionally, preventative measures are particularly important in hospital 
and specifically neonatal intensive care units, because RSV is highly conta- 
gious. Prevention of nosocomial spread requires strict enforcement of the fol- 
lowing precautions: hand washing; isolation of RSV infected infants; and 
changing of gloves, gowns, and masks between patients. 

COMPREHENSION QUESTIONS 

[36.1] Which of the following paramyxoviruses lacks an envelope viral 
attachment protein with hemagglutinin activity? 

A. Parainfluenza virus 

B. Mumps virus 

C. Measles virus 

D. Respiratory syncytial virus 



CLINICAL CASES 263 

[36.2] An 8-month-old infant is brought to the emergency room with a sus- 
pected RSV infection. Which of the following clinical illnesses would 
you be most concerned about this child having as a result of infection 
with this virus? 

A. Bronchiolitis 

B. Encephalitis 

C. Meningitis 

D. Pancreatitis 

E. Pharyngitis 

[36.3] Which of the following statements most accurately describes the chemical 
and physiologic properties of RSV? 

A. RSV is a nonenveloped virus with a single-stranded, negative- 
sense RNA genome. 

B. Newly formed RSV particles are released via host cell lysis. 

C. RSV infects erythroid precursor cells via fusion of its viral enve- 
lope glycoproteins with the host cytoplasmic membrane. 

D. Transcription of the RSV genome occurs in the nucleus of the host 
cell, while protein synthesis, replication, and assembly occur in the 
cytoplasm. 

E. RSV is sensitive to detergents and is inactivated by changes in 
temperature and pH. 

Answers 

[36. 1] D. Respiratory syncytial virus, differs from other paramyxoviruses in 
that it does not have a hemagglutinin protein in its viral envelope; 
answers A, B, and C all have viral envelope proteins with hemagglu- 
tinin activity. 

[36.2] A. Bronchiolitis is a common clinical manifestation of RSV infection 
in infants, which results from inflammation and plugging of the bronchi 
and bronchioles with mucous and necrotic tissue; answers B, C, D, 
and E are incorrect as they are not symptoms specific to infection 
with RSV. 

[36.3] E. RSV is an enveloped virus and is inactivated by many detergents 
as well as changes in temperature and pH.; answers A, B, C, and D 
are incorrect: (A) RSV is an enveloped virus with a single-stranded, 
negative-sense RNA genome; (B) RSV virions are released by bud- 
ding from the host cell membrane, not by cell lysis; (C) RSV infects 
respiratory epithelial cells, not erythroid precursor cells, via fusion 
with the host membrane; (D) RSV is an RNA virus, and transcription, 
protein synthesis, replication, and assembly of new virions occurs in 
the cytoplasm of the host cell. 



264 CASE FILES: MICROBIOLOGY 



MICROBIOLOGY PEARLS 

*♦* RSV is highly contagious and is the primary cause of respiratory 
tract infections in infants under 1 year of age. 

*♦* Clinical manifestations: respiratory symptoms including rhinitis, pneu- 
monia, and blockage of airways leading to respiratory distress. 

*♦* Vaccination for RSV is available for children at high-risk for the 
disease. 
Treatment with ribavirin in severe cases or in high-risk infants 
exposed to the virus. 



REFERENCES 

Brooks GF, Butel JS, Morse SA. Jawetz, Melnick, & Adelberg's Medical Microbiology, 

23rd ed. New York: McGraw-Hill, 2004:558-60. 
Ryan JR, Ray CG. Sherris Medical Microbiology, 4th ed. New York: McGraw-Hill, 

2004:503-6. 



♦ CASE 37 



You are asked to consider being vaccinated with smallpox vaccine to serve as 
a first-responder in the event of a biological warfare attack. After considering 
the risks and benefits, you consent. You are given the vaccine by the standard 
technique — a small, bifurcated needle is used to create multiple punctures in the 
skin overlying your deltoid. The area is covered, and you are instructed not to 
touch the actual site. In 2 days, a small papule and erythema appear at the vac- 
cine site. A few days later, multiple vesicles are noted. These progress to form 
larger pustules. In approximately 2 weeks, the whole vaccine site has formed 
a scab and this subsequently falls off in another week. When complete recov- 
ery has occurred, you have a scar left at the vaccine site. 

^ What is the actual virus used as the smallpox vaccine? 

^ Why must the virus used in the vaccine carry or encode its own 
enzymes for DNA and mRNA synthesis? 



266 CASE FILES: MICROBIOLOGY 

ANSWERS TO CASE 37: SMALLPOX 

Summary: A physician has received the smallpox vaccine. 

Actual virus used as the smallpox vaccine: Vaccinia, which is a form 
of the cowpox virus. 

^ Reason variola carries or encodes its own enzymes for DNA 

and mRNA synthesis: Variola virus must produce its own enzymes 
for DNA and mRNA synthesis because viral replication occurs entirely 
in host cell cytoplasm, and therefore it cannot use the enzymes located 
in the host nucleus. 

CLINICAL CORRELATION 

Variola, the virus that causes smallpox, is a member of the poxvirus family. 
Smallpox is a highly contagious and severe disease that once caused high mor- 
tality in human populations. It was discovered in 1796 by Edward Jenner that 
the closely related but less virulent cowpox virus could confer resistance to 
smallpox. This discovery, along with the fact that humans were the only reser- 
voir for variola, eventually led to an effective global vaccination program, using 
the vaccinia virus as the live viral vaccine. Vaccinia shares antigenic determi- 
nants with variola but primarily causes clinical disease in nonhuman animals. 
Rare, but potentially severe, adverse events such as postvaccinial encephalitis, pro- 
gressive vaccinia necrosum, or fetal vaccinia can occur after vaccination, primarily 
in persons with suppressed immunity, severe allergies, eczema, or pregnant 
women. Additionally, smallpox vaccination is also contraindicated for persons in 
close contact with individuals with the conditions listed. 

Because of worldwide vaccination and disease control efforts, the last case 
of indigenously acquired smallpox was seen in Somalia in 1977. The World 
Health Organization declared that smallpox was eradicated in 1980. Routine 
smallpox vaccination was discontinued after 1980, as the risk of vaccination 
was thought to outweigh the risk of acquiring smallpox. Concerns about the 
risk of smallpox being used as a bioterror weapon have led to the reinstitution 
of vaccination programs, primarily among military, public health, and safety 
workers. 

APPROACH TO SUSPECTED SMALLPOX INFECTION 
Objectives 

1. Be able to describe the characteristics of the virus. 

2. Be able to describe the strategies for prevention and treatment of the 
infection. 



CLINICAL CASES 267 

Definitions 

Bifurcated needle: A specialized needle that forks into two prongs at its 
distal end; the prongs use capillary action to administer a specific 
amount of smallpox vaccine via multiple inoculations at the same site. 

Maculopapular: The clinical presentation combination of both macules 
(rash) with papules (lesions). 

Guarnieri's inclusion bodies: Electron-dense intracytoplasmic acidophilic 
inclusions within infected cells which serve as assembly sites for new 
smallpox virions. 

DISCUSSION 

Characteristics of Smallpox That Impact Transmission 

Variola is a member of the poxvirus family, and a member of the genera 
Orthopoxvirus. There are several diseases caused by orthopoxviruses: vari- 
ola, vaccinia, cowpox, and monkeypox. Variola is the causative agent of 
smallpox, a virulent human virus that causes high mortality, while cowpox and 
monkeypox are zoonotic viruses causing accidental cutaneous infections in 
humans. Vaccinia is a form of the cowpox virus and has been used effectively 
as a live viral vaccine against smallpox disease. Poxviruses are the largest 
and most complex viruses known. Poxviruses are enveloped and contain a 
linear, double-stranded DNA genome, which is fused at both ends. They are 
the only DNA viruses, which replicate entirely in the host cell cytoplasm. 
Because of this, poxviruses must carry and/or encode all of the proteins 
required for both DNA and mRNA synthesis. 

Transmission of the smallpox virus occurs via inhalation of infected res- 
piratory droplets, exposure to infectious skin lesions, or through contact with 
contaminated fomites. Once inhaled, initial replication of the virus occurs in 
the respiratory tract, where the virus binds to a target cell surface receptor and 
the envelope fuses with the cell membrane. The core of the virus is then 
released into the cellular cytoplasm where DNA replication and transcription 
takes place. New virions are assembled in cytoplasmic inclusions, referred to as 
Guarnieri inclusion bodies. Unlike other enveloped viruses, poxviruses assem- 
ble their own viral membranes around these viral inclusions instead of acquiring 
them from host membranes. The new viral particles are then released either by 
cell lysis or exocytosis. After initial infection of the respiratory tract occurs, 
the virus spreads through lymphatic channels causing primary viremia and 
infection of reticuloendothelial cells. Viral replication in these cells causes a 
secondary viremia and results in clinical manifestations of the skin and inter- 
nal organs. Variola virus exists as at least two strains — variola major and 
variola minor. Variola major is associated with high mortality rates (20-50 
percent), whereas variola minor is associated with a mortality rate of less 
than 1 percent. 



268 CASE FILES: MICROBIOLOGY 

Diagnosis 

Diagnosis of smallpox is typically made by clinical presentation. Clinical 
smallpox has an incubation period of approximately 2 weeks, followed by an 
abrupt onset of malaise, fever, chills, and myalgia. A few days post-onset, a 
characteristic maculopapular rash begins to develop and progresses in a cen- 
trifugal pattern over the head and extremities. During approximately a 2-week 
period, the rash progresses to a single crop of maculopapular lesions to firm 
vesicles, then to pustules that scab and slowly heal. The high mortality asso- 
ciated with this smallpox results from either the overwhelming primary viral 
infection or from potential secondary bacterial superinfection. 

Highly suspected cases of smallpox should be referred immediately to the 
Centers for Disease Control, where new variola and orthopox PCR tests are 
available. 

Treatment and Prevention 

As previously discussed, successful global vaccination efforts have eliminated 
naturally acquired cases of smallpox worldwide, with routine smallpox vaccina- 
tions ending in 1980 in the United States. However, new concerns of biological 
weapons development have led to the testing of old vaccine stocks and the devel- 
opment of new stocks for use primarily among military, public health, and safety 
workers. Chemo therapeutic agents such as methisazone or cidofovir may have 
some efficacy as prophylaxis against smallpox infection; however, currently there 
are no treatments available for use in established smallpox disease. 

COMPREHENSION QUESTIONS 

[37.1] Which of the following statements describes a characteristic that 
enabled the worldwide eradication of smallpox in 1980? 

A. The inactivated smallpox vaccine is easily prepared and safe. 

B. Smallpox has no known reservoir outside of humans. 

C. Mass vaccination of the world was possible as a result of easy 
administration of the vaccine in the field. 

D. Subclinical smallpox infections were also inhibited through world- 
wide mass vaccinations. 

E. All stocks of smallpox virus were destroyed worldwide in 1979. 

[37.2] Due to the potential of a bioterrorist threat, emergency health-care 
responders in New York City are being considered for smallpox vacci- 
nation. Which of the following would be a candidate for vaccination? 

A. Household contact is breast-feeding 

B. Mild asthma 

C. Is pregnant 

D. Has eczema 

E. Household contact is HIV-positive 



CLINICAL CASES 269 

[37.3] A college student is reading about the Middle Ages and notices that 
many people during that era contracted a deadly disease with similar 
symptoms including acute fever, chills, and myalgia followed by a 
characteristic rash with small blister-like lesions. Those that did not die 
from the illness were left with disfiguring scars. The inciting agent has 
a double-stranded linear DNA genome that replicates in the cytoplasm. 
Which of the following agents is the most likely culprit? 

A. Varicella virus 

B. Herpes simplex virus 

C. Rubeola virus 

D. Papilloma virus 

E. Variola virus 

Answers 

[37.1] B. Smallpox has no known reservoir outside of humans, which was 
one of the factors which enabled its eradication; answers A, C, D, and 
E are all incorrect: the vaccination for smallpox consists of a live vac- 
cinia virus and does not contain smallpox virus; mass vaccination of 
the world was not performed or required because there are no known 
nonhuman reservoirs of smallpox and subclinical infections do not 
occur. Thus, large numbers of vaccinations occurring in many popu- 
lations, such as in the United States, along with strict epidemiologic 
reporting of smallpox cases worldwide allowed for immunization of 
those exposed and the elimination of smallpox disease. Not all stocks 
of smallpox virus were destroyed, and there are still two locations 
where smallpox virus strains are held: one in Atlanta, and one in 
Moscow. 

[37.2] B. Smallpox is a live attenuated vaccine and is contraindicated for 
individuals or those who are household contacts who are immuno- 
compromised or who may be susceptible to the adverse effects of the 
vaccine. Those with eczema and similar skin conditions, infection 
with HIV, transplant patients, those on high dose corticosteroids, 
those patients who are pregnant or who are breast-feeding are a par- 
tial listing of patients for whom the vaccine is contraindicated. 

[37.3] E. Smallpox (variola virus) killed many people during the Middle 
Ages. The clinical presentation was that of fever, malaise, and myal- 
gia followed by pus-filled or vesicular rash, which often left disfigur- 
ing scars. 



270 CASE FILES: MICROBIOLOGY 



MICROBIOLOGY PEARLS 

*♦* Variola is a poxvirus and etiologic agent of smallpox. 

*♦* Vaccinia is a form of the cowpox virus and has been used effectively 
as a live viral vaccine against smallpox disease. 
Clinical manifestations of smallpox: a severe rash followed by a sin- 
gle crop of maculopapular lesions that transition into vesicles and 
pustules, and then slowly crust and heal. The lesions are at the 
same stage. 
Contraindications for vaccinia vaccination: suppressed immunity, 
severe allergies, eczema, pregnancy, or close contact with such 
persons. 



_ 



REFERENCES 

Brooks GF, Butel JS, Morse SA. Jawetz, Melnick, & Adelberg's Medical Microbiology, 

23rd ed. New York: McGraw-Hill, 2004:561-6. 
Cono J, Casey CG, Bell DM: Smallpox vaccination and adverse reactions. Guidance 

for clinicians. MMWR Recomm Rep 2003;52(RR-4):1. 
Ryan JR, Ray CG. Sherris Medical Microbiology, 4th ed. New York: McGraw-Hill, 

2004:525-7. 
Centers for Disease Control and Prevention. Acute, generalized vesicular or pustular 

rash illness testing protocol in the United States, http://www.bt.cdc.gov/agent/small- 

pox/diagnosis/rashtestingprotocol.asp 



♦> CASE 38 



A 65-year-old man comes to your office for the evaluation of lower back pain. 
For the past 3 days, he has had a sharp, burning pain in his left lower back, 
which would radiate to his flank and, sometimes, all the way around to his 
abdomen. The pain comes and goes, feels like an "electric shock," is unrelated 
to activity, and can be severe. He has had no injury to his back and has no 
history of back problems in the past. He denies fever, urinary symptoms, or 
gastrointestinal symptoms. His examination today, including careful back 
and abdominal examination, is normal. You prescribe a nonsteroidal anti- 
inflammatory drug for the pain. The next day, he returns to your office stating 
that he has had an allergic reaction to the medication because he's developed 
a rash. The rash is in the area where he had the pain for which he was seen the 
day before. On examination now, he has an eruption consisting of patches of 
erythema with clusters of vesicles extending in a dermatomal distribution from 
his left lower back to the midline of his abdomen. 

^ What is the cause of this rash? 

^ What is the mechanism for the dermatomal distribution of the rash? 



272 CASE FILES: MICROBIOLOGY 

ANSWERS TO CASE 38: VARICELLA -ZOSTER 

Summary: A 65-year-old man has a painful, dermatomal rash. 

^ Cause of this rash: The most likely cause of this man's rash is 
reactivation of varicella-zoster virus, causing the appearance of 
shingles. 

^ Mechanism for the dermatomal distribution of the rash: The 

dermatomal distribution of this rash is caused by reactivation of a latent 
varicella infection of a dorsal root ganglion with viral spread along the 
pathway of the nerve distribution. 

CLINICAL CORRELATION 

Varicella-zoster virus (VZV) is the causative agent of both chickenpox and 
shingles. Primary infection with chickenpox occurs mostly in children, with 
90 percent of the population acquiring antibodies to VZV by age 10. After pri- 
mary infection, the virus becomes latent in the dorsal root ganglia, where it 
may be reactivated later in life. Reactivation of VZV infection results in the 
unilateral eruption of a painful rash known as herpes zoster or shingles. 

APPROACH TO SUSPECTED VZV INFECTION 
Objectives 

1. Be able to describe the characteristics of the virus. 

2. Be able to describe the strategies for prevention and treatment of the 
infection. 

Definitions 

Dermatome: An area of skin served by one sensory spinal nerve. 
Neuropathic pain: Pain disseminating from the peripheral nervous system. 

DISCUSSION 

Characteristics of VZV That Impact Transmission 

VZV is a member of the Alphaherpesvirinae subfamily of the herpesviruses, 
which also include HSV-1 and HSV-2. Similar to other herpesviruses, VZV is 
a large, enveloped virus with a double-stranded DNA genome. As an 
enveloped virus, VZV is sensitive to drying and many detergents, necessitating 
its spread from person to person via respiratory droplets or direct contact 
with skin lesions. Initial VZV infection and replication occur in the epithelium 
of the respiratory tract, followed by viremic spread to the skin The virus 



CLINICAL CASES 273 

binds to specific receptors, and the viral envelope fuses with the cell membrane. 
The capsid delivers the genome to the host cell nucleus where transcription and 
replication occur. 

VZV can cause both lytic and latent infections. In lytic infections, new 
virions are assembled in the host nucleus, acquire an envelope from the 
nuclear or Golgi membrane, and are released by exocytosis or lysis of the host 
cell. In latent infections, the viral genome is not replicated, and only certain 
viral genes are transcribed. Latent infection of dorsal root or cranial nerve gan- 
glia can occur during the initial infection. The virus spreads by viremia or 
lymphatic dissemination to the reticuloendothelial system. A secondary viremia 
then occurs, which disseminates VZV to the skin and other organs. VZV can 
also form syncytia and spread directly from cell to cell. 

Diagnosis 

Viremic spread to the skin results in classic varicella infection or chickenpox. 
Typically, crops of vesicles and pustules form on erythematous bases, start- 
ing on the head and trunk and progressing centripetally to the extremities. The 
appearance of these lesions is often described as "dewdrops on a rose petal." 
Both humoral and cell-mediated immunity contribute to control of the infec- 
tion. VZV is a common childhood disease, and infection usually confers life- 
long immunity against future disseminated disease. However, reactivation of 
latent VZV infections of nerve root ganglia may result and is classically 
described as herpes zoster or shingles. The causes of the reactivation are not 
entirely known, but it tends to be more common in older persons as cellular 
immunity decreases, in immunosuppressed individuals or in otherwise immune- 
competent individuals during times of emotional stress. The reactivated virus 
replicates and is released along the dermatomal distribution of the nerve, 
causing the characteristic unilateral vesicular eruption of shingles. The rash 
is frequently preceded by pain along the course of a sensory nerve days to 
weeks prior to the onset of rash. Neuropathic pain may continue to persist for 
weeks or months after the rash clears, indicating damage to the nerve root. 
Secondary bacterial infections may also complicate reactivation. Reactivations 
of VZV tend to be infrequent and sporadic. 

Similar to HSV-1 and HSV-2, VZV can be diagnosed by examining a 
Tzanck smear of cells scraped from vesicular lesions for the presence of 
multinucleated giant cells. However, direct fluorescent-antibody staining 
of vesicular lesion scrapings remains the most rapid, sensitive, and specific 
assay for diagnosing VZV infections. 

Treatment and Prevention 

Several viral DNA polymerase inhibitors are available for treating VZV 
infections including: acyclovir, famciclovir, and valacyclovir. Treatment 
with these drugs has shown to be effective in reducing fever and skin lesions 



274 CASE FILES: MICROBIOLOGY 

if treatment is begun within 3 days of onset of infection, prior to the eruption 
of lesions. These drugs have also shown some efficacy in reducing viral dis- 
semination in immunocompromised patients. Additionally, analgesics and other 
pain killers can help with the patient's neuralgia. 

Prevention of infection spread involves respiratory and contact isolation 
of infected patients. Passive immunization of high-titer varicilla-zoster 
immunoglobulin (VZIg) can be administered to immunocompromised patients 
if given within 3 days of exposure. This treatment is effective only for inhibit- 
ing primary infection in high-risk patients. More recently, a live vaccine has 
been used in the United States since 1995 to prevent primary childhood 
infections. A single dose has been shown to be 80 percent effective in children 
1-13 years of age, and two doses have been shown to be 70 percent effective 
in adults. A new, live-attenuated virus vaccine, Zostavax, is recommended to 
reduce the incidence of shingles for adults over the age of 60. 

COMPREHENSION QUESTIONS 

[38.1] A Tzanck smear is obtained from a scraping of a patient's skin lesion, 
and analysis of the smear shows the presence of multinucleated giant 
cells. Which of the following viruses are known to cause this type cyto- 
pathic effect in infected cells? 

A. Cytomegalovirus 

B. Epstein-Barr virus 

C. Herspes simplex virus- type 2 

D. Human papillomavirus 

E. Human herpesvirus 8 

[38.2] A 3-year-old girl presented to her pediatrician's office with fever, 
swollen lymph nodes, and a vesicular rash on her chest and upper arms. 
The vesicles were at various stages of development: some were newly 
forming, while some were crusted over. Which of the following infec- 
tious agents is the most likely cause of this girl's rash? 

A. Smallpox 

B. Parvovirus B19 

C. Epstein-Barr virus 

D. Measles virus 

E. Varicella-zoster virus 

[38.3] Based on information provided in the previous question, which of the 
following clinical specimens should be collected to confirm diagnosis 
of VZV infection? 

A. Saliva 

B. Blood 

C. Vesicle fluid 

D. Cerebrospinal fluid 

E. Urine 



CLINICAL CASES 275 

Answers 

[38.1] C. HSV-1, HSV-2, and VZV are all known to produce multinucleated 
giant cells resulting in a positive Tzanck smear, whereas CMV, EBV, 
HPV, and human herpesvirus 8 do not. 

[38.2] E. VZV produces a vesicular rash commonly seen in children, and 
different "crops" of vesicles generally appear on the head and trunk 
then moving outward; answers A, B, C, and D are incorrect, smallpox 
infection produces a vesicular rash with all lesions being at the same 
stage of development, whereas parvovirus B19, EBV, and the measles 
virus does produce a rash, but not consisting of vesicular lesions. 

[38.3] C. VZV-specific antigens or viral DNA can be detected in vesicle 
fluid leading to a definitive diagnosis of VZV infection; answers A, 
B, D, and E are incorrect: CMV can be detected in saliva, blood, and 
urine; VZV is not commonly detected in CSF specimens. 



MICROBIOLOGY PEARLS 

Primary lytic infection: chickenpox or varicella; recurrent latent 
infection: shingles or zoster. 

Clinical manifestations: unilateral eruption of a painful rash in a sin- 
gle dermatome. 

Prevention of varicella is by immunization of children and adults 
over the age of 60, respiratory and contact isolation of infected 
persons. 

Treatment: viral DNA polymerase inhibitors such as acyclovir, fam- 
ciclovir, and valacyclovir. 



REFERENCES 

Fields BN, Rnipe DM, Howley PM, et al. Herpesviridae. Fields Virology, 3rd ed. 

Philadelphia, PA: Lippincott-Raven, 1996:2525^11. 
Murray PR, Rosenthal KS, Pfaller MA. Medical Microbiology, 5th ed. St. Louis, 

MO: Mosby, 2005:550-3. 
Ryan JR, Ray CG. Sherris Medical Microbiology, 4th ed. New York: McGraw-Hill, 

2004:562-6. 



This page intentionally left blank 



♦> CASE 39 



A 42-year-old woman with chronic asthma presents for evaluation of a cough. 
She has had severe asthma for most of her life and currently uses both inhaled 
and oral corticosteroids, oral leukotriene modifiers, and inhaled albuterol to 
manage her symptoms. While in the process of tapering down her dose of oral 
steroids, she developed a cough productive of brown mucous and, occasion- 
ally, blood. She has had a low-grade fever as well. Her asthma control has been 
significantly worsened since she developed the cough. On examination, she 
has a temperature of 37.7°C (99.9°F) and a respiratory rate of 22 breaths per 
minute, and her saturation of oxygen is slightly low (96 percent on room air). 
She is coughing frequently. Her head and neck exam is unremarkable. Her pul- 
monary examination is notable for diffuse expiratory wheezing. A chest x- 
ray shows a lobular infiltrate that is reminiscent of a cluster of grapes. A 
complete blood count (CBC) shows a mildly elevated white blood cell count 
with a markedly elevated eosinophil count. A microscopic examination of her 
sputum is also notable for the presence of numerous eosinophils. 

^ What organism is most likely causing her cough? 

^ What is the characteristic morphology of this organism seen on 
microscopic examination? 



278 CASE FILES: MICROBIOLOGY 



ANSWERS TO CASE 39: Aspergillus 

Summary: A 42-year-old asthmatic woman has allergic bronchopulmonary 
aspergillosis. 

^ Most likely etiologic agent: Aspergillus fumigatus 

^ Characteristic morphology of this organism seen on microscopic 
examination: Septate hyphae with 45° angle branching 



CLINICAL CORRELATION 

Aspergillus is a ubiquitous fungal organism that is capable of causing disease 
in both healthy and immunocompromised hosts. Infection occurs following 
either inhalation of the organism into the respiratory tract or introduction 
through the skin via a wound or surgery. Aspergillus fumigatus causes about 
90 percent of invasive disease in humans, with A. flavus causing approxi- 
mately 10 percent. Other Aspergillus species can cause disease but are less 
common. Aspergillus primarily infects the lungs and may cause a hypersensi- 
tivity reaction, chronic necrotizing pneumonia, aspergillomas ("fungal balls"), 
or systemic infection. Aspergillus can also cause keratitis and sinusitis. The 
hypersensitivity reaction, known as allergic bronchopulmonary aspergillosis 
(ABPA), is seen primarily in chronic asthmatics and persons with cystic fibro- 
sis (CF). Approximately 25 percent of asthmatics and about half of patients 
with CF are allergic to Aspergillus, although the percentages that develop 
symptomatic disease are much lower. ABPA causes a cough productive of 
brown mucous plugs and, often, blood. Examination of the mucous will reveal 
eosinophils and the characteristic fungus. The symptoms initially tend to be 
mild but become more severe as the patient ages. Repeated episodes may 
cause bronchiectasis and chronic fibrotic pulmonary disease. 

Systemic disease most often occurs in patients who are severely immuno- 
compromised such as subsequent to bone marrow transplantation. 



APPROACH TO THE SUSPECTED Aspergillus PATIENT 
Objectives 

1 . Know the morphology, environmental sources, and pathogenic proper- 
ties of Aspergillus species. 

2. Know the clinical syndromes and diseases associated with Aspergillus 
infections. 



CLINICAL CASES 279 

Definitions 

Allergic bronchopulmonary aspergillosis (ABPA): A hypersensitivity 

response to inhaled Aspergillus in patients with underlying asthma or 

lung disease. 
Aspergilloma: A fungal ball most commonly in the sinus or within an old 

tuberculous cavity. 
Bronchiectasis: Chronic inflammation of the bronchi with dilatation and 

loss of elasticity of the walls. 

DISCUSSION 

Characteristics of Aspergillus Species 

Aspergillus species is found in every country in the world, and its primary 
habitat is decomposing vegetation. It is an opportunistic pathogen of ani- 
mals and humans that causes a spectrum of disease ranging from allergic 
bronchopulmonary disease to disseminated disease in severely immuno- 
suppressed patients. There are more than 40 species of Aspergillus, not all of 
which cause disease in humans. Therefore species identification is helpful in 
determining the clinical significance of an isolate. Aspergillus fumigatus is 
responsible for the majority of serious infections as a result of these organisms; 
however, A. terreus and A. flavus can be associated with disease in patients on 
cancer chemotherapy. 

A virulence factor common to most Aspergillus species is mycotoxin pro- 
duction. One of the toxins, gliotoxin, can affect phagocytosis by macrophages 
as well as induce apoptosis. 

Several factors contribute to the ability of A. fumigatus to cause infection. 
Aspergillus fumigatus grows more readily at normal human body temperature 
than other Aspergillus species. It has a very small spore size, which allows the 
spores to penetrate deep into the lung. It also is the most rapidly growing of all 
Aspergillus species. 

Diagnosis 

Diagnosis of allergic aspergillosis is usually made clinically, although these 
patients may have positive respiratory cultures for Aspergillus. Patients typ- 
ically have a long-standing history of asthma with a history of infiltrates on 
chest x-ray. Other diagnostic criteria include presence of specific antibody to 
Aspergillus as well as elevated levels of IgE in the serum and peripheral 
blood eosinophils. The lack of systemic symptoms helps differentiate ABPA 
from Aspergillus pneumonia or disseminated disease. Diagnosis of dissemi- 
nated disease is by culture of the organism from a normally sterile site and/or 



280 CASE FILES: MICROBIOLOGY 

demonstration of hyphae invading blood vessels in a tissue biopsy. Disseminated 
disease can also be presumptively diagnosed by presence of antibody or galac- 
tomannan antigen in the patient's serum. 

Fungal hyphae can be seen on direct smear using KOH or calcofluor 
white, which is a more sensitive fluorescent stain. Aspergillus hyphae can be 
identified by their frequent septae, and branching at regular intervals at a 
45° angle (Figure 39-1); however, these characteristics are not specific or 
diagnostic for Aspergillus. Definitive diagnosis would be made by micro- 
scopic observation of the fungus after culture of the organism. Aspergillus 
species can be cultured from sputum or bronchoalveolar lavages of infected 
patients. The fungus grows rapidly on most laboratory media including blood 
agar, although a more selective media such as Sabouraud agar is commonly 
used to culture fungus. Growth is enhanced by incubation at room temperature 
versus 35 °C (95 °F). Visualization of the characteristic structure with a conid- 
iophore, a vesicle to which the phialides are attached would confirm the diag- 
nosis of Aspergillus. Although speciation can be preliminarily made by the 
color of the front and reverse of the colony on Sabouraud dextrose agar and 
their microscopic features, A. fumigatus is differentiated from the others by 
growth at a temperature at or above 50°C (122°F). 




Figure 39-1. Aspergillus fumigatus. Frequent septa with branching pattern is 
characteristic. 



CLINICAL CASES 281 

Treatment and Prevention 

Treatment of ABPA is usually not warranted; however because this is a hyper- 
sensitivity reaction, systemic corticosteroids are effective treatment, whereas 
inhaled corticosteroids are not. Therapy for invasive aspergillosis is with ampho- 
tericin B, itraconazole, or voriconazole. Antifungal agents may be used for 
prophylaxis of patients who are severely immunocompromised to prevent dis- 
seminated disease, particularly bone marrow transplant patients. These patients 
should also be protected from exposure to the organism by use of air filters. 

COMPREHENSION QUESTIONS 

[39.1] A biopsy of an infected lung from a 76-year-old woman who suffered 
a third-degree burn 2 months ago revealed uniform hyphae with regu- 
larly spaced septation and a parallel arrangement. No yeast cells were 
observed. Which of the following is the most probable diagnosis? 

A. Actinomycosis 

B. Aspergillosis 

C. Blastomycosis 

D. Cryptococcosis 

E. Zygomycosis 

[39.2] Which of the following is the probable source of infection in 
aspergillosis in the patient in Question 39.1? 

A. Contact with an infected animal 

B. Implantation 

C. Ingestion 

D. Inhalation 

E. Water used in preparing lemonade 

[39.3] An examination of sputum for a suspected case of fungal infection may 
reveal hyphae in which of the following? 

A. Aspergillosis 

B. Cryptococcosis 

C. Histoplasmosis 

D. Paracoccidioidomycosis 

E. Sporotrichosis 



282 CASE FILES: MICROBIOLOGY 

Answers 

[39.1] B; [39.2] B. Aspergillosis is a spectrum of diseases that may be caused 
by a number of Aspergillus species. These species are widespread in 
nature. Aspergillus species grow rapidly in vivo and in vitro and bear 
long conidiophores with terminal vesicles on which phialides produce 
chains of conidia. In healthy individuals, alveolar macrophages are able 
to phagocytize and destroy the conidia. Macrophages from immuno- 
compromised patients have a diminished ability to do this. In the lung, 
conidia swell and germinate to produce hyphae that have a tendency to 
invade preexisting cavities (abnormal pulmonary space as a result of 
tuberculosis, sarcoidosis, or emphysema). Sputum and lung tissue spec- 
imens produce colonies which are hyaline, septate and uniform in width. 
Blastomyces and Cryptococcus from yeast cells, while Zygomycoses 
species have hyphae that are sparsely septate. Actinomyces may be con- 
sidered a branching bacterium. Aspergillus tends to invade either via 
inhalation or implantation through skin wounds. In this patient with a 3rd 
degree [full-thickness skin] burn, implantation through the wound would 
be the most likely source of infection. 

[39.3] A. Cryptococcosis, histoplasmosis, paracoccidioidomycosis and 
sporotrichosis are all caused by dimorphic fungi. At 37°C (98°F), the 
yeast form predominates. Aspergillosis, on the other hand, is caused 
by an organism that produces only hyphae (no-yeast component). 



MICROBIOLOGY PEARLS 

*♦* Aspergillus is commonly found in the environment and cause a spec- 
trum of disease ranging from allergic bronchopulmonary disease 
to disseminated disease. 

*♦* Microscopically aspergillus has septated, hyphae that branch at 45° 
angles and a vesicle with condida in either a single row (uniser- 
ate) or a double row (biserate). 
Although steroids are used to treat allergic disease, disseminated 
disease is difficult to treat and has a high mortality rate in severely 
immunosuppressed patients such as following bone marrow 
transplant. 



_ 



REFERENCES 

Denning DW. Aspergillus species. In: Mandell GL, Bennett JE, Dolin R, eds. 

Principles and Practice of Infectious Diseases, 6th ed. Philadelphia, PA: Churchill 

Livingstone, 2005:2958-73. 
Murray PR, Rosenthal KS, Pfaller MA. Opportunistic mycoses In: Murray PR, 

Rosenthal KS, Pfaller MA. Medical Microbiology, 5th ed. St. Louis, MO: Mosby, 

2005:779-800. 



♦♦♦ CASE 40 



A 52-year-old man presents to the physician's office for the evaluation of a 
cough and fever. He has had these symptoms for approximately a week. He 
has also noted a sharp chest pain that is worse when he coughs or takes a deep 
breath. He has had some associated fatigue, headaches, achy joints, and 
sweatiness at night. He has been using an over-the-counter flu medication, 
which helps to reduce the cough, but he wanted to be checked because his 
symptoms are lingering. He has no history of pulmonary diseases and has 
never smoked cigarettes. He has had no exposure to ill contacts. His only 
recent travel was a weeklong golf vacation to Phoenix, which he took 3 weeks 
ago. On examination, he is comfortable appearing and in no respiratory dis- 
tress. His temperature is 37.7°C (99.9°F), and his vital signs are otherwise nor- 
mal. His pulmonary examination is notable for some faint expiratory wheezing 
and crackles in the left upper lung field. The remainder of his physical exam- 
ination is unremarkable. A chest x-ray shows hilar adenopathy. A CBC shows 
a normal total white blood cell count but with a high percentage of circulating 
eosinophils. Microscopic examination of a fresh sputum sample treated with 
KOH reveals numerous spherules. 

^ What organism is the likely cause of this patient's symptoms? 

^ For this organism, how do spherules form and what is their role in 
propagating infection? 



284 CASE FILES: MICROBIOLOGY 

ANSWERS TO CASE 40: BLASTOMYCOSIS, 
COCCIDIODOMYCOSIS, AND HISTOPLASMOSIS 

Summary: A 52-year-old man who recently traveled to Phoenix complains of 
a cough, fatigue, and night sweats. A chest x-ray shows hilar adenopathy. The 
sputum reveals numerous spherules. 

^ Organism most likely to cause his symptoms: Coccidioid.es immitis. 

^ How do spherules form and what is their role in propagating 

infection: Inhaled arthroconidia lose their hydrophobic outer wall and 
remodel into spherical cells, or spherules. Nuclear division and cell 
multiplication occur and multiple septae develop within the circular 
cell, dividing it into endospore-containing compartments. The external 
wall of the spherule thins as growth occurs and then ruptures, releasing 
multiple spores and propagating the infection. 

CLINICAL CORRELATION 

Coccidioides immitis is a dimorphic fungus endemic in the western hemi- 
sphere. It is typically found in semiarid climates within the north and south 40° 
latitudes. Common endemic regions in the United States include the San 
Joaquin Valley, southern Arizona, and southwestern Texas. Transmission 
occurs by inhalation of the arthroconidia from the soil. The arthroconidia are 
taken into the bronchioles, where they form a spherule. When symptoms do 
occur, they usually start 1-3 weeks after exposure and typically include cough, 
fever, and fatigue. Chest pain, dyspnea, arthralgias, and skin rashes may occur 
as well. Most infections are self-limited, but it can take several weeks to 
months for symptomatic resolution. A small percentage of infections result in 
progressive pulmonary disease or chronic pulmonary complications, and an 
even smaller percentage may result in dissemination outside of the lung the 
most common site being the skin. Other areas of dissemination include the 
bones, joints, and the central nervous system (CNS). Most patients who develop 
disseminated disease have an underlying risk factor of severe immunosup- 
pression, including those infected with HIV. 

APPROACH TO THE SUSPECTED Coccidoides PATIENT 
Objectives 

1. Know the morphology, growth, and reproductive characteristics of 
C. immitis. 

2. Know the sources of infection, modes of transmission, and clinical dis- 
eases associated with C. immitis infection. 



CLINICAL CASES 285 

Definitions 

Dyspnea: Shortness of breath or difficulty breathing. 

Dimorphic fungi: Fungi that grow as a mold at room temperature and in 

the environment and as yeast at 35°C (95°F) or in the body. 
Arthroconidia: Barrel-shaped structures that are the mold and infectious 

form of C. immitis. 

DISCUSSION 

Characteristics of Coccidioides Species 

Coccidioides is one of several systemic and dimorphic fungi. Histoplasma 

capsulatum, Blastomyces dermatitidis, and Paracoccidioides braziliensis are 
the others. Cryptococcus neoformans is also a systemic fungus but is not 
dimorphic. Sporothrix schenckii is dimorphic but not usually systemic. These 
fungi are commonly found in the environment in differing parts of the world 
and are transmitted by the aerosol route. In the majority of cases, fungi are 
inhaled into the lungs, and disease is unrecognized because patients remain 
asymptomatic. 

Coccidioides immitis grows in the soil as mycelia hy apical extension. 
Maturation results in the development of arthroconidia, which have a 
hydrophobic outer layer and can remain viable for a long period of time. They 
form fragile attachments to adjacent cells that are easily broken. Physical 
trauma, even a mild wind, can break these attachments and result in airborne 
dissemination of arthroconidia. If inhaled, the arthroconidia can deposit in the 
lung, where they lose their hydrophobic outer wall. The cell remodels into a 
spherical form known as a spherule. Within the spherule, cells multiply and 
septae form that divide the spherule into multiple compartments. These com- 
partments contain endospores that are released as the spherule grows and 
eventually ruptures. The endospores are capable of generating new spherules 
or reverting to mycelial growth if removed from the site of an infection. 

Spherule growth and rupture result in a host inflammatory response that 
includes the action of neutrophils and eosinophils. T lymphocytes also play an 
important role in the control of C. immitis infection. Most infections with 
this organism are asymptomatic or cause mild, nonspecific upper respiratory 
symptoms that are not diagnosed. 

Diagnosis 

Initial preliminary diagnosis is made by consistent clinical symptoms in a 
patient with recent travel to a Coccidioides endemic area of the country. 
Definitive diagnosis is made hy direct observation of spherules with sub- 
sequent culture of the organism in a specimen, usually of respiratory origin. 
Direct examination can be made using either KOH or calcofluor white stains. 



286 CASE FILES: MICROBIOLOGY 

Coccidioides immitis is a dimorphic fungus that forms spherules in the 
patient (35°C [95°F]) and arthroconidia in the environment (room tempera- 
ture). The arthroconidia are the infectious form and can be transmitted in 
the laboratory if proper biosafety precautions are not adhered to. Coccidioides 
immitis grows rapidly (within 1 week) on routine laboratory media. Colonies 
appear as a white fluffy mold, whose appearance is indistinguishable from the 
other dimorphic fungi, including Histoplasma capsulatum and Blastomyces 
dermatitidis. Coccidioides immitis can be specifically identified by immunod- 
iffusion of extracted C. immitis antigen and commercially prepared antibody 
or by DNA probes specific for C. immitis RNA. 

In cases in which culture is not possible, or negative, serology or skin test- 
ing may be helpful for diagnosis. The disadvantage of both is that a positive 
conversion may last for life and make diagnosis of a current infection difficult. 

Treatment and Prevention 

Treatment is not usually provided to patients with uncomplicated respiratory 
disease without risk factors for dissemination. Patients with complicated dis- 
ease are treated with either an azole or amphotericin B. 

COMPREHENSION QUESTIONS 

[40.1] A 35 -year-old man is HIV antibody-positive and has a CD4 count of 
50 cells/mm 3 (normal: 500-1000 cells/mm 3 ). He has had a fever of 
38.3°C (101°F) for a few weeks and "feels tired all the time." He has 
no other symptoms, and findings on physical examination are normal. 
Complete blood count, urinalysis, and chest x-ray are normal. A bone 
marrow biopsy reveals granulomas, and a culture grows an organism 
that is a budding yeast at 37°C (98.6°F), but produces hyphae and 
tuberculated chlamydospores at 25°C (77°F). Of the following, which 
is the most likely cause? 

A. Aspergillus fumigatus 

B. Coccidioides immitis 

C. Cryptococcus neoformans 

D. Histoplasma capsulatum 

E. Mucor species 

[40.2] A 4-year-old girl who lives in Bakersfield, CA, has had a low-grade 
fever. Skin tests performed for the first time give the following results: 

Tuberculin (PPD) Positive (10 mm induration) 

Coccidioidin test Positive (15 mm induration) 

Dick test Positive (with erythema) 

Dick control test (heated toxin) Negative (no erythema) 

Schick test Negative (no erythema) 

Schick control test (heated toxin) Negative (no erythema) 



CLINICAL CASES 287 

The test results suggest which of the following? 

A. The patient has been exposed to Coccidioides imtnitis. 

B. The patient has been immunized against Coccidioides immitis. 

C. The patient has had scarlet fever. 

D. The patient has IgG antibody to Mycobacterium tuberculosis. 

E. The patient lacks immunity to Cory neb acterium diphtheriae. 

[40.3] A 50-year-old immunocompromised woman is diagnosed as having 
meningitis. A latex agglutination test on the spinal fluid for capsular 
polysaccharide antigen is positive. Of the following organisms, which 
one is the most likely cause? 

A. Aspergillus fumigatus 

B. Cryptococcus neoformans 

C. Histoplasma capsulatum 

D. Nocardia asteroides 

E. Toxoplasma gondii 

[40 .4] Which of the following is the most common portal of entry in Blastomyces 
dermatitidis infection? 

A. Genitourinary tract 

B. Lymphatic system 

C. Mouth 

D. Respiratory tract 

E. Skin 

Answers 

[40.1] D. An HIV-positive individual may have normal immune capacity as 
measured by laboratory parameters, but still be more at risk for 
opportunistic organisms. Respiratory infections may be caused by 
fungi, bacteria, or viruses. As a result, laboratory results may be cru- 
cial in determining the exact organism causing an infection. In this 
case, the bone marrow biopsy revealed a budding yeast form at 37°C 
(98.6°F), but hyphae and tuberculated chlamydospores at room tem- 
perature (25°C [77°F]). In disseminated histoplasmosis, bone marrow 
cultures are often positive. Tuberculate macroconidia are characteris- 
tic for H. capsulatum 's mycelial form. 

[40.2] A. The Dick and Schick tests are related to streptococcal infections, 
specifically scarlet fever. The young girl has been exposed to a 
Mycobacterium, most likely M. tuberculosis, but the positive reaction 
observed is based on a cellular immune reaction, not one mediated by 
antibodies. The location is a region where Coccidioides is endemic 
and should be one of the suspected pathogens to be considered. No 
vaccine is available for C. immitis. Therefore, a positive coccidioidin 
test indicates that the young girl has been exposed to the agent and 
has developed a cellular immune reaction in response. 



288 CASE FILES: MICROBIOLOGY 

[40.3] B. Cryptococcus neoformans is a yeast characterized by a thick poly- 
saccharide capsule. It occurs worldwide in nature and in very large 
numbers in pigeon feces. Cryptococcus infection is usually associ- 
ated with immunosuppression. Tests for capsular antigen can be per- 
formed on cerebrospinal fluid and serum. The latex agglutination test 
for cryptococcal antigen is positive in 90 percent of patients with 
cryptococcal meningitis. With effective treatment (amphotericin B 
and possibly flucytosine), the antigen titer usually drops except for 
AIDS patients. 

[40.4] D. Blastomyces dermatitidis grows as a mold culture, producing sep- 
tate hyphae and conidia. In a host, it converts to a large, singly bud- 
ding yeast cell. It is endemic in North America. Human infection is 
initiated in the lungs. Diagnosis may be difficult because no skin or 
serologic tests exist. Chronic pneumonia is a common presentation. 
Sputum, pus, exudates, urine, and lung biopsy material can be exam- 
ined microscopically, looking for thick walled yeast cells with broadly 
attached buds. It may also be cultured. 



MICROBIOLOGY PEARLS 



*♦* Coccidioides immitis is a dimorphic, systemic fungus, commonly 
found in the soil of arid areas. 

*♦* Most patients exposed to the arthroconidia of C. immitis develop an 
asymptomatic or respiratory infection. Disseminated disease 
occurs rarely in severely immunosuppressed patients. 
I *♦* Person-to-person transmission of C. immitis is not known to occur. 



REFERENCES 

Galgiani J. Coccidioides immitis. In: Mandell GL, Bennett JE, Dolin R, eds. 

Principles and Practice of Infectious Diseases, 6th ed. Philadelphia, PA: 

Churchill Livingstone, 2005:3040-51. 
Murray PR, Rosenthal KS, Pfaller MA. Systemic mycoses In: Murray PR, 

Rosenthal KS, Pfaller MA. Medical Microbiology, 5th ed. St. Louis, MO: 

Mosby, 2005:765-78. 



♦> CASE 41 



A 28-year-old woman presents complaining of 2-days of itchy vaginal dis- 
charge. One week ago you saw and treated her for a urinary tract infection 
(UTI) with sulfamethoxazole and trimethoprim (SMX-TMP). She completed 
her medication as ordered and developed the vaginal discharge shortly there- 
after. She denies abdominal pain, and her dysuria has resolved. She is not cur- 
rently taking any medications. On examination, she is comfortable appearing 
and has normal vital signs. Her general physical examination is normal. A 
pelvic examination reveals a thick, curd-like, white discharge in her vagina 
that is adherent to the vaginal sidewalls. There is no cervical discharge or cer- 
vical motion tenderness, and bimanual examination of the uterus and adnexa 
is normal. 

^ What is the most likely cause of these symptoms? 

^ What are the most likely reservoirs of this organism in this patient? 



290 CASE FILES: MICROBIOLOGY 

ANSWERS TO CASE 41: Candida 

Summary: A 28-year-old woman who recently took antibiotics now presents 
with a vaginal discharge consistent with candidiasis. 

^ Most likely etiologic organism: Candida albicans 

^ Most likely reservoirs of infection: Gastrointestinal (GI) and vaginal 
colonization 

CLINICAL CORRELATION 

Most Candida infections come from the host's endogenous flora. Both 
humoral and cell-mediated immune functions play a role in defense against 
Candida infections. Those with impaired or suppressed immunity are predis- 
posed to more severe or diffuse disease. Neutropenic persons, such as those on 
chemotherapy or posttransplant patients, are at risk for severe disease, which 
disseminates in the blood stream. In contrast, AIDS patients often develop 
oral, pharyngeal, or esophageal candidiasis but rarely disseminated disease. 
Intact skin also plays a key role in preventing cutaneous infections, because 
breaks in the skin of even healthy hosts may result in Candida skin infections. 
The use of antibiotics is probably the most significant predisposing factor for 
the development of Candida infections. Antibiotics that suppress the growth of 
the normal host bacterial flora can allow Candida to proliferate. This is a fre- 
quent contributing cause of the development of vaginal candidiasis in women 
and Candida diaper dermatitis in infants. 

Candida is the cause of a wide range of infections from oral lesions (thrush) 
to disseminated disease including endocarditis and meningitis. Candida albi- 
cans is the most common cause of vaginitis. Predisposing factors include dia- 
betes, previous antimicrobial use, pregnancy, and use of oral contraceptives. 
Although the pathogenesis and the virulence mechanisms of Candida infec- 
tion is unclear, the presence of pseudohyphae seems to indicate active disease 
versus colonization. Pseudohyphae are able to adhere to epithelial cells, then 
the blastoconidia. 

APPROACH TO THE SUSPECTED Candida PATIENT 
Objectives 

1 . Know the morphology, reservoirs, and reproduction of Candida species. 

2. Know the clinical syndromes, risk factors, and routes of transmission 
of diseases associated with Candida infection. 



CLINICAL CASES 291 

Definitions 

Neutropenia: A decrease in the number of neutrophils circulating in the 
blood to less than 2.0 x 10 9 /L, with significant neutropenia being less 
than 0.5 x 10 9 /L. 

Thrush: Form of oral candidiasis in which a membrane forms in the oral 
cavity consisting of Candida, desquamated cells and white blood cells 
and debris. The appearance is of a creamy white, curd-like exudative 
plaque on the tongue and in the mouth. 

DISCUSSION 
Characteristics of Candida 

Candida are yeasts that exist as both sexual and asexual forms, reproduce 
by budding and form blastospores, which are small, thin-walled ovoid cells. 
Blastospores, pseudohyphae may be seen on examination of clinical speci- 
mens. There are over 150 species of Candida, 9 of which appear to cause dis- 
ease in humans. Candida albicans is the most common cause of human 
candidiasis. It can be found in soil and on inanimate objects and foods. It is 
also found in the normal flora of the human GI tract, vagina, and skin. 

Diagnosis 

Diagnosis of vaginitis is made by a combination of physical examination and 
testing the vaginal exudate. Ruling out other causes of vaginitis may be aided 
by determining the pH of the exudates as well as stain and culture of the mate- 
rial. Yeast cells are larger than bacteria and can be visualized easily by 
direct wet preparation of the exudates with KOH. Candida will grow with 
24-48 hours on most routine laboratory media; however, Sabouraud dextrose 
agar can be used to inhibit the normal flora bacteria in cultures for mucosal 
candidiasis. Candida colonies are smooth and creamy, although some species 
may be dry and can be identified as yeast by a wet preparation. Candida species 
produce round or oval blastoconidia, and some species also produce pseudo- 
hyphae (chains of elongated blastoconidia), as in Figure 41-1. Preliminary 
differentiation of Candida albicans from the other Candida species can be 
made by observation of the presence of a germ tube. Candida albicans will 
make a germ tube after several hours incubation in the presence of serum. 
Candida albicans can also be differentiated from other yeast based on their 
microscopic morphology on corn meal agar. Candida albicans produce 
chlamydospores, large rounded structures in the middle of the pseudohyphae. 
Yeasts that are germ tube-negative can be further identified by assimilation of 
different substrates. Several commercial kits are available that identify 
Candida to the species level. 



292 CASE FILES: MICROBIOLOGY 




Figure 41-1. Candida albicans. Pseudohyphae noted on microscopy. 
{Reproduced, with permission, Brooks G, Butel J, Morse S. Jawetz, Melnick, 
& Adelberg 's medical microbiology, 23rd ed. New York: McGraw-Hill, 2004:646.) 



Treatment and Prevention 

Therapy for Candida vaginitis is usually topical antifungal agent such as 
nystatin or clotrimazole. It is not uncommon for patients to remain or be 
recurrently colonized with vaginal yeast after appropriate therapy. This may or 
may not lead to a symptomatic recurrence. Oral or intravenous therapy with 
either an azole antifungal such as fluconazole, amphotericin B, or the new 
agent, caspofungin, are used for treatment of disseminated infections with 
Candida. The agent of choice is dependent on the species of Candida isolated 
and the susceptibility of the isolate to the antifungal. Prophylaxis for Candida 
infections is not routinely recommended because of the selection of strains of 
Candida that are resistant to antifungal agents. The only population in which 
some benefit has been seen with prophylaxis is in bone marrow transplant 
patients. Partly as a result of the increased use of fluconazole, the incidence of 
Candida species not albicans, or the species more likely to be fluconazole 
resistant have also increased. 

COMPREHENSION QUESTIONS 

[41.1] Candida albicans can be differentiated from other Candida species on 
cornmeal agar by its unique ability to form which of the following? 

A. Arthrospores 

B. Aseptate hyphae 

C. Chlamydospores 

D. Germ tubes 

E. Tuberculate macroconidia 



CLINICAL CASES 293 

[41.2] A young man in his mid-twenties presented with mucosal lesions in his 
mouth. Based on his CD4 cell count and other signs during the past 
few months, he was diagnosed as having AIDS. Which of the follow- 
ing is the most likely etiology of the oral lesions? 

A. Aspergillus 

B . Candida 

C. Cryptococcus 

D. Mucor 

E. Rhizopus 

[41.3] Which of the following morphologic structures is not associated with 
Candida albicans? 

A. Chlamydospore 

B. Hyphae 

C. Pseudohyphae 

D. Sporangium 

E. Yeast 

[41.4] Which of the following is the main reason that individuals taking tetra- 
cycline often develop candidiasis? 

A. Candida albicans is capable of degrading the antibiotic. 

B . The action of the antibiotic is neutralized by the protein of C. albicans. 

C. The antibiotic damages the host mucous membrane. 

D. The antibiotic is nutritionally favorable for the growth of C. albicans. 

E. The normal bacterial flora is drastically altered by tetracycline. 

Answers 

[41.1] C. Although multiple Candida species may cause disease in humans, 
C. albicans is the most frequent species identified. Chlamydospores 
(chlamydoconidia) are round, thick-walled spores formed directly 
from the differentiation of hyphae in which there is a concentration 
of protoplasm and nutrient material. They may be intercalary (within 
the hyphae) or terminal (end of hyphae). Germ tubes appear as 
hyphal-like extensions of yeast cells, usually without a constriction 
at the point of origin from the cell. Approximately 75 percent of the 
yeasts recovered from clinical specimens are C albicans, and the 
germ-tube test can usually provide identification within 3 hours. 
The morphologic features of yeasts on cornmeal agar containing 
Tween 80 allow for the differentiation of C. albicans from five other 
Candida species. 



294 CASE FILES: MICROBIOLOGY 

[41.2] B. The risk factors for cutaneous and mucosal candidiasis include 
AIDS, pregnancy, diabetes, young or old age, birth control pills, and 
trauma. Oral thrush can occur on the tongue, lips, gums, or palate. It 
may be patchy to confluent, and it forms whitish lesions composed of 
epithelial cells, yeasts, and pseudohyphae. Oral thrush commonly 
occurs in AIDS patients. Although the other genera listed may be 
opportunistic, only Candida routinely presents with mucosal lesions. 

[41.3] D. A sporangium is a sac enclosing spores, seen in certain fungi, but 
not Candida species. Spores produced within a sporangium, usually 
located at the tip of a long hyphal stalk are released by rupture of the 
sporangial wall. All other options (chlamydospore, hyphae, pseudo- 
hyphae, and yeasts) are routinely observed in C. albicans cultures, 
depending on conditions of growth. 

[41.4] E. Patients with compromised host defenses are susceptible to ubiq- 
uitous fungi to which healthy people are exposed but usually resist- 
ant. Candida and related yeasts are part of the normal microbial flora, 
but are kept at low numbers by faster-growing normal flora bacteria. 
If broad-spectrum antimicrobials are used, much of the usual flora bac- 
teria may be eliminated. No longer held in check, the opportunist yeast 
may become more predominant and opportunistic. Discontinuation of 
use of the broad-spectrum antibiotic is an important first step in 
patient management, allowing for reestablishment of the normal or 
usual flora and control of the yeast species. 



MICROBIOLOGY PEARLS 

*♦* Candida albicans is a germ tube — positive yeast that is the most 

common cause of vaginitis. 
*♦* Candida albicans can be distinguished from the other Candida 

species by formation of a germ tube after incubation in serum. 
*♦* The incidence of Candida species other than albicans has increased 

because of the increased use of azoles and their propensity to 

develop resistance to those antifungal agents. 



REFERENCES 

Edwards JE. Candida species. In: Mandell GL, Bennett JE, Dolin R, eds. Principles 

and Practice of Infectious Diseases, 6th ed. Philadelphia, PA: Churchill Livingstone, 

2005:2938-57. 
Murray PR, Rosenthal KS, Pfaller MA. Opportunistic mycoses. In: Murray PR, 

Rosenthal KS, Pfaller MA. Medical Microbiology, 5th ed. St. Louis, MO: Mosby, 

2005:779-800. 



$► CASE 42 



♦> 



A 32-year-old man with known AIDS is brought to the emergency room with 
headache and fever for the past 3 days. According to family members who are 
with him, he has been confused, forgetful, and irritable for a few weeks prior 
to the onset of these symptoms. They state that he has advanced AIDS with a 
low CD4 count and has had bouts of Pneumocystis pneumonia, candidal 
esophagitis, and Kaposi sarcoma. He is on multiple medications, although they 
don't know whether he is actually taking them. On examination, he is cachetic 
and frail appearing. He is confused and only oriented to his name. His tem- 
perature is 37.8°C (100°F), and his other vital signs are normal. Examination 
of his cranial nerves is normal. He has minimal nuchal rigidity. Cardiovascular, 
pulmonary, and abdominal examinations are normal. He is hyperreflexic. A 
head CT scan is normal. A report of the microscopic examination of his cere- 
brospinal fluid obtained by lumbar puncture comes back from the laboratory 
and states that there were numerous white blood cells, predominantly lym- 
phocytes, and no organisms identified on Gram stain but a positive India ink 
test. 

^ What organism is the likely cause of this illness? 

^ What characteristic of this organism is primarily responsible for its 
virulence? 



296 CASE FILES: MICROBIOLOGY 

ANSWERS TO CASE 42: Cryptococcus neoformans 

Summary: A 32-year-old male with advanced AIDS presents with meningitis. 
The India ink test is positive. 

^ Most likely etiology for this man's meningitis: Cryptococcus neoformans. 

^ Characteristic of this organism is primarily responsible for its 
virulence: Cryptococcus neoformans is known characteristically to 
produce a mucopolysaccharide capsule. This is a key feature of this 
organism's virulence, because it is antiphagocytic and also interferes 
with leukocyte migration to sites of infection. 

CLINICAL CORRELATION 

Cryptococcus neoformans is an encapsulated monomorphic fungi that commonly 
causes chronic meningitis in immune-suppressed individuals and occasionally in 
immune-competent persons. The lungs are the primary site of infection, although 
the organism appears to have specific affinity for the brain and meninges on sys- 
temic spread. Cryptococcus neoformans is the leading cause of fungal menin- 
gitis and is an important cause of mortality in AIDS patients. 

APPROACH TO SUSPECTED Cryptococcus neoformans 
INFECTION 

Objectives 

1. Be familiar with the characteristics and disease presentation of 
C. neoformans fungi. 

2. Know the methods of diagnosis of infection with C. neoformans. 

3. Be able to describe the treatment and prevention of infection. 

Definitions 

Meningitis: Inflammation of the meninges. 

Nuchal rigidity: Stiffness of the neck associated with meningitis. 

Cachetic: Weight loss or wasting because of disease or illness. 

DISCUSSION 

Characteristics of Cryptococcus neoformans That Impact 
Transmission 

Cryptococcus neoformans is an encapsulated yeast, 4-6 mm in diameter, 

which is distributed globally. The most common serotypes are found in high 
concentrations in pigeon and other bird droppings, although they do not 



CLINICAL CASES 297 

appear to cause disease in these hosts. The most common route of transmis- 
sion to humans is via aerosolization of the organism followed by inhalation 
into the lungs. Direct animal-to-person transmission has not been shown. 
Unlike other systemic fungi, C. neoformans is monomorphic, not dimorphic, 
and grows as budding yeast cells at both 25 °C (77°F) in culture and at 37°C 
(98.6°F) in tissues. When grown in culture, C. neoformans produces white- or 
tan-colored mucoid colonies in 2-3 days on a variety of common fungal media. 
Microscopically, the organism appears as spherical budding yeast, surrounded 
by a thick capsule. Cryptococcus neoformans differs from the other nonpath- 
ogenic cryptococcal strains by its ability to produce phenol oxidase and growth 
at 37°C (98.6°F). 

The capsule is an important virulence factor of Cryptococcus, and it con- 
sists of long, unbranched polysaccharide polymers. Capsule production is nor- 
mally repressed in environmental settings and is stimulated by physiologic 
conditions in the body. The capsule is antiphagocytic, because of its large 
size and structure and has also been shown to interfere with antigen presenta- 
tion and the development of T-cell-mediated immune responses at sites of 
infection. This suppression of an immune response can allow for multiplica- 
tion of the organism and promotion of its spread outside the respiratory tract. 
Once outside the lung, the organism appears to have an affinity for the central 
nervous system (CNS), possibly because of its ability to bind C3 and the low 
levels of complement found in the CNS. 

Diagnosis 

Inhalation of these aerosolized yeast cells leads to a primary pulmonary infec- 
tion. The infection may be asymptomatic or may result in a flu-like respiratory 
illness or pneumonia. Commonly, cryptococcal pulmonary infection is identi- 
fied only as an incidental finding on a chest x-ray being performed for other 
reasons. Often the infection and resulting lesions appear suspicious for a 
malignancy, only to be diagnosed properly after surgical removal. The most 
commonly diagnosed cryptococcal disease is meningitis, which results from 
hematogenous spread of the organism from the lung to the meninges. It occurs 
most commonly in persons with AIDS or those who are immunosuppressed 
for other reasons, but it can occasionally occur in persons without underlying 
conditions. Outside the lungs, C. neoformans appears to have a preference for 
the cerebrospinal fluid (CSF), but disseminated disease can also cause infec- 
tions of the skin, eye, and bone. Cryptococcal meningitis may be insidious in 
its onset, slowly causing mental status changes, irritability, or confusion that 
occurs over weeks to months, or it can occur acutely, with immediate changes 
in mentation and meningeal symptoms. Clinical disease may present with 
intermittent headache, irritability, dizziness, and difficulty with complex cere- 
bral functions and may even be mistaken as psychoses. Seizures, cranial nerve 
signs, and papilledema may appear in late clinical course. 



298 CASE FILES: MICROBIOLOGY 

A diagnosis of C. neoformans infection is made primarily by clinical pres- 
entation and examination of CSF for increased pressure, increased number of 
white cells, and low glucose levels. Serum and CSF specimens should also be 
tested for polysaccharide capsular antigen by latex agglutination or enzyme 
immunoassay. Another classic test for C. neoformans is the India ink test, 
which is an easy and rapid test that is positive in approximately 50 percent of 
patients with cryptococcal disease. A drop of India ink is placed on a glass 
slide and mixed with a loopful of CSF sediment or a small amount of isolated 
yeast cells. A cover slip is added and the slide is examined microscopically for 
encapsulated yeast cells that exclude the ink particles. 

Treatment and Prevention 

Cryptococcus neoformans infections can be treated with antifungal agents 
such as amphotericin B or fluconazole. Amphotericin B is a broad-spectrum 
chemo-therapeutic agent and is the most effective drug for severe systemic 
mycoses. However, it is an extremely nephrotoxic agent to which all patients 
have adverse reactions such as fever, chills, dyspnea, hypotension, and nausea. 
Fluconazole is less toxic than amphotericin B and produces fewer side effects; 
however, resistance to fluconazole has been shown to occur. AIDS patients 
with cryptococcosis are required to continue lifelong suppressive therapy with 
fluconazole to prevent relapse of fungal infection. 

COMPREHENSION QUESTIONS 

[42. 1] A 32-year-old man who lives in downtown Philadelphia presents to his 
physician with a 4-day history of terrible headache, fever, and stiff 
neck. He has always been in good health and attributes this to his 
healthy eating habits and his daily running through the city parks near 
his apartment. The physician suspects the man may have cryptococcal 
meningitis and collects CSF for examination. Which of the following 
results would you most likely expect from this patient's CSF studies? 

A. Elevated CSF pressure with increased white cell counts 

B. Elevated polymorphonuclear cells with high protein levels 

C. Elevated lymphocytes with normal glucose levels 

D. Normal CSF pressure with a positive Gram stain reaction 

E. Normal CSF pressure with negative Gram stain reaction 

[42.2] Which of the following laboratory tests would best definitely diagnose 
cryptococcal infection in the above patient? 

A. Quelling reaction capsular swelling 

B. Latex agglutination test for polysaccharide capsular antigen 

C. Ouchterlony test for fungal infection 

D. India ink test for the presence of capsulated yeast 

E. Gram stain reaction 



CLINICAL CASES 299 

[42.3] A 35-year-old man with AIDS presents to the local clinic with com- 
plaints of nausea, vomiting, confusion, fever and staggering gait. A 
lumbar puncture is performed, and an organism with a halo is noted 
with India ink preparation. What drug would be most beneficial? 

A. Ketoconazole and amphotericin B 

B. Fluconazole and amphotericin B 

C. Nystatin and ketoconazole 

D. Nystatin and miconazole 

E. Griseofulvin 

[42.4] A 34-year-old white homeless man in New York city is brought in by 
the police to the emergency room because he was found wandering the 
streets confused with a staggering gait. On physical exam, he is noted 
to have acne like lesions over a large part of his body accompanied 
with skin ulcers. He is febrile and has some cranial nerve deficits. A 
short time later the man becomes short of breath, which was deter- 
mined to be caused by severe cerebral edema compressing the medulla. 
Which of the following is the most likely causative agent? 

A. Histoplasma capsulatum 

B. Coccidioides immitis 

C. Exophiala werneckii 

D. Sporothrix schenckii 

E. Cryptococcus neoformans 

Answers 

[42.1] A. Meningitis caused by C. neoformans infection typically results in 
increased CSF pressure with an increased number of white cells and 
low glucose levels; answers B, C, D, and E are incorrect: both (B) and 
(D) appropriately describe meningitis caused by a bacterial agent 
such as Neisseria meningitides; (C) appropriately describes meningi- 
tis caused by a viral agent such as herpes simplex virus; (E) describes 
normal CSF findings. 

[42.2] B. answers A, C, D, and E are incorrect: (A) is a test useful for diag- 
nosing Streptococcus pneumoniae and uses capsule-specific antibody 
to cause capsule swelling; (C) is an immunodiffusion test useful in 
diagnosing Histoplasma and Blastomyces fungal infections; (D) does 
provide rapid diagnosis of Cryptococcus neoformans; however, this 
assay is positive in only 50 percent of cryptococcal cases; (E) the 
Gram stain is more useful in diagnosing bacterial infections, because 
it would show the presence of yeast cells, but the capsule would not 
be visible. 



300 CASE FILES: MICROBIOLOGY 

[42.3] B. The organism present is C. neoformans, the usual treatment for 
cryptococcosis is amphotericin B and fluconazole. The other drugs 
listed are not indicated for cryptococcosis. Ketoconazole is usually 
used for chronic mucocutaneous candidiasis. Nystatin is used for can- 
didiasis, and griseofulvin is indicated for dermatophytes of the hair, 
skin, and nails. Miconazole is used for topical fungal infections, oral 
thrush, and vaginitis. 

[42.4] E. The symptoms described including acne-like lesions, skin ulcers, 
fever, confusion, staggering gait and cranial nerve deficits are a clas- 
sic example of Cryptococcus neoformans infection. In some patients 
the cerebral edema progresses to a fatal stage compressing the medulla 
reducing respiratory efforts. The other yeast listed do not cause cere- 
bral edema. Exophiala werneckii causes tinea nigra characterized by 
dark patches on the hands and soles of the feet. Sporothrix schenckii 
is usually associated with a prick from a rose thorn. Coccidioides 
immitis is associated with the desert southwest. Histoplasma capsu- 
latum is usually associated with a Mississippi river valley history, as 
well as lesions that calcify. 



MICROBIOLOGY PEARLS 



*♦* Cryptococcus neoformans is transmitted via aerosolized pigeon or 
bird droppings. 

*♦* Clinical manifestations: headache, altered mental state, nuchal rigid- 
ity, often associated with AIDS. 

*♦* Identification: clinical symptoms, examination of CSF for increased 
pressure and number of white cells with low glucose levels, and 
a positive capsular antigen latex agglutination and India ink tests. 

*♦* Current treatment: amphotericin B or fluconazole. 



REFERENCES 

Brooks GF, Butel JS, Morse SA. Jawetz, Melnick, & Adelberg's Medical Microbiology, 

23rd ed. New York: McGraw-Hill, 2004:647-9. 
Murray PR, Rosenthal KS, Pfaller MA. Medical Microbiology, 5th ed. St. Louis,MO: 

Mosby, 2005:779-800. 
Ryan JR, Ray CG. Sherris Medical Microbiology, 4th ed. New York: McGraw-Hill, 

2004:647-9. 



t* CASE 43 



♦> 



A 29-year-old woman comes into the clinic for evaluation of a cough. Her 
symptoms started a few weeks ago and have progressively worsened. The 
cough is not productive. She has had intermittent, low-grade fevers and feels 
short of breath. She has tried some over-the-counter cough medications, which 
don't seem to help. She smokes approximately a half-pack of cigarettes a day. 
She denies any history of pulmonary diseases. On examination, her tempera- 
ture is 37.5°C (99.5°F), pulse is 100 beats per minute, respiratory rate is 
26 breaths per minute, and oxygen saturation is 89 percent on room air. Her 
blood pressure is normal, but when applying the blood pressure cuff, you 
notice numerous scars in her antecubital region consistent with "needle 
tracks." In general, she is a thin woman who appears to be in moderate respira- 
tory distress and is coughing frequently. Her head and neck examination is nor- 
mal. Her lung examination is notable for decreased breath sounds and rhonchi 
in all fields. Her cardiovascular and abdominal examinations are normal. A chest 
x-ray shows a bilateral interstitial infiltrate with a "ground-glass" appearance. 
She confides that she is HIV positive. 

^ What organism is the likely cause of her symptoms? 

^ Describe the sexual phase of reproduction of this organism. 



302 CASE FILES: MICROBIOLOGY 

ANSWERS TO CASE 43: Pneumocystis jiroveci 

Summary: A 29-year-old woman intravenous drug user who is HIV positive 
has an interstitial pneumonia. 

^ Most likely etiologic agent: Pneumocystis jiroveci. 

^ Sexual phase reproduction of P. jiroveci: Haploid trophic forms 
conjugate to form diploid zygotes that become sporocysts; sporocysts 
undergo meiosis and mitosis to form the spore case that contains eight 
haploid spores. The spores are released by rupture of the spore case wall. 

CLINICAL CORRELATION 

Pneumocystis is an opportunistic organism found primarily in the lungs of 
humans and other animals. The reservoir of the organism in the environment 
is at this point unknown. Transmission of the organism is from person to per- 
son by respiratory droplet inhalation into the lungs. It is unclear whether dis- 
ease results from the reactivation of a latent infection or acquisition of a new 
infection. The cellular immune system is primarily responsible for host 
defenses, with alveolar macrophages and CD4 cells playing a particularly 
important role. In HIV patients, the risk of developing symptomatic disease 
from Pneumocystis is highly correlated to the number of circulating CD4 cells, 
with the highest risk in those persons with CD4 counts below 200/mm 3 . The 
use of corticosteroids or other immunosuppressive drugs, treatment for malig- 
nancies, or severe malnutrition are risk factors for disease in non-HIV infected 
people. Classic Pneumocystis pulmonary infection is an interstitial pneumonia 
with plasma cell infiltrates. Typical symptoms are nonproductive cough, fever, 
dyspnea, and hypoxia. Chest x-rays commonly show a bilateral interstitial 
infiltrate extending from the hilum with a "ground-glass" appearance. In severely 
immunosuppressed patients Pneumocystis can disseminate most commonly to 
the thyroid, liver, bone marrow, lymph nodes, or spleen. 

APPROACH TO THE SUSPECTED Pneumocystis 
PATIENT 

Objectives 

1 . Know the life cycle, morphology, and reproduction of Pneumocystis. 

2. Know the epidemiology, modes of transmission, and clinical syn- 
dromes associated with Pneumocystis infection. 



CLINICAL CASES 303 

Definitions 

Hypoxia: Reduction of oxygen supply to the tissues despite adequate blood 

perfusion. 
Dyspnea: Shortness of breath leading to labored breathing. 

DISCUSSION 

Characteristics of Pneumocystis 

Pneumocystis was originally characterized as a trypanosome; however, 
advanced molecular biological techniques have shown it to be closely related to 
fungi. It is unusual among fungi because it lacks ergosterol in its cell mem- 
branes and is insensitive to many antifungal drugs. Its life cycle has both sex- 
ual and asexual components. The trophic form of Pneumocystis is small and 
often seen in clusters. It multiplies asexually by binary fission and sexually by 
conjugation of haploid trophic forms to diploid cells that become sporocysts. 
These uninuclear cells undergo miosis then mitosis to form a spore case, which 
contains eight haploid spores. The spores are released by rupture of the cell 
wall, although the cyst wall remains and can be identified as empty structures. 
Pneumocystis is thought to be ubiquitous in the environment, and most 
adults have been exposed to the organism during childhood and develop an 
asymptomatic infection. Pneumocystis is found in many mammalian species and 
is not thought to cross species lines. Pneumocystis that infects humans was 
recently renamed P. jirovecii. 

Diagnosis 

The diagnosis is confirmed by the presence of the organisms in sputum or 

bronchial samples obtained by bronchoalveolar lavage or other techniques, 
such as sputum induced by respiratory therapy. Pneumocystis can be identified 
microscopically by using numerous stains, such as methenamine silver, 
Giemsa, chemofluorescent agents such as calcofluor white, or specific 

immunofluorescent monoclonal antibodies. The monoclonal antibody fluores- 
cent stain increases the sensitivity and specificity of the test. The diagnostic 
stage seen is usually the cyst form. The organism cannot be grown in culture. 

Treatment and Prevention 

Treatment for Pneumocystis is usually with sulfamethoxazole and trimetho- 
prim (SMX-TMP); however, in allergic patients there are other options such 
as dapsone or pentamidine. Prophylaxis with SMX-TMP is recommended 
for severely immunosuppressed patients including HIV patients with a CD4 
count of less than 200 cells/mm 3 . 



304 CASE FILES: MICROBIOLOGY 

COMPREHENSION QUESTIONS 

[43. 1 ] Pneumocystis jiroveci is now considered a fungus. Which of the following 
statements accurately describes this organism? 

A. In immunocompromised patients the organism invades blood ves- 
sels causing thrombosis and infarction. 

B. It grows best in a culture medium containing tissue fluid. 

C. It is now classified as a fungus because it grows into septate hyphae 
in Sabouraud agar. 

D. It is sensitive to antifungal agents such as amphotericin B. 

E. Methenamine silver stain is used to visualize the organism in the 
clinical specimen. 

[43.2] Which of the following statements best describes the laboratory diag- 
nosis of Pneumocystis jiroveci? 

A. India ink stain of bronchoalveolar lavage material 

B. KOH stain of lung biopsy tissue 

C. Growth of the organism on Sabouraud agar 

D. Methenamine silver stain of induced sputum 

[43.3] Pneumocystis jiroveci produces disease under what conditions listed 
below? 

A. In individuals with CD4 lymphocyte counts above 400/|jL 

B. In the presence of immunosuppression 

C. Infection in early childhood 

D. Prophylaxis with SMX-TMP 

Answers 

[43.1] E. Pneumocystis jiroveci is often reported as the organism responsible 
for the described case. Pneumocystis carinii is found in rats, whereas 
P. jirovecii is found in humans. These species are not grown in the 
laboratory and do not respond to traditional antifungal chemotherapy. 
Being found primarily in the lungs, respiratory infections occur in 
immunocompromised individuals, and dissemination is rare. 
Specimens of bronchoalveolar lavage, lung biopsy, or induced spu- 
tum are stained (e.g., Giemsa or methenamine silver) and examined 
for cysts or trophozoites. 

[43.2] D. Because Pneumocystis species are not able to be grown in the lab- 
oratory, staining procedures constitute the primary diagnostics tech- 
niques used. See the answer to Question 43.1 for further discussion. 

[43.3] B. Pneumocystis jiroveci and P. carinii are present in the lungs of 
many animals, including humans. This organism rarely causes dis- 
ease except in immunocompromised hosts. No other natural reservoir 
has ever been demonstrated, and the mode of infection is unclear. 
Transmission by aerosols may be possible. 



CLINICAL CASES 305 



MICROBIOLOGY PEARLS 

*♦* Pneumocystis that infects humans was recently renamed from P. carinii 
to P. jirovecii. 

*♦* Pneumocystis has a predilection for the lungs of humans and animals. 

*♦* Diagnosis of Pneumocystis is made by induced sputum or bron- 
choscopy with microscopic visualization of the cyst forms with 
either Papanicolaou, Giemsa, silver stain, or monoclonal antibodies. 



REFERENCES 

Murray PR, Rosenthal KS, Pfaller MA. Opportunistic mycoses. In: Murray PR, 

Rosenthal KS, Pfaller MA. Medical Microbiology, 5th ed. St. Louis, MO: 

Mosby, 2005:664-72. 
Thomas CF, Limper AH. Pneumocystis pneumonia. NEJM 2004;350:2487-98. 
Walter PD. Pneumocystis carinii. In: Mandell GL, Bennett JE, Dolin R, eds. 

Principles and Practice of Infectious Diseases, 5th ed. Philadelphia, PA: Churchill 

Livingstone, 2000:2781-95. 



This page intentionally left blank 



♦> CASE 44 



A 44-year-old woman presents to the physician's office for evaluation of skin 
growths on her right arm. She reports that a few weeks ago she developed 
some small, red bumps on her right palm, which seemed to come together into 
a larger nodule. This then ulcerated, but it never was painful. She has been put- 
ting topical antibiotic on this area, and it seemed to be improving. However, in 
the past week she has noticed new growths extending up her forearm that 
appear just like the original lesion. She denies having skin lesions anywhere 
else, denies systemic symptoms such as fever, and has no history of anything 
like this before. She has no significant medical history and takes no medications. 
She is employed as a florist and floral arranger. On examination, she is comfort- 
able appearing and has normal vital signs. On her right palm you see a circular, 
1 -cm -diameter, ulcerated area with a surrounding red, raised border. There are 
two identical appearing, but smaller, lesions on the forearm. Microscopic exam- 
ination of a biopsy taken from one of the lesions reveals numerous white blood 
cells and cigar-shaped yeast forms. 

^ What is the most likely infectious cause of these lesions? 

^ What is the most likely route by which this infection was 
transmitted? 



308 CASE FILES: MICROBIOLOGY 

ANSWERS TO CASE 44: Sporothrix schenckii 

Summary: A 44-year-old florist has painless, ulcerated lesions on her right 
hand and arm. 

^ Most likely infectious etiology: Sporothrix schenckii. 

^ Most likely route by which this infection was transmitted: Most 

likely mechanism of infection is inoculation into the skin via a puncture 
of the hand with an infected plant (most likely a rose thorn). 

CLINICAL CORRELATION 

Cutaneous sporotrichosis results from the inoculation of the organism into the 
skin via a puncture or other minor trauma. Most cases occur in persons with 
occupational or avocational exposure to infected material, such as in garden- 
ing or farming. The most common exposures are to rose thorns and sphagnum 
moss. The initial lesions are usually in areas that are prone to trauma, such as 
the extremities. They are often erythematous papules or nodules, which then 
ulcerate. Secondary lesions develop along the lines of lymphatic drainage. The 
lesions are usually painless, can wax and wane, and systemic symptoms are 
rare. Extracutaneous infections with S. schenckii have occurred, most com- 
monly involving the joints, particularly hand, elbow, ankle, or knees. Cases of 
pulmonary sporotrichosis as well as meningitis have been described. Invasive 
and disseminated disease may occur in the severely immunosuppressed, par- 
ticularly patients with advanced HIV disease. 

APPROACH TO THE SUSPECTED SPOROTRICHOSIS 
PATIENT 

Objectives 

1 . Know the morphologic characteristics of the yeast and mycelial forms 
of 5". schenckii. 

2. Know the common sources, routes of transmission, and clinical syn- 
dromes associated with S. schenckii infections. 

Definitions 

Dematiaceous fungi: Fungi with dark colored (brown or black) conidia 

and/or hyphae. 
Lymphadenitis: Inflammation of the lymph node(s). 



CLINICAL CASES 309 



DISCUSSION 

Characteristics of Sporothrix schenckii 

Sporothrix schenckii is a dimorphic fungus that is most often isolated from 
soil, plants, or plant products. When cultured at 37°C (98.6°F) or in vivo, it 
exists as cigar-shaped yeast. At lower temperatures, it exists as a white, fuzzy 
mold that on further incubation develops a brown pigment. The hyphal form 
has numerous conidia, which develop in a rosette pattern at the ends of conid- 
iophores. The fungus is found in the soil and on vegetation in all parts of the 
world, but most commonly in the tropical regions of North and South America. 
Transmission from animals to man has also been rarely described. 

Diagnosis 

Skin lesions associated with sporotrichosis can resemble those of other infec- 
tious and noninfectious entities, such as other fungal infections, Mycobacterium 
infections, or collagen vascular diseases. Diagnosis can be made by culture of 
biopsy material or demonstration of the characteristic cigar-shaped yeast 
forms on microscopic examination of a biopsy specimen. Multiple attempts at 
biopsy and culture may be required to recover the organism. 

Sporothrix schenckii grows well within several days to several weeks on 
routine fungal media such as Sabouraud dextrose agar. Colonies initially are 
small and white to cream color that eventually turn brown to black. Laboratory 
confirmation of S. schenckii can be established by demonstration of character- 
istic mold structures after culture at room temperature. The rosette formation 
of the conidia on the conidiophore is characteristic, but not diagnostic. 
Conversion from the hyphal form to the yeast form on subculture of a speci- 
men at 37°C (98.6°F) can aid in the specific identification of the fungus. 

Treatment and Prevention 

Cutaneous sporotrichosis is usually treated orally with either a saturated 
solution of potassium iodide or an antifungal agent such as itraconazole. 

Extracutaneous or disseminated disease is difficult to treat, but usually treated 
with itraconazole. Patients with concomitant HIV and sporotrichosis are usu- 
ally treated prophylactically for the rest of their life with oral itraconazole. 



310 CASE FILES: MICROBIOLOGY 

COMPREHENSION QUESTIONS 

[44.1] Which of the following fungi is most likely to cause cutaneous disease? 

A. Aspergillus fumigatus 

B. Candida albicans 

C. Cryptococcus neoformans 

D. Histoplasma capsulatum 

E. Sporothrix schenckii 

[44.2] A woman who pricked her finger while pruning some rose bushes 
develops a local pustule that progressed to an ulcer. Several nodules 
then developed along the local lymphatic drainage. The most reliable 
method to identify the etiologic agent is which of the following? 

A. Culture of the organism in the laboratory 

B. Gram stain of smear prepared from the lesion 

C. India ink preparation 

D. Skin test for delayed hypersensitivity 

E. Stain the culture with potassium iodide 

Answers 

[44.1] E. Aspergillus, Cryptococcus, and Histoplasma infections routinely 
involve the respiratory system and form cellular components recog- 
nizable in the diagnostic laboratory. Candida species are usually 
endogenous flora that may be opportunistic under the right circum- 
stances (e.g., immunocompromised patient). Cutaneous and systemic 
infections are possible under these conditions. S. schenckii is typi- 
cally introduced into the skin by trauma, often related to outdoor 
activities and/or plants. About three-fourths of the cases are lympho- 
cutaneous, with multiple subcutaneous nodules and abscesses along 
the lymphatics. 

[44.2] A. The most reliable method of diagnosing S. schenckii is by culture. 
Specimens are usually biopsy materials or exudate from granulose or 
ulcerative lesions and are usually streaked on a selective medium such 
as Sabouraud agar containing antibacterial antibiotics. Initial incuba- 
tion is usually 25°C-30°C (77°F-86°F), followed by growth at 35°C 
(95°F) and confirmation by conversion to the yeast form. Staining pro- 
cedures are usually nonspecific unless fluorescent antibody. 



CLINICAL CASES 311 



MICROBIOLOGY PEARLS 

*♦* Sporothrix schenckii is a dimorphic fungus found in the soil of many 
areas of the world and associated with skin lesions following 
traumatic implantation most commonly from rose thorns. 

*♦* Cutaneous sporotrichosis is commonly treated with oral potassium 
iodide. 

*♦* Extracutaneous sporotrichosis, although rare, occurs in severely 
immunocompromised patients such as those with HIV. 



REFERENCES 

Fitzpatrick FB, Johnson RA, Polano MK, et al. Color Atlas and Synopsis of Clinical 

Dermatology, 2nd ed. New York: McGraw-Hill, 1992. 
Gorbach SL, Bartlett JG, Blacklow NR. Infectious Diseases, 2nd ed. Philadelphia,PA: 

W.B. Saunders, 1998. 
Harmon EM, Szwed F. Aspergillosis. eMedicine, 2002. http://www.emedicine.com/ 

med/topic 1 74.html 
Mandell GL, Bennett JE, Dolin R, eds. Principles and Practice of Infectious 

Diseases, 5th ed. Philadelphia, PA: Churchill Livingstone, 2000. 
Murray PR, Rosenthal KS, Pfaller MA. Medical Microbiology, 5th ed. St. Louis, 

MO: Mosby, 2005:738,799. 
Rex JH, Okhuysen PC. Sporothrix schenckii. In: Mandell GL, Bennett JE, Dolin R, 

eds. Principles and Practice of Infectious Diseases, 6th ed. Philadelphia, PA: 

Churchill Livingstone, 2005:2695-702. 
Shafazand S, Doyle R, Ruoss S, et al. Inhalational anthrax: epidemiology, diagno- 
sis and management. Chest 1999; 116(5): 1369-76. 
Fodar K. University of Wisconsin-Madison Dept. of Bacteriology, 2002; http:www. 

bact.wisc.edu/bact330/lymelecture 



This page intentionally left blank 



i* CASE 45 



♦> 



An 8-year-old boy, a child of immigrants from El Salvador who moved to the 
southwestern United States 3 months ago, presents to the emergency room 
with abdominal pain and vomiting. He was ill for a day, but his symptoms have 
worsened in the past few hours, and his parents panicked when they saw a 
worm in his vomitus. He has no significant medical history and has taken no 
medications. On examination, he appears very ill and in obvious pain. His 
temperature is 37.7°C (99.9°F), his pulse is 110 beats per minute, and his 
blood pressure is normal. His mucous membranes are dry, but his head and 
neck exam is otherwise normal. He is tachycardic, and his lungs are clear. His 
abdomen has high-pitched, tinkling bowel sounds on auscultation and is dif- 
fusely tender to palpation. There is, however, no rebound tenderness. An 
abdominal x-ray shows air-fluid levels consistent with a small bowel obstruc- 
tion. His parents saved and brought in the worm that he vomited. It is 5 inches 
long and reddish-yellow in color. 

^ What organism is most likely responsible for the patient's illness? 

^ How did the patient become infected? 



314 CASE FILES: MICROBIOLOGY 

ANSWERS TO CASE 45: ASCARIASIS 

Summary: An 8-year-old boy with a small bowel obstruction and who vomited 
a worm. 

^ Most likely organism responsible for the patient's illness: The 

nematode, Ascaris lumbricoides. 

^ How did the patient become infected: By ingesting eggs of the parasite 
that have matured in an external environment to contain a larval form. 

CLINICAL CORRELATION 

Infections with Ascaris are frequently asymptomatic or may be accompanied 
by numerous symptoms. The pathology that underlies symptoms is condi- 
tioned by the number of worms harbored and the sequential location of larvae 
in the lungs and adult worms in the intestine. 

Larvae released when ingested eggs hatch make a so-called "heart-lung cir- 
cuit" in which they are carried via the blood to various tissues (Figure 45-1). 
Migrating larvae elicit eosinophilia and granulomatous lesions. During the larval 
migration stage, symptoms are associated mainly with larvae present in the liver 
and lungs. Larvae that reach the lungs may cause pneumonitis. Patients may 
cough up exudates that are blood-tinged and contain polymorphonuclear leuko- 
cytes, mainly eosinophils. Fever may accompany pneumonitis, which usually dis- 
appears when the larvae move to the small intestine, as part of their life cycle. 
Ascaris is highly allergenic, stimulating inordinately high IgE levels. Therefore, 
pneumonitis may be especially severe in individuals who are sensitized by a pri- 
mary infection and encounter a challenge or secondary infection. Thus, increased 
severity of pneumonitis is caused by immune-mediated inflammation. 

Gastrointestinal (GI) symptoms depend on the number and location of adult 
worms in the GI tract. Light infection may cause GI upset, colic, and loss of 
appetite and impair digestion or absorption of nutrients. In heavy infections, 
the adult worms may ball up in the small intestine, resulting in physical 
obstruction, a rare but serious occurrence. From their usual location in the 
small intestine, adult worms infrequently migrate down and out of the anus or 
up and through the mouth or nose. Adult worms may cause symptoms by 
migrating to obstruct the bile ducts or penetrate into the gallbladder or liver. 
Adult worms may be seen in the stool or vomitus. 



CLINICAL CASES 



375 



ADULT: 
Small and 

large 
intestines 




Strongyloides 

Figure 45-1. Life cycle of parasitic nematodes, including Ascaris lumbri- 
coides. (With permission from Barron S, ed. Medical Microbiology, 4th ed. 
Galveston, TX: University of Texas Medical Branch, 1996.) 



316 CASE FILES: MICROBIOLOGY 

APPROACH TO SUSPECTED Ascaris INFECTION 
Objectives 

1 . Learn the life cycle of A. lumbricoides and the epidemiology and clin- 
ical course of infection. 

2. Be able to describe three basic aspects of infection: transmission, diag- 
nosis, and treatment/prevention. 

Definitions 

Nematode: A helminth characterized by a cylindrical body and separate 
sexes, in contrast to cestodes and trematodes (tapeworms and flukes) 
that are flatworms and generally hennaphroditic. 

"Giant intestinal roundworm": Common name for Ascaris lumbricoides. 

Egg (ova), larva, and adult: Sequential developmental stages in life cycle 
of helminths. 

Larva: Preadult or juvenile stage of helminths; nematodes have four larval 
stages in their life cycle prior to reaching the adult stage. Each stage is 
preceded by a molt or shedding of the cuticle or "skin." 

DISCUSSION 
Characteristics of Ascaris 

Ascaris is one of several nematodes that infect the GI tract and, like all of 
these, develops through egg, larva, and adult stages. Ascaris lumbricoides is 
very similar in size and life cycle to Ascaris suum, which is the species found 
in pigs. Parasitologists debate the point that there is only one species of Ascaris 
that infects both pigs and humans. If a human ingests eggs of A. suum, the lar- 
vae will migrate to the lungs and die, but in the process can cause a serious form 
of "ascaris pneumonia." Adult worms of A. suum do not develop in the human 
intestine. Other ascarids that infect humans are Toxocara canis and Toxocara 
cati, which are parasites of dogs and cats, respectively. Although humans can 
become infected by ingesting eggs of these species, neither worm develops to 
maturity. After hatching in the intestine, Toxocara larvae migrate chronically in 
visceral tissue, giving rise to the condition termed "visceral larvae migrans." 

Infection Transmission 

Typical of nematodes, A. lumbricoides has separate sexes and a life cycle that 
involves egg, larva, and adult stages. Growth involves four larval stages. 
Transition from one stage to another and to the adult stage is preceded by a 
molt. A person becomes infected by ingesting eggs that are usually acquired 
through hand-to-mouth transmission from the soil or via contaminated food or 



CLINICAL CASES 317 

water. Eggs hatch in the duodenum, releasing larvae that penetrate the small 
intestinal wall to enter the bloodstream. Larvae are carried hematogenously 

to various organs. On reaching the lungs, they penetrate into the alveoli. 

Larvae reside in the lung for approximately 3 weeks, growing and advancing 
to a subsequent larval stage. They then migrate up the bronchi and trachea, 
where they are then swallowed and returned to the small intestine. In the intes- 
tine, larvae develop to the adult stage and reach reproductive maturity in 
approximately 2 months. Adults can survive for up to 2 years and grow to 15-35 
cm in length. A single female worm lays approximately 200,000 eggs daily that 
are passed in the feces. Eggs can survive for years in the soil, tolerating a wide 
range of temperatures and other environmental variables. Under optimal con- 
ditions, eggs reach an infective, larvated stage in 2-3 weeks. Development is 
arrested at this stage until the egg is ingested. 

Ascariasis is a chronic disease of the small intestine and can be trans- 
mitted as long as adult worms are in the intestine and feces containing Ascaris 
eggs are allowed to contaminate the environment. Ascariasis is more prevalent 
in tropical climates but is found in temperate regions of the world. It is most 
common where sanitation is poor and where human feces are used as fertilizer 
in agricultural practices. Infections occur in all ages but are more common in 
children. In endemic areas, most of the population has some worm burden. 
With the exception of pinworm infection, ascariasis is the most common 
helminthic infection of humans on a global scale. 

Diagnosis 

A presumptive diagnosis may be based on clinical symptoms. However, symp- 
toms are not pathognomonic. The pneumonitis phase of infection cannot be 
diagnosed as Ascara-induced because it is a generalized symptom that pre- 
cedes the intestinal phase of infection by several weeks. Intestinal symptoms 
are normally absent or mild and, in most cases, go undetected. 

A definitive diagnosis is based on identifying egg or adult stages. As in 
this case, the first clue of infection may be adult worms that are vomited or 
passed in a stool. Intestinal obstruction, especially in children, often prompts 
medical attention. Because of its uniquely large size, an adult A. lumbricoides 
is unmistakable. The adult females are reddish-yellow in color and can meas- 
ure up to 7-13 inches long (males are generally shorter) and one-fourth of an 
inch in diameter. Although a definitive diagnosis can be made by identifying 
adult worms, ascariasis is more commonly diagnosed by identifying eggs in 
the stool. Both fertile and infertile eggs are passed in the stool. Because female 
worms pass thousands of eggs daily, diagnosis can be made by direct exami- 
nation of a stool sample without the need for specialized laboratory techniques 
to concentrate eggs. The eggs are ovoid, measure 45 by 75 |im and have a thick 
transparent inner shell covered by an albuminous coat that is wrinkled and 
usually stained light brown by bile pigments. Unfertilized eggs are commonly 
seen in a stool. These are more elongated, measuring 40 by 90 |im. The inside 



318 



CASE FILES: MICROBIOLOGY 



of the unfertilized egg is amorphous instead of containing a well defined sin- 
gle cell of the fertilized egg. 

Treatment and Prevention 

Adult worms are the target of several drugs available to treat ascariasis. The 
drug of choice is mebendazole, a benzimidazole derivative that has a high 
therapeutic index. The effectiveness of treatment can be assessed by the dis- 
appearance of eggs in stool samples and alleviation of symptoms. In cases of 
intestinal obstruction, the first action should be directed at ridding the patient 
of worms through the use of chemotherapeutic agents, such as mebendazole. 
If the worms can be dislodged the patient may void them in the stool. If the 
bolus of worms cannot be dislodged, surgery may be an option. If surgery is 
chosen, it is imperative that the worms be killed or paralyzed with a drug such 
as piperazine prior to physically removing them from the intestine. The reason 
for this procedure is that a live, active worm releases an aerosol of eggs 
through its uterine pore. Eggs are invisible to the naked eye. However, if eggs 
fall on exposed viscera they elicit granulomatous lesions and adhesions that 
can lead to severe complications. Preventive measures relate to instructing the 
patient on how the infection was acquired and on the proper disposal of feces 
to avoid soil contamination and reinfection. 

Synopsis 

The synopsis of ascariasis is presented in Table 45-1. 



Table 45-1 



Synopsis of Ascariasis 



Infective stage Location of 

of etiologic agent Pathogenic stage 



Laboratory diagnosis based 
on identification of: 



Drugs used 
in treatment 



Egg of: 
Ascaris 
lumbricoides 



Lungs 

(larvae, transient) 



Small intestine 
(adult, chronic) 




Mebendazole 



exam) 



Adult 
(vomitus; 
passed in 
stool) 



CLINICAL CASES 319 

COMPREHENSION QUESTIONS 

[45. 1] A definitive diagnosis of ascariasis can be made by observing which of 
the following? 

A. An eosinophilia in a differential white blood cell count 

B. Motile larvae in a stool sample 

C. Larvae in x-ray of lungs 

D. An adult worm passed during a bowel movement 

[45.2] Ascariasis is most effectively treated with which of the following drugs? 

A. Mebendazole 

B. Metronidazole 

C. Niclosamide 

D. Praziquantel 

[45.3] A person presents to his physician complaining of chronic GI symp- 
toms. A diagnosis of A. lumbricoides is made. Human nematodes 
infect individuals via different routes. This patient is most likely 
infected by which of the following? 

A. Larvae penetrating unprotected skin 

B. Ingesting larvated eggs 

C. Eating uncooked pork 

D. Internal autoinfection 

Answers 

[45. 1] D. Identification of adult worms passed by an individual can ensure a 
definitive diagnosis. Although an eosinophilia accompanies ascaria- 
sis, this condition can be caused by other infections and by various 
allergic conditions. Motile larvae in a stool are indicative of infection 
with Strongyloides stercoralis. Nematode larvae cannot be seen in an 
x-ray. 

[45.2] A. Mebendazole is a broad spectrum antihelminthic that is the drug 
of choice for treating Ascaris and several other intestinal nematodes. 
Metronidazole is used primarily to treat infections with protozoa, 
such as Giardia lamblia and Trichomonas vaginalis. Niclosamide is 
the drug of choice in treating most adult cestode or tapeworm infec- 
tions. Praziquantel is used to treat infections with blood flukes or 
schistosomes and can also be used to treat adult tapeworms. 

[45.3] B. Ascaris is acquired by ingesting infective eggs, as are the whip- 
worm, Trichuris trichiura, and the pinworm, Enterobius vermicularis. 
Filariform larvae penetrating unprotected skin is the route by which a 
person becomes infected by other intestinal nematodes, hookworms 
and Strongyloides; eating uncooked pork could lead to trichinosis; and 
internal autoinfection is caused only by Strongyloides. 



320 CASE FILES: MICROBIOLOGY 



MICROBIOLOGY PEARLS 

*♦* The fertilized eggs are unsegmented when laid by the female worm. 
*♦* Mebendazole, one of several available benzimidazole compounds 

used to treat nematodes, is the drug of choice in treating Ascaris 

infection. 



REFERENCES 

Centers for Disease Control. DPDx. Laboratory identification of parasites of public 
concern. Ascariasis. 2007. http://www.dpd.cdc.gov/dpdx/HTML/Ascariasis.htm. 

Centers for Disease Control. DPDx. Laboratory identification of parasites of public 
concern. Image Library. Ascariasis. 2007. http://www.dpd.cdc.gov/dpdx/ 
HTML/ImageLibrary/Ascariasis_il.htm. 

Medical Letter on Drugs and Therapeutics. Drugs for parasitic infections. New 
Rochelle, NY. 2004. http://www. medletter.com/freedocs/parasitic.pdf. This ref- 
erence has been updated and "superseded by the special report Drugs for 
Parasitic Infections, which can be purchased (on-line) for S25." This reference 
has been provided because it is a comprehensive and clinically useful reference 
that is regularly updated and could be of value to those who are involved in treat- 
ing parasitic infections. 



♦> CASE 46 



A 32-year-old man with known HIV is brought to the hospital with diarrhea. 
He has had between 15 and 25 watery stools a day for the past 2 weeks. He 
has had a low grade fever and felt very fatigued, but denies vomiting. He has 
not passed any blood in his stool. He says that he has lost 8 lb in this time 
frame. He is on a "triple therapy cocktail" of AZT, 3TC, and a protease 
inhibitor for his HIV. His last CD4 cell count was 150 cells/mm 3 . On exami- 
nation, his temperature is 37.2°C (98.9°F), pulse is 110 beats per minute, 
blood pressure is 95/75 mm Hg, and respiratory rate is 24 breaths per minute. 
In general, he appears cachectic. His eyes are dry and sunken. His mucous 
membranes are moist. His cardiovascular exam is notable for tachycardia, and 
he has orthoscopic changes on sitting up from lying down. His abdomen has 
hyperactive bowel sounds, but is soft and only mildly tender. His stool is heme 
negative. A modified acid-fast stained stool sample reveals multiple red and 
pink, round oocysts. 

^ What is the most likely cause of diarrhea? 

^ How is this infection most commonly acquired? 



322 CASE FILES: MICROBIOLOGY 

ANSWERS TO CASE 46: CRYPTOSPORIDIOSIS 

Summary: A 32-year-old man with HIV and diarrhea. A modified acid-fast 
stained stool sample reveals multiple red and pink, round oocysts. 

^ Most likely etiologic agent: Cryptosporidium parvum. 

^ The most common mode of acquiring this infection: Ingestion of 
oocysts in contaminated water or food or fecal-oral transmission from 
infected animals or person to person. 

CLINICAL CORRELATION 

Cryptosporidium parvum belongs to a group of protozoans known as coccidi- 
ans. The infective oocyst is approximately 3-8 |im in diameter. The parasite, 
on emerging from the oocyst, attaches to the surface of intestinal epithelium 
where it multiplies both asexually and sexually. The parasite causes changes 
in the mucosa that include crypt hyperplasia and villous atrophy. Associated 
with its presence in the intestine, and after an incubation period of approxi- 
mately 2-12 days, is an acute illness characterized by nausea, abdominal 
cramps, weight loss, anorexia, malaise, low-grade fever, and diarrhea. The fre- 
quency of diarrheal episodes and voluminous fluid loss is often debilitating. 
During infection there may be periods in which symptoms are absent. Although 
any individual can acquire the infection, children in day care centers and indi- 
viduals with AIDS or HIV infection represent populations that are especially 
vulnerable to cryptosporidiosis. In immunocompetent individuals, the disease 
will resolve on its own, usually within 7-14 days. In immunocompromised hosts, 
the disease is generally more severe and chronic, sometimes lasting for life. Five 
to ten percent of patients with AIDS acquire infection with Cryptosporidium. 
Persistence of infection in HIV-infected individuals is closely associated 
with CD4 lymphocyte counts of less than 180 cells/mm 3 . Protracted diar- 
rhea may lead to dehydration, wasting, and death. Severe intestinal distress, 
usually in immunodeficient individuals, is sometimes associated with pul- 
monary and tracheal cryptosporidiosis that is associated with coughing and 
low-grade fever. The strains that infect the intestine and lungs are, to date, 
indistinguishable. 



CLINICAL CASES 323 

APPROACH TO SUSPECTED Cryptosporidium INFECTION 
Objectives 

1. Learn the life cycle of C. parvum and the epidemiology and clinical 
course of cryptosporidiosis, and compare this disease with those 
caused by related organisms, such as Cyclospora and Isospora. 

2. Be able to describe the three basic aspect of infection: transmission, 
diagnosis, and treatment/prevention. 

Definitions 

Coccidia: The specific taxonomic group of protozoans to which C. parvum 

belongs. 
Oocyst: The stage in the life cycle of C. parvum that transmits the disease 

and is also sought in making a definitive diagnosis. Each oocyst contains 

four sporozoites. 
Sporozoite: The stage released from the oocyst following ingestion and 

which initiates infection. 
Zoonosis: A disease that is transmitted from lower vertebrate hosts to humans. 
Acid-fast stain: A type of stain that renders oocysts highly visible in a fecal 

sample. It is used to support the microscopic diagnosis of the parasite. 

DISCUSSION 

Characteristics of Cryptosporidium That Impact 
Transmission 

Cryptosporidium species are ubiquitous, worldwide enteric pathogens of 
humans and multiple other animal species. Of the many species of the genus, 
C. parvum is responsible for most clinical disease in humans and other mam- 
mals. The life cycle of C. parvum occurs within a single host and, like other 
coccidia, involves sexual and asexual reproduction. Misdiagnosis with 
cyclosporiasis may be made, in part because clinical symptoms are similar. 
Protracted, watery diarrhea is the hallmark of infection. Cyclospora 
cayetanensis, which infects humans, has a worldwide distribution. Another 
related organism that causes diarrhea in humans is Isospora belli. 

The small intestine is the usual host habitat for Cryptosporidium, where 
it lives in a unique intraepithelial niche. The life cycle is initiated with the 
ingestion of oocysts that contain four sporozoites. When oocysts are ingested, 
they undergo excystation as the outer wall is removed by digestive processes. 
Sporozoites that are released attach to the host's intestinal epithelial cells and 
become surrounded by a host-derived membrane, making them intracellular but 



324 CASE FILES: MICROBIOLOGY 

extracytoplasmic. Sporozoites undergo multiple fission to form meronts that 
contain multiple merozoites. The merozoites are released to infect other cells. 
Following another round of asexual division and on release of the second and 
subsequent generations of merozoites, they penetrate new cells to form 
gametes. Most gametes undergo enlargement into macrogametes (female). 
Some become microgametocytes that undergo fission multiple times to form 
sperm-like microgametes (male). Microgametes leave the microgametocyte 
and fertilize a macrogamete to form a zygote. The zygote then becomes cov- 
ered by a wall, forming an oocyst that is highly resistant to chemical and phys- 
ical changes in the internal and external environment. Sporozoites develop 
within the oocyst that are sloughed, along with intestinal epithelial cells, and 
voided in the feces. Because oocysts are passed in the feces in a sporulated 
stage (i.e., contain sporozoites), they are immediately infective and can retain 
infectivity for long periods because of their protective wall. In having oocysts 
that are immediately infective, Cryptosporidium is different from Cyclospora, 
which has oocysts that require 1-2 weeks to develop to an infective stage. 

Presumably ingestion of one oocyst can initiate an infection that can be 
contracted by eating contaminated food or drinking contaminated water. 
Touching the stool of infected individuals or animals or anything contaminated 
with feces and then touching your mouth can also initiate infection. 

Infection can be transmitted by ingestion of oocysts passed in feces of 
infected humans or animals. Thus, infection can be transmitted from one per- 
son to another or from animals to humans, from eating and drinking food or 
water contaminated with fecal material or from transfer of oocysts from con- 
taminated material to the mouth or from person to person. 



Diagnosis 

A definitive diagnosis is based on identifying oocysts in a fecal sample. A 

technique, such as sugar flotation, is used to concentrate the oocysts and acid- 
fast staining is used to identify them. A fluorescent antibody technique is 
also available to stain the isolated oocysts, augmenting visualization. Oocysts 
contain four sporozoites. It is important to make a differential diagnosis with 
Cyclospora oocysts, which are similar in size but are not sporulated when 
passed. Cryptosporidium oocysts contain four sporozoites and are approxi- 
mately 8 \im in diameter. When sporulated, the oocysts of Cyclospora are the 
same size, but contain two sporocysts, each with two sporozoites. Isospora 
oocysts can also be found in the stool; however, they can be differentiated by 
their larger size, 15 by 30 |im ovoid. Like Cyclospora, oocysts of Isospora are 
excreted in an unsporulated stage, and, after becoming sporulated, contain two 
sporocysts, each with two sporozoites. Pulmonary infections of 
Cryptosporidium are diagnosed by biopsy and staining (Table 46-1). 



CLINICAL CASES 325 

Table 46-1 

DIFFERENTIATION OF HUMAN COCCIDIAN OOCYSTS 



SPECIES 


NUMBER 

OF 

SPOROCYSTS 


NUMBER 

OF 

SPROROZOITES 


OOCYSTS 
SPORULATED 
IN STOOL 


SIZE OF 

OOCYST 

|iM 


Cryptosporidium 
parvum 





4 


Yes 


4-8 


Cyclospora 
cayetanensis 


2 


4 


No 


4-8 


Isospora belli 


2 


4 


No 


12x30 



Treatment and Prevention 

As noted, infection is self-limiting in immunocompetent hosts and chronic in 
immunosuppressed individuals. While nitazoxanide is effective in the treat- 
ment of immunocompetent hosts, it has not proven so for immunosu- 
pressed patients. Because of massive fluid loss, due to diarrhea, infected 
individuals may require rehydration therapy. 

Because there is no effective agent to treat infection specifically in 
immunosuppressed patients, the best measure to control infection is avoid- 
ance of situations that are conducive to transmission. Thus, knowledge about 
sources of infection and how infection is transmitted is the key to prevention. 
Most surface water, such as streams, lakes, and rivers contain some 
Cryptosporidium oocysts. Many public supplies of treated and filtered water 
derived from these sources are contaminated with low levels of oocysts. 
Cryptosporidiosis can be prevented by thoroughly washing hands before 
eating and after any contact with animals or soil or after changing diapers. In 
people with weakened immune systems, cryptosporidiosis can be life- 
threatening. These individuals must take extra precautions to drink only water 
that has been purified; wash with purified water; cook all food; do not swim in 
lakes, rivers, streams, or public pools; avoid sexual practices that might 
involve contact with stool; and avoid touching farm animals. 



Synopsis 

The synopsis of cryptosporidiosis is presented in Table 46-2. 



326 



CASE FILES: MICROBIOLOGY 



Table 46-2 





Synopsis of 


Cryptosporidiosis 




Infective stage 


Location of 


Laboratory diagnosis based 


Drugs used 


of etiologic agent 


Pathogenic stage 


on identification of: 


in treatment 


Oocyst of: 


Small and large 


/""vfafc 


Nitazoxanide 


Cryptosporidium 
parvum 


intestine 


Oocyst (stool exam) 





COMPREHENSION QUESTIONS 

[46.1] A 33-year-old woman has chronic diarrhea. A fecal sample is obtained. 
Microscopic identification of which of the following stages of the 
organism would provide the strongest evidence for cryptosporidiosis? 

A. Cyst 

B. Oocyst 
C Egg 

D. Sporocysts 

E. Merozoites 

[46.2] A 24-year-old male scientist is diagnosed with chronic cryptosporidio- 
sis. He asks about the epidemiology of this disorder. Which of the fol- 
lowing accurately describes the disease or the etiologic agent? 

A. Is self-limiting in immunocompromised patients 

B. Reproduces sexually and asexually in different hosts 

C. Can be acquired through sporozoites transmitted by an insect vector 

D. Is transmitted through drinking water contaminated with animal 
feces 

E. Is the only human parasite that produces oocysts 



CLINICAL CASES 327 

[46.3] Chronic, debilitating cryptosporidiosis is most likely to affect which of 
the following individuals? 

A. Dairy farmers 

B. Individuals with AIDS 

C. Infants placed in day care centers 

D. Zoo animal handlers 

E. Hikers who drink from streams and lakes 

Answers 

[46.1] B. Oocyst is the correct answer. Egg and cyst stages are not part of 
the life cycle of Cryptosporidium. Eggs are produced by helminths, 
and cyst stages are produced by other intestinal protozoans, such as 
Entamoeba and Giardia. Merozoites occur within infected epithelial 
cells but are not the target of diagnostic tests or procedures. Sporocysts 
are not found in the life cycle of Cryptosporidium as a feature of the 
oocyst. Sporocysts are a feature of the genus Cyclospora and impor- 
tant in differential diagnosis. 

[46.2] D. Infection can be acquired from oocysts transferred from farm ani- 
mals. In immunocompromised individuals, such as those with AIDS 
or cancer patients being treated with immunosuppressive agents, 
infection is not self-limiting but rather chronic and sometimes life- 
threatening. The life cycle of Cryptosporidium involves both asexual 
and sexual reproduction, but both forms occur in a single host. 
Sporozoites are stages in the life cycle that are released from ingested 
oocysts. There is no insect that serves as a biological or mechanical 
vector in the life cycle. Oocysts are also produced by other species of 
coccidians that infect humans, such as Isospora, Cyclospora, 
Toxoplasma, and Plasmodium. Toxoplasma oocysts only occur in 
feline hosts; Plasmodium species cause malaria in which only the 
mosquito definitive host harbors oocysts; Isospora and Cyclospora are 
intestinal coccidian parasites that produce oocysts that are passed in 
the feces and must be considered in making a differential diagnosis. 

[46.3] B. All individuals (A-E) are susceptible to infection. However, per- 
sons that are at high risk of severe, protracted infection are those with 
AIDS, or those who have cancer or organ transplants who are treated 
with drugs that weaken the immune system, or individuals who are 
genetically immunodeficient. 



328 CASE FILES: MICROBIOLOGY 



MICROBIOLOGY PEARLS 

*♦* Often there is misdiagnosis between cryptosporidiosis and cyclospo- 

riasis. 
*♦* Immunologically compromised patients with cryptosporidiosis do 

not respond to specific therapy with nitazoxanide. 



REFERENCES 

Centers for Disease Control. DPDx. Laboratory identification of parasites of 
public concern. Cryptosporidiosis. 2007. http://www.dpd.cdc.gov/dpdx/HTML/ 
Cryptosporidiosis.htm. 

Centers for Disease Control. DPDx. Laboratory identification of parasites of public 
concern. Image Library. Cryptosoridiosis. 2007. http://www.dpd.cdc.gov/ 
dpdx/HTML/ImageLibrary/Cryp tosporidiosis_il.htm. 

Medical Letter on Drugs and Therapeutics. Drugs for parasitic infections. New 
Rochelle, NY. 2004. http://www.medletter.com/freedocs/parasitic.pdf. This ref- 
erence has been updated and "superseded by the special report Drugs for 
Parasitic Infections, which can be purchased (on-line) for S25." This reference 
has been provided because it is a comprehensive and clinically useful reference 
that is regularly updated and could be of value to those who are involved in treat- 
ing parasitic infections. 



♦> CASE 47 



A 4-year-old girl is brought to the physician's office by her mother because of 
anal itching. The mother has noticed her daughter scratching and rubbing her 
anal area frequently for the past few days. Her anal area has been getting red 
and raw from all the scratching. Mother has used some petrolatum and hydro- 
cortisone cream, but it hasn't helped much. The child has not had any obvious 
skin rashes and is not scratching any other part of her body. She has not had 
diarrhea. She takes no medications and has no significant medical history. She 
attends day care 4 days a week. On examination, she is a well-appearing child. 
Her vital signs and general examination are normal. Examination of her peri- 
anal area reveals some erythema and excoriation from scratching. You perform 
a microscopic examination of a sample collected by touching the perianal 
region with a piece of clear cellophane tape. 

^ What diagnostic finding are you likely to see on this microscopic 
examination? 

^ What is the organism responsible for this infection? 



330 CASE FILES: MICROBIOLOGY 

ANSWERS TO CASE 47: ENTEROBIASIS OR 
PINWORM INFECTION 

Summary: A 4-year-old girl has perianal pruritus. The diagnosis is made by 
microscopic examination of a sample collected by touching the perianal region 
with a piece of clear cellophane tape. 

^ Diagnostic finding likely to see on this microscopic examination: 

Thin-walled, ovoid eggs that are flattened on one side and contain a 
nematode larva. 

^ Organism responsible for this infection: Enterobius vermicularis. 

CLINICAL CORRELATION 

Enterobius vermicularis, commonly called the pinworm, is the most common 
cause of helminthic infections in the United States and is endemic around the 
world. Humans are the only known host for E. vermicularis, but other verte- 
brates can be infected with different species of this nematode. Adult worms, 
approximately 1 cm in length, white and thread-like in appearance, inhabit the 
large intestine. Gravid females migrate to the perianal and perineal regions at 
night to lay eggs that are immediately infective. Infection is more common in 
children than adults and is often asymptomatic. However, a variety of symptoms 
are ascribed to pinworms. Atypically, worms are sometimes found in an 
inflamed appendix and there are rare reports of worms reaching the genital tract 
and producing vaginitis. By far the most common signs of infection occur in 
children and include restless sleep and tiredness during the day. However, more 
common symptoms consist of anal or perianal itching because of the adult 
worms crawling on the skin. The eggs can also cause local itching, which may 
be more intense in secondary infections as a result of allergic reactions to their 
antigenic coating. Frequent scratching results in transfer to the hands and areas 
under the fingernails. Eggs are frequently transferred to clothing, bedding, toys, 
and dust, where they can survive for several weeks. Through hand-to-mouth 
transmission, the eggs are ingested and hatch in the duodenum. Larvae released 
from eggs reside in the cecum and reach adult stage in about a month. Infections 
are acute, generally lasting 4-8 weeks. Considering the relatively short duration 
of a single infection, chronic enterobiasis is caused by reinfection. 



CLINICAL CASES 331 

APPROACH TO THE SUSPECTED Enterobius INFECTION 
Objectives 

1 . Learn the life cycle of E. vermicularis and the epidemiology and clin- 
ical course of infection. 

2. Be able to describe three basic aspects of infection: transmission, diag- 
nosis, and treatment/prevention. 



Definitions 

Pinworm: Common name for Enterobius vermicularis. 

Cervical alae: An extension of a lateral cuticular protuberance or lateral 
line on the body surface of the pinworm that extends to the head region 
and appears microscopically as a "flared" region or collar. Adult worms 
are identified, in part, by the presence of cervical alae which are promi- 
nent structures when examined microscopically. 

Nocturnal migration: Refers to the tendency of pinworms to migrate at 
night from the colon, out the anus, to the perianal and perineal regions 
to deposit eggs. 

Larvated egg: Refers to eggs that contain a larval stage and are deposited 
by pinworms on the skin. 



DISCUSSION 

Characteristics of Enterobiasis That Impact Transmission 

A patient acquires infection by ingesting the pinworm eggs containing 

infective larvae. Ingested eggs hatch in the small intestine releasing larvae 
that migrate to the cecal area and mature into adult male and female worms 
that are free or insecurely attached to the mucosa. The period between inges- 
tion of eggs to maturation takes approximately 3-4 weeks. Following copula- 
tion, the female pinworms produce eggs. Rather than release eggs in the 
bowel, the female worms migrate out the anus onto the surrounding skin 
and release eggs. Worm migration usually occurs at night. Each female 
will lay thousands of microscopic, larvated eggs. Pinworm eggs are infective 
within a few hours after being deposited on the skin. They can survive up to 2 
weeks on clothing, bedding, or other objects. Individuals can become infected 
after accidentally swallowing infective pinworm eggs from contaminated sur- 
faces or fingers. The duration of a single infection is 4-8 weeks. 



332 CASE FILES: MICROBIOLOGY 

Diagnosis 

Although Enterobius is an intestinal parasite, eggs are rarely found during lab- 
oratory examinations of stools. If a person is suspected of having pinworms, 
the so-called "scotch tape test" should be used to identify the parasite. 
Transparent adhesive tape, sometime attached to the end of tongue depressor 
or "pinworm paddle," is pressed in the anal region. This procedure involves the 
help of a patient or parents of suspected children. The tape is then transferred 
to a glass slide, sticky side down. The slide should then be examined micro- 
scopically for eggs. Pinworm eggs are approximately 20 x 50 \im characteris- 
tically flattened on one side and usually contain an active larva. Because 
bathing or having a bowel movement may remove eggs, the scotch tape 
impression should be made on awakening in the morning. In children, samples 
taken from under the fingernails may also contain eggs because scratching of 
the anal area is common. 

A definitive diagnosis may also be made on recovery and identification of 
adult worms seen directly in bedclothes or around the anal area. The female 
pinworm has a sharply pointed tail and anterior alae that form a collar-like 
structure around the mouth. The female worm is about 1 cm long with a 
diameter approximately 0.5 mm. In female worms that are gravid, the uterus 
filled with easily identifiable eggs is a common feature. 

Treatment and Prevention 

Highly effective drugs in the treatment of enterobiasis are pyrantel pamoate 
and mebendazole, given as a single dose, with a repeat dose administered 
2 weeks later. Mebendazole is a broad spectrum antinematode agent that has 
a high therapeutic index. Close family contacts of infected individuals should 
be treated as well. If reinfection occurs, the source of the infection should be 
identified. Therefore, playmates, schoolmates, close contacts outside the 
house, and household members should be considered. Each infected person 
should receive the two-dose treatment and, if necessary, more than two doses. 
In short, the importance of determining infection in the entire family or con- 
tacts should be explained in terms of the life cycle of the worm and personal, 
and group hygiene should be stressed. 

Synopsis 

The synopsis of enterobiasis is presented in Table 47-1. 



CLINICAL CASES 



333 



Table 47-1 



Synopsis of Enterobiasis 


Infective stage 


Location of 


Laboratory diagnosis based 


Drugs used 


of etiologic agent 


pathogenic stage 


on identification of: 


in treatment 


Egg of : 


Large intestine 


/^ jf 


Mebendazole 


Enterobius 




ffMjfoNpi /fy 




vermicularis 




Egg or Adult 
(anal swab) 





COMPREHENSION QUESTIONS 

[47.1] In which of the following life cycle stages is enterobiasis transmitted? 

A. Larva 

B. Egg 

C. Adult 

D. Cyst 

E. Oocyst 

[47.2] Which of the following is the drug of choice in treating enterobiasis? 

A. Mebendazole 

B. Metronidazole 

C. Piperazine 

D. Praziquantel 

E. Chloroquine 



334 CASE FILES: MICROBIOLOGY 

[47.3] A parent of a child suffering from disturbed sleep and restlessness calls 
the family physician and states that her child is once again infected 
with pinworms and asks if she can administer the same medicine that 
was used to cure an earlier infection. After the physician is convinced 
that the pinworm infection is the problem, she advises on giving the 
same treatment and provides direction on how to clean up the environ- 
ment to prevent further reinfection. The physician should have been 
convinced by which of the following facts? 

A. The parent knew that reinfection was a possibility. 

B. The parent had collected worms from bed linen and accurately 
described them. 

C. The parent described symptoms of enterobiasis. 

D. The parent had the child's stool examined by her veterinarian who 
identified telltale eggs. 

E. The parent noted that the family's pet cat continued to sleep on the 
child's bed. 

Answers 

[47.1] B. The egg stage is the stage transmitted from person to person. A lar- 
val form (A) is found inside the egg but does not escape to initiate 
infection until the egg is ingested. Adult forms (C) live in the intes- 
tine but are not the stage directly responsible for transmission of the 
infection. Cyst (D) and oocyst (D) stages are not a part of the life 
cycle of E. vermicularis. 

[47.2] A. Mebendazole is the most appropriate of several available benzimi- 
dazole compounds to treat enterobiasis. This drug of choice is a 
highly effective, broad-spectrum antihelminthic. Metronidazole (B) 
is used to treat various protozoan infections, but is not efficacious in 
treating pinworms. Piperazine (C) is an anthelminthic that was used 
prior to the discovery of mebendazole to treat enterobiasis and is less 
effective, and its dose regimens are more complicated. Praziquantel 
(D) is effective in treating tapeworm and fluke (flatworms) infections 
but is not useful against pinworms or other nematodes. Chloroquine 
is a potent antimalarial drug but of no use against helminths. 

[47.3] B. Finding and identifying adult pinworms is one way to make a 
definitive diagnosis. Reinfection (A) is a definite possibility but not 
convincing evidence that the child is actually infected. Symptoms 
described (C) are associated with enterobiasis but are only presump- 
tive, not definitive, evidence of infection. A stool exam (D) is not an 
appropriate or effective method to diagnose enterobiasis. Cats (E) are 
in no way associated with transmission of infection. 



CLINICAL CASES 335 



MICROBIOLOGY PEARLS 

*♦* The egg (larvated) is the infective stage. 

*♦* The life cycle is direct, meaning that the adults develop from larvae 
without leaving the gastrointestinal tract. Adult worms are the 
primary cause of pathology. 

*♦* Pamental pamoate and mebendazole, a broad spectrum anti- 
helminthic, are drugs of choice. 



REFERENCES 

Centers for Disease Control. DPDx. Laboratory identification of parasites of 
public concern. Enterobiasis. 2007. http://www.dpd.cdc.gov/dpdx/HTML/ 
Enterobiasis.htm. 

Centers for Disease Control. DPDx. Laboratory identification of parasites of public 
concern. Image Library. Enterobiasis. 2007. http://www.dpd.cdc.gov/dpdx/ 
HTML/ImageLibrary/Enterobiasis_il.htm. 

Medical Letter on Drugs and Therapeutics. Drugs for parasitic infections. New 
Rochelle, NY. 2004. http://www.medletter.com/freedocs/parasitic.pdf. This ref- 
erence has been updated and "superseded by the special report Drugs for 
Parasitic Infections, which can be purchased (on-line) for S25." This reference 
has been provided because it is a comprehensive and clinically useful reference 
that is regularly updated and could be of value to those who are involved in treat- 
ing parasitic infections. 



This page intentionally left blank 



♦> CASE 48 



A 50-year-old woman presents to your office with the complaints of fever, 
chills, nausea, and vomiting for the past 5 days. She is especially concerned 
because she just returned from a 3-week long church mission trip to central 
Africa during which she did not take the recommended malaria prophylaxis. 
She was careful about using insect repellent and wearing long-sleeved cloth- 
ing, but she did not take the recommended weekly dose of mefloquine because 
it made her nauseous. Starting a few days after her return, she has had episodes 
of shaking chills followed by fever spikes as high as 39.7°C (103. 5°F) and 
then profuse sweating. After these episodes she would feel so exhausted that 
she would sleep for hours. These severe episodes have been occurring every 
other day. In between these episodes, she has had low-grade fever, myalgias, 
nausea, vomiting, and diarrhea. On examination, she appears very fatigued and 
pale. Her temperature is 37.7°C (99.9°F), pulse 100 beats per minute, blood 
pressure 1 10/80 mm Hg, and respiratory rate 18 breaths per minute. Other than 
signs of dehydration, her examination is unremarkable. A complete blood 
count shows her to be anemic. She has elevated blood urea nitrogen, creati- 
nine, and lactate dehydrogenase levels. A thin-blood smear is sent to the labo- 
ratory, which shows erythrocytes with ring forms at the periphery of the cell 
and multiple erythrocytes with three or four ring forms present. 

^ What is the most likely etiology of her infection? 

^ What findings on the thin blood smear are specific for this organism? 



338 CASE FILES: MICROBIOLOGY 

ANSWERS TO CASE 48: MALARIA 

Summary: A 50-year-old woman has fever and body aches. A thin-blood smear 
shows erythrocytes with ring forms at the periphery of the cell and multiple 
erythrocytes with three or four ring forms. 

^ Most likely etiology of her infection: Plasmodium falciparum. 

^ Findings on the thin blood smear are specific for this organism: 

Multiple ring forms in a single erythrocyte and ring forms located at the 
periphery of the erythrocytes. 

CLINICAL CORRELATION 

Malaria is caused by one of the four species of plasmodia involved in a human- 
mosquito-human life cycle. Plasmodia are coccidian parasites of erythrocytes. 
Their life cycle involves asexual reproduction in humans and sexual reproduc- 
tion in the mosquito. Human infection is initiated by the bite of an infected 
mosquito, which introduces sporozoites into the bloodstream. The sporozoites 
travel to the liver where they mature and reproduce asexually by schizogony. 
Plasmodium ovale and P. vivax may also establish a dormant — hypnozoite — 
stage in the liver; P. falciparum and P. malariae are incapable of this. On com- 
pletion of the hepatic growth and reproductive stage, merozoites are released 
from hepatocytes and infect erythrocytes, initiating the erythrocytic cycle. 
Asexual reproduction continues, resulting in rupture of erythrocytes and 
release of more infectious merozoites. The classic symptoms of malaria relate 
to the paroxysm of shaking chills, fever, and sweating and correspond with the 
cyclical lysis of erythrocytes and release of merozoites. Plasmodium vivax, 
P. ovale, and P. falciparum species of malaria tend to produce paroxysms 
in 48-hour cycles (tertian malaria), whereas P. malariae causes paroxysms in 
72-hour cycles (quartan malaria). 

A series of paroxysms of decreasing intensity constitutes a primary malar- 
ial attack. After the primary attack, parasites tend to disappear from the blood. 
In infections with P. falciparum or P. malariae this would constitute a cure. In 
P. vivax and P. ovale infections, relapses may occur as a result of hypnozoites 
persisting in the liver. 

Complicating pathologic changes such as anemia, hepatomegaly, and 
splenomegaly may occur. In the case of falciparum malaria, capillaries are 
blocked by infected erythrocytes that typically tend to become "sticky" and 
sequestered in capillary beds. Erythrocyte destruction leads to anemia. 
Capillary blockage leads to ischemia, anoxia, and subsequent organ damage. 
This is the basis for cerebral symptoms and kidney damage that leads to black 
water fever, a condition in which hemoglobin and erythrocytes appear in the 
urine; "blackwater fever" is associated with a poor prognosis. Falciparum 
malaria is the most virulent and lethal form of malaria, sometimes called 
malignant tertian malaria. 



CLINICAL CASES 339 

APPROACH TO THE SUSPECTED MALARIA 
INFECTION 

Objectives 

1 . Learn the life cycle of Plasmodium species and the epidemiology and 
clinical course of infection. 

2. Be able to describe the three basic aspects of infection: transmission, 
diagnosis, and treatment/prevention. 



Definitions 

Relapse malaria: Infection derived from hypnozoites (hypnos = sleeping; 
zoites = animals) or residual liver stages that persist after a primary 
infection with P. vivax and P. ovale. 

Applique form: Parasite on the periphery of erythrocytes, as in P. falci- 
parum infection. 

Ring stage: Stage in the life cycle of Plasmodium in an erythrocyte con- 
sisting of a thin ring of protoplasm with a nucleus at one side. 

Schizogony: Asexual division or "splitting" carried out by all Plasmodium 
species. 

Blackwater fever: A dangerous complication of malaria, especially falci- 
parum, characterized by passage of red to black urine and associated 
with high mortality. 



DISCUSSION 

Characteristics of Plasmodium That Impact Transmission 

Plasmodium is a genus in the phylum Apicomplexa, which contains other 
human parasites such as Toxoplasma, Cryptosporidium, Cyclospora, and 
Isospora. All of these organisms belong to a phylogenetic class in which all 
species are parasitic. 

Malaria infections are endemic in tropical developing countries. 
Although endogenous malaria has occurred in the United States, most cases 
are imported by travelers. There are numerous species of Plasmodium, but 
only four species cause human malaria — P. falciparum, P. vivax, P. malariae, 
and P. ovale. All species are transmitted by an infected anopheline mosquito. 
Plasmodium sporozoites are the infective forms injected into the bloodstream 
when the mosquito takes a blood meal. The sporozoites circulate in the blood- 
stream and then invade hepatocytes to initiate a preerythrocytic cycle. In the 
liver parenchymal cells the parasite multiplies asexually by a process called 



340 CASE FILES: MICROBIOLOGY 

schizogony or splitting. Asexual reproduction gives rise to multiple individual 
stages or merozoites. These merozoites become blood-borne and invade ery- 
throcytes to initiate the erythrocytic cycle. In the case of P. vivax and P. ovale, 
the liver phase can be sustained for years by sporozoite-derived dormant stages 
known as hypnozoites. It is the prevalence of the hypnozoites that leads to 
relapses of malarial symptoms, possibly occurring several years after the first 
acute disease has been cured. 

When merozoites parasitize erythrocytes, their development takes two routes. 
Some merozoites develop into micro (male) and macro (female) gametocytes. 
When a female anopheline mosquito bites an infected person and ingests the 
gametocytes, fertilization of the macrogametocyte by the microgametocyte 
takes place in the mosquito with the subsequent and sequential formation of 
diploid zygotes, oocysts, and, eventually, sporozoites. Sporozoites travel to 
enter the salivary glands of the mosquito where they are capable of initiating 
a new infection when the mosquito takes a blood meal. Through the second route 
in the erythrocytic cycle, the parasite develops successively through ring, tropho- 
zoite and schizont stages. As a result of schizogony, the erythrocyte breaks open 
and releases many new merozoites. These parasites then infect more erythro- 
cytes, repeat the development cycle, ultimately causing the destruction of mas- 
sive numbers of erythrocytes. The characteristic chill, fever, and sweating 
paroxysm occur when the parasites are released from the erythrocytes. 
Because the release of parasites becomes synchronized and periodic, the parox- 
ysms are also periodic, occurring at 48 or 72 hours depending on the species. 
The destruction of erythrocytes and release of cell and parasite debris elicit host 
responses that contribute to pathologic changes. 



Diagnosis 

Diagnosis is made by finding the characteristic organisms on thick and thin 

blood smears. Differential diagnosis rests on knowing the specific morpho- 
logic characteristics of each species, which are revealed in a thin blood smear 
(Table 48-1). Plasmodium vivax and P. ovale appear as ring shapes, and in 
other advanced stages of development in enlarged erythrocytes that contain 
numerous granules, known as Schiiffner dots. Plasmodium malariae has 
characteristic "band or bar" pattern and do not enlarge the host erythrocytes. 
Plasmodium falciparum can be identified by the presence of multiple ring 
forms within a single erythrocyte, in contrast to other plasmodia that will have 
only one ring form per erythrocyte. Plasmodium falciparum ring forms also 
tend to occur at the periphery of the erythrocyte; these "applique" forms are 
distinctive for this species. Mixed infections with more than one species of 
Plasmodium may occur. 



CLINICAL CASES 



341 



Table 48-1 

APPEARANCE OF MALARIA PARASITES IN BLOOD SMEARS 



CHARACTERISTIC 


Plasmodium SPECIES 


Crescent or sausage shapes present 


P. falciparum 


Trophozoites older than ring stages 


P. vivax, P. malariae, P. ovale 


Schizonts with more than 12 nuclei 


P. vivax 


Enlarged erythrocytes 


P. vivax 


Band forms 


P. malaria 


Ring stages only 


P. falciparum 


Fimbriated oval cell 


P. ovale 



Treatment and Prevention 

From the perspective of patient management, drugs to treat malaria fall into 
three categories: prophylactic, schizonticidal, and antirelapse. 

Prophylactics are designed to prevent infection by attacking the sporozoite 
stages or preventing the development of clinical symptoms by preventing 
schizogony in the erythrocytic cycle. Schizonticidal compounds may be used 
in prophylactic measures and to affect a clinical cure in an acute infection. 
Antirelapse drugs are directed against hypnozoite stages, as in vivax infection. 
A radical cure in P. vivax and P. ovale infections requires the use of drugs that 
eradicate both the erythrocytic and exoerythrocytic schizonts in the liver. 

Chloroquine is a schizonticidal compound and drug of choice in treating 
clinical cases of malaria. Mefloquine, referred to in the case presentation, is used 
in prophylaxis and also to treat chloroquine resistant strains of Plasmodium. 
Drug resistance of certain strains of Plasmodium is a practical problem. In this 
case, back-up drugs are quinine or chemically related mefloquine, or a combi- 
nation of sulfadoxine (a sulfonamide) and pyrimethamine (pyrimidine deriva- 
tive). Antirelapse or tissue schizonticidal drugs are aimed at hypnozoites (liver 
schizonts). The drug of choice in this category is primaquine. 



Synopsis 

The synopsis of malaria is presented in Table 48-2. 



342 



CASE FILES: MICROBIOLOGY 



Table 48-2 



Synopsis of Malaria 



Infective Location of 

stage of pathogenic 

etiologic agent stage 



Laboratory diagnosis 

based 
on identification of: 



Drugs used 
in treatment 



Sporozoite Liver 

(injected by (primary 

mosquito) of: schizogony) 

P. vivax Erythrocytes 

P. malariae (Schizogonic 

P. falciparum cycle) 
P. ovale 




Blood smears 

(Stages in erythrocytes) 



Prophylaxis 
Mefloquine 

Acute 
Chloroquine 

Chloroquine 
Resistant 
Quinine or 
Pyrimethamine 
+Sulfadoxine 

Antirelapse 
Primaquine 



COMPREHENSION QUESTIONS 

[48.1] Microscopic examination of a thin blood smear from a patient sus- 
pected of having malaria reveals numerous normal size erythrocytes 
without stippling but with ring stages, many with multiple ring stages 
and applique forms. Several erythrocytes show developing tropho- 
zoites that are spread across the erythrocytes in a band fashion. Which 
of the following is the most likely cause of infection? 

A. Plasmodium vivax 

B. Plasmodium malariae 

C. Plasmodium ovale 

D. Plasmodium falciparum 

E. A mixed infection with two Plasmodium species 

[48.2] A patient in California was diagnosed with malaria acquired through a 
blood transfusion. A discussion of this case by physicians included the 
following statements. Which statement is correct? 

A. The infected blood used in transfusion contained sporozoites. 

B. The patient should be treated with chloroquine and primaquine. 

C. The patient should be treated to eradicate the stages responsible for 
symptoms. 

D. The blood donor had chloroquine-resistant malaria. 

E. The patient would not be infective to mosquitoes. 



CLINICAL CASES 343 

[48.3] Cerebral malaria most commonly attends infection with which of the 
following? 

A. Any two species of Plasmodium 

B. Plasmodium malariae 

C. Plasmodium falciparum 

D. Plasmodium ovale 

E. Plasmodium vivax 

Answers 

[48. 1] E. Multiple ring stages and applique forms are indicative of P. falci- 
parum; several erythrocytes show developing trophozoites that are 
spread across the erythrocytes in a band fashion that is indicative of 
P. malariae infection. Normal size erythrocytes without stippling 
(Schiiffner dots) would exclude P. vivax and P. ovale. 

[48.2] C. The primary goal should be to treat the patient to eliminate the ery- 
throcytic cycle that is the cause of symptoms. This would constitute 
a radical cure because the liver phase only occurs if infection is initi- 
ated by sporozoites. Thus, treating with primaquine (B) is not neces- 
sary because the patient will not harbor hypnozoites. Likewise, (A) is 
not correct because sporozoites are only acquired from mosquitoes. 
Transfusion malaria is caused by schizonts and merozoites present in 
the transferred blood. There is no way to know that the malaria is 
chloroquine resistant until after treatment with chloroquine (D) and 
the patient, although not likely to occur, could transmit the infection 
to mosquitoes (E) because gametocytes would be present in the blood 
and susceptible Anopheles species occur in the United States. 

[48.3] C. Cerebral malaria involves the clinical manifestations of Plasmodium 
falciparum malaria that induce changes in mental status and coma 
and is accompanied by fever. Without treatment, cerebral malaria 
is fatal in 24-72 hours and the mortality ratio is between 25 to 50 
percent. The common histopathologic finding is the sequestration of 
parasitized and nonparasitized red blood cells in cerebral capillaries 
and venules. 



L 



MICROBIOLOGY PEARLS 

In a laboratory diagnosis, in which blood smears are treated with 
Giemsa or Wright stain plasmodia can be identified when the 
nucleus and cytoplasm are seen. 

Chemotherapy is directed at erythrocytic stages to provide a clinical 
cure and hypnozoites in the liver to affect a radical cure. Treatment 
of specific cases remains an active area of continuous investigation. 



344 CASE FILES: MICROBIOLOGY 

REFERENCES 

Centers for Disease Control. DPDx. Laboratory identification of parasites of public 
concern. Malaria. 2007 http:// www.dpd.cdc.gov/dpdx/HTML/Malaria.htm 

Centers for Disease Control. DPDx. Laboratory identification of parasites of public 
concern. Image Library. Malaria. 2007. http://www.dpd.cdc.gov/dpdx/HTML/ 
ImageLibrary/Malaria_il.htm 

Medical Letter on Drugs and Therapeutics. Drugs for parasitic infections. New 
Rochelle, NY. 2004. http://www.medletter.com/freedocs/parasitic.pdf. This ref- 
erence has been updated and "superseded by the special report Drugs for 
Parasitic Infections, which can be purchased (on-line) for S25." This reference 
has been provided because it is a comprehensive and clinically useful reference 
that is regularly updated and could be of value to those who are involved in treat- 
ing parasitic infections 

Centers for Disease Control. Malaria. Part 2: Treatment: General Approaches and 
Treatment: uncomplicated malaria.2007. http://www.cdc.gov/malaria/diagnosis_ 
treatment/clinicians2.htm 



♦> CASE 49 



A 40-year-old man presents for a routine examination. He is generally feeling 
well but complains of some mild dysuria and increasing urinary frequency. He 
has never had a urinary tract infection (UTI) and thought that the increasing 
urinary frequency was a normal part of aging. He has not seen any blood in his 
urine but says that the urine does appear darker than it used to look. He has no 
other complaints and his review of systems is otherwise entirely negative. He 
has no significant medical or family history. He smokes a pack of cigarettes a 
day and denies alcohol use. He is an immigrant from Egypt who has lived in 
the United States for 3 years. His vital signs and physical examination, includ- 
ing genital and prostate exams, are normal. A urinalysis shows many red blood 
cells, a few white blood cells, and oval-shaped parasite eggs with terminal 
spines. 

^ What organism is the likely cause of his hematuria? 

How does this organism gain entry into humans? 



346 CASE FILES: MICROBIOLOGY 



ANSWERS TO CASE 49: SCHISTOSOMIASIS 

Summary: A 40-year-old Egyptian man has hematuria. A urinalysis shows 
many red blood cells, a few white blood cells, and oval-shaped eggs with ter- 
minal spines. 

^ Organism likely cause of his hematuria: Schistosoma haematobium 

^ Method organism gains entry into humans: Penetration through 
intact skin by the cercarial stage of the organism 



CLINICAL CORRELATION 

Schistosomiasis is a human disease syndrome caused by infection with one of 
several parasitic trematodes or flukes of the genus Schistosoma. These para- 
sites are known commonly as blood flukes because the adult worms live in 
blood vessels of the definitive host. The human disease syndrome is character- 
ized by dermatitis that is caused by entry of the infective stage and by acute and 
chronic systemic symptoms caused by host responses to eggs deposited by adult 
worms. Schistosoma haematobium, S. mansoni, and S. japonicum are the major 
species that infect over 200 million humans in Asia, Africa, the Middle East, and 
South America. Schistosoma japonicum is considered a zoonotic infection. In 
addition to these three major species, others, such as S. mekongi and S. interca- 
latum, also with potential zoonotic properties, rarely infect humans. 
Schistosomes with avian or nonhuman mammalian hosts can cause severe der- 
matitis or swimmer's itch in humans, but not infection with adult worms. 

The disease syndrome parallels the development of the parasite in the defin- 
itive host. Swimmer's itch, an allergic dermatitis, is caused shortly after 
humans make skin contact with microscopic, infective larval forms called 
cercariae that live in an aquatic environment. Following exposure to human 
schistosomes, the dermatitis is mild and may go unnoticed. However, when 
exposed to cercariae of schistosomes that normally infect birds, swimmer's 
itch can present as an itchy maculopapular rash. Cercariae penetrate the intact 
skin, enter the circulation and migrate to the liver where they mature into 
adult male and female worms. The adult worms migrate via the bloodstream 
to their final locations. Schistosoma mansoni and S. japonicum descend to the 
mesenteric veins and S. haematobium to the vesical plexus. Gravid female 
worms may release 300-3000 eggs per day over a 5-10 year life span. Eggs of 
S. haematobium can work their way through the wall of the urinary bladder 
into the lumen and are eliminated in urine, while eggs of S. mansoni and S. 
japonicum work their way through the walls of the small intestine and colon 
and are voided in feces. Acute schistosomiasis or Katayama syndrome 
develops one to two months after initial infection and includes fever, chills, 
abdominal pain, lymphadenopathy, and hepatomegaly and splenomegaly 
The pathophysiologic etiology of acute schistosomiasis is not known. 



CLINICAL CASES 347 

However, the association of its manifestations with heavy infection suggests 
that it is a form of serum sickness as a result of circulating antigen-antibody 
complexes. Chronic schistosomiasis results from the inflammatory response to 
eggs, with granulomas, fibrosis, and scar tissue occurring at the site where 
eggs are deposited in tissues. Chronic schistosomiasis results from the inflam- 
matory response to the presence of eggs, with granulomas, fibrosis, and scar- 
ring around the eggs. Eggs in the bowel wall may result in symptoms of 
abdominal pain, diarrhea, and blood in the stool. Schistosomiasis of the blad- 
der can cause hematuria, dysuria, frequent urination, and a reduction in blad- 
der capacity. With intestinal schistosomiasis, the liver is frequently involved 
as a result of eggs being carried by the portal circulation and becoming trapped 
in the liver. Pathology involves inflammation and fibrosis, leading to cir- 
rhosis with resulting portal hypertension, splenomegaly, ascites, and 
abdominal and esophageal varices. Ectopic lesions may rarely be associated 
with eggs reaching the brain and/or lungs. 

APPROACH TO THE SUSPECTED SCHISTOME 
INFECTION 

Objectives 

1 . Learn the life cycle of blood flukes and the epidemiology and clinical 
course of infection. 

2. Be able to describe three basic aspects of infection: transmission, diag- 
nosis, and treatment/prevention. 

Definitions 

Cercaria: Infective, aquatic larval form of schistosomes; characterized by 
a forked tail. 

Miracidium: Ciliated larval form of schistosomes (and other flukes) that 
escapes from the egg and infects a snail intermediate host. 

Intermediate host: The host in the life cycle of a helminth that harbors the 
larval stage(s) of the parasite. 

Bilharziasis: A synonym for schistosomiasis. 

Swimmer's itch: Dermatitis in humans caused by cercariae penetrating the 
skin, commonly involving cercariae of schistosomes that parasitize birds 
or mammals but which cannot complete their life cycle in humans. 

Dioecious fluke: A fluke that has separate sexes, as opposed to hermaph- 
roditic flukes. 



348 CASE FILES: MICROBIOLOGY 



DISCUSSION 

Characteristics of Schistosomes That Impact Transmission 

The life cycle of all human schistosomes is similar, except in fine details. Eggs 
voided in feces or urine hatch in fresh water, releasing ciliated miracidia 

that penetrate a snail intermediate host. The species of snail varies with the 
species of schistosome. Miracidia undergo morphologic development through 
other larval stages, eventually reproducing asexually. The product is hundreds 
of cercariae with forked tails that emerge from the snail and swim freely. On 
contacting humans that enter their environment, the cercariae penetrate the 
intact skin, losing their tails in the process, and enter the circulation and are 
disseminated to all parts of the body. 

The cercariae, now termed schistosomula, are carried via the portal vein 
into the intrahepatic portal system where they mature in approximately 3-4 
weeks. After maturing, the worms migrate against the blood current and move 
into branches of veins that drain the urinary bladder (S. hematobium) or the 
lower ileum and cecum (S. japonicum) or the colon (5*. mansoni). Female 
worms lay several hundred eggs per day. The eggs of each species have char- 
acteristic morphology. All eggs leaving the host contain a fully developed 
miracidium. 



Diagnosis 

Infections can be definitively diagnosed by finding characteristic eggs in the 
urine or feces. Eggs of S. hematobium are in the shape of an elongated oval 
with a terminal spine; S. mansoni eggs are also oval shaped but with a distinct 
lateral spine; and S. japonicum eggs are round to oval with a short lateral spine 
or knob that often is unseen. In suspected cases where stools are negative, eggs 
of S. mansoni may be seen in microscopic examination of a rectal biopsy. 

Treatment and Prevention 

Praziquantel is the drug of choice in the treatment of human schistosomiasis 
and is effective against all human species. Because the drug has been reported 
to be effective as a prophylactic, larval stages and adult forms are presumed 
to be susceptible. The treatment of cercarial dermatitis is symptomatic. 
Prevention is based on avoiding skin exposure to groundwater in endemic 
areas. 

Synopsis 

The synopsis of schistosomiasis is presented in Table 49-1. 



CLINICAL CASES 



349 



Table 49-1 



Synopsis of Schistosomiasis 


Infective stage 
of etiologic agent 


Location of 
pathogenic stage 


Laboratory diagnosis based 
on identification of: 


Drugs used 
in treatment 


Cercariae: 
Schistosoma 


Adults: 

Mostly inferior 
mesenteric veins 


y^^ 


Praziquantel 


mansoni 




V ^^ 




Schistosoma 




Eggs (stool exam, rectal biopsy) 




japonicum 




y*^^\ 




Schistosoma 
hematobium 


Mostly superior 
mesenteric veins 


Eggs (stool exam, rectal biopsy) 






Vesicular veins 


^^ 




Eggs (urine) 


Swimmer's Itch — 
Cercarial dermatitis 
(Various species of 
non human 
schistosomes) 


Skin 


Clinical diagnosis 


None specific 



COMPREHENSION QUESTIONS 

[49.1] Which of the following is a host in the life cycle of all trematodes that 
infect humans? 

A. Flea 

B. Mosquito 

C. Mollusk 

D. Flour weevil 

E. Sandfly 



350 CASE FILES: MICROBIOLOGY 

[49.2] An oil field worker who has lived in Brazil for 10 years has mild gas- 
trointestinal symptoms. Brazil is the only country ever visited by the 
patient outside of the United States. The patient is diagnosed by his physi- 
cian of having 5*. mansoni because of which of the following? 

A. Round eggs with a prominent terminal spine were observed in a 
rectal biopsy. 

B. Blood was detected in the stool. 

C. Nonoperculated eggs with a miracidium inside were observed in 
stool samples. 

D. Eggs were found in a urine sample. 

E. Symptoms were relieved by treatment with praziquantel. 

[49.3] A 12-year-old boy reports feeling tingling and itching of his legs 30 
minutes after swimming in a lake. Over the next day, small papules 
develop followed by blisters of the legs. Dermatitis due to schistosome 
infection is diagnosed. What larval stage most likely caused the infection? 

A. Filariform larva 

B. Cysticercus 

C. Cercaria 

D. Miracidium 

E. Sparganum 

Answers 

[49.1] C. Snails are mollusks. All flukes have snails as intermediate hosts. 
Fleas, mosquitoes, flour weevils, and sand flies serve as intermediate 
or definitive hosts to various helminth and protozoan parasites, but 
not to flukes. 

[49.2] C. Eggs with a miracidium inside is indicative of a fluke infection; 
nonoperculated eggs are characteristic of schistosomes and differen- 
tiate them from all other human flukes, which have eggs with oper- 
cula. Also, S. mansoni is the only human schistosome endemic to 
Brazil and to the western hemisphere. A round egg with a terminal 
spine (A) is characteristic of S. hematobium but would not be 
expected in a rectal biopsy; furthermore, S. hematobium is not 
endemic to the western hemisphere. Blood in the stool (B) may be a 
finding in S. mansoni infection but would not be a definitive diagno- 
sis. Finding eggs in a urine sample (D) is consistent with S. hemato- 
bium but not S. mansoni infection; again, S. hematobium is not 
endemic to the western hemisphere. Vague gastrointestinal symptoms 
are not pathognomonic of schistosome infections. Praziquantel (E) is 
used to effectively treat all intestinal tapeworms of humans, as well as 
schistosomes. Symptoms could have been caused by adult tapewonns. 



CLINICAL CASES 351 

[49.3] C. Forked-tail cercariae are infective for humans. A filariform larva is 
the infective stage for hookworm and strongyloides, both nematode 
parasites. A cysticercus larva (B) is the infective stage of Taenia species 
of tapeworms. A miracidium (D) is the stage of flukes that infects 
snails, not humans. A sparganum larva (E) is involved in the life cycle 
of pseudophyllidean tapeworms, such as the broad fish tapeworm. 



MICROBIOLOGY PEARLS 



*♦* Swimmer's itch, a symptom of schistosomiasis, is also transmitted 

by bird schistosomes. 
*♦* Egg stages cause pathology in blood fluke infections and are critical 

in diagnosis of infection. 
*♦* Praziquantel is effective for all species of human schistosomes. 



REFERENCES 

Centers for Disease Control. DPDx. Laboratory identification of parasites of 
public concern. Schistosomiasis. 2007. http://www.dpd.cdc.gov/dpdx/HTML/ 
Schistosomiasis.htm. 

Centers for Disease Control. DPDx. Laboratory identification of parasites of public 
concern. Graphic Library. Schistosomiasis. 2007. http://www.dpd.cdc.gov/dpdx/ 
HTML/ImageLibrary/Schistosomiasis_il.htm. 

Medical Letter on Drugs and Therapeutics. Drugs for parasitic infections. New 
Rochelle, NY. 2004. http://www.medletter.com/freedocs/parasitic.pdf. This ref- 
erence has been updated and "superseded by the special report Drugs for 
Parasitic Infections, which can be purchased (on-line) for S25." This reference 
has been provided because it is a comprehensive and clinically useful reference 
that is regularly updated and could be of value to those who are involved in treat- 
ing parasitic infections. 



This page intentionally left blank 



♦> CASE 50 



A 19-year-old woman presents to the physician's office for the evaluation of 
an itchy vaginal discharge that she has had for about a week. She has had no 
fever, abdominal pain, or dysuria. She became involved with a new sexual 
partner approximately 3 weeks ago. She takes birth control pills but does not 
regularly use condoms during intercourse. Her partner is asymptomatic. On 
examination, her vital signs are normal, and a general physical examination is 
unremarkable. On pelvic examination, her external genitalia are normal. After 
inserting a speculum you see a bubbly, thin, yellow vaginal discharge. Her 
cervix is erythematous but without discharge. She has no cervical motion or 
uterine or adnexal tenderness. A wet mount of the vaginal discharge examined 
microscopically reveals numerous motile, flagellated, pear-shaped organisms 
along with numerous white blood cells. 

^ What is the most likely infectious cause of her vaginal discharge? 

^ What is the most likely source of her infection? 



354 CASE FILES: MICROBIOLOGY 

ANSWERS TO CASE 50: TRICHOMONIASIS 

Summary: A 19-year-old woman with vaginal discharge, which on microscopy 
reveals numerous motile, flagellated, pear-shaped organisms along with 
numerous white blood cells. 

^ Most likely infectious cause of her vaginal discharge: Trichomonas 
vaginalis 

^ Most likely source of her infection: Sexual contact with infected but 
asymptomatic partner 

CLINICAL CORRELATION 

Trichomonas vaginalis is a motile, pear-shaped protozoan with four flagella 
and an undulating membrane. It multiplies by binary fission and exists only 
in its trophozoite form; no cyst form has been identified. It is a common cause 
of both symptomatic and asymptomatic infections. Many infected women are 
asymptomatic or have only a small amount of thin vaginal discharge. Others 
develop symptomatic disease with vaginal inflammation, itching, and copi- 
ous vaginal discharge. The discharge may be white, yellow, or green, and 
bubbles are often seen. Cervical inflammation with punctate hemorrhages may 
produce a "strawberry cervix." The vast majority of infections in men are 
asymptomatic, although urethritis, prostatitis, and epididymitis can occur. The 
parasite is almost always passed by sexual contact, although fomite transmis- 
sion has been documented. The diagnosis is most often made by the micro- 
scopic evaluation of a sample of vaginal discharge in a saline wet mount. 
Flagellated, motile trichomonads will be visible in most symptomatic infec- 
tions. The diagnosis can also be made by the identification of organisms on 
Pap smears, by culture of the vaginal discharge, or by the use of specific mon- 
oclonal antibody stains or nucleic acid probes. This infection is usually treated 
with oral metronidazole, and both partners should be treated to prevent rein- 
fection. Because of its route of transmission, the identification of infection 
with Trichomonas should prompt the consideration of evaluation for other sex- 
ually transmitted diseases. 

APPROACH TO THE SUSPECTED Trichomonas 
INFECTION 

Objectives 

1. Learn the life cycle of Trichomonas vaginalis and the epidemiology 
and clinical course of infection. 

2. Be able to answer the three basic aspects of infection: transmission, 
diagnosis, and treatment/prevention. 



CLINICAL CASES 355 

Definitions 

Trophozoite: Feeding stage of protozoans, in contrast to the vegetative cyst 
stage that may be present in the life cycles of some protozoa. 

Axostyle: A hyaline rod-like structure that runs through the length of T. 
vaginalis and exits at the posterior end. 

Fomite: A substance other than food that may harbor and transmit infec- 
tious agents. 

DISCUSSION 

Characteristics of Trichomonas That Impact Transmission 

The life cycle of trichomonads in general is the simplest of protozoan life 
cycle because the organism exists only as a trophozoite that divides by binary 
fission. Transmission is presumed to be by direct transfer of trophozoites 
because a cyst stage does not exist. 

Sexual intercourse is considered the usual means of transmitting this 
infection that is common worldwide. The organism is transmitted cyclically 
from a woman to a man and back to the same or another woman. Infected men, 
who play a key role in transmission, are usually asymptomatic. Trichomonas 
vaginalis trophozoites in vaginal discharge are known to live for 30 minutes or 
more on toilet seats, supporting the possibility that some infections could be 
acquired through fomites such as towels and toilet seats. However, this means 
of transmission is not well supported by evidence. 

Diagnosis 

If infection with T. vaginalis is suspected, a first step is to diagnose infection by 
microscopically examining a wet mount preparation of vaginal discharge 
from the patient. The live parasite appears as a pear-shaped trophozoite with 

active flagella that give it motility. Sometime the undulating membrane pro- 
vides a waving movement. Propagation and concentration of the organism in 
culture is a possibility if wet mounts are negative. However, examination of wet 
mounts is usually sufficient to find and identify the organism. 

Treatment and Prevention 

Metronidazole is effective in treating 71 vaginalis. Treatment of both sexual 
partners is recommended to prevent reinfection. Using condoms correctly and 
consistently will lower the risk of individuals contracting trichomoniasis and 
other sexually transmitted diseases. 

Synopsis 

The synopsis of vaginal trichomoniasis is presented in Table 50-1. 



Table 50-1 



Synopsis of Vaginal Trichomoniasis 


Infective stage 


Location of 


Laboratory diagnosis based 


Drugs used 


of etiologic agent 


pathogenic stage 


on identification of: 


in treatment 


Trophozoite 


Genito-urinary tract 


^-ST^ 


Metronidazole 


Trichomonas 




[\/^-v'.'o\ 




vaginalis 




Trophozoite (vaginal 
discharge) 





COMPREHENSION QUESTIONS 

[50.1] Trichomoniasis is transmitted by which stage? 

A. Cyst 

B. Oocyst 
C Egg 

D. Sporozoite 

E. Trophozoite 

[50.2] What is the drug of choice in treating vaginal trichomoniasis? 

A. Metronidazole 

B. Mebendazole 

C. Mefloquine 

D. Niclosamide 

E. Niridazole 

[50.3] Laboratory diagnosis of vaginal trichomoniasis is most commonly 
made by which of the following methods? 

A. Identifying cyst stages in an iodine stained preparation of vaginal 
secretion. 

B. Finding trophozoites in a saline wet mount of vaginal discharge. 

C. Using an acid-fast stain to highlight the parasite. 

D. Staining a thin blood smear with common blood stains. 

E. Testing for specific antibodies against T. vaginalis in the patient's 
serum. 



CLINICAL CASES 357 

Answers 

[50.1] E. Trichomonas vaginalis exists only as a trophozoite; no cyst stage 
has been identified. A cyst (A), oocyst (B) and sporozoite (D) are 
stages involved in transmitting other protozoan infections, and an egg 
(C) is the means of transmission in a number of helminth infections. 

[50.2] A. Metronidazole is the drug of choice. Mebendazole (B) is a broad- 
spectrum antinematode agent. Mefloquine (C) is used as a prophylac- 
tic drug to prevent malaria and also used to treat chloroquine-resistant 
clinical malaria. Niclosamide is a broad-spectrum agent effective in 
the treatment of adult tapeworm infections. Niridazole (E) is a drug 
used to treat schistosomiasis if praziquantel is not available. 

[50.3] B. Trophozoites are usually visible in saline mounts of vaginal dis- 
charge or scrapings. Cysts (A) are not present in the T. vaginalis life 
cycle, and iodine is used primarily to observe cysts of intestinal pro- 
tozoa. Acid-fast stains (D) are used to search for oocysts of coccidian 
intestinal parasites, such as Cryptosporidium and Cyclospora. Thin 
blood smears are used to diagnose malaria. Serologic diagnoses (E) 
are helpful in the diagnosis of several "deep tissue" parasites but are 
not used in diagnosing T vaginalis. 



MICROBIOLOGY PEARLS 



*♦* Trichomonas vaginalis is an important sexually transmitted para- 
site throughout the world. 

*♦* Only trophozoite stages occur and frequently are difficult to find in 
a saline wet mount. 

*♦* Males and females are host, although males are generally asymptomatic. 

*♦* Metronidazole is the drug of choice for treating trichomoniasis. 



REFERENCES 

Centers for Disease Control. DPDx. Laboratory identification of parasites of 
public concern. Trichomoniasis. 2007. http://www.dpd.cdc.gov/dpdx/HTML/ 
Trichomoniasis .htm . 

Centers for Disease Control. DPDx. Laboratory identification of parasites of public 
concern. Image Library. Trichomoniasis. 2007. http:www.dpd.cdc.gov/dpdx/ 
HTML/ImageLibrary/Trichomoniasis_il.htm. 

Medical Letter on Drugs and Therapeutics. Drugs for parasitic infections. New 
Rochelle, NY. 2004. http://www.medletter.com/freedocs/parasitic.pdf. This ref- 
erence has been updated and "superseded by the special report Drugs for 
Parasitic Infections, which can be purchased (on-line) for S25." This reference 
has been provided because it is a comprehensive and clinically useful reference 
that is regularly updated and could be of value to those who are involved in treat- 
ing parasitic infections. 



This page intentionally left blank 



SECTION IV 



Listing of Cases 



Listing by Case Number 

Listing by Disorder (Alphabetical) 



Copyright © 2008 by The McGraw-Hill Companies, Inc. Click here for terms of use. 



This page intentionally left blank 



LISTING OF CASES 



361 



LISTING BY CASE NUMBER 



CASE NO. 


DISEASE 


1 


Bacillus anthracis 


2 


Bacteroides fragilis 


3 


Borrelia burgdorferi 


4 


Campylobacter jejuni 


5 


Chlamydia trachomatis 


6 


Clostridia 


7 


Corynebacterium diphtheriae 


8 


Enterococcus faecalis 


9 


Escherichia coli 


10 


Helicobacter pylori 


11 


Haemophilus influenzae 


12 


Klebsiella pneumoniae 


13 


Listeria monocytogenes 


14 


Mycobacterium 


15 


Mycoplasma 


16 


Neisseria 


17 


Proteus mirabilis 


18 


Pseudomonas aeruginosa 


19 


Salmonella/Shigella 


20 


Streptococcus 


21 


Staphylococci 


22 


Treponema pallidum 


23 


Vibrio cholerae 


24 


Adenovirus 


25 


Cytomegalovirus 


26 


Epstein-Barr virus 


27 


Hepatitis viruses 


28 


Herpes simplex viruses 


29 


Human immunodeficiency virus 


30 


Human papillomavirus 


31 


Molluscum contagiosum 


32 


Mumps 


33 


Parvoviruses 


34 


Poliovirus 


35 


Rotavirus 


36 


Respiratory syncytial virus 


37 


Smallpox 


38 


Varicella-zoster virus 


39 


Aspergillus 


40 


Blastomycosis, coccidioidomycosis. 




and histoplasmosis 


41 


Candida 


42 


Ciyptococcus neoformans 


43 


Pneumocystis jiroveci 



CASE PAGE 



28 

34 

40 

46 

52 

58 

64 

70 

76 

84 

90 

96 

102 

108 

116 

122 

1 30 

136 

142 

150 

156 

164 

172 

178 

184 

190 

196 

208 

214 

222 

230 

236 

242 

248 

254 

260 

266 

272 

278 

284 

290 
296 

302 



362 



CASE FILES: MICROBIOLOGY 



CASE NO. 


DISEASE 


44 


Sporothrix schenckii 


45 


Ascariasis 


46 


Cryptosporidiosis 


47 


Enterobiasis or pinworm infection 


48 


Malaria 


49 


Schistosomiasis 


50 


Trichomoniasis 



CASE PAGE 



308 
314 
324 
332 
340 
348 
356 



LISTING BY DISORDER (ALPHABETICAL) 



CASE NO. 


DISEASE 


24 


Adenovirus 


45 


Ascariasis 


39 


Aspergillus 


1 


Bacillus anthracis 


2 


Bacteroides fragilis 


40 


B lastomycosis, coccidiodomycosis. 




and histoplasmosis 


3 


Borrelia burgdorferi 


4 


Campylobacter jejuni 


41 


Candida 


5 


Chlamydia trachomatis 


6 


Clostridia 


7 


Corynebacterium diphtheriae 


42 


Cryptococcus neofortnans 


46 


Cryptosporidiosis 


25 


Cytomegalovirus 


47 


Enterobiasis or pinworm infection 


8 


Enterococcus faecalis 


26 


Epstein-Barr virus 


9 


Escherichia coli 


11 


Haemophilus influenzae 


10 


Helicobacter pylori 


27 


Hepatitis viruses 


28 


Herpes simplex viruses 


29 


Human immunodeficiency virus 


30 


Human papillomavirus 


12 


Klebsiella pneumoniae 


13 


Listeria monocytogenes 


48 


Malaria 


31 


Molluscum contagiosum 


32 


Mumps 


14 


Mycobacterium 


15 


Mycoplasma 



CASE PAGE 



178 
314 
278 
28 
34 
284 

40 

46 

290 

52 

58 

64 

296 

324 

184 

332 

70 

190 

76 

90 

84 

196 

208 

214 

222 

96 

102 

340 

230 

236 

108 

116 



LISTING OF CASES 363 



CASE NO. DISEASE CASE PAGE 

122 
242 
302 
248 
130 
136 
260 
254 
142 
348 
266 
308 
156 
150 
164 
356 
272 
172 



16 


Neisseria 


33 


Parvoviruses 


43 


Pneumocystis jiroveci 


34 


Poliovirus 


17 


Proteus mirabilis 


18 


Pseudomonas aeruginosa 


36 


Respiratory syncytial virus 


35 


Rotavirus 


19 


Salmonella/Shigella 


49 


Schistosomiasis 


37 


Smallpox 


44 


Sporothrix schenckii 


21 


Staphylococci 


20 


Streptococcus 


22 


Treponema pallidum 


50 


Trichomoniasis 


38 


Varicella-zoster virus 


23 


Vibrio cholerae 



This page intentionally left blank 



♦> INDEX 



Note: Page numbers followed by for t indicate figures or tables, respectively. 



A 

Abacavir, 16? 
ABCs, 172 
Abdominal pain 

in Bacteroides fragilis infection, 33-34 

in Campylobacter jejuni gastroenteritis, 
45-46 

in Clostridium difficile colitis, 51 

in cryptosporidiosis, 321 

in Helicobacter pylori infection, 83 

in hepatitis C, 201 

retro vir-related, 18 

in rotavirus infection, 253, 256 

in Salmonella gastroenteritis, 141—142 

in schistosomiasis, 346-347 

in Vibrio parahaemolyticus infection, 172 
Abdominal varices, in schistosomiasis, 347 
Abortive poliomyelitis, 249—250 
Abscess 

in Campylobacter fetus infection, 46 

in granulomatosis infantiseptica, 102 

in Klebsiella pneumoniae infection, 96 

in Proteus infection, 130 
Acid tolerance response gene, 

Salmonella, 143 
Acid-fast stain 

for Cryptosporidium, 323 

for mycobacteria, 109 
Acquired immune deficiency syndrome 

(AIDS), 214, 215. See also Human 
immunodeficiency virus (HIV) infection 
Acyclovir, 17, Ylt 

for herpes simplex virus, 211 

for varicella-zoster virus, 273 
Adefovir, 16? 

Adenopathy, in syphilis, 163 
Adenovirus, 4/ 177-182, 180/ 
Adenyl cyclase, Vibrio choleras and, 173 
Adherence protein PI, Mycoplasma pneumoniae, 

117 
Adhesin 

Escherichia coli, 11 

Pseudomonas aeruginosa, 131 



Adult stage 

Ascaris lumbricoides, 316-317 

Enterobius vermicularis, 331 

helminths, 315/ 316 

Schistosoma haematobium, 348 
Aerobes 

Enterococcus faecalis, 69-73 

Mycoplasma pneumoniae, 1 15-120 

Neisseria species, 121-126 

Pseudomonas aeruginosa, 29, 135-140 
AIDS (acquired immune deficiency syndrome), 
214, 215. See also Human immunodefi- 
ciency virus (HIV) infection 
Albendazole, 22? 
Allergic bronchopulmonary aspergillosis, 

278-279 
Allylamine antifungals, 19?, 21 
Alphaherpesvirinae, 272 

herpes simplex virus, 191?, 207-212, 210/ 

varicella-zoster virus, lit, 18, 191?, 271-275 
Amantadine, 16, 16? 
Amebiasis, antiparasitic agents for, 22? 
Amikacin, 111 
Aminoglycosides, 15? 

for enterococci, 72 

for Pseudomonas aeruginosa, 138 
Aminopenicillins, 14? 
Ammonia, urease and, 85 
Amoxicillin, 14? 

for Lyme disease, 42 

for Salmonella, 144 

for typhoid fever, 144 
Amoxicillin-clavulanate, for Haemophilus 

influenzae, 92 
Amphotericin B, 19-21, 19?, 22? 

for aspergillosis, 281 

for Candida albicans, 292 

for Coccidioides immitis, 286 

for Cryptococcus neoformans, 298 
Ampicillin, 14? 

for enterococci, 72 

for leptospirosis, 41? 

for Listeria, 104 



Copyright © 2008 by The McGraw-Hill Companies, Inc. Click here for terms of use. 



366 



INDEX 



Amprenavir, 16? 
Anaerobes, 34 

Bacteroides fragilis, 33-38 

Clostridium difficile, 57-61 

Haemophilus influenzae, 89-94 

Listeria monocytogenes, 101-105 

oxygen tolerant, 35 

Proteus species, 129-133 

Salmonella, 141-147 

Staphylococcus aureus, 67, 155-161, 
158/ 

strict, 35 

Vibrio species, 171—176 
Anemia 

in malaria, 338 

in parvovirus B19 infection, 243 

in pneumococcal pneumonia, 150 
Anogenital warts, 222t, 224 
Anopheline mosquito, 339 
Anorexia 

in cryptosporidiosis, 322 

in Klebsiella pneumoniae pneumonia, 96 

in typhoid fever, 142 
Anterior fontanelle, 103 
Anterior horn cells, poliovirus and, 249 
Anthrax, 27-32 
Antibacterial agents, 13-15, 14f, \5t. See also 

specific agents 
Antibacterial resistance. See Resistance, 

antibacterial 
Antibiotic-associated colitis, 59 
Antibody(ies) 

Aspergillus, 279 

Borrelia burgdorferi, 42 

Coccidioides immitis, 286 

Epstein-Barr virus, 192 

hepatitis A, 199 

hepatitis B, 199 

hepatitis C, 201 

heterophile, 190, 192 

human immunodeficiency virus, 216 

poliovirus, 249 
Anticestode agents, 22? 
Antifungal agents, 19-21, 19?. See also specific 

agents 
Antigen 

anthrax toxin, 29 

cytomegalovirus, 185 

Epstein-Barr virus, 191 

hepatitis B, 199, 200/ 202 

hepatitis C, 201 

hepatitis D, 201 

human immunodeficiency virus, 216 

mumps, 237 

parvovirus B19, 353 

rotavirus, 255, 256 
Antigen-antibody complex 

Mycoplasma pneumoniae, 117 

in schistosomiasis, 347 
Antihelminthic agents, 22t 
Antimicrobial agents, 13 

antibacterial. See Antibacterial agents 

antifungal. See Antifungal agents 



antiparasitic. See Antiparasitic agents 

antiviral. See Antiviral agents 
Antiparasitic agents, 21-22, 22? 
Antitoxin, diphtheria, 66 
Antitrematode agents, 22? 
Antitreponemal antibody stain, 165 
Antiviral agents, 16-19, 16?. See also specific 

agents 
Apicomplexa, 5 
Aplastic crisis, parvovirus B19 infection 

and, 244 
Appendicitis, 47 

Applique form, Plasmodium falciparum, 339 
Arenaviridae, 4/ 

Argyll Robertson pupil, 165, 167 
Arthralgia 

in Coccidioides immitis infection, 283-284 

in hepatitis, 197 

in Klebsiella pneumonia pneumonia, 96 

in parvovirus B19 infection, 243 
Arthritis 

Lyme disease and, 40 

in parvovirus B19 infection, 243 

septic, 121-126 

streptococcal, 150 
Arthroconidia, Coccidioides immitis, 

284-285 
Asbestosis, 108 

Ascaris lumbricoides, 313-320, 314/ 319? 
Ascaris suum, 316 
Ascites, schistosomiasis and, 347 
Aseptic meningitis, poliovirus in, 250 
Aspergilloma, 278-279 
Aspergillus flavus, 278-279 
Aspergillus fumigatus, 277-282, 280/ 
Aspergillus terreus, 279 
Asthma, Aspergillus fumigatus infection in, 

277-278 
Atovaquone, 22? 
Attenuated live poliovirus vaccine, 

248-249 
Atypical lymphocytes, 190 
Atypical pneumonia, Chlamydia pneumoniae 

in, 54 
Axostyle, Trichomonas vaginalis, 355 
Azidothymidine (AZT), 18 
Azithromycin, 15? 

for Chlamydia trachomatis, 55 

for Haemophilus influenzae, 92 

for Mycobacterium avium-intra.cellula.re, 111 

for Streptococcus pneumoniae, 152 
Azole antifungals, 19?, 20-21, 22?. See also 
specific agents 

for Candida albicans, 292 

for Coccidioides immitis, 286 

for invasive aspergillosis, 281 
Azotemia, 173 
AZT (azidothymidine), 18 
Aztreonam, 13, 15? 



B 



Bacillus anthracis, 27-32 
Bacillus cereus, 28, 105 



INDEX 



367 



Bacteremia 

in Campylobacter jejuni gastroenteritis, 46 

in Corynebacterium jeikeium infection, 64 

in Entewcoccus faecalis infection, 70 

in Neisseria gonorrhoeae infection, 122 

in Proteus infection, 130 

in Pseudomonas aeruginosa infection, 136 

in Salmonella infection, 142 
Bacteria, 4-5, 5/ 

Bacillus anthracis, 27-32 

Bacillus cereus, 28, 105 

Bacteroides fragilis, 33-38 

Borrelia burgdorferi, 39^14, 41r 

Campylobacter jejun i, 45-50 

Chlamydia trachomatis, 51—56 

Clostridium, 57-61 

Corynebacterium, 63-67 

Entewcoccus faecalis, 69-73 

Haemophilus, 89—94 

Helicobacter pylori, 83-87 

Klebsiella pneumoniae, 95—100 

Listeria monocytogenes, 101-105 

Mycobacterium species, 107-113 

Mycoplasma, 115-120 

Neisseria, 121—126 

Proteus, 129-133 

Pseudomonas aeruginosa, 135-140 

Salmonella, 141-147 

Shigella, 143-147 

Staphylococcus, 67, 155-161, 158/ 

Streptococcus, 149—154 

Treponema pallidum, 163—170 

U reaplasma urealyticum, 118 

Vibrio, 171-176 
Bacteroides bile esculin (BBE) agar, 34, 35 
Bacteroides fragilis, 33—38 
B-cell lymphoma, 192 
BCG vaccine, 1 1 1 

Beef, Escherichia coli infection and, 76 
Benzathine penicillin, for syphilis, 167 
(i-lactam antibiotics, 15? 

for Bacteroides fragilis, 36 

for Entewcoccus faecalis, 11 
(i-lactamase production 

Klebsiella pneumoniae and, 97 

Neisseria gonorrhoeae and, 123 

Pseudomonas aeruginosa and, 137 
Betaherpesvirinae, 184 
Bifurcated needle, 267 
Bile solubility, streptococci, 152 
Bilharziasis, 347 
Biofilm, 157 

Bioterrorism, Bacillus anthracis and, 30 
Birds, Chlamydophila psittaci and, 54 
Bismuth, for Helicobacter pylori, 86 
Blackwater fever, 338, 339 
Bladder 

infection. See Urinary tract infection 

Schistosoma haematobium in, 346 
Blastomyces dermatitidis, 285, 286 
Blastospore, 291 

Blindness, Chlamydia trachomatis and, 
53-54 



Blood agar 

for Bacteroides fragilis, 35 

for Campylobacter jejuni, 47 

for Staphylococcus aureus, 159 

for Streptococcus pneumoniae, 152 

for Vibrio cholerae, 174 
Blood culture 

for Klebsiella pneumoniae, 97 

for Listeria monocytogenes, 103 

for Salmonella and Shigella, 144 

for Staphylococcus aureus, 159 
Blood fluke, 345-351 
Blood smears, for Plasmodium, 340 
Blood transfusion, hepatitis C infection 

and, 196 
Blurred vision, voriconazole-related, 20 
Bone marrow suppression 

fluey to sine-related, 20 

ganciclovir-related, 18 

retro vir-related, 18 
Borrelia burgdorferi, 39^14, Alt 
Botulism, 58 

Bronchiectasis, Aspergillus fumigatus and, 279 
Bronchiolitis, respiratory syncytial virus and, 

260,261 
Bronchitis 

Haemophilus species in, 90 

Streptococcus pneumoniae in, 150 
Bronchoalveolar lavage 

for aspergillosis, 280 

for Pneumocystis jiroveci, 303 
Bull's eye skin lesion, 40 
Bunyaviridae, 4/ 
Burkholderia cepacia, 137 
Burkitt lymphoma, 192 



Cachetic (definition), 296 

Calcofluor white stain 
for Aspergillus, 280 
for Coccidioides immitis, 285 
for Pneumocystis jiroveci, 303 

Caliciviridae, Af 202 

Campylobacter coli, 46 

Campylobacter fetus, 46 

Campylobacter jejuni, 45-50 

Candida albicans, 289-294, 292? 

Capsid, 3, 4/ 
adenovirus, 178 
cytomegalovirus, 184 
rotavirus, 255 

Capsule 

of Bacillus anthracis, 29 
of Bacteroides fragilis, 35 
of Cryptococcus neoformans, 297 
of Haemophilus influenzae, 91 
of Klebsiella pneumoniae, 97 
of Neisseria meningitidis, 123 
of Pseudomonas aeruginosa, 137 
of Streptococcus, 151 

Carbapenems, 13, 15? 

Carboxypenicillin, 14? 

Cardiolipin antibody, 167 



368 



INDEX 



Cary-Blair medium, 47 

Caspofungin, for Candida albicans, 292 

Catalase production 

by Helicobacter pylori, 85 

by Listeria, 103 

by Staphylococcus aureus, 159 
Catheter-related infection, 69-70 
Cavitary lung lesion 

in Klebsiella pneumoniae pneumonia, 96 

in tuberculosis, 108 
CCFA medium, 59 
CD4 T cells 

human immunodeficiency virus and, 214 

Pneumocystis jiroveci and, 302 
Cefadroxil, 14? 
Cefazolin, Ut 
Cefepime, 14? 

for Klebsiella pneumoniae, 98 
Cefotaxime, 14? 

for Haemophilus influenzae, 92 

for pneumococcal disease, 152 
Cefotetan, 14r 
Cefoxitin, 14? 
Ceftazidime, 14? 
Ceftriaxone, 14? 

for Haemophilus influenzae, 92 

for Lyme disease, 42 

for Neisseria gonorrhoeae, 124 

for pelvic inflammatory disease, 55 
Cefuroxime, 14? 
Cell-mediated immunity 

Candida and, 290 

Cryptococcus neoformans and, 297 

cytomegalovirus and, 185 

herpes simplex virus and, 209 

mumps virus and, 237 

Pneumocystis jiroveci and, 302 
Cellulitis, Vibrio vulnificus in, 172 
Cephalexin, 14? 
Cephalosporins, 13, 14?, 15? 

for Haemophilus influenzae, 92 

fox Klebsiella pneumoniae, 97 
Cercaria, 346, 347, 348 
Cerebrospinal fluid, Cryptococcus neoformans 

and, 298 
Cerebrospinal fluid culture 

for Haemophilus influenzae, 91 

for herpes simplex virus, 210 

for Listeria monocytogenes, 103 
Cervical adenitis, in adenovirus infection, 179 
Cervical alae, of pinworm, 331 
Cervical motion tenderness. See Chandelier sign 
Cervix 

Chlamydia trachomatis and, 54 

high-grade squamous intraepithelial lesion, 
221-222 

human papillomavirus and, 221-224, 222/ 

Neisseria gonorrhoeae and, 122 

strawberry, 354 
Cestodes, antiparasitic agents for, 22? 
Chancre, 164 
Chancroid, 90 
Chandelier sign, 52, 54 



Chest pain, in Coccidioides immitis infection, 

284 
Chickenpox, 272 
Children 

chickenpox in, 272 

mumps in, 235-239 

parvovirus B19 in, 241—246 

pinworm infection in, 329—335 

respiratory syncytial virus infection in, 
259-262 

rotavirus infection in, 253—258 

viral conjunctivitis in, 177—178 
Chills 

amphotericin B-related, 20 

in malaria, 338 

in pneumococcal pneumonia, 150 

in schistosomiasis, 346 

in smallpox, 268 

in typhoid fever, 142 
Chlamydia trachomatis, 5 1—56, 53? 
Chlamydophila pneumoniae, 54 
Chlamydophila psittaci, 54 
Chloroquine, 22?, 341 
Chocolate agar 

for Haemophilus influenzae, 91 

for Neisseria gonorrhoeae, 124 

for Streptococcus pneumoniae, 152 
Cholera, 171-176 

Chorioretinitis, cytomegalovirus and, 185 
Chronic obstructive pulmonary disease 

(COPD), 97 
Cidofovir, 17?, 268 
Cilastatin, 13 
Ciliophora, 5 
Ciprofloxacin, 15?, 30, 31 
Cirrhosis 

hepatitis C and, 201 

schistosomiasis and, 347 
Clarithromycin, 15?, Ill 
Clindamycin, 15? 

for malaria, 22? 

for staphylococcal infection, 159 
Clinical database, 6 
Clostridium botulinum, 58, 67 
Clostridium difficile, 57-61 
Clostridium perfringens, 58 
Clostridium tetani, 58, 67 
Clotrimazole, 19?, 292 
Cloxacillin, 14? 
Coagulase, Staphylococcus aureus production 

of, 157 
Coccidian parasites, 323 
Coccidioides immitis, 283-288 
Cold agglutinins, for Mycoplasma pneumoniae, 

117 
Cold enrichment, 102 
Cold sore, 207-208 
Colitis 

antibiotic-associated, 59 

pseudomembranous, 58, 59 
Complement fixation testing, for Mycoplasma 

pneumoniae, 117 
Condyloma acuminatum, 222?, 223 



INDEX 



369 



Condyloma latum, 166, 166/ 
Congenital infections 

cytomegalovirus, 185 

parvovirus B19, 24 

Treponema pallidum, 165 
Conjunctivitis 

adenovirus, 177-178 

Chlamydia trachomatis, 53, 54 

definition, 178 

in herpes simplex virus infection, 209 

Neisseria gonorrhoeae and, 122 
Cor pulmonale, respiratory syncytial virus 

and, 261 
Cord factor, Mycobacterium tuberculosis, 110 
Corkscrew motility, flagellum, 85 
Coronaviridae, 4/ 
Corticosteroids, for allergic bronchopulmonary 

aspergillosis, 281 
Corynebacterium diphtheriae, 63-67 
Corynebacterium jeikeium, 63-67 
Coryza, in hepatitis, 197 
Cough 

in adenovirus infection, 179 

in allergic bronchopulmonary aspergillosis, 
278 

in Coccidioides immitis infection, 284 

in Mycoplasma pneumoniae pneumonia, 1 17 

in pneumococcal pneumonia, 150 

in Pneumocystis jiroveci pneumonia, 302 

in respiratory syncytial virus infection, 261 
Cowpox virus, 266-267 
Coxsackie adenovirus receptor, 179 
Cryotherapy, for wart removal, 225 
Cryptococcus neoformans, 285, 295—300 
Cryptosporidium parvum, 22t, 321-328, 

325?, 326? 
Curettage, for Molluscum contagiosum 

lesions, 231 
Cutaneous anthrax, 27-32 
Cutaneous diphtheria, 64 
Cutaneous infections 

candidal, 290 

herpes simplex virus, 209 
Cutaneous sporotrichosis, 308 
Cyclospora cayetanensis, 22t, 323, 324, 325? 
Cystic fibrosis 

Aspergillus fumigatus infection in, 277-278 

Pseudomonas aeruginosa infection in, 137 
Cystitis, 77. See also Urinary tract infection 
Cytolytic toxins, Staphylococcus aureus, 157 
Cytomegalovirus, 18, 183-187, 191? 
Cytotoxin 

Clostridium difficile, 59 

Helicobacter pylori, 85 

D 

Dane particle, 197, 199 
Dapsone, 22? 

for leprosy, 1 1 1 

for Pneumocystis jiroveci, 303 
Darkfield microscopy, for Treponema pallidum, 

165, 167 
Deafness, cytomegalovirus infection and, 185 



Dehydration 

in cholera, 172, 173 

in rotavirus infection, 256 
Delavirdine, I6t 
Delta antigen, 201 
Dematiaceous fungi, 308 
Dermatitis 

diaper, 290 

in schistosomiasis, 346 
Dermatome, 272 
Diaper dermatitis, candidal, 290 
Diarrhea 

antibiotic-associated, 59 

in Campylobacter jejuni gastroenteritis, 
45^6 

in Clostridium difficile infection, 57-61 

in cryptosporidiosis, 322 

in Escherichia coli infection, 76 

in rotavirus infection, 256 

in Salmonella infection, 142 

in schistosomiasis, 347 

in Vibrio cholerae infection, 172 
Dick test, 287 
Dicloxacillin, 14?, 159 
Didanosine, 16f, 18 
Differential diagnosis, 29 
Dimorphic fungi, 285 
Dioecious fluke, 347 
Diphtheria, 63-67 
Direct fluorescent-antibody staining, for varicella 

zoster, 273 
Disseminated intravascular coagulation (DIC), 

123 
Diverticulitis, Bacteroides fragilis and, 34 
DNA polymerase inhibitors, 16, 273 
DNA probes 

for adenovirus, 179 

for Coccidioides immitis, 286 

for Mycobacteria, 110 
DNA viruses, 4/ 

adenovirus, 177-182, 180/ 

cytomegalovirus, 18, 183-187, 191* 

Epstein-Barr virus, 189-194, 19lt 

hepatitis B, 16t, 199-200, 200/ 

herpes simplex virus, 19 If, 207-212, 210/ 

human papillomavirus, 221-226, 222?, 224/ 

Molluscum contagiosum, 229-233 

parvovirus B19, 241-246 

poxviruses, 230-231, 267 

varicella-zoster virus, 17?, 18, 191?, 271-275 
Doxycycline, 15?, 22t 

for cholera, 174 

for leptospirosis, 41? 

for Lyme disease, 42 

for malaria, 22t 

for pelvic inflammatory disease, 55 

for syphilis, 167 
Dysphagia, diphtheria and, 64 
Dyspnea 

in Coccidioides immitis infection, 284, 285 

in pneumococcal pneumonia, 150 

in Pneumocystis jiroveci pneumonia, 302, 303 
Dysuria, in schistosomiasis, 347 



3 70 



INDEX 



E 

Ear infection. See Otitis externa; Otitis media 
Early antigens, Epstein-Barr virus, 191-192 
EBV (Epstein-Barr virus), 189-194, 191? 
Ecthyma gangrenosum, 29, 136 
Edema factor, anthrax toxin, 29 
Efavirenz, 16/ 

Efflux pumps, antibiotic resistance and, 152 
Eggs 

Ascaris lumbricoides, 316-318 

Enterobius vermicularis, 331 

of helminths, 316 

larvated, 331 

Schistosoma species, 348, 349/ 
Elderly 

Pseudomonas aeruginosa infection in, 
135-136 

respiratory syncytial virus infection in, 261 
Electrocautery 

for Molluscum contagiosum, 231 

for warts, 225 
Elek test, 65 

Elementary body, C. trachomatis, 52, 53, 53/ 
ELISA. See Enzyme-linked immunoassay 
Empyema, in Klebsiella pneumoniae 

pneumonia, 97 
En anthem, 242 
Encephalitis 

Epstein-Barr virus and, 192 

in herpes simplex virus infection, 18, 209 
Endocarditis 

Campylobacter fetus, 46 

candidal, 290 

enterococcal, 70 

Pseudomonas, 136 

streptococcal, 150 
Endocytosis, receptor- mediated, 179, 180/ 
Endoflagella, Borrelia burgdorferi, 41 
Endospore, Coccidioides immitis, 285 
Endotoxin A, Pseudomonas aeruginosa, 137 
Enfuvirtide, 16/ 
Enteric fever, 144-145 
Enteroaggregative E. coli, 76 
Enterobacteriaceae 

Escherichia coli, 47, 75-81, lit, 78/ 

Klebsiella pneumoniae, 95—100 

Proteus mirabilis, 129-133 

Salmonella, 47, 141-147 

Shigella, 47, 143-147 
Enterobius vermicularis (pin worm), 315/ 

329-335, 333/ 
Enterococcus faecaUs, 69-73 
Enterococcus faecium, 71 
Enterohemorrhagic Escherichia coli, 47, 76 
Enteroinvasive Escherichia coli, 76 
Enterotoxigenic Escherichia coli, 76 
Enterotoxins 

Clostridium difficile, 59 

rotavirus, 255 

Staphylococcus aureus, 156, 157 

Vibrio cholerae, 173 
Enzyme immunoassay 

for Clostridium difficile, 60 

for Cryptococcus neoformans, 298 



for Mycoplasma pneumoniae, 111 

for rotavirus, 256 
Enzyme-linked immunoassay (ELISA) 

for adenovirus, 179 

for Borrelia burgdorferi, 42 

for Epstein-Barr virus, 193 

for hepatitis D, 201 

for human immunodeficiency virus, 214, 216 

for parvovirus B 19, 244 

for respiratory syncytial virus, 262 
Eosinophilia, in Ascaris lumbricoides infection, 

314 
Eosinophilic cytoplasmic inclusions, 230, 231 
Epididymitis 

Chlamydia trachomatis in, 54 

Trichomonas vaginalis in, 354 
Epiglottitis, Haemophilus influenzae in, 91 
Epithelial cells 

adenovirus and, 179 

Epstein-Barr virus and, 191-192 

human papillomavirus and, 222 

mumps virus and, 237 

rotavirus and, 255 
Epstein-Barr virus (EBV), 189-194, 191/ 
Ertapenem, 13, 15/ 

Erythema infectiosum (fifth disease), 241-246 
Erythema migrans, 40, 42 
Erythrocyte sedimentation rate, 136 
Erythroid progenitor cells, 242 
Erythromycin, 15/ 

for Campylobacter jejuni infection, 48 

for Chlamydia trachomatis, 55 

for diphtheria prophylaxis, 66 

for Haemophilus ducreyi, 92 

for Lyme disease, 41/ 

for Mycoplasma pneumoniae, 118 

for syphilis, 167 

for Ureaplasma infections, 118 
Eschar, 28, 29 

Escherichia coli, 47, 75-81, 78/ 
Esophageal varices, schistosomiasis and, 347 
Etaquine, 22/ 
Ethambutol, 111 
Exanthem, 242 

Exfoliative toxins, Staphylococcus aureus, 157 
Exotoxins 

of Corynebacterium diphtheriae, 65, 67 

of Escherichia coli, 11 

of Pseudomonas, 137 
Exudate 

definition, 52 

in diphtheria, 64 
Eye, Chlamydia trachomatis and, 53-54 



Facultative anaerobes 

Haemophilus influenzae, 89-94 
Listeria monocytogenes, 101-105 
Proteus species, 129-133 
Staphylococcus aureus, 67, 155-161, 158/ 
Vibrio species, 171—176 
Famciclovir, 17/, 18 

for herpes simplex virus, 211 
for vari eel la-zoster virus, 273 



INDEX 



371 



Fansidar, 22/ 
Fatigue 

in Coccidioides immitis infection, 284 

in Epstein-Barr virus infection, 190 

in hepatitis, 197 

in Lyme disease, 40 
Fecal leukocytes, in gastroenteritis, 47, 

143, 144 
Fecal-oral transmission 

of polio virus, 249 

of rotavirus, 255 
Fever 

in abortive poliomyelitis, 250 

in adenovirus infection, 179 

amphotericin B-related, 20 

in Ascaris lumbricoides infection, 314 

in Bacteroides fragilis infection, 33 

in Campylobacter jejuni gastroenteritis, 
45^6 

in Coccidioides immitis infection, 284 

in cryptosporidiosis, 322 

in diphtheria, 64 

in Enterococcus faecalis infection, 69 

in Epstein-Barr virus infection, 190, 192 

in Haemophilus influenzae meningitis, 91 

in hepatitis, 197 

in herpes simplex virus infection, 209 

in Klebsiella pneumonia pneumonia, 96 

in Lyme disease, 40 

in malaria, 338 

in mumps, 236, 237 

in Mycoplasma pneumoniae pneumonia, 
117 

in parvovirus B19 infection, 242, 243 

in pneumococcal pneumonia, 150 

in Pneumocystis jiroveci pneumonia, 302 

in Pneumocystis pneumonia, 302 

retro vir-related, 18 

in rotavirus infection, 256 

in Salmonella infection, 142 

in schistosomiasis, 346 

in smallpox, 268 

in streptococcal infection, 150 

in typhoid fever, 142 

in Vibrio parahaemolyticus infection, 172 
Fibrosis, in schistosomiasis, 347 
Fifth disease, 241-246 
Filopod, of Listeria monocytogenes, 103 
Filoviridae, 4/ 

Fimbriae, of Proteus species, 131 
Flagella 

of Borrelia burgdorferi, 41 

of Campylobacter jejuni, 47 

of Helicobacter pylori, 84 

of Proteus species, 131 

of Pseudomonas aeruginosa, 137 

of Trichomonas vaginalis, 354 
Flank pain, in nephrolithiasis, 129 
Flat wart, 222/, 224 
Flatworms, 5 
Flaviviridae, Af 200, 202 
Fluconazole, 19/ 

for Candida albicans, 292 

for Cryptococcus neoformans, 298 



Flucytosine, 19, 19/, 20, 21 
Flukes 

antiparasitic agents for, 22? 

dioecious, 347 

Schistosoma haematobium, 345-351 
Fluorescein, 137 
Fluorescent antibody stain 

for Cryptosporidium, 324 

for varicella zoster, 273 
Fluorescent treponemal antibody-absorption 

test (FTA-ABS), 164 
Fluorinated pyrimidine nucleoside analog, 18 
Fluoroquinolones, 15/ 

for Escherichia coli, 78 

for Haemophilus influenzae, 92 

for Klebsiella pneumoniae, 98 

for Neisseria gonorrhoeae, 124 

for Pseudomonas aeruginosa, 138 

for Salmonella, 144 

for Shigella, 145 

for Streptococcus pneumoniae, 152 

for typhoid fever, 144 

for Vibrio vulnificus, 174 
Fomites 

in adenovirus transmission, 179 

definition, 355 

in Molluscum contagiosum transmission, 
231 

in mumps virus transmission, 237 

in rotavirus transmission, 255 
Fomivirsen, 17/ 
Food poisoning 

Bacillus cereus, 28 

Campylobacter jejuni, 46 

Listeria monocytogenes, 102, 104 

Salmonella, 142 

Staphylococcus aureus, 155-161 
Foscarnet, lit, 186 
Fulminant hepatitis, 197 
Fungi, 5 

antifungal agents for, 19—21, 19/ 

Aspergillus, 277-282, 280/ 

Blastomyces dermatitidis, 285, 286 

Candida albicans, 289-294, 292/ 

Coccidioides immitis, 283-286 

Cryptococcus neoformans, 285, 
295-300 

dematiaceous, 308 

Histoplasma capsulatum, 285, 286 

Paracoccidioides braziliensis, 285 

Pneumocystis jiroveci, 301—305 

Sporothrix schenckii, 285, 307-311 
Furuncles, 29 
Fusion inhibitors, 16/ 



Gamete, Cryptosporidium, 324 
Gametocyte, Plasmodium, 340 
Gammaherpesvirinae, 190 
Ganciclovir, 17/, 18, 186 
Gardasil, 225 
Gastric biopsy, 84 
Gastric ulcers, 83-84, 86 
Gastritis, type B, 85 



372 



INDEX 



Gastroenteritis 
adenovirus in, 179 

Campylobacter jejuni in, 45-50 

differential diagnosis, 47 

Escherichia coli in, 76 

Salmonella and Shigella in, 141-146 

Staphylococcus aureus in, 155-161 

in typhoid fever, 142 

Vibrio parahaemolyticus in, 172 
Gastrointestinal tract 

antifungal effects on, 20 

Ascaris lumbricoides and, 313—321, 314/ 

Campylobacter jejuni and, 45-50 

cholera and, 171-176 

Clostridium difficile and, 57-61 

Cryptosporidium parvum and, 321-328 

Escherichia coli and, 76 

rotavirus and, 253—258 

Salmonella and Shigella infection and, 
141-146 

Schistosoma haematobium and, 345—351 
Gatifloxacin, 15? 

Genital herpes. See Herpes simplex virus 
Genital warts. See Human papillomavirus 
Ghost cell, 109 
Giant cells 

herpes simplex virus and, 210 

varicella-zoster virus and, 273 
Giant intestinal roundworm. See Ascaris 

lumbricoides 
Giardiasis, antiparasitic agents for, 22? 
Giemsa stain, for Pneumocystis jiroveci, 303 
Gingivostomatitis, in herpes simplex infection, 

208, 209 
Gonorrhea, 121-126 
gp41 product, HIV, 216 
gpl20 viral receptor, HIV, 216 
gpl60 product, HIV, 216 
Gram-negative bacteria, 5, 5/ 

aztreonam for, 13, 15/ 

Bacteroides fragilis, 33-38 

Campylobacter jejuni, 45-50 

cephalosporins for, 14? 

Chlamydia trachomatis, 51-56 

Escherichia coli, 75-81 

Haemophilus influenzae, 89-94 

Helicobacter pylori, 83-87 

Klebsiella pneumoniae, 95—100 

Neisseria, 121—126 

Proteus species, 129-133 

Pseudomonas aeruginosa, 135-140 

Salmonella, 141-147 

Shigella, 141-147 

Treponema pallidum, 163—170 

Vibrio c holer ae, 171-176 
Gram-positive bacteria, 5, 5/ 

Bacillus anthracis, 27-32 

cephalosporins for, 14? 

Clostridium difficile, 57-61 

Corynebacterium diphtheriae, 63-67 

Enterococcus faecalis, 69-73 

Listeria monocytogenes, 101-105 

Staphylococcus aureus, 155-161 



Streptococcus species, 149-154 

vancomycin for, 13 
Grand-mal seizure, 91 
Granuloma, mycobacterial, 109 
Granulomatosis infanti septic a, 102 
Granulomatous lesions, in Ascaris 
lumbricoides infection, 314 
Gravida, definition, 103 
Griseofulvin, 19, 19?, 20-21 
Guarnieri inclusions, 231, 267 
Guillain-Barre syndrome, 46, 47, 49 
Gumma, 167 

H 

H antigen, Escherichia coli, 78, 78/ 
HAART (highly active an tiretro viral therapy), 

215,217 
Haemophilus aphrophilus, 90 
Haemophilus ducreyi, 90, 92 
Haemophilus influenzae, 89-94 
Haemophilus parainfluenzae, 90 
Haemophilus paraphrophilus, 90 
Hairy leukoplakia, 192 
Head and neck tumors, human papillomavirus 

and, 222? 
Headache 

in abortive poliomyelitis, 250 

amphotericin B-related, 20 

in cryptococcal meningitis, 297 

famciclovir-related, 18 

griseofulvin-related, 20 

in Haemophilus influenzae meningitis, 91 

in hepatitis, 197 

in Klebsiella pneumonia pneumonia, 96 

in Lyme disease, 40 

in Mycoplasma pneumoniae pneumonia, 
117 

retro vir-related, 18 

in typhoid fever, 142 
Heart failure, itraconazole-related, 20 
Heat shock protein, Helicobacter pylori, 84 
Hektoen enteric agar, 144 
Helicobacter pylori, 83-87 
Helminths, 5 

antiparasitic agents for, 22? 

Ascaris lumbricoides, 313-320 

Enterobius vermicularis (pinworm), 315/ 
329-335, 333? 

life cycle, 314/ 316 
Hemagglutinin, in adenovirus, 179 
Hemagglutinin-neuraminidase protein, 236 
Hematin medium, for Haemophilus 

influenzae, 91 
Hematoxylin-eosin stain, for Helicobacter 

pylori, 86 
Hematuria, in schistosomiasis, 345-346 
Hemoconcentration, in cholera, 173 
Hemolysin, 131 
Hemolysin production 

Escherichia coli, 77—78 

Proteus species, 131 
Hemolytic anemia, in Epstein-Barr virus 
infection, 192 



INDEX 



3 73 



Hemolytic uremic syndrome (HUS) 

Escherichia coli and, 76, 77 

shiga toxin and, 144 
Hepadnaviruses, 199 
Hepatitis A virus immunoglobulin, 199 
Hepatitis B surface antigen, 199, 200/ 202 
Hepatitis viruses, 195-204 

antiviral for, 16? 

comparison, 198? 

hepatitis A, 197-199 

hepatitis B, 199-200, 200/ 

hepatitis C, 16?, 195-196,200-201 

hepatitis D, 201-202 

hepatitis E, 197,202 

hepatitis G, 202 

serology, 202-203 
Hepatocellular carcinoma 

hepatitis B and, 199-200 

hepatitis C and, 201 
Hepatomegaly 

in malaria, 338 

in schistosomiasis, 346 
Hepatosplenomegaly 

in cytomegalovirus infection, 185 

definition, 143, 184 

in Epstein-Barr virus infection, 190, 192 

in typhoid fever, 142 
Hepatotoxicity, of antifungal agents, 20 
Herpes encephalitis, 18, 209 
Herpes simplex vims, 19k, 207-212, 210/ 
Herpes zoster (shingles), 18, 271—272 
Herpesviruses, 4/ 19 If 

antiviral agents for, 17-18, 17? 

cytomegalovirus, 183-187 

Epstein-Barr virus, 189-194 

herpes simplex virus, 19 If, 207-212, 210/ 

varicella-zoster virus, lit, 18, 191/, 271-275 
Herpetic whitlow, 208, 209 
Heterophile antibodies, 190, 192 
Highly active antiretroviral therapy (HAART), 

215,217 
Histoplasma capsulation, 285, 286 
HMG CoA reductase inhibitors, azole 

antifungals and, 20 
Hodgkin disease, Epstein-Barr virus and, 192 
Hookworm, 3 14/ 

Human immunodeficiency virus (HIV) 
infection, 213-220 

antiviral agents for, 16, 16? 

candidiasis in, 290 

Coccidioides infection in, 284 

Cryptococcus neoformans infection in, 
295-300 

Cryptosporidium parvum infection in, 
321-322 

Mycobacterium infection in, 108 

Pneumocystis jiroveci infection in, 301-305 
Human papillomavirus (HPV), 221—226, 222?, 

224/ 
Hyaluronidase, Clostridium difficile production 

of, 59 
Hydrogen peroxide, pneumococcal production 
of, 151 



Hydronephrosis, 131 
Hydrops fetalis, 242 
Hyperbilirubinemia, in hepatitis, 197 
Hyphae, 5 

of Aspergillus, 280, 280/ 

of Sporothrix, 309 
Hypnozoites, Plasmodium, 340 
Hypokalemia, in cholera, 171, 173 
Hypotension, amphotericin B-related, 20 
Hypovolemia, in cholera, 173 
Hypoxia 

definition, 303 

in Pneumocystis pneumonia, 302 

in respiratory syncytial virus infection, 261 

I 

Icosadeltahedral nucleocapsid. See 

Nucleocapsid, icosadeltahedral 
Idoxuridine, 17?, 18 
Imipenem, 13, 15? 

for Bacteroides fragilis, 36 

for Klebsiella pneumoniae, 98 

for Pseudomonas aeruginosa, 138 
Immune globulin, for cytomegalovirus, 186 
Immune stimulant therapy, for human 

papillomavirus, 225 
Immunofluorescence testing 

for Borrelia burgdorferi, 42 

for mumps virus, 237 

for Mycoplasma pneumoniae, 117 

for Pneumocystis jiroveci, 303 

for respiratory syncytial virus, 262 
Immunoglobulin 

anti-RSV, 262 

varicella-zoster, 274 
Immunoglobulin A protease, 92-93, 151 
Immunoglobulin G antibody 

mumps and, 238 

parvovirus B19 and, 244 
Immunoglobulin M antibody 

Epstein-Barr virus and, 192 

hepatitis A virus and, 199 

hepatitis D virus and, 202 

mumps virus and, 238 

parvovirus B19 and, 244 
Inactivated poliomyelitis vaccine (IPV), 

248-249 
Inclusion bodies 

in Chlamydia trachomatis, 53, 54 

Guamieri, 231, 267 

in Molluscum contagiosum, 231 

in smallpox, 267 
India ink stain 

for anthrax, 30 

for Cryptococcus neoformans, 298 
Indinavir, 16? 
Indirect immunofluorescence, for Borrelia 

burgdorferi, 42 
Indole test, for Escherichia coli, 78 
Infant botulism, 58 
Infants 

chlamydial conjunctivitis in, 54 

cytomegalovirus infection in, 185 



3 74 



INDEX 



Infants (Cont.): 

diaper dermatitis in, 290 

parvovirus B19 infection in, 24 

respiratory syncytial virus infection in, 
259-262 

Treponema pallidum infection in, 165 
Infectious mononucleosis, 189-194 
Infectious subviral particle, 255 
Influenza, antiviral agents for, 16, 16? 
Inhalation anthrax, 29 
Insect repellents, 42 
Interferon alfa, 16/, 196 
Intermediate host, 347 
Internalin, Listeria monocytogenes production 

of, 103 
Interstitial infiltrates 

in aspergillosis, 299 

in Pneumocystis pneumonia, 302 
Interstitial lymphocytic pneumonia, Epstein- 

Barr virus infection and, 192 
Intestinal schistosomiasis, 347 
Intracerebral calcifications, cytomegalovirus 

and, 185 
Intravenous drug use, hepatitis C infection 

and, 196 
Intussusception, 254, 256 
Invirase, 19 

Iodinated thymidine analog, 18 
Iodoquinol, 22/ 
Irritable bowel syndrome, 47 
Isoniazid, for tuberculosis, 1 10-1 1 1 
Isospora belli, 323, 324, 325/ 
Itraconazole, 19r, 20 

for invasive aspergillosis, 281 

for Sporothrix schenckii, 309 
Ixodes tick, 40, 42 

J 

Jaundice 

cytomegalovirus and, 185 

in hepatitis, 197 
Jembec medium, 124 
Joint pain, in Lyme disease, 40 

K 

Kaposi sarcoma, 213-214 
Kaposi sarcoma-related virus, 19 If 
Katayama syndrome, 346—347 
Keratitis, Aspergillus in, 278 
Keratoconjunctivitis, in herpes simplex virus 

infection, 209 
Ketoconazole, 19/ 
Ketolides, 15/ 
Kidney stones, 131 
Kinyoun stain, for Mycobacterium, 

109, 110 
Kissing disease. See Infectious 

mononucleosis 
Kissing lesion, 231 
Klebsiella oxytoca, 98 
Klebsiella ozaenae, 96 
Klebsiella pneumoniae, 95-100 
Klebsiella rhinoscleroma, 96 



KOH preparation 
for Aspergillus, 280 
for Candida, 291 
for Coccidioides, 285 

Koilocyte, 223 



Lamivudine, 16/ 
Langhans cells, 108, 109 
Larva 

Ascaris lumbricoid.es, 316-317 

Enterobius vermicular is, 331 

helminths, 315/ 316 

Schistosoma haematobium, 348 
Larvate egg, 331 
Laryngeal obstruction, in Epstein-Barr virus 

infection, 192 
Laryngeal papilloma, 222/, 224 
Latex agglutination 

for Cryptococcus neoformans, 298 

for rotavirus, 256 
Lecithinase, of Bacillus anthracis, 29-30 
Leishmaniasis, antiparasitic agents for, 22/ 
Lentivirinae, 214 
Leprosy, 1 1 1 
Leptospira, Alt 

Lethal factor, anthrax toxin, 29 
Leukocyte(s) 

fecal, 47, 143, 144 
Leukocyte esterase, 70 

Enterococcus faecalis and, 71 

Escherichia coli and, 78 

Proteus species and, 131 
Levofloxacin, 15/ 
Lincomycins, 15/ 
Linezolid, 15/, 72 
Lipopolysaccharide, 78/ 175 
Liquid nitrogen, for Molluscum contagiosum, 

231 
Listeria monocytogenes, 101-105 
Liver 

antifungal agents and, 20 

cytomegalovirus infection and, 185 

Epstein-Barr virus infection and, 190, 192 

hepatitis and. See Hepatitis viruses 

malaria and, 338 

schistosomiasis and, 346 

typhoid fever and, 142 
Lockjaw, 58 
Lowen stein- Jen sen agar, for Mycobacteria, 

109, 110 
Lung 

diphtheria and, 64 

interstitial infiltrates, 279, 302 

normal flora, 90, 122 

pneumonia. See Pneumonia 

pneumonitis, hi Ascaris lumbricoid.es 
infection, 317 
Lyme disease, 39^14 
Lymphadenitis, 190, 308 
Lymphadenopathy 

definition, 178 

in diphtheria, 64 



INDEX 



375 



in Epstein-Barr virus infection, 192 

in Haemophilus ducreyi infection, 90 

in herpes simplex virus infection, 209 

in schistosomiasis, 346 

in typhoid fever, 142 
Lymphocytes, atypical, 190 
Lymphocytosis, 190 
Lymphogranuloma venereum, 54 
Lymphoid tissue 

adenovirus and, 179 

poliovirus and, 249 
Lympho trophic (definition), 184 
Lysogenic bacteriophage, 65 

M 

M cell, of Salmonella, 142, 143 
MacConkey agar 

for Escherichia coli, 78 

for Klebsiella pneumoniae, 97 

for Proteus species, 131 

for Pseudomonas aeruginosa, 138 

for Salmonella, 144 

for Shigella, 144 
Macrogamete, Cryptosporidium, 324 
Macrogametocyte, Plasmodium, 340 
Macrolide antibiotics, \5t 

for Haemophilus influenzae, 92 

for Streptococcus pneumoniae, 152 
Macule, 165 
Maculopapular rash 

with amoxicillin, in Epstein-Barr virus 
infection, 192 

definition, 267, 268 

in parvovirus B19 infection, 243 

in smallpox, 268 

in syphilis, 166 

in typhoid fever, 142 
Malaise 

in Campylobacter jejuni gastroenteritis, 45^-6 

in cryptosporidiosis, 322 

in Epstein-Barr virus infection, 192 

in hepatitis C, 201 

in Klebsiella pneumonia pneumonia, 96 

in mumps, 237 

in Mycoplasma pneumoniae pneumonia, 1 17 

in smallpox, 268 
Malaria, 337-344, 341?, 342? 

antiparasitic agents for, 22? 

Burkitt lymphoma and, 192 
Malignant otitis externa, 135-136 
Malignant tertian malaria, 338 
Martin Lewis medium, 124 
Mebendazole, 22t 

for Ascaris lumbricoides, 318 

for pinworm infection, 332 
Mefloquine, 22?, 341 
Meningitis, 91, 296 

aseptic, 250 

Campylobacter fetus and, 46 

candidal, 290 

Cryptococcus neoformans in, 295-300 

Epstein-Barr vims in, 192 

fungal, 295-300 



in granulomatosis infantiseptica, 102 

Haemophilus influenzae in, 89-94 

Listeria monocytogenes in, 101-105 

poliovirus in, 250 

Proteus species in, 130 

Pseudomonas in., 136 

Streptococcus pneumoniae in, 150 
Meningococcemia, 124 
Mental retardation, cytomegalovirus and, 185 
Meronts, 324 
Meropenem, 13, 15?, 98 
Merozoites 

Cryptosporidium, 324 

Plasmodium, 340 
Metabolic acidosis, in cholera, 173 
Methacycline, 15? 
Methenamine silver stain, for Pneumocystis 

jiroveci, 303 
Methisazone, 268 
Metronidazole, 22? 

for Bacteroides fragilis, 36 

for Clostridium difficile, 60 

for trichomoniasis, 354, 355 
Mezlocillin, 14? 
Miconazole, 19? 
Microaerophilic organisms 

Borrelia burgdorferi, 39^14, 41? 

Campylobacter jejun i, 45-50 

definition, 85, 165 

Helicobacter pylori, 83-87 

Treponema pallidum, 163—170 
Microbiology 

approach to disease, 6 

approach to learning, 3—5 

approach to reading, 6-9 
Microcephaly, in cytomegalovirus infection, 

184, 185 
Micrococcaceae, 157 
Microgamete, Cryptosporidium, 324 
Microgametocyte, Plasmodium, 340 
Microhemagglutination test, Treponema 

pallidum, 164 
Minocycline, 15? 
Miracidium, 347, 348 
Mitral valve endocarditis. See Endocarditis 
MMR (measles, mumps, rubella) vaccine, 

235-236 
Molds, 5. See also Fungi 
Molluscum bodies, 230, 231 
Molluscum contagiosum, 229-233 
Monobactam antibacterials, 13, 15? 
Monoclonal antibody testing 

for anthrax, 30 

for Pneumocystis, 303 
Monospot test, for Epstein-Barr virus, 193 
Motor cortex, poliovirus and, 250 
Moxifloxacin, 15? 

for Klebsiella pneumoniae, 98 
Mucinase, Helicobacter pylori 'production of, 85 
Mucous membranes 

herpes simplex virus and, 209 

human papillomavirus and, 223 

secondary syphilis and, 166 



3 76 



INDEX 



Multiple drug resistance, 13. See also 

Resistance, antibacterial 
Mumps, 235-239 
Mupirocin, intranasal, 159 
Muscle cells, poliovirus and, 249 
Myalgia 

in hepatitis, 197 

in Klebsiella pneumonia pneumonia, 96 

in smallpox, 268 

in typhoid fever, 142 
Mycobacterium abscessus, 109, 110 
Mycobacterium africanum, 109 
Mycobacterium avium-intracellulare, 108—111 
Mycobacterium bovis, 109, 111 
Mycobacterium cheionae, 109, 110 
Mycobacterium fortuitum, 109, 110 
Mycobacterium gordonae, 110 
Mycobacterium haemophilum, 109, 110 
Mycobacterium kansasii, 108, 110 
Mycobacterium leprae, 108, 109 
Mycobacterium szulgai, 109 
Mycobacterium tuberculosis, 107—113 
Mycobacterium ulcerans, 109 
Mycoplasma hominis, 117—118 
Mycoplasma pneumoniae, 115-120 
Mycotoxin, Aspergillus species, 279 
Myelosuppression 

fluey to sine-related, 18 

ganciclovir-related, 18 
Myocarditis, diphtheria and, 64 

N 
NAD medium, 91 
Nafcillin, 14/, 159 
Naftifine, 19?, 20 
Nasopharyngeal carcinoma, Epstein-Barr virus 

and, 192 
Natamycin, 19/ 
Natural penicillins, 13, 14/ 
Nausea 

in cryptosporidiosis, 322 

in hepatitis, 197 

in Salmonella infection, 142 
Necrosis, in diphtheria, 65 
Necrotizing fasciitis, streptococcal, 150 
Neisseria gonorrhoeae, 121-126 
Neisseria meningitidis, 122, 123 
Nelfinavir, 16/ 
Nemathelminths, 5 
Nematodes. See Helminths 
Nephrolithiasis, 131 
Nephrotoxicity 

of amphotericin B, 19-20 

of foscarnet, 18 

of vidarabine, 18 
Neuropathic pain, 272 
Neutropenia, 29 1 
Nevirapine, 16/ 
Newborn 

Chlamydia trachomatis infection in, 54 

granulomatosis infantiseptica in, 102 

hepatitis B in, 199-200 

herpes simplex virus infection in, 209-210 



Listeria monocytogenes infection in, 
101-105 ' 

Neisseria gonorrhoeae conjunctivitis in, 
122, 123 

Proteus meningitis in, 130 
Nitazoxanide, for Cryptosporidium parvum, 

325 
Nocturnal migration, 331 
Nonchromogens, 109 
Nongonococcal urethritis, Mycoplasma 

hominis in, 117-118 
Nonnucleoside reverse transcriptase inhibitors, 

16/ 
Nonparalytic poliomyelitis, 249 
Normal flora 

Bacteroides fragilis, 34 

Enterococcus faecalis, 71 

Escherichia coli, 76 

Haemophilus species, 90 

Neisseria meningitidis, 122 

Proteus species, 130 
Nosocomial infection 

Enterococcus faecalis, 70 

Klebsiella pneumoniae, 96 

Pseudomonas aeruginosa, 136 

respiratory syncytial virus, 261 
Nuchal rigidity, 91,296 

Nuclear antigens, Epstein-Barr virus, 191-192 
Nucleocapsid, icosadeltahedral 

of adenovirus, 178 

of cytomegalovirus, 184 

of Epstein-Ban - virus, 191 

of herpes simplex virus, 209 

of poliovirus, 249 
Nucleoside reverse transcriptase inhibitors, \6t 
Nucleotide inhibitors, 16/ 
Nystatin, 19/, 20,21,292 

O 

O antigen 

of Escherichia coli, 78, 78/ 

of Vibrio cholerae, 173 
Obligate pathogens 

Chlamydia trachomatis, 51-56, 53/ 

Treponema pallidum, 163—170, 166/ 
Obtunded (definition), 173 
Occult blood, in gastroenteritis, 144 
Ocular trachoma, 53-54 
Ofloxacin, 15/ 
Onychomycosis, 20 
Oocyst 

of Cryptosporidium parvum, 323—324, 
325/, 326/ 

of Cyclospora, 324, 325/ 

of Isospora, 324, 325/ 
Oophoritis, 236 

Opa proteins, of Neisseria gonorrhoeae, 123 
Ophthalmia neonatorum, 123 
Opportunistic infections 

Aspergillus fumigatus, 277—282, 280/ 

Klebsiella pneumoniae, 95—100 

Pneumocystis jiroveci, 301—305 

Pseudomonas aeruginosa, 29, 135-140 



INDEX 



377 



Optochin susceptibility, of streptococci, 152 

Oral cavity, thrush in, 291 

Oral papilloma, 222? 

Oral polio vaccine (OPV), 248-249 

Oral rehydration therapy 
for cholera, 174 
for cryptosporidiosis, 325 
for rotavirus infection, 256 

Orchitis, 236 

Orthomyxoviridae, 4/ 

Orthopoxvirus, 267 

Oseltamivir, 16? 

Otitis externa, Pseudomonas aeruginosa in, 
135-136, 138 

Otitis media, 260 

Haemophilus species in, 90 
respiratory syncytial virus in, 259 
Streptococcus pneumoniae in, 150 

Owl's eyes, 185 

Oxacillin, 14? 

Oxazolidones, 15? 

Oxidase 

Helicobacter pylori and, 85 
Neisseria gonorrhoeae and, 124 

Oxygen tolerant anaerobes, 35 

Oxy tetracycline, 15? 



p24 protein, HIV, 216 

Pain 

abdominal. See Abdominal pain 

arthralgia See Arthralgia 

joint, 40 

neuropathic, 272 
Pancreatic damage 

didano sine-related, 18 
PantonValentineLeukocidin (PVL), 156 
Papanicolaou (Pap) smear 

for cervical neoplasia detection, 221-222, 
225 

for Trichomonas vaginalis, 354 
Papilloma, 223, 224, 224/ 
Papillomaviridae, 223 
Papovaviridae, 4/ 
Papule, 52 

in cutaneous diphtheria, 64 

in Molluscum contagiosum, 230 
Para, definition, 103 
Paracoccidioides braziliensis, 285 
Paralysis, in diphtheria, 64 
Paralytic poliomyelitis, 249 
Paramyxoviridae, 4/ 237, 261 
Parasites, 5 

antiparasitic agents for, 21-22, 22? 

Ascaris lumbricoides, 313-320, 314/ 319? 

Chlamydia trachomatis, 51-56, 53? 

Cryptosporidium parvum, 22?, 321-328, 
325?, 326? 

Enterobius vermicularis, 315/ 329-335, 
333? 

Plasmodium species, 337-344, 341?, 342? 

Schistosoma species, 345-351, 349? 

Trichomonas vaginalis, 353-358, 356? 



Paromomycin, 22? 

Parotitis, 236 

Parvoviridae, 4/ 

Parvovirus B 19, 241-246 

Paul-Bunnell antigen, Epstein-Barr virus 

and, 192 
PCR. See Polymerase chain reaction 
Pelvic inflammatory disease 

Chlamydia trachomatis in, 51-56 

Mycoplasma hominis in, 117 

Neisseria gonorrhoeae infection and, 122 
Penciclovir, 17?, 18 
Penicillin(s), 13, 14?, 15? 

mechanisms of action, 13 

for meningococcemia, 124 

for Neisseria gonorrhoeae, 124 

for pneumococcal disease, 152 

for Proteus species, 132 

for Pseudomonas, 138 

for syphilis, 41?, 167 
Penicillin binding protein (PBP2), of 

Staphylococcus aureus, 158 
Penicillin G, 14?, 42 
Penicillin resistance, pneumococci, 152. See 

also Resistance, antibacterial 
Penicillin V, 14? 
Pentamidine, 22?, 303 
Periauricular (definition), 136 
Peripheral neuropathy, didanosine-related, 18 
Peritonitis 

Bacteroides fragilis in, 33-38 

Campylobacter fetus in, 46 
Petechiae, 242 
Phagocytosis 

Bacteroides fragilis, 35 

Streptococcus, 151 
Pharyngeal candidiasis, 290 
Pharyngeal diphtheria, 63-67 
Pharyngitis 

in abortive poliomyelitis, 250 

in adenovirus infection, 179 

in Epstein-Barr virus infection, 190, 192 

in hepatitis, 197 

in herpes simplex virus infection, 209 

streptococcal, 150 
Phospholipase enzymes, of Helicobacter 

pylori, 85 
Picornaviridae, 4/ 

hepatitis A, 197-199 

poliovirus, 248-252 
Pili, of Bacteroides fragilis, 123 
Pinworm (Enterobius vermicularis), 315/ 

329-335, 333? 
Piperacillin, 14? 

for Bacteroides fragilis, 36 

for Pseudomonas aeruginosa, 138 
Piperazine, for Ascaris lumbricoides, 318 
Plasmodium species, 337-344, 341?, 342? 
Platy helminths, 5 
Pleural effusion, in pneumococcal pneumonia, 

150 
Pneumocystis carinii. See Pneumocystis 
jiroveci 



3 78 



INDEX 



Pneumocystis jiwveci, 22t, 301—305 

Pneumolysin, 151 

Pneumonia 

Aspergillus, 278 

atypical, 54 

Chlamydia trachomatis, 54 

immunization, 152 

interstitial lymphocytic, 192 

Klebsiella pneumoniae, 95—100 

Mycoplasma pneumoniae, 1 15-120 

neonatal, 54 

Pneumocystis jiwveci, 30 1—305 

Streptococcus pneumoniae, 150 
Pneumonitis, in Ascaris lumbricoides infection, 

317 
Pneumovirus, 261 
Poikilocytosis, 223 
Poliomyelitis, 247-252 
Poliovirus, 248-252 
Polyene macrolides, 19r 
Polymerase chain reaction (PCR). See also 

Reverse transcriptase polymerase chain 
reaction 

for adenovirus, 179 

for cytomegalovirus, 185 

for hepatitis C, 196 

for parvovirus B19, 244 
Polyribitol phosphate capsule, of Haemophilus 

influenzae, 91 
Polysaccharide capsule 

of Cryptococcus neoformans, 297 

of Haemophilus influenzae, 91 

of Klebsiella pneumoniae, 97 

of Neisseria meningitidis, 122 

of Pseudomonas aeruginosa, 137 

of Streptococcus pneumoniae, 151 
Por proteins, of Neisseria gonorrhoeae, 123 
Porphyromas, 35 

Portal hypertension, schistosomiasis and, 347 
Portal vein, schistosomula and, 348 
Positive tilt test, 142 
Postpartum fever, Mycoplasma hominis in, 

117-118 
Potassium hydroxide preparation. See KOH 

preparation 
Potassium iodide, for Sporothrix schenckii, 309 
Poxviruses, 4/ 

Molluscum contagiosum, 229-233 

smallpox, 265-270 
PPD (purified protein derivative), 109, 110 
Praziquantel, 22t, 348 
Preauricular adenopathy, 178 
Pregnancy 

cytomegalovirus infection during, 185 

hepatitis E during, 202 

parvovirus B19 infection during, 24 

Treponema pallidum during, 165 
Prevotella, 35 
Primaquine, 22f, 341 
Primary syphilis, 165—166 
Prodrome, herpes simplex virus infection, 208 
Prokaryote, 41 



Prophylaxis 

antiparasitic agents for, 22t 

for candidiasis, 292 

for malaria, 341 

for Pneumocystis carinii, 303 
Prostatitis, Trichomonas vaginalis in, 354 
Protease inhibitors, 16, \6t, 19 
Protective antigen, anthrax toxin, 29 
Protein synthesis 

Corynebacterium diphtheriae inhibition 
of, 65 

Pseudomonas aeruginosa inhibition of, 137 
Protein tegument layer, capsid. See Tegument 

layer, capsid 
Proteus mirabilis, 129-133 
Proteus vulgaris, 132 
Proton pump inhibitors, for Helicobacter 

pylori, 86 
Protozoa, 5 

Cryptosporidium parvum, 321-328 

Trichomonas vaginalis, 353-358, 356? 
Pruritus, in pinworm infection, 1 16 
Pseudohyphae, 5, 291, 292/ 
Pseudo membrane, 65 

Clostridium difficile, 59 

Corynebacterium diphtheria, 64, 65 
Pseudomembranous colitis, 58, 59 
Pseudomonas aeruginosa, 29, 135—140 
Psittacosis, 54 
Pulmonary infiltrates 

in aspergillosis, 279 

in Pneumocystis jiwveci pneumonia, 302 
Purified protein derivative (PPD), 109, 110 
Pyelonephritis, 77. See also Urinary tract 
infection 

Escherichia coli in, 76 

Mycoplasma hominis in, 117 
Pyocyanin, 137 
Pyorubin, 137 
PYRtest, 71 

Pyrantel pamoate, 22t, 332 
Pyrazinamide, 111 
Pyrimethamine, 22t, 341 



Quartan malaria, 338 

Quinidine, 22/ 

Quinine, 22r, 341 

Quinolones. See Fluoroquinolones 

Quinupristin/dalfopristin, 72 

R 

Rapid plasmin reagin (RPA), for Treponema 
pallidum, 164, 167 

Rash 

in Coccidioides immitis infection, 284 

in cytomegalovirus infection, 185 

in granulomatosis infantiseptica, 102 

in Haemophilus influenzae meningitis, 91 

in Lyme disease, 39^14 

in parvovirus B19 infection, 243 

in shingles, 271-275 



INDEX 



3 79 



in smallpox, 265-270 

in syphilis, 163—165 

in typhoid fever, 142 
Reassortment, viral, 254 
Receptor- mediated endocytosis, 179, 180/ 
Recombinant interferon. See Interferon alfa 
Reduction- modifiable (Rmp) proteins, 

Neisseria gonorrhoeae, 123 
Relapse malaria, 339 
Reoviridae, 4/ 255 
Repellants, tick, 42 
Resistance, antibacterial 

Bacteroides species, 36 

Enterococcus faecalis, 71 

Escherichia coli, 79—80 

Klebsiella pneumoniae, 97 

Listeria, 104 

mechanisms, 13 

multiple drug, 13 

Mycoplasma pneumoniae, 117 

Neisseria gonorrhoeae, 123 

pneumococci, 152 

Proteus vulgaris, 132 

Pseudomonas, 137 

Staphylococcus aureus, 158 
Respiratory diphtheria, 63-67 
Respiratory distress, in respiratory syncytial 

virus infection, 261 
Respiratory flora, normal 

Haemophilus species, 90 

Neisseria meningitidis, 122 
Respiratory infections 

adenovirus, 4/ 177-182, 180/ 

Aspergillus fumigatus, 277—282, 280/ 

Coccidioides immitis, 283-288 

Cryptococcus neoformans, 285, 295-300 

Haemophilus species, 89-94 

Klebsiella species, 95-100 

Mycobacterium species, 107-113 

Mycoplasma pneumoniae, 1 15-120 
Respiratory syncytial virus (RSV), 

259-264 
Reticulate body, C. trachomatis, 53, 53? 
Retrovir, 18 
Retroviruses, 4/ 215 
Reverse transcriptase, human 

immunodeficiency virus, 215 
Reverse transcriptase inhibitors, 16, 217 
Reverse transcriptase polymerase chain 
reaction (RT-PCR). See also 
Polymerase chain reaction 

for hepatitis C vims, 201 

for human immunodeficiency virus, 214, 
216 

for respiratory syncytial vims, 262 
Rhabdoviridae, 4/ 
Rheumatic fever, 150 
Rhonchi, 116 
Ribavirin, 16? 
Rifampin 

for leprosy, 1 1 1 

for staphylococcal infection, 159 

for tuberculosis, 1 1 1 



Right otitis media, 256 

Rimantadine, 16, 16f 

Ring stage, Plasmodium, 339 

Ritonavir, \6t 

Rmp (reduction-modifiable protein), Neisseria 

gonorrhoeae, 123 
RNA viruses, 4/ 

hepatitis A vims, 197-199 

hepatitis D vims, 201-202 

hepatitis E virus, 197, 202 

human immunodeficiency virus. See Human 
immunodeficiency virus 

mumps virus, 235—239 

poliovirus, 248-252 

respiratory syncytial virus, 259-264 

rotavirus, 253-258 
Rose spots, in typhoid fever, 143 
Rotavirus, 253-258 
Roundworms, 5 

RSV (respiratory syncytial virus), 259-264 
RT-PCR. See Reverse transcriptase polymerase 

chain reaction 
Runyon categories, Mycobacterium, 109 



Sabouraud agar 

for Aspergillus, 280 

for Candida, 291 

for Sporothrix schenckii, 309 
Saline wet mount, for Trichomonas vaginalis, 

354, 355 
Salmonella, 47, 141-147 
Salmonella paratyphi, 142, 143 
Salmonella typhi, 142, 143 
Salmonella-secreted invasion proteins, 

143 
Saquinavir, 16f, 19 
Sarcomastigophora, 5 
Schick test, 287 
Schistosoma haematobium, 22t, 345-351, 

349r 
Schistosoma intercalatum, 346 
Schistosoma japonicum, 346, 348, 349? 
Schistosoma mansoni, 346, 348, 349? 
Schistosomula, 348 
Schizogony, 339 
Schizonticidal compounds, 341 
Schliffner dots, 340 
Scotch tape test, 332 
Scotochromogens, mycobacterial, 109 
Secondary syphilis, 163-164, 166, 166/ 
Secretory IgA protease, 151 
Seizure 

in cryptococcal meningitis, 297 

in Haemophilus influenzae meningitis, 
89,91 
Sepsis 

Campylobacter gastroenteritis and, 46 

Escherichia coli and, 76 

Pseudomonas aeruginosa and, 137 
Septic arthritis 

Campylobacter fetus and, 46 

Neisseria gonorrhoeae and, 121-126 



380 



INDEX 



Serologic testing 

for Borrelia burgdorferi, 42 

for Coccidioides immitis, 286 

for Helicobacter pylori, 86 

for hepatitis viruses, 202—203 

for human immunodeficiency virus, 216-217 

for mumps virus, 237—238 

for Mycoplasma pneumoniae, 117 

for parvovirus B19, 244 

for syphilis, 164, 167 
Serotyping 

herpes simplex virus, 210 

Vibrio cholerae, 173 
Sexually transmitted diseases 

chancroid, 90 

Chlamydia trachomatis, 51-56, 53f 

hepatitis B, 199-200, 200/ 

hepatitis C, \6t, 195-196,200-201 

human papillomavirus (HPV), 221-226, 
222/, 224/ 

syphilis, 163-170 

Trichomonas vaginalis, 353-358, 356f 
Sheep blood agar, 9 1 
Shellfish, Vibrio infections and, 173 
Shiga toxin, 144 
Shigella, 47, 143-147 
Shigella boydii, 144 
Shigella dysenteriae, 143—144 
Shigella flexneri, 144 
Shigella sonnei, 144 
Shingles (herpes zoster), 271-275 
Sickle cell disease, parvovirus B19 infection 

in, 244 
Silicosis, 108 
Sinusitis 

Aspergillus in, 278 

Bacteroides fragilis in, 34 

Streptococcus pneumoniae in, 150 
Skin lesion 

in ecthyma gangrenosum, 136 

in gonorrhea, 122 

in herpes simplex virus infection, 
209-210,210/ 

in Molluscum contagiosum, 231 

in Sporothrix schenckii infection, 307-309 

in syphilis, 163—167 
Skirrow medium, 47 
Slapped cheek appearance, in parvovirus B19 

infection, 243 
Smallpox, 265-270 
Snail, in schistosomiasis, 348 
Sore throat. See Pharyngitis 
Sorivudine, 18 
Spectinomycin, \5t 
Spherule, fungal, 284 
Spider bites, anthrax vs., 29 
Spinal cord, poliovirus and, 248-252 
Spirochetes, 40, 41r 

Borrelia burgdorferi, 39^14, 41r 

Leptospira, 41? 

Treponema pallidum, 163—170 
Splenic rupture, in Epstein-Barr virus 
infection, 192 



Splenomegaly. See also Hepato splenomegaly 

in malaria, 338 

schistosomiasis and, 347 
Sporangium, 294 
Spore-forming bacteria 

Bacillus anthracis, 27-32 

Clostridium difficile, 57-61 
Sporocyst 

Cyclospora, 324 

Pneumocystis jiroveci, 302, 303 
Sporothrix schenckii, 285, 307—311 
Sporozoite, 323 

Cryptosporidium, 323-324 

Plasmodium, 339-340 
Sputum culture 

for aspergillosis, 280 

for Klebsiella pneumoniae, 97 

for Pneumocystis jiroveci, 303 
Stain 

acid-fast 

for Cryptosporidium, 323 
for mycobacteria, 109 

an ti treponemal antibody, 165 

calcofluor white 
for Aspergillus, 280 
for Coccidioides immitis, 285 
for Pneumocystis jiroveci, 303 

direct fluorescent-antibody, 273 

Giemsa, 303 

hematoxylin-eosin, 86 

India ink 

for anthrax, 30 

for Cryptococcus neoformans, 298 

Kinyoun, 109 

methenamine silver, 303 

Warthin- Starry silver, 86 

Ziehl-Neelsen, 109 
Staphylococcus aureus, 67, 155-161, 

' 158/ 
Staphylococcus epidermitis, 156, 159 
Staphylococcus lugdunensis, 159 
Staphylococcus protein A, 159 
Staphylococcus saprophytics, 159 
Stavudine, 16r, 18 
Stool culture 

for Campylobacter, 47 

for Clostridium difficile, 59 

for Cryptosporidium parvum, 324 

for herpes simplex virus, 210 

for rotavirus, 256 

for Salmonella and Shigella, 144 
Strawberry cervix, 354 
Streptococcus, 149-154 

Enterococcus vs., 71 

Epstein-Barr virus infection vs., 
190 

Listeria monocytogenes vs., 102 
Streptococcus agalactiae, 150 
Streptococcus bovis, 73 
Streptococcus pneumoniae, 150, 151 
Streptococcus pyogenes, 150 
Strict anaerobes, 35 
Strongyloid.es, 314/ 



INDEX 



381 



Subclinical infection 

cytomegalovirus, 184 

hepatitis C, 196, 201 

poliovirus, 249 
Succinic acid, Bacteroides fragilis and, 35 
Sulfadiazine, 15/, 22? 
Sulfadoxine, 341 
Sulfamethizole, 15/ 
Sulfamethoxazole, 15/, 22/. See also 

Trimethoprim/sulfamethoxazole 
Sulfanilamide, 15/ 
Sulfixoxazole, 15/ 
Sulfonamides, 15/ 
Superantigen, 157 
Superoxide dismutase, Bacteroides fragilis 

and, 35 
Swarmer cells, Proteus species, 11 
Swimmer's itch, 34, 347 
Syncytia, 208 
Syphilis, 163-170 



Tabes dorsalis, 165, 167 
Tachycardia, 70, 254 
Tachypnea, 261 
Tazobactam, 36 
Tegument layer, capsid 

in cytomegalovirus, 185 

in Epstein-Barr virus, 191 
Telithromycin, 15/ 
Tellurite media, 66 
Tenofovir, 16/ 
Terbinaflne, 19/, 20 
Tertian malaria, 338 
Tertiary syphilis, 166-167 
Tetanus, 58 
Tetracycline, 15/ 

for Chlamydia trachomatis, 55, 118 

for Lyme disease, 41/ 

for Mycoplasma pneumoniae, 118 

for Vibrio vulnificus, 174 
Thayer Martin medium, 124 
Thiabendazole, 22/ 
Thiosulfate citrate bile salts sucrose (TCBS) 

agar, for Vibrio, 174 
Thrombocytopenia 

cytomegalovirus and, 185 

in Epstein-Barr virus infection, 192 
Thrush, 291 
Ticarcillin, 14/, 138 
Ticks, as disease vector, 40, 42, 44 
Tilt test, positive, 142 
Togaviridae, 4/ 
Toll-like receptor, 1 1 2 
Tongue, hairy leukoplakia of, 192 
Toxic shock syndrome toxin. Staphylococcus 

aureus, 157 
Toxocara canis, 316 
Toxocara cati, 316 
Toxoplasmosis, 22/ 
Tracheobronchitis, Mycoplasma pneumoniae 

in, 116 
Trachoma, 53-54 



Transplacental transfer. See Congenital 
infections 

Transposons, 70 

Traveler's diarrhea, 81 

Trematodes 

antiparasitic agents for, 22/ 
Schistosoma species, 345-351 

Treponema pallidum, 41/, 163-170, 166/ 

Triazolam, 20 

Trichomonas vaginalis, 353-358, 356? 

Trifluridine, 17/, 18 

Trimethoprim, 15/ 

Trimethoprim/sulfamethoxazole, 22/ 
for Escherichia coli, 78 
for Pneumocystis proved, 303 
for Salmonella, 144 
for staphylococcal infection, 159 
for typhoid fever, 144 

Trophozoite, 355 

Tuberculosis, 107-113 

Type B gastritis, 85 

Typhoid fever, 142 

Tzanck smear 

for herpes simplex virus, 210 
for vari eel la-zoster virus, 273 

U 

Umbilicate lesions, 230 

Upper endoscopy, 85 

Urea breath test, 86 

Ureaplasma urealyticum, 118 

Urease 

Helicobacter pylori and, 85, 86 
Proteus mirabilis and, 130 

Ureidopenicillins, 14/ 

Urethritis 

Chlamydia trachomatis in, 54 
Mycoplasma hominis in, 117—118 
Neisseria gonorrhoeae in, 122 
Trichomonas vaginalis in, 354 

Urinalysis 

for Enterococcus faecalis, 71 
for Escherichia coli, 78 
for Proteus species, 131 
for Schistosoma species, 348 

Urinary tract infection 

Enterococcus faecalis in, 69-72 
Escherichia coli in, 75—81 
Klebsiella pneumoniae in, 96 
Proteus mirabilis in, 129-133 
Pseudomonas aeruginosa in, 136 
Staphylococcus saprophytics in, 159 

Urine culture 

for Enterococcus faecalis, 71 
for Escherichia coli, 78 
for Proteus species, 131 



V factor medium, 9 1 
Vaccine 

adenovirus, 180 

anthrax, 30 

BCG, 111 



382 



INDEX 



Vaccine (Cont): 

diphtheria, 66 

Haemophilus influenzae type B, 93 

hepatitis A, 199 

hepatitis B, 200 

human immunodeficiency virus, 217 

human papillomavirus, 225 

Lyme disease, 42 

meningococcus, 124 

MMR, 235-236 

pneumococcal, 152 

polio, 248, 250 

rotavirus, 256 

smallpox, 265-266 

typhoid fever, 145 

varicella-zoster virus, 274 
Vaccinia, 266 
Vaginitis 

candidal, 289-294, 292? 

in trichomoniasis, 354 
Valacyclovir, 17, 17? 

for herpes simplex virus, 211 

for vari eel la-zoster virus, 273 
Vancomycin, 13, 15? 

for Clostridium difficile, 60 

for enterococci, 72 

for staphylococcal infection, 159 
Varicella-zoster virus, 17/, 18, 191?, 271-275 
Varices, schistosomiasis and, 347 
Variola major, 267 
Variola minor, 267 
Variola virus, 265-270 
Venereal Disease Research Laboratory 

(VDRL), 164, 167 
Vesicular lesions 

in herpes simplex virus infection, 208 

in shingles, 273 
Vibrio cholerae, 171—176 
Vibrio parahaemolyticus, 172-174 
Vibrio vulnificus, 172-174 
Vidarabine, 18 

Viral hepatitis. See Hepatitis viruses 
Virus(es), 3, 4/ 

adenovirus, 4/ 177-182, 180/ 

antiviral agents for, 16-19, 16?. See also 
specific agents 

cytomegalovirus, 18, 183-187, 191* 

Epstein-Barr virus, 189-194, 19lt 

herpes. See Herpesviruses 

human immunodeficiency virus. See Human 
immunodeficiency virus 



human papillomavirus, 221-226, 222?, 224/ 

mumps, 235-239 

parvovirus B 19, 241-246 

poliovirus, 248-252 

poxviruses, 230-231 

respiratory syncytial virus, 259-264 

rotavirus, 253-258 

smallpox, 265-270 

varicella-zoster virus, 17?, 18, 191?, 271-275 

variola virus, 265—270 
Visceral larvae migrans, 316 
Vomiting 

amphotericin B-related, 20 

in rotavirus infection, 256 

in Salmonella infection, 142 
Voriconazole, 19?, 20,281 

W 

Walking pneumonia, 116 

Warthin- Starry silver stain, 86 

Warts, 222?, 224-225, 224/ 

Weil syndrome, 41? 

Western blot test, for human immunodeficiency 

virus, 216 
Wet mount, for Trichomonas, 354, 355 
Whipworm, 3 14/ 
Wild poliovirus, 248 
Window period, in hepatitis B, 202-203 
Wool-sorter's disease, 28, 29 
Worm infestations 

Ascaris lumbricoides, 313-320, 314/ 319? 
Enterobius vermicularis, 315/ 329-335, 

333? 
Wound infections, Vibrio vulnificus in, 172 



X-factor medium, for Haemophilus influenzae, 
91 



Yeasts. See also Fungi 

Candida albicans, 289-294, 292? 

Cryptococcus neoformans, 285, 295-300 
Yersinia, 47 



Zalcitabine, 16? 

Zanamivir, 16? 

Zidovudine, 16?, 18 

Ziehl-Neelsen stain, fox Mycobacterium, 109 

Zoonosis, 323